You are on page 1of 168

Pan Malayan Insurance Corp. v. CA (Hilario) 2.

  Through the negligence of Erlinda Fabie’s (FABIE) unknown driver (he


April 3, 1990 | Cortes, J. | Subrogation was driving a pick-up), the insured car was hit and suffered damages worth
P42, 052.
PETITIONER: Pan Malayan Insurance Corp. 3.   Being the insurance company, Pan Malayan paid for the cost of repair of the
RESPONDENTS: Court of Appeals, Erlinda Fabie and her unkown driver insured car and was therefore subrogated to the rights of CANLUBANG
against the driver of the pick-up and his employer Erlind Fabie.
SUMMARY: Pan Malayan (PANMALAY) insured a Mitsubishi Colt Lancer. The 4.   Despite repeated demands, Fabie failed and refused to pay the claim of
car was registered under the name of Canlubang Automotive Resources Corporation PANMALAY, so PANMALAY filed a complaint for damages before the
(CANLUBANG). Through the negligence of Erlinda Fabie’s (FABIE) unknown RTC of Makati against Fabie and her driver.
driver (he was driving a pick-up), the insured car was hit and suffered damages 1.   PANMALAY said that the damage caused to the insured car was
worth P42, 052. Being the insurance company, Pan Malayan paid for the cost of settled uner the “own damage” coverage of the insurance policy,
repair of the insured car and was therefore subrogated to the rights of CANLUBANG and submitted a copy of the insurance policy and the Release of
against the driver of the pick-up and his employer Erlind Fabie. Despite repeated Claim and Subrogation Receipt executed by CANLUBANG in
demands, Fabie failed and refused to pay the claim of PANMALAY, so favor of PANMALAY.
PANMALAY filed a complaint for damages before the RTC of Makati against Fabie 5.   FABIE filed a Motion to Dismiss saying PANMALAY had no cause of
and her driver. PANMALAY said that the damage caused to the insured car was action against them, and claimed that the “own damage” clause of the
settled uner the “own damage” coverage of the insurance policy, and submitted a insurance policy precluded subrocation since indemnification in Art. 2207
copy of the insurance policy and the Release of Claim and Subrogation Receipt of the Civil Code1 was made on the assumption that there was no
executed by CANLUBANG in favor of PANMALAY. FABIE filed a Motion to wrongdoer or no third party at fault.
Dismiss saying PANMALAY had no cause of action against them, and claimed that 6.   RTC dismissed PANMALAY’s complaint for no cause of action, and this
the “own damage” clause of the insurance policy precluded subrogation since was upheld by the CA. So now, PANMALAY comes before the SC.
indemnification in Art. 2207 of the Civil Code was made on the assumption that
there was no wrongdoer or no third party at fault. RTC dismissed PANMALAY’s ISSUE/s:
complaint for no cause of action, and this was upheld by the CA. So now, 1.   WON PANMALAY (insurer) may institute an action to recover the amound
PANMALAY comes before the SC. Issue: WON PANMALAY (insurer) may it had paid CANLUBANG (assured) in settlement of an insurance claim
institute an action to recover the amound it had paid CANLUBANG (assured) in against FABIE? – YES, PANMALAY has cause of action because payment
settlement of an insurance claim against FABIE? YES, because payment by the by the insurer to the assured operates as an equitable assignment to the
insurer to the assured operates as an equitable assignment to the assured of all assured of all remedies with the latter may have against the third party
remedies with the latter may have against the third party whose negligence or whose negligence or wrongful act caused the loss.
wrongful act caused the loss. The right of subrogation is not dependent upon, nor
does it grow out of, any privity of contract or upon written assignment of claim-- it RULING: WHEREFORE, in view of the foregoing, the present petition is
accrues simply upon payment of the insurance claim by the insurer. See ratio 3 GRANTED. Petitioner's complaint for damages against private respondents is hereby
for exceptions to that rule (but none of them apply here J). REINSTATED. Let the case be remanded to the lower court for trial on the merits.

RATIO:
DOCTRINE: Payment by the insurer to the assured operates as an equitable 1.   Payment by the insurer (PANMALAYA) to the assured (CANLUBANG)
assignment to the assured of all remedies with the latter may have against the third operates as an equitable assignment to the assured (CANLUBANG) of
party whose negligence or wrongful act caused the loss. The right of subrogation all remedies which the latter may have against the third party whose
accrues simply upon payment of the insurance claim by the insurer. negligence or wrongful act caused the loss.

                                                                                                                       
FACTS: 1
 Art.  2207.  If  the  plaintiff's  property  has  been  insured,  and  he  has  received  indemnity  from  
1.   Pan Malayan (PANMALAY) insured a Mitsubishi Colt Lancer. The car was
the  insurance  company  for  the  injury  or  loss  arising  out  of  the  wrong  or  breach  of  contract  
registered under the name of Canlubang Automotive Resources Corporation complained   of,   the   insurance   company   shall   be   subrogated   to   the   rights   of   the   insured  
(CANLUBANG). against  the  wrongdoer  or  the  person  who  has  violated  the  contract.  If  the  amount  paid  by  
the   insurance   company   does   not   fully   cover   the   injury   or   loss,   the   aggrieved   party   shall   be  
entitled  to  recover  the  deficiency  from  the  person  causing  the  loss  or  injury.  
2.   The right of subrogation is not dependent upon, nor does it grow out of, any
privity of contract or upon written assignment of claim-- it accrues simply
upon payment of the insurance claim by the insurer.
3.   Exceptions to this rule:
1.   If the assured by his own act releases the wrongdoer or third party
liable for the loss or damage, from liability, the insurer's right of
subrogation is defeated; and
2.   Where the insurer pays the assured the value of the lost goods
without notifying the carrier who has in good faith settled the
assured's claim for loss, the settlement is binding on both the
assured and the insurer, and the latter cannot bring an action
against the carrier on his right of subrogation; and
3.   Where the insurer pays the assured for a loss which is not a risk
covered by the policy, thereby effecting "voluntary payment", the
former has no right of subrogation against the third party liable for
the loss. (But see ratio 5)
4.   (re: why the lower court is wrong in interpreting “own cause” clause; just in
case sir asks) It must be emphasized that the lower court's ruling that the
"own damage" coverage under the policy implies damage to the insured car
caused by the assured itself, instead of third parties, proceeds from an
incorrect comprehension of the phrase "own damage" as used by the
insurer. When PANMALAY utilized the phrase "own damage" — a phrase
which, incidentally, is not found in the insurance policy — to define the
basis for its settlement of CANLUBANG's claim under the policy, it simply
meant that it had assumed to reimburse the costs for repairing the
damage to the insured.
5.   Even if PANMALAY could not be deemed subrogated to the rights of its
assured under Article 2207 of the Civil Code, PANMALAY would still
have a cause of action against private respondents. In the pertinent case of
Sveriges Angfartygs Assurans Forening v. Qua Chee Gan, the Court ruled
that the insurer who may have no rights of subrogation due to
"voluntary" payment may nevertheless recover from the third party
responsible for the damage to the insured property under Article 1236
of the Civil Code.
002 FEDERAL EXPRESS CORPORATION v. AMERICAN HOME FACTS:
ASSURANCE COMPANY and PHILAM INSURANCE COMPANY, 1.   SMITHKLINE Beecham (SMITHKLINE) of Nebraska, USA delivered to
INC (HIRANG) Burlington Air Express (BURLINGTON), an agent of Federal Express
August 18, 2004 |Panganiban, J. | Subrogation Corporation, a shipment of 109 cartons of veterinary biologicals for
delivery to consignee SMITHKLINE and French Overseas Company in
Makati City, Metro Manila.
PETITIONER: FEDERAL EXPRESS CORPORATION 2.   The shipment reads REFRIGERATE WHEN NOT IN TRANSIT and
RESPONDENTS: AMERICAN HOME ASSURANCE COMPANY and PHILAM PERISHABLE stamp marked on its face
INSURANCE COMPANY, INC 3.   BURLINGTON insured the cargoes in the amount of $39,339.00 with
American Home Assurance Company (AHAC)
SUMMARY: SMITHKLINE, USA delivered to BURLINGTON (Agent of 4.   BURLINGTON turned over the custody of said cargoes to FEDERAL
FEDERAL EXPRESS) 109 cartons of veterinary biologicals for delivery to EXPRESS which transported the same to Manila.
consignee SMITHKLINE and French Overseas Company in Makati City, Metro 5.   They were shipped in two batches. The first one consist of 92 cartons and
Manila. The same was insured by BURLINGTON in the amount of $39,339.00 with the next one came 2 days after which consist of 17 cartons. These were
American Home Assurance Company (AHAC). BURLINGTON then subsequently immediately stored at Cargohaus Inc’s warehouse.
turned over the goods to FEDERAL EXPRESS which then shipped the cargoes to 6.   Twelve (12) days after the cargoes arrived in Manila, a non-licensed
the consignee SMITHKLINE and French Overseas Company in Makati City, Metro customs broker, DARIO C. DIONEDA (DIONEDA) who was assigned by
Manila. The cargoes was then stored in Cargohaus’ warehouse. The cargo reads GETC (Custom broker Corporation hired by SMITHKLINE) to facilitate
REFRIGERATE WHEN NOT IN TRANSIT and PERISHABLE stamp marked on the release of the subject cargoes, found out, that the goods were stored
its face. 12 days after the shipment, DARIO C. DIONEDA, the broker of the GETC only in a room with two (2) air conditioners running, to cool the place
(Custom Broker Corporation hired by SMITHKLINE to release the goods) found out instead of a refrigerator
that the goods were stored inside a room with only 2 aircondition units. Upon 7.   When he asked an employee of Cargohaus why the cargoes were stored in
instructions for GETC, Dioneda did not proceed with the withdrawal of the vaccines the cool room only, the latter told him that the cartons where the vaccines
but instead samples were taken to the Bureau of Animal Industry and there it was were contained specifically indicated therein that it should not be subjected
discovered that the ELISA reading of vaccinates sera are below the positive to hot or cold temperature.
reference serum. Because of the unusable vaccines, SMITHKLINE abandoned the 8.   Upon instructions from GETC, Dioneda did not proceed with the
shipment and filed a claim against AHAC thorught its representative in the withdrawal of the vaccines but instead samples were taken to the Bureau
Philippines, PHILAM. PHILAM paid SMITHKLINE and subsequently filed a case of Animal Industry and there it was discovered that the ELISA reading
for recovery of damages against FEDERAL EXPRESS and Cargohaus, imputing of vaccinates sera are below the positive reference serum.
negligence in handling the cargo. FEDERAL EXPRESS contends that PHILAM has 9.   As a result, SMITHKLINE abandoned the shipment and declared a total
no personality to sue and thus, it has no cause of action against it. The issue in this loss for the unusable shipment of vaccines. SMITHKLINE then filed a
case is WoN, PHILAM has the capacity to sue and the court held in the affirmative. claim against AHAC through its representative in the Philippines, Philam
The SC held that upon receipt of the insurance proceeds, SMITHKLINE executed a Insurance, the whole amount of the cargo.
subrogation Receipt in favor of PHILAM. PHILAM was thus authorized to file 10.   PHILAM paid SMITHKLINE the whole amount and subsequently it
claims and begin suit against any such carrier, vessel, person, corporation or (PHILAM) filed an action for damages against FEDERAL EXPRESS and
government. Upon payment to the consignee of an indemnity for the loss of or Cargohaus imputing negligence in handling the cargo.
damage to the insured goods, the insurers entitlement to subrogation pro tanto --
11.   The Trial Court ruled in favor of PHILAM. FEDERAL EXPRESS and
being of the highest equity -- equips it with a cause of action in case of a contractual
Cargohaus were held to be solidarily liable.
breach or negligence. Further, the insurers subrogatory right to sue for recovery
12.   The CA also ruled in favor of PHILAM stating that the goods were
under the bill of lading in case of loss of or damage to the cargo is jurisprudentially
delivered to FEDERAL EXPRESS in good condition as provided for in the
upheld.
shipping receipts. FEDERAL EXPRESS was not able to discharge the
presumption that the goods were indeed delivered to them in good condition
DOCTRINE: In the exercise of its subrogatory right, an insurer may proceed against
thus, creating a presumption that they were the one’s negligent in handling
an erring carrier. To all intents and purposes, it stands in the place and in substitution
the goods.
of the consignee. A fortiori, both the insurer and the consignee are bound by the
contractual stipulations under the bill of lading
13.   FEDERAL EXPRESS contends that PHILAM has no personality to Burlington. This document has thus been duly indorsed in blank and is
sue and thus, no cause of action against it - because the payment made deemed a bearer instrument.
to SMITHKLINE was erroneous.
2.   Since the Certificate was in the possession of Smithkline, the latter had the
ISSUE/s: right of collecting or of being indemnified for loss of or damage to the
-WoN payment made to SMITHKLINE was erroneous – NO, SMITHKLINE is insured shipment, as fully as if the property were covered by a special
the holder of the Certificate of Insurance and under this, it had the right of policy in the name of the holder. Hence, being the holder of the Certificate
collecting or of being indemnified for loss of or damage to the insured shipment and having an insurable interest in the goods, Smithkline was the proper
-WoN PHILAM had the personality to sue – YES, Upon receipt of the insurance payee of the insurance proceeds.
proceeds, Smithkline executed a subrogation Receipt in favor of respondents. The (CLAIM ALREADY PRESCRIBED)
latter were thus authorized to file claims and begin suit against any such carrier,
vessel, person, corporation or government. (SUBROGATION ISSUE) 1.   In this jurisdiction, the filing of a claim with the carrier within the time
-WoN Federal Express is liable for the damage to or loss of the insured limitation therefor actually constitutes a condition precedent to the accrual
products? – NO, SC stated that the reason they are not liable is because of the fact of a right of action against a carrier for loss of or damage to the goods. The
that the claim has already prescribed. The filing of the claim within the time limit is a shipper or consignee must allege and prove the fulfillment of the condition.
condition precedent to the accrual of a right of action against a carrier for loss of or If it fails to do so, no right of action against the carrier can accrue in favor
damage to the goods. of the former. The aforementioned requirement is a reasonable condition
precedent; it does not constitute a limitation of action
RULING: WHEREFORE, the Petition is GRANTED, and the assailed 2.   In this case, even from the very start of the trial, PHILAM never submitted
Decision REVERSED insofar as it pertains to Petitioner Federal Express any evidence to prove that it indeed made a complaint in writing when it
Corporation. No pronouncement as to costs. found out that the goods were damaged
3.   According to the Airway bill (contract of carriage), the person entitled
RATIO: to delivery must make a complaint in writing in case of visible and
(SUBROGATION ISSUE) other damages within 14 days from the receipt of the goods.
1.   Upon receipt of the insurance proceeds, Smithkline executed a subrogation 4.   Nothing from the records show that PHILAM indeed made a complaint.
Receipt in favor of PHILAM. The latter was thus authorized to file claims 5.   The requirement of giving notice of loss of or injury to the goods is not an
and begin suit against any such carrier, vessel, person, corporation or empty formalism. The fundamental reasons for such a stipulation are (1) to
government. inform the carrier that the cargo has been damaged, and that it is being
2.   Upon payment to the consignee of an indemnity for the loss of or damage to charged with liability therefor; and (2) to give it an opportunity to examine
the insured goods, the insurers entitlement to subrogation pro tanto -- being the nature and extent of the injury. This protects the carrier by affording it
of the highest equity -- equips it with a cause of action in case of a an opportunity to make an investigation of a claim while the matter is fresh
contractual breach or negligence. Further, the insurers subrogatory right to and easily investigated so as to safeguard itself from false and fraudulent
sue for recovery under the bill of lading in case of loss of or damage to the claims
cargo is jurisprudentially upheld 6.   When an airway bill -- or any contract of carriage for that matter -- has a
3.   In the exercise of its subrogatory right, an insurer may proceed against an stipulation that requires a notice of claim for loss of or damage to goods
erring carrier. To all intents and purposes, it stands in the place and in shipped and the stipulation is not complied with, its enforcement can be
substitution of the consignee. A fortiori, both the insurer and the consignee prevented and the liability cannot be imposed on the carrier. To stress,
are bound by the contractual stipulations under the bill of lading notice is a condition precedent, and the carrier is not liable if notice is not
given in accordance with the stipulation. Failure to comply with such a stipulation
OTHER ISSUES: bars recovery for the loss or damage suffered.
(ERRONEOUS PAYMENT TO SMITHKLINE)
1.   The Certificate specifies that loss of or damage to the insured cargo is
payable to order x x x upon surrender of this Certificate. Such wording
conveys the right of collecting on any such damage or loss, as fully as if the
property were covered by a special policy in the name of the holder itself.
At the back of the Certificate appears the signature of the representative of
03 FIREMAN’S FUND INSURANCE CO vs. JAMILA (LAGUILLES) 5.   The lower court dismissed the complaint as to Jamila on the ground that
April 7, 1976 | Aquino, J. | Art. 2207 there was no allegation that it had consented to the subrogation, and,
therefore, Fireman’s Fund had no cause of action against it.
PETITIONER: Fireman’s Fund Insurance Company and Firestone Tire and 6.   The lower court also dismissed the complaint as to First Quezon City
Rubber Company of the Philippines Insurance Co on the ground of res judicata. It appears that the same action
RESPONDENTS: Jamila & Company, Ic. and First Quezon City Insurance Co was previously filed which was dismissed because of the failure of the
Inc. plaintiffs to appear at pre-trial.
7.   Firestone and Fireman’s Fund moved for the reconsideration of the order of
SUMMARY: Jamila contracted to supply security guards to Firestone for dismissal, and the lower court set aside its order of dismissal. It sustained
which Jamila assumed responsibility for the acts of its security guards. With plaintiff’s contention that there was no res judicata as to First Quezon City
this, First Quezon City Insurance Co (surety) executed a bond in the sum of Insurance Co because the case was dismissed without prejudice.
P20,000 to guarantee Jamila’s obligations under the contract. Firestone 8.   Due to inadvertence, the lower court did not state why it set aside its prior
properties worth P19,000 were lost allegedly due to the acts of its employees order dismissing the complaint with respect to Jamila.
who connived with Jamila’s security guards. Fireman’s Fund as insurer, paid to 9.   Jamila, upon noticing that the order had obliterated its victory without any
Firestone the amount of the loss and Fireman’s Fund was subrogated reason therefore, filed a motion for reconsideration. It originally moved for
(substituted) to Firestone’s right to get reimbursed from Jamila. However, the dismissal of the complaint on the ground of lack of cause of action. Its
Jamila and its surety failed to pay. The lower court dismissed the complaint as contention was based on two grounds:
to Jamila on the ground that the latter never consented to the subrogation, hence a.   that the complaint did not allege that Firestone, pursuant to the
there is no cause of action. The issue is WoN the complaint states a cause of contractual stipulation quoted in the complaint, had investigated
action against Jamila. the loss and that Jamila was represented in the investigation
b.   That Jamila did not consent to the subrogation of Fireman’s Fund
The SC held that yes, there is a cause of action because as insurer, Fireman’s to Firestone’s right to get reimbursement from Jamila and its
Fund is entitled to go after the person or entity that violated its contractual surety.
commitment. Subrogation is a normal incident of indemnity insurance. Upon 10.   The lower court sustained the second ground. However, Jamila in its MR
payment of the loss, the insurer is entitled to be subrogated pro tanto to any invoked the first ground which was never passed upon by the lower court.
right of action which the insured may have against the third person whose However, the lower court completely ignored the first ground and reverted
negligence or wrongful act caused the loss. The loss in the first instance is that to the second ground which was relied upon in its order.
of the insured but after reimbursement, it becomes the loss of the insurer. 11.   The lower court reiterated that Fireman’s Fund had no cause of action
against Jamila because Jamila did not consent to the subrogation.
DOCTRINE: Upon payment of the loss, the insurer is entitled to be subrogated 12.   Plaintiffs filed a second motion for reconsideration and called the lower
pro tanto to any right of action which the insured may have against the third court’s attention to the fact that the issue of subrogation was of no moment
person whose negligence or wrongful act caused the loss. because Firestone, the subrogor, is a party-plaintiff and could sue directly
Jamila in its own right.
FACTS: 13.   The plaintiffs cited Art. 2207 of the Civil Code which provides:
1.   Jamila or the Veterans Philippine Scouts Security Agency contracted to “If the plaintiff’s property has been insured, and he has received
supply security guards to Firestone for which Jamila assumed responsibility indemnity from the insurance company for the injury or loss arising out
for the acts of its security guards. of the wrong or breach of contract complained of, the insurance
2.   First Quezon City Insurance Co., Inc. executed a bond in the sum of company shall be subrogated to the rights of the insured against the
P20,000.00 to guarantee Jamila’s obligations under the contract. wrongdoer or the person who has violated the contract.”
3.   However, properties of Firestone valued at P11,925.00 were lost allegedly 14.   Firestone and Fireman’s Fund contend that the trial court’s dismissal of
due to the acts of its employees who connived with Jamila’s security their complaint is contrary to the aforementioned provision which provides
guards. for legal subrogation.
4.   Fireman’s Fund, as insurer, paid to Firestone the amount of the loss and
Fireman’s Fund was subrogated to Firestone’s right to get reimbursement ISSUE/s:
from Jamila, and Jamila and its surety, First Quezon City Insurance Co, 1.   WoN the complaint of Firestone and Fireman’s Fund states a cause of
failed to pay the amount of the loss in spite of repeated demands. action against Jamila – YES, because as insurer, Fireman’s Fund is entitled
to go after the person or entity that violated its contractual commitment. equitable assignment to the insurer of the property and all remedies which
the insured may have for the recovery thereof.

RULING: Finding the trial court's order of dismissal to be legally untenable, the
same is SET ASIDE with costs against defendant-appellee Jamila & Co., Inc.

RATIO:
1.   Jamila is of the opinion that legal subrogation under Art. 2207 requires the
debtor’s consent; that legal subrogation takes place in the cases mentioned
in Art. 1302 of the Civil Code and the instant case is not among the three
cases enumerated in that article.
2.   The Court holds that Fireman’s Fund’s action against Jamila is sanctioned
by Art. 22071. As the insurer, Fireman’s Fund is entitled to go after the
person or entity that violated its contractual commitment to answer for
the loss insured against.
3.   The lower court erred in applying to this case the rules on novation. The
plaintiffs in alleging in their complaint that Fireman’s Fund became a party
in interest in this case by virtue of a subrogation right given in its favor by
Firestone, were not relying on the novation by change of creditors as
contemplated in Arts. 1291, and 1300 to 1303 of the Civil Code, but rather
on Art. 2207.
4.   Subrogation has been referred to as the doctrine of substitution. It is an arm
of equity that may guide or even force one to pay a debt for which an
obligation was incurred but which was in whole or in part paid by another.
5.   Subrogation is founded on principles of justice and equity, and its operation
is governed by principles of equity. It rests on the principle that substantial
justice should be attained regardless of form, that is, its basis is the doing of
complete, essential, and perfect justice between all the parties without
regard to form.
6.   Subrogation is a normal incident of indemnity insurance. Upon payment of
the loss, the insurer is entitled to be subrogated pro tanto to any right of
action which the insured may have against the third person whose
negligence or wrongful act caused the loss.
7.   The loss in the first instance is that of the insured but after reimbursement, it
becomes the loss of the insurer.
8.   Although many policies now provide for subrogation, and thus determine
the rights of the insurer in this respect, the equitable right of subrogation as
the legal effect of payment inures to the insurer without any formal
assignment or any express stipulation to that effect in the policy.
9.   When the insurance company pays for the loss, such payment operates as an

                                                                                                                       
1
  Art.  2207.  If  the  plaintiff’s  property  has  been  insured,  and  he  has  received  indemnity  from  
the  insurance  company  for  the  injury  or  loss  arising  out  of  the  wrong  or  breach  of  contract  
complained   of,   the   insurance   company   shall   be   subrogated   to   the   rights   of   the   insured  
against  the  wrongdoer  or  the  person  who  has  violated  the  contract.”  
004 FF. Cruz & Co., Inc v. CA (Marcos) the contents thereof.
Aug. 29, 1988 | Cortes, J. | Subrogation / Art. 2207 7.   Subsequently, the family filed an action for damages against FF. Cruz.
8.   The CFI ruled in favor of the Mable family, and later on the CA affirmed
PETITIONER: F.F. Cruz and Co., Inc. the decision of the trial court but reduced the award for damages.
RESPONDENTS: The Court of Appeals, Gregorio Mable as substituted by his 9.   Both Courts did not deduct the P35,000 from the award of damages.
wife Luz Almonte Mable and children Doming, Leonidas, Ligaya, Elena, 10.   FF. Cruz filed a petition for review with the SC after its MR with the CA
Gregorio, Jr., Salome, Antonio, and Bernardo all surnamed Mable. was dnid.
11.   FF. Cruz contends that the CA erred in not deducting the P35,000 recovered
SUMMARY: The manufacturing shop of FF. Cruz and the house of Mable was insurance from the award of damages, in awarding excessive and unproved
situated adjacent with each other. Mable repeatedly requested that a firewall be damages, and in applying the doctrine of res ipsa loquitur.
constructed between the shop and their house but such request fell on deaf ears.
A fire broke in the shop and it then spread to the residence. Both the shop and ISSUE/s:
the house were razed to the ground. The Mable family was able to recover 1.   WoN FF. Cruz is still liable to pay for the 35,000 recovered by the Mable
P35,000 from its insurance on the house. The family then filed an action for Family from its insurance – No, because Art. 2207 of the Civil Code
damages against FF. Cruz. The CFI ruled in favor of the family, and later on the provides that it should be deducted from the amount of award as damages.
CA affirmed the decision. Both Courts did not deduct the P35,000 from the
award of damages. The issue before the SC is WoN FF. Cruz is still liable to pay RULING: WHEREFORE, in view of the foregoing, the decision of the Court of
for the 35,000 recovered by the Mable Family from its insurance – No, because Appeals is hereby AFFIRMED with the following modifications as to the damages
Art. 2207 of the Civil Code provides that it should be deducted from the amount awarded for the loss of private respondents' house, considering their receipt of
of award as damages. Having been indemnified by their insurer, The Mable P35,000.00 from their insurer.
family are only entitled to recover the deficiency from FF. Cruz. The insurer, if it
is so minded, may seek reimbursement of the amount it indemnified the family RATIO:
from FF. Cruz. Under Article 2207, the real party in interest with regard to the 1.   Doctrine of res ipsa loquitur: Where the thing which caused the injury
indemnity received by the insured is the insurer. Whether or not the insurer complained of is shown to be under the management of the defendant or his
should exercise the rights of the insured to which it had been subrogated lies servants and the accident is such as in the ordinary course of things does not
solely within the former's sound discretion. happen if those who have its management or control use proper care, it
affords reasonable evidence, in the absence of explanation by the defendant,
DOCTRINE: The insurer, if it is so minded, may seek reimbursement of the that the accident arose from want of care.
amount it indemnified the family from FF. Cruz. This is the essence of its right 2.   The facts of the case likewise call for the application of the doctrine,
to be subrogated to the rights of the insured, as expressly provided in Article considering that in the normal course of operations of a furniture
2207. Upon payment of the loss incurred by the insured, the insurer is entitled to manufacturing shop, combustible material such as wood chips, sawdust,
be subrogated pro tanto to any right of action which the insured may have paint, varnish and fuel and lubricants for machinery may be found thereon
against the third person whose negligence or wrongful act caused the loss. 3.   It must also be noted that negligence or want of care on the part of
petitioner or its employees was not merely presumed.
4.   The Court of Appeals found that petitioner failed to construct a firewall
FACTS: between its shop and the residence of private respondents as required by a
1.   The furniture manufacturing shop of FF. Cruz in Caloocan was situated city ordinance; that the fire could have been caused by a heated motor or a
adjacent to the resident of respondent Mable family. lit cigarette; that gasoline and alcohol were used and stored in the shop; and
2.   Sometime in Aug 1971, Mable approached Cruz, the plant manager, to that workers sometimes smoked inside the shop
request that a firewall be constructed between the shop and the residence. 5.   In the instant case, with more reason should petitioner be found guilty of
3.   The request was repeated several times but they fell on deaf ears. negligence since it had failed to construct a firewall between its property
4.   In the morning of Sept. 6, 1974, fire broke in the shop which then spread to and private respondents' residence which sufficiently complies with the
the house of the Mable family. Both the shop and the house were razed to pertinent city ordinances. The failure to comply with an ordinance
the ground. The cause of the conflagration was never discovered. providing for safety regulations had been ruled by the Court as an act of
5.   The National Bureau of Investigation found specimens from the burned negligence
structures negative for the presence of inflammable substances. 6.   [INSURANCE TOPIC] While this Court finds that petitioner is liable for
6.   The Mable family collected P35,000.00 on the insurance on their house and
damages to private respondents as found by the Court of Appeals, the fact
that private respondents have been indemnified by their insurer in the
amount of P35,000.00 for the damage caused to their house and its
contents has not escaped the attention of the Court.
7.   Hence, the Court holds that in accordance with Article 22072 of the
Civil Code the amount of P35,000.00 should be deducted from the
amount awarded as damages.
8.   Having been indemnified by their insurer, The Mable family are only
entitled to recover the deficiency from FF. Cruz.
9.   The insurer, if it is so minded, may seek reimbursement of the amount
it indemnified the family from FF. Cruz. This is the essence of its right
to be subrogated to the rights of the insured, as expressly provided in
Article 2207. Upon payment of the loss incurred by the insured, the
insurer is entitled to be subrogated pro tanto to any right of action
which the insured may have against the third person whose negligence
or wrongful act caused the loss
10.   Under Article 2207, the real party in interest with regard to the
indemnity received by the insured is the insurer
11.   Whether or not the insurer should exercise the rights of the insured to
which it had been subrogated lies solely within the former's sound
discretion.
12.   Since the insurer is not a party to the case, its identity is not of record and
no claim is made on its behalf, the private respondent's insurer has to claim
his right to reimbursement of the P35,000.00 paid to the insured.

                                                                                                                       
2
  Art. 2207. If the plaintiffs property has been insured, and he has received indemnity from the insurance
company for the injury or loss arising out of the wrong or breach of contract complained of, the insurance
company is subrogated to the rights of the insured against the wrongdoer or the person who violated the
contract. If the amount paid by the insurance company does not fully cover the injury or loss, the
aggrieved party shall be entitled to recover the deficiency from the person causing the loss or injury.    
 
005 RIZAL SURETY & INSURANCE CO v. MANILA RAILROAD violated the contract.”
COMPANY AND MANILA PORT SERVICES (MATSUMURA) In this case, Rizal Surety cannot recover from Manila Railroad and Manila Port Services
April 25, 1968 | Fernando, J. | Subrogation an amount greater than that to which the consignee could lawfully lay claim.

PETITIONER: Rizal Surety & Insurance Company


FACTS
RESPONDENTS:Manila Railroad Company and Manila Port Service
1.   On or before November 29, 1960 the vessel, SS Flying Trader, loaded on
SUMMARY: On November 29, 1960, the SS Flying Trader loaded for shipment several Board at Genoa, Italy for shipment to Manila, Philippines.
cargo from Genoa Italy to Manila. Among the cargo was 6 cases of OMH containing 2.   Among the cargo was 6 cases of OMH, Special Single Colour Offset Press
Special Single Colour Offset Press Machines. When the vessel arrived in Manila, the Machine where Bill of Lading No. 1 was issued. Such machine was
cargo was discharged completely and in good order to the custody of Manila Port services consigned to Suter, Inc.
(Arrastre operator). While the cargo was being lifted and loaded into Manila Port 3.   When the vessel arrived at the Port of Manila on January 16, 1961, the
Services’ truck via crane, one of the cases was dropped by the crane resulting to the cargo was discharged complete and in good order into the custody of
damage of one of the Special Single Colour Offset Press Machines. Rizal Surety (insurer)
Manila Port Service as arrastre operator.
paid P16,500 to Suter, Inc (consignee) representing the cost of the replacement and repair
of the machine, plus P180.70 to the International Adjustment Bureau as adjuster’s fee. 4.   In the course of the handling, one of the six cases identified as Case No.
2143 containing the OMH, Special Single Colour Offset Press was dropped
Clause 15 of the management contract between Suter, and Manila Railroad Co and by the crane as it was being lifted and loaded of the Manila Port Service
Manila Port Services appeared in the delivery permit of the cargo. The permit stated that into the consignee's truck. Because of this, the machine was heavily
“presented subject to all the terms and conditions of the Management Contract between damaged.
the Bureau of Customs and Manila Port Service and amendments thereto or alterations 5.   Rizal Surety, as insurer, paid
thereof, particularly but not limited to paragraph 15 thereof limiting the Company
liability to P500.00 per package, unless the value of the goods is otherwise, specified, a.   P16,500.00 to consignee, Suter, Inc. — representing damages by
declared or manifested and the corresponding arrastre charges have been paid, . . .” way of costs of replacement parts and repairs to put the machine in
working condition,
Rizal Surety now filed a suit against Manila Railroad and Manila Port Services to recover b.   P180.70 to the International Adjustment Bureau — as adjuster's
the amount it paid to Suter arguing that as the insurance company it has been subrogated fee for the survey conducted on the damaged cargo
to the rights of the insured. The lower court ruled that Rizal Surety is entitled to recovery 6.   In total, Rizal Surety paid a total of P16,680.70 representing its liability
but only to the extent of P500 as stated in Clause 15. Hence, this appeal by Rizal Surety
under the insurance contract. The arrastre charges in this particular
arguing that it should be entitled to recover the full amount.
shipment was paid on the weight or measurement basis whichever is higher,
The SC ruled that Rizal Surety is only entitled to P500. The literal language of Article and not on the value thereof.
2207 does not warrant such an interpretation. It is clear that in the event that the property 7.   Clause 15 of the management contract appeared "at the dorsal part of the
has been insured and the Insurance Company has paid the indemnity for the injury or loss Delivery Permit" and was "used in taking delivery of the subject shipment
sustained, it "shall be subrogated to the rights of the insured against the wrong-doer or the from the Manila Port Service and Manila Railroad Co.’s custody and
person who has violated the contract.” In this case, the management contract is clear that control, issued in the name of consignee's broker," contained what was
the amount is limited to P500 per package. While yes the management contract stated
referred to as "an important notice."
that a greater value could be recovered when it stated “unless the value of the goods is
otherwise specified, declared, or manifested”, Suter (consignee) failed to do so. Hence, 8.   Such permit is “presented subject to all the terms and conditions of the
since the insurer is subrogated merely to the rights of the consignee its recovery Management Contract between the Bureau of Customs and Manila Port
necessarily should be limited to what was recoverable by the insured -in this case P500 Service and amendments thereto or alterations thereof, particularly but not
because no other value was declared. limited to paragraph 15 thereof limiting the Company liability to P500.00
per package, unless the value of the goods is otherwise, specified, declared
DOCTRINE: Under Article 2207, in the event that the property has been insured and the or manifested and the corresponding arrastre charges have been paid, . . .”
Insurance Company has paid the indemnity for the injury or loss sustained, it "shall be
subrogated to the rights of the insured against the wrong-doer or the person who has
9.   Rizal Surety filed a suit against Manila Railroad and Manila Port Services said payment — is not subject to the provisions of said stipulation, and that
to recover the amount it paid to consignee, Suter, Inc. based on the Civil the same is furthermore invalid. The lower court correctly rejected this
Code wherein it is tested that the Insurance Company "shall be subrogated pretense because, having taken delivery of the shipment aforementioned by
to the rights of the insured”. Rizal Surety argues that it is entitled to the virtue of a delivery permit, incorporating thereto, by reference, the
amount paid by it in full, by virtue of the insurance contract. provisions of said management contract, particularly paragraph 15 thereof,
10.   The lower court ruled that the Manila Railroad and Manila Port Services the gist of which was set forth in the permit, the consignee became bound
should jointly and severally pay Rizal Surety P500 with legal interest. by said provisions, and because it could have avoided the application of said
11.   Rizal Surety appealed the decision arguing that under the civil code, it could maximum limit of P500.00 per package by stating the true value thereof in
recover in full. Hence, this petition. (Note, the case didn’t mention what its claim for delivery of the goods in question, which admittedly, the
else happened in the lower court. It went straight to the SC decision) consignee failed to do.
ISSUE/S: 6.   Rizal Surety and Insurance Company having been subrogated merely to the
1.   W/N Rizal Surety is entitled to recover the full amount - NO because Rizal rights of the consignee its recovery necessarily should be limited to what
Surety having been subrogated merely to the rights of the consignee (Suter, was recoverable by the insured.
Inc) which is only P500 per package. Rizal Surety could not recover more 7.   The lower court therefore did not err when in the decision appealed from, it
because Suter as the consignee did not declare a higher amount which, limited the amount which defendants were jointly and severally to pay
under Management Contract, should have been done by Suter if it wishes to plaintiff-appellant to "Five Hundred Pesos (P500.00) with legal interest
be entitled to a greater amount. thereon from January 31, 1962, the date of the filing of the complaint, . . . "

RULING: WHEREFORE, the decision appealed from is affirmed. With costs


against Rizal Surety and Insurance Company.

RATIO:
1.   The literal language of Article 2207 does not warrant such an interpretation.
It is clear that in the event that the property has been insured and the
Insurance Company has paid the indemnity for the injury or loss sustained,
it "shall be subrogated to the rights of the insured against the wrong-doer or
the person who has violated the contract.”
2.   In this case, Rizal Surety cannot recover from Manila Railroad and Manila
Port Services an amount greater than that to which the consignee could
lawfully lay claim.
3.   The management contract is clear. The amount is limited to Five Hundred
Pesos (P500.00).
4.   Such a stipulation has invariably received the approval of this Court from
the leading case of Bernabe & Co. v. Delgado Bros., Inc. Such a decision
was quoted with approval in the following subsequent cases: Atlantic
Mutual Insurance Co. v. Manila Port Service, Insurance Service Co. of
North America v. Manila Port Service, Insurance Company of North
America v. U.S. Lines, Co., and Insurance Company of North America v.
Manila Port Service.
5.   Rizal Surety maintains that, not being a party to the management contract,
the consignee — into whose shoes plaintiff had stepped in consequence of
PIONEER INSURANCE V CA (ARMAND) cover the loss, then the aggrieved party is the one entitled to recover the
July 28, 1989| Gutierrez, J. | Subrogation deficiency. Evidently, under this legal provision, the real party in interest with regard
PETITIONER: Pioneer Insurance & Surety Corporation to the portion of the indemnity paid is the insurer and not the insured
RESPONDENTS: Hon. Court of Appeals, Border Machinery & Heavy Equipment FACTS:
Inc, Constancio M. Maglana and Jacob S. Lim 7.   This case involves consolidated petitions, one of which came from the
SUMMARY: Lim is an owner-operator of Southern Airlines (SAL). Japan Domestic decision of the CA which modified the decision of the then CFI of Manila.
Airlines (JDA) and Lim entered into a sales contract. Pioneer Insurance and Surety 8.   In 1965, Jacob S. Lim (petitioner in G.R. No. 84157) was engaged in the
Corp. as surety executed its surety bond in favor of JDA on behalf of its principal airline business as owner-operator of Southern Air Lines (SAL) a single
Lim. Border Machinery and Heacy Equipment Co, Inc., Francisco and Modesto proprietorship.
Cervantes, and Constancio Maglana contributed funds based on the misrepresentation 9.   On May 17, 1965, at Tokyo, Japan, Japan Domestic Airlines (JDA) and
of Lim that they will form a new corporation to expand his business. They executed Lim entered into and executed a sales contract for the sale and purchase of
two separate indemnity agreements in favor of Pioneer, one signed by Maglana and two (2) aircrafts and one (1) set of necessary spare parts for the total agreed
the other jointly signed by Lim for SAL, Bormaheco and the Cervanteses. The price of US $109,000.00 to be paid in installments.
indemnity agreements stipulated that the indemnitors principally agree and bind 10.   On May 22, 1965, Pioneer Insurance and Surety Corporation (Pioneer) as
themselves jointly and severally to indemnify and hold and save Pioneer from and surety executed and issued its Surety Bond No. 6639 in favor of JDA, in
against any/all damages, losses, etc. of whatever kind and nature may incur in behalf of its principal, Lim, for the balance price of the aircrafts and spare
consequence of having become surety. Lim executed in favor of Pioneer a deed of parts.
chattel mortgage as security. Upon default on the payments, Pioneer paid for him and 11.   It appears that Border Machinery and Heavy Equipment Company, Inc.
filed a petition for the foreclosure of chattel mortgage as security. Maglana, (Bormaheco), Francisco and Modesto Cervantes (Cervanteses) and
Bormaheco and the Cervantes’s filed cross-claims against Lim alleging that they Constancio Maglana contributed some funds used in the purchase of the
were not privies to the contracts signed by Lim and for recovery of the sum of money above aircrafts and spare parts. The funds were supposed to be their
they advanced to Lim for the purchase of the aircrafts. The decision was rendered contributions to a new corporation proposed by Lim to expand his airline
holding Lim liable to pay. The issues are (1) Whether or not the CA erred when it business.
dismissed the appeal of Pioneer on the sole ground that it had already collected 12.   They executed two (2) separate indemnity in favor of Pioneer, one signed
the proceeds of the reinsurance on its bond in favor of the JDA and that it by Maglana and the other jointly signed by Lim for SAL, Bormaheco and
cannot represent a reinsurer to recover the amount from herein respondents – the Cervanteses. The indemnity agreements stipulated that the indemnitors
NO. Pioneer was already paid by the reinsurers the amount of P295,000, and had principally agree and bind themselves jointly and severally to indemnify
already foreclosed the mortgaged chattels for the amount of 37,050, thus Pioneer is and hold and save harmless Pioneer from and against any/all damages,
already overpaid by P33,383, and has no more claim against the defendants. Pioneer, losses, costs, damages, taxes, penalties, charges and expenses of whatever
having foreclosed the chattel mortgage on the planes and spare parts, no longer has kind and nature which Pioneer may incur in consequence of having become
any further action against the defendants as indemnitors to recover any unpaid surety upon the bond/note and to pay, reimburse and make good to Pioneer,
balance. The indemnity agreement was ipso jure extinguished upon the foreclosure of its successors and assigns, all sums and amounts of money which it or its
the chattel mortgage. These defendants, as indemnitors, would be entitled to be representatives should or may pay or cause to be paid or become liable to
subrogated to the right of Pioneer should they make payments to the latter. pay on them of whatever kind and nature.
(2) Whether or not Maglana, Bormaheco and the Cervanteses must share in the 13.   On June 10, 1965, Lim doing business under the name and style of SAL
loss of the venture in proportion to their contribution - NO. There was no de facto executed in favor of Pioneer as deed of chattel mortgage as security for
partnership. Ordinarily, when co-investors agreed to do business through a the latter's suretyship in favor of the former. It was stipulated therein that
corporation but failed to incorporate, a de facto partnership would have been formed, Lim transfer and convey to the surety the two aircrafts. The deed was duly
and as such, all must share in the losses and/or gains of the venture in proportion to registered with the Office of the Register of Deeds of the City of Manila
their contribution. But in this case, it was shown that Lim did not have the intent to and with the Civil Aeronautics Administration pursuant to the Chattel
form a corporation with Maglana et al. Mortgage Law and the Civil Aeronautics Law (Republic Act No. 776).
DOCTRINE: In general a reinsurer, on payment of a loss, acquires the same rights 14.   Lim defaulted on his subsequent installment payments prompting JDA to
by subrogation as are acquired in similar cases where the original insurer pays a loss. request payments from the surety. Pioneer paid a total sum of P298,626.12.
If a property is insured and the owner receives the indemnity from the insurer, it is 15.   Pioneer then filed a petition for the extrajudicial foreclosure of the said
provided in said article that the insurer is deemed subrogated to the rights of the chattel mortgage before the Sheriff of Davao City. The Cervanteses and
insured against the wrongdoer and if the amount paid by the insurer does not fully
Maglana, however, filed a third party claim alleging that they are co-owners represent a reinsurer to recover the amount from herein respondents –
of the aircraft. NO. Pioneer was already paid by the reinsurers the amount of P295,000,
16.   On July 19, 1966, Pioneer filed an action for judicial foreclosure with an and had already foreclosed the mortgaged chattels for the amount of 37,050,
application for a writ of preliminary attachment against Lim and thus Pioneer is already overpaid by P33,383, and has no more claim against
respondents, the Cervanteses, Bormaheco and Maglana. the defendants.
17.   In their Answers, Maglana, Bormaheco and the Cervanteses filed cross- 3.   Whether or not Maglana, Bormaheco and the Cervanteses must share
claims against Lim alleging that they were not privies to the contracts in the loss of the venture in proportion to their contribution - NO. There
signed by Lim and, by way of counterclaim, sought for damages for being was no de facto partnership. Ordinarily, when co-investors agreed to do
exposed to litigation and for recovery of the sums of money they advanced business through a corporation but failed to incorporate, a de facto
to Lim for the purchase of the aircrafts in question. partnership would have been formed, and as such, all must share in the
18.   After trial, a decision was rendered holding Lim liable to pay Pioneer but losses and/or gains of the venture in proportion to their contribution. But in
dismissed Pioneer's complaint against all other defendants. this case, it was shown that Lim did not have the intent to form a
19.   As stated earlier, the appellate court modified the trial court's decision in corporation with Maglana et al.
that the plaintiffs complaint against all the defendants was dismissed. In all
other respects the trial court's decision was affirmed. RULING: WHEREFORE, the instant petitions are DISMISSED. The questioned
20.   The appellate court ruled that Pioneer's contention that it is representing the decision of the Court of Appeals is AFFIRMED.
reinsurer to recover the amount from defendants, hence, it instituted the
action is utterly devoid of merit. Plaintiff did not even present any evidence RATIO:
that it is the attorney-in-fact of the reinsurance company, authorized to 6.   As for the first case, the payment to the petitioner made by the reinsurers
institute an action for and in behalf of the latter. Moreover, the appellate was not disputed in the appellate court. Considering this admitted payment,
court ruled that Pioneer cannot be considered as the real party in interest as the only issue that cropped up was the effect of payment made by the
it has already been paid by the reinsurer the sum of P295,000.00 — the bulk reinsurers to the petitioner. Therefore, the petitioner's argument that the
of defendants' alleged obligation to Pioneer. It also ruled that in addition to respondents had no interest in the reinsurance contract as this is strictly
the said proceeds of the reinsurance received by plaintiff Pioneer from its between the petitioner as insured and the reinsuring company pursuant to
reinsurer, the former was able to foreclose extra-judicially one of the subject Section 91 (should be Section 98) of the Insurance Code has no basis.
airplanes and its spare engine, realizing the total amount of P37,050.00 7.   Hence the applicable law is Article 2207 of the new Civil Code, to wit:
from the sale of the mortgaged chattels. Adding the sum of P37,050.00, to Art. 2207. If the plaintiffs property has been insured, and he has received indemnity from the
insurance company for the injury or loss arising out of the wrong or breach of contract
the proceeds of the reinsurance amounting to P295,000.00, it is patent that
complained of, the insurance company shall be subrogated to the rights of the insured against
plaintiff has been overpaid in the amount of P33,383.72 considering that the the wrongdoer or the person who has violated the contract. If the amount paid by the insurance
total amount it had paid to JDA totals to only P298,666.28. To allow company does not fully cover the injury or loss, the aggrieved party shall be entitled to recover
plaintiff Pioneer to recover from defendants the amount in excess of the deficiency from the person causing the loss or injury.
P298,666.28 would be tantamount to unjust enrichment 8.   Interpreting the aforesaid provision, we ruled in the case of Phil. Air Lines,
21.   Pioneer contends that (1) it is at a loss where respondent court based its Inc. v. Heald Lumber Co. which we subsequently applied in Manila
finding that Pioneer was paid by its reinsurer, (2) even assuming that it was Mahogany Manufacturing Corporation v. Court of Appeals:
Note that if a property is insured and the owner receives the indemnity from the insurer, it is
paid by its reinsurer, none of the respondents had any interest in the matter
provided in said article that the insurer is deemed subrogated to the rights of the insured against
since reinsurance is between the petitioner and the reinsurer pursuant to the wrongdoer and if the amount paid by the insurer does not fully cover the loss, then the
Sec. 91 of the Insurance Code, (3) pursuant to the indemnity agreements, aggrieved party is the one entitled to recover the deficiency. Evidently, under this legal
the petitioner is entitled to recover from respondents Bormaheco and provision, the real party in interest with regard to the portion of the indemnity paid is the
insurer and not the insured. (Emphasis supplied)
Maglaa and (4) the principle of unjust enrichment is not applicable since
whatever amount he would recover from the co-indemnitor will be paid to 9.   It is clear from the records that Pioneer sued in its own name and not as an
the insurer. attorney-in-fact of the reinsurer. Accordingly, the appellate court did not
commit a reversible error in dismissing the petitioner's complaint as
ISSUE/s: against the respondents for the reason that the petitioner was not the
2.   Whether or not the CA erred when it dismissed the appeal of Pioneer real party in interest in the complaint and, therefore, has no cause of
on the sole ground that it had already collected the proceeds of the action against the respondents.
reinsurance on its bond in favor of the JDA and that it cannot 10.   Nevertheless, the Pioneer argues that the appeal as regards the counter
indemnitors should not have been dismissed on the premise that the
evidence on record shows that it is entitled to recover from the counter effect partners inter se, and their rights as members of the company to the
indemnitors. It does not, however, cite any grounds except its allegation that property acquired by the company will be recognized.
respondent "Maglanas defense and evidence are certainly incredible" 14.   A partnership relation between certain stockholders and other stockholders,
11.   On the other hand, we find the trial court's findings on the matter replete who were also directors, will not be implied in the absence of an agreement,
with evidence to substantiate its finding that the counter-indemnitors are so as to make the former liable to contribute for payment of debts illegally
not liable to the petitioner, to wit: contracted by the latter
Pioneer Insurance, knowing the value of the aircrafts and the spare parts involved, agreed to 15.   In his answer, the petitioner denied having received any amount from
issue the bond provided that the same would be mortgaged to it, but this was not possible respondents Bormaheco, the Cervanteses and Maglana but the trial and
because the planes were still in Japan and could not be mortgaged here in the Philippines. As
soon as the aircrafts were brought to the Philippines, they would be mortgaged to Pioneer appellat court found that petitioner received various amounts from
Insurance to cover the bond, and this indemnity agreement would be cancelled. Pioneer, Bormaheco and Magnlana in the ownership of subject airplanes.
having foreclosed the chattel mortgage on the planes and spare parts, no longer has any 16.   It is therefore clear that the petitioner never had the intention to form a
further action against the defendants as indemnitors to recover any unpaid balance of the
corporation with the respondents despite his representations to them. This
price. The indemnity agreement was ipso jure extinguished upon the foreclosure of the chattel
mortgage. These defendants, as indemnitors, would be entitled to be subrogated to the gives credence to the cross-claims of the respondents to the effect that they
right of Pioneer should they make payments to the latter. were induced and lured by the petitioner to make contributions to a
Pioneer's election of the remedy of foreclosure precludes any further action to recover any proposed corporation which was never formed because the petitioner
unpaid balance of the price. SAL or Lim, having failed to pay the second to the eight and last
reneged on their agreement.
installments to JDA and Pioneer as surety having made of the payments to JDA, the alternative
remedies open to Pioneer were as provided in Article 1484 of the New Civil Code, known as 17.   Applying therefore the principles of law earlier cited to the facts of the case,
the Recto Law. necessarily, no de facto partnership was created among the parties which
Pioneer exercised the remedy of foreclosure of the chattel mortgage both by extrajudicial would entitle the petitioner to a reimbursement of the supposed losses of the
foreclosure and the instant suit. Such being the case, as provided by the aforementioned proposed corporation. The record shows that the petitioner was acting on
provisions, Pioneer shall have no further action against the purchaser to recover any unpaid
balance and any agreement to the contrary is void. The restructuring of the obligations of SAL his own and not in behalf of his other would-be incorporators in transacting
or Lim, thru the change of their maturity dates discharged these defendants from any liability the sale of the airplanes and spare parts.
as alleged indemnitors. The change of the maturity dates of the obligations of Lim, or SAL
extinguish the original obligations thru novations thus discharging the indemnitors. These
restructuring of the obligations with regard to their maturity dates, effected twice, were done
without the knowledge, much less, would have it believed that these defendants Maglana (sic).
Pioneer's official Numeriano Carbonel would have it believed that these defendants and
defendant Maglana knew of and consented to the modification of the obligations. But if that
were so, there would have been the corresponding documents in the form of a written notice to
as well as written conformity of these defendants, and there are no such document. The
consequence of this was the extinguishment of the obligations and of the surety bond secured
by the indemnity agreement which was thereby also extinguished.
Pioneer's liability as surety to JDA had already prescribed when Pioneer paid the same.
Consequently, Pioneer has no more cause of action to recover from these defendants, as
supposed indemnitors, what it has paid to JDA. By virtue of an express stipulation in the
surety bond, the failure of JDA to present its claim to Pioneer within ten days from default of
Lim or SAL on every installment, released Pioneer from liability from the claim. Therefore,
Pioneer is not entitled to exact reimbursement from these defendants thru the indemnity
12.   As for the second case wherein the petioner is Jacob S. Lim, he theorizes
that as a result of the failure of respondents Bormaheco, Spouses Cervantes,
Constancio Maglana and petitioner Lim to incorporate, a de
facto partnership among them was created, and that as a consequence of
such relationship all must share in the losses and/or gains of the venture in
proportion to their contribution. Petitioner then questions the appellate
court’s findings ordering him to reimburse certain amounts given by
respondents as their contributions to the intended corporation.
13.   Where persons associate themselves together under articles to purchase
property to carry on a business, and their organization is so defective as to
come short of creating a corporation within the statute, they become in legal
007 ORIENTAL ASSURANCE v. ONG (MERILLES)
October 11, 2017 | Leonen, J. | Subrogation FACTS:
1.   JEA Steel Industries imported from South Korea 72 aluminum-zinc-alloy-
coated steel sheets in coils. These steel sheets were transported to Manila on
PETITIONER: Oriental Assurance Corporation
board the vessel M/V Dooyang Glory as evidenced by a Bill of Lading.
RESPONDENTS: Manuel Ong, doing business under the name of Western
2.   Upon arrival of the vessel at the Manila South Harbor on June 10, 2002, the
Pacific Transport Services and/or Asian Terminals, Inc.
72 coils were discharged and stored in Pier 9 under the custody of the
NOTE THE RELATIONSHIPS: Oriental = insurer; JEA = insured/consignee; Asian
arrastre contractor, Asian Terminals, Inc (respondent company in this case).
Terminals = consignor 3.   From the storage compound, the coils were loaded on the trucks on Manuel
Ong (respondent) and delivered to JEA Steel’s plant in Trece Martirez,
SUMMARY: JEA Steels imported from South Korea 72 coils. When it arrived Cavite.
at Manila, it was stored under the custody of Asian Terminals. From storage it 4.   Eleven (11) of the coils ''were found to be in damaged condition, dented or
was loaded on trucks of Manuel Ong to be delivered at the plant of JEA Steel. their normal round shape deformed.”
However, upon arrival at the plant, 11 coils were found to be in damaged 5.   JEA Steel filed a claim with Oriental for the value of the 11 damaged coils,
condition. pursuant to Marine Insurance Policy No. OAC/M-12292.
6.   Oriental paid JEA Steel the sum of ₱521,530.16 and subsequently
JEA claimed insurance from Oriental Assurance. Oriental paid JEA the sum of demanded indemnity from Ong and Asian Terminals (respondents), but
₱521,530.16. Oriental demanded indemnity form Ong and Asian Terminals, but they refused to pay.
they refused. Hence, a complaint was filed for collection of sum of money. Ong 7.   Oriental filed a complaint before the RTC of Manila for sum of money
alleged that the coils were already damaged when he received it. Asian argues against Ong and Asian Terminals.
hat it had exercised due diligence in caring for the consigned goods. The RTC 8.   Ong countered that the 1l coils were already damaged when they were
dismissed the case, which the CA affirmed. The CA stated that the action of loaded on board his trucks and transported to the consignee.
Oriental has already prescribed. 9.   Asian Terminals claimed that it exercised due diligence in handling the
cargo, that the cargo was released to the consignee's representative in the
The issue relevant to our topic is wether the claim against Asian Terminals, Inc. same condition as when received from the vessel, and that the damages
is barred by prescription. The court ruled that NO, the action is not barred. were sustained while in the custody of the vessel or the customs broker.
Under the Management Contract between JEA and Asian Terminal, claims must a.   Asian Terminals further argued that Oriental's claim was barred for
be filed within 15 days from the issuance of the Gate Pass. However, also under the latter's failure to file a notice of claim within the 15-day period
the Management Contract, the consignee (JEA) had thirty (30) days from provided in the Gate Pass and in Article VII, Section 7.01 of the
receipt of the cargo to request for a certificate of loss from Asian Terminals. Contract for Cargo Handling Services (Management Contract)
Oriental claimed damages 17 days after last receipt of the goods or within the 30 between the Philippine Ports Authority and Asian Terminals
day period. Although not a party to the contract, Oriental was subrogated to the b.   Asian Terminals added that its liability, if any, should not exceed
rights of JEA. ₱5,000.00, pursuant to said Section 7.01.
10.   RTC dismissed the complaint as it found no preponderance of evidence to
If the insured property is destroyed or damaged through the fault or establish that Ong and Asian Terminals were the ones responsible for the
negligence of a party other than the assured, then the insurer, upon damages. The MR was likewise denied.
payment to the assured, will be subrogated to the rights of the assured to 11.   The CA dismissed Oriental’s appeal on the ground that its claim had
recover from the wrongdoer to the extent that the insurer has been already prescribed.
obligated to pay. As subrogee, Oriental merely stepped into the shoes of the a.   CA also found that the coils were already damaged when it was
consignee and may only exercise those rights that the consignee may have loaded in Ong’s trucks
against the wrongdoer who caused the damage. b.   Hence, the legal presumption of negligence applies against Asian
Terminals unless it is able to prove that it exercised extraordinary
DOCTRINE: As subrogee, Oriental merely stepped into the shoes of the diligence in the handling of the cargo.
consignee and may only exercise those rights that the consignee may have c.   The CA held that as an arrastre operator, Asian Terminals was
against the wrongdoer who caused the damage. bound to observe the same degree of care required of common
carriers.
d.   The CA ultimately ruled that while Asian Terminals failed to rebut of Appeals in passing upon this issue.
the presumption of negligence against it, it cannot be held liable to 2.   [IMPORTANT] ISSUE #2 ON WHETHER THE CLAIM IS BARRED
pay the value of the damaged coils because Oriental's claim was BY PRESCRIPTION:
filed beyond the 15-day prescriptive period stated in the Gate Pass a.   Oriental contends that it was not aware of the provisions of the
e.   An MR was also denied. Gate Pass or the Management Contract, neither of which it was a
12.   Hence, this petition before the SC. party to. Consequently, it cannot be bound by the stipulation
limiting the liability of Asian Terminals.
ISSUE/s: b.   Asian Terminals counters that "the provisions of the Management
1.   Whether or not the Court of Appeals gravely erred in passing upon the issue Contract and the Gate Pass are binding on Oriental as insurer-
of prescription even though it was not an assigned error in the appeal – NO, subrogee and successor-in-interest of the consignee.”
the issue of prescription is closely related to, and determinant of, the c.   THE COURT AGREES WITH ASIAN TERMINALS. The fact
propriety of the lower court's ruling, absolving respondents from liability that Oriental is not a party to the Gate Pass and the Management
for the damaged goods and dismissing Oriental's complaint. Contract does not mean that it cannot be bound by their provisions.
2.   Whether or not the claim against Asian Terminals, Inc. is barred by Oriental is subrogated to the rights of the consignee simply upon
prescription – NO, the claim was received within the 30 day period to its payment of the insurance claim.
request for a certificate of loss, damage, or injury from the arrastre operator. d.   Citing GSIS v Manila Railroad: The SC has repeatedly held that,
3.   Whether or not the Court of Appeals gravely erred in ruling that Manuel by availing himself of the services of the arrastre operator and
Ong is not liable for the damage of the cargo – NO. Manuel Ong is not taking delivery therefrom in pursuance of a permit and a pass
liable, Asian Terminal knows of the defect prior to it being loaded in his issued by the latter, which were "subject to all the terms and
truck. conditions" of said management contract, including, inter alia, the
requirement thereof that "a claim is filed with the Company within
RULING: WHEREFORE, the Petition for Review is GRANTED. The February 19, 15 days from the date of arrival of the goods", the consignee - and,
2009 Decision and August 25, 2009 Resolution of the Court of Appeals in CA-G.R. hence, the insurer, or plaintiff herein, as successor to the rights of
CV No. 89311 are SET ASIDE. Respondent Asian Terminals, Inc. is ORDERED to the consignee - became bound by the provisions of said contract.
pay petitioner Oriental Assurance Corporation the amount of ₱55,000.00, with e.   Citing Summa Insurance v CA: However, a management contract,
interest at the legal rate of six percent (6%) per annum from the date of finality of which is a sort of a stipulation pour autrui within the meaning of
this judgment until fully paid. Article 1311 of the Civil Code, is also binding on a consignee
because it is incorporated in the gate pass and delivery receipt
RATIO: which must be presented by the consignee before delivery can be
1.   ISSUE #1 ON RULING ON PRESCRIPTION (can skip): effected to it. The insurer, as successor-in-interest of the consignee,
a.   Oriental submits that the "Court of Appeals cannot rule on the is likewise bound by the management contract.
issue of prescription as this was not included in the assignment of f.   The fact that Oriental is not a party to the Gate Pass and the
errors nor was this properly argued by any of the parties in their Management Contract does not mean that it cannot be bound by
respective briefs filed before the Court of Appeals. their provisions. Oriental is subrogated to the rights of the
b.   On the other hand, Asian Terminals counters that the Court of consignee simply upon its payment of the insurance claim.
Appeals properly reviewed the issue of prescription even though it g.   Art. 2207 of the Civil Code is founded on the well-settled
was not raised in Oriental's appeal brief. This issue is closely principle of subrogation. If the insured property is destroyed
related to the liability of Asian Terminals for the damaged or damaged through the fault or negligence of a party other
shipment, the first error in Oriental's appeal. Moreover, Asian than the assured, then the insurer, upon payment to the
Terminals asserts that it raised the issue of prescription before the assured, will be subrogated to the rights of the assured to
trial court, although it was not resolved. recover from the wrongdoer to the extent that the insurer has
c.   THE COURT AGREES WITH ASIAN TERMINALS. The issue been obligated to pay.
of prescription is closely related to, and determinant of, the i.   Payment by the insurer to the assured operates as an
propriety of the lower court's ruling, absolving respondents from equitable assignment to the former of all remedies which
liability for the damaged goods and dismissing Oriental's the latter may have against the third party whose
complaint. Thus, the Court finds no error on the part of the Court negligence or wrongful act caused the loss,
ii.   The right of subrogation is not dependent upon, nor does fact, Ong and Asian Terminals were sued in the alternative because
it grow out of, any privity of contract or upon written petitioner was uncertain against whom it was entitled for relief.
assignment of claim. d.   Furthermore, there was no proof of Ong's bad faith.
iii.   It accrues simply upon payment of the insurance claim e.   Mere allegation cannot take the place of evidence.
by the insurer f.   Besides, Ong's assertion that the loading of the cargo on the trucks
h.   As subrogee, Oriental merely stepped into the shoes of the was undertaken by Asian Terminals and the unloading of the same
consignee and may only exercise those rights that the consignee cargo was undertaken by the consignee at its warehouse remains
may have against the wrongdoer who caused the damage. unrequited
i.   Under the express terms of the Management Contract, the g.   Asian Terminals caused the inspection of the shipment before they
consignee had thirty (30) days from receipt of the cargo to request were loaded on Ong's trucks. At the consignee's warehouse, the
for a certificate of loss from the arrastre operator. inspection was done in the presence of the consignee's authorized
j.   Upon receipt of such request, the arrastre operator would have representative.
15 days to issue a certificate of loss, either actually or h.   Thus, Ong is not obliged to inform the consignee or Asian
constructively. Terminals about the damaged coils as they would have presumably
k.   From the date of issuance of the certificate of loss or where no known about them.
certificate was issued, from the expiration of the 15-day period, 5.   ASIAN TERMINALS IS FOUND LIABLE AND ORDERED TO PAY
the consignee has 15 days within which to file a formal claim ORIENTAL P55,000 with interests.
with the arrastre operator.
l.   In other words, the consignee had 45 to 60 days from the date of
last delivery of the goods within which to submit a formal claim to
the arrastre operator.
m.   The consignee's claim letter was received by Asian Terminals
on July 4, 2002, or 17 days from the last delivery of the goods,
still within the prescribed 30-day period to request a certificate
of loss, damage, or injury from the arrastre operator.
3.   LIABILITY OF ASIAN TERMINALS:
a.   Evidence showed that Upon Asian Terminal’s request, Ultraphil
conducted surveys which sowed that 11 coils were damaged before
it was loaded don Ong’s truck.
b.   Asian Terminals request for the cargo survey shows that it had
knowledge of the damage of the shipment while in its possession
and that the survey was sought specifically to ascertain the nature
and extent of the damage.
c.   Thus, Asian Cargos cannot escape liability for the damaged coils,
simply by its own act of not sending a representative, after it had
contracted for the survey of the shipment.
4.   ISSUE #3 ON LIABILITY OF MANUEL ONG:
a.   Both the Court of Appeals and the Regional Trial Court found that
the 11 coils were already damaged before the coils were loaded on
Ong's truck. Hence, Ong could not be responsible for the damaged
shipment.
b.   However, petitioner asserts that Ong should be held solidarily
liable with Asian Terminals for acting in bad faith when it did not
apprise the consignee or Asian Terminals about the damaged coils.
THE COURT FINDS THIS CONTENTION UNTENABLE.
c.   This issue was never raised by petitioner in the lower courts. In
001 UNITED DOCTORS MEDICAL CENTER v. BERNADAS 13.   On July 17, 1986, Cesario Bernadas (Cesario) was an orderly3 in United
(PELIÑO) Doctors Medical Center’s (UDMC) housekeeping department; he was
December 13, 2017 | Leonen, J. | Insurance and Retirement Benefits eventually promoted as a utility man.
14.   UDMC and the rank-and-file employees had a collective bargaining agreement
(CBA) where the rank-and-file employees were entitled to optional retirement
PETITIONER: United Doctors Medical Center benefits (ORB).
RESPONDENT: Cesario Bernadas, represented by Leonila Bernadas a.   The provision on the CBA reads: “Sec. 1. Retirement and Severance Pay.
The CENTER shall grant each employee retirement and severance pay in
SUMMARY: Cesario was an employee of UDMC. UDMC and the rank-and-file
accordance with law. It shall also continue its present policy on optional
employees had a CBA where these employees were entitled to ORB. Based on this
optional retirement policy, an employee who has rendered at least 20 years of service is retirement.”
entitled to optionally retire and the pay would be equal to a retiree’s salary for 11 days per b.   Based on this optional retirement policy, an employee who has rendered at
year of service. Cesario died of a freak accident. So Leonila, his wife, filed a Complaint least 20 years of service is entitled to optionally retire and the retirement
for payment of retirement benefits, damages, and atty’s fees with the NLRC. She was pay is equal to a retiree’s salary for 11 days per year of service.
already able to get 180k insurance proceeds based on the CBA. The LA dismissed her c.   Employees are also provided insurance where UDMC pays the
complaint, saying that since Cesario didn’t apply for ORB, then the beneficiaries are not premiums and the family members would be the beneficiaries.
entitled. She appealed to NLRC, and they reversed the LA. NLRC said that since the 15.   October 20, 2009, Cesario, 53 years at that time, died in a “freak accident”
optional retirement plan was never presented, it cast a doubt on whether an application is while working in a doctor’s residence.
required before an employee could be entitled, hence the doubt should be resolved in 16.   His wife, Leonila Bernadas (Leonila) filed a Complaint for payment of
favor of Cesario. On appeal to the CA, CA sustained the NLRC, saying that the retirement benefits, damages, and attorney’s fees with the NLRC. And Leonila
retirement benefits and insurance are 2 distinct benefits. Hence, this petition. The issue in and her son also claimed and were able to receive insurance proceeds of Php
this case is whether or not Cesario is entitled to receive his ORB despite receiving 180k under the CBA.
insurance benefits. The SC answered in the affirmative. The SC distinguished between 17.   Labor Arbiter: dismissed Leonila’s complaint.
insurance and retirement plans, wherein a contract of insurance – indemnity; agreement a.   Cesario should have applied for ORB during his lifetime, the benefits being
whereby one undertakes for a consideration to indemnify another against loss, optional. But since he didn’t, then his beneficiaries are not entitled to claim
damage or liability arising from an unknown or contingent event, while retirement his ORB.
plans are those which serve to secure loyalty and efficiency on the part of employees, 18.   Leonila appealed to the NLRC.
and to increase continuity of service and decrease the labor turnover, by giving to the a.   NLRC reversed the Labor Arbiter.
employees some assurance of security as they approach and reach the age at which b.   It found that the optional retirement plan was never presented so it cast a
earning ability and earnings are materially impaired or at an end. Hence, grant of doubt on whether or not the plan required an application for ORB before an
insurance proceeds will not necessarily bar the grant of retirement benefits; these are employee could become entitled to them. And considering the
2 separate and distinct benefits that an employer may provide to its employees. In “constitutional mandate to afford full protection to labor”, NLRC resolved
addition, Cesario is also entitled to ORB because CBA does not mandate that an the doubt in favor of Cesario.
application must first be filed before the benefits may vest, hence, this ambiguity c.   UDMC filed a MR but was denied.
should be resolved in favor of the retiree. 19.   UDMC subsequently filed a Petition for Certiorari with the CA.
a.   CA rendered a decision sustaining the ruling of the NLRC.
DOCTRINE: Contract of insurance – indemnity; agreement whereby one undertakes b.   CA held that the retirement plan and insurance are 2 separate and distinct
for a consideration to indemnify another against loss, damage or liability arising benefits that were granted to employees and that Leonila’s receipt of the
from an unknown or contingent event, while retirement plans are those which serve insurance proceeds did not bar her from being entitled to the retirement
to secure loyalty and efficiency on the part of employees, and to increase continuity benefits under the CBA.
of service and decrease the labor turnover, by giving to the employees some c.   UDMC filed a MR but was denied as well.
assurance of security as they approach and reach the age at which earning ability and 20.   Hence, this petition.
earnings are materially impaired or at an end. Hence, grant of insurance proceeds 21.   UDMC argues:
will not necessarily bar the grant of retirement benefits; these are 2 separate and a.   Cesario’s beneficiaries do not have legal capacity to apply for his ORB
distinct benefits that an employer may provide to its employees since Cesario never applied for it in his lifetime, and even assuming that he
was already qualified to apply for optional retirement 3 years prior to his
FACTS: death, he never did. Hence, no basis for Cesario’s beneficiaries to be
12.   This is a Petition for Review on Certiorari assailing the Decision and Resolution
of the CA, sustaining the NLRC’s finding.                                                                                                                        
3
Merriam-Webster defined this as a person who works in a hospital and does various jobs (such as
moving patients or cleaning)
entitled to his optional retirement benefits. b.   Voluntary4: by agreement between the employer and employee – This is
b.   To grant them these ORB on top of the life insurance benefits would usually embodied in a CBA.
amount to double compensation and unjust enrichment. c.   Voluntary: given by the employer – Announced company policy or
22.   Leonila argues: impliedly as in a failure to contest the employee’s claim for retirement
a.   Had her husband died under normal circumstances, he would have applied benefits.
for the ORB. 6.   These retirement plans are not meant to be a replacement to the compulsory
b.   The fact that he was unable to apply before his death is a procedural retirement scheme under social security laws.
technicality that should be set aside so that “full protection to labor” is
afforded and “the ends of social and compassionate justice” are met. On whether Cesario is entitled to receive his optional retirement benefits despite his
untimely death
ISSUE/s: 1.   Issue in this case concerns the second type of retirement plan.
2.   WON Leonila, as her husband’s representative, may claim his optional 2.   The terms and conditions of a CBA constitute the law between the parties, but
retirement benefits. – YES, she is a beneficiary of Cesario. the CBA involved in this case does not provide for the terms and conditions of
3.   WON Cesario can receive the ORB despite already receiving insurance the “present policy on optional retirement”.
benefits. – YES, insurance and retirement plans are two separate benefits and a.   Leonila merely alleged before the Labor Arbiter that UDMC grants an
receiving one is not a bar to receiving the other. employee a retirement or separation equivalent to 11 days per year of
4.   WON Cesario is entitled to receive his ORB despite his untimely death. – YES, service after serving for at least 20 years, which was not deisputed by
CBA does not mandate that an application must first be filed before the benefits UDMC. Therefore, doubt arises as to what UDMC’s optional retirement
may vest, hence, this ambiguity should be resolved in favor of the retiree. package actually entails.
3.   It is settled that doubts must be resolved in favor of labor.
RULING: WHEREFORE, the petition is DENIED. The Decision and Resolution of the a.   Retirement laws should be liberally construed and administered in favor of
CA are AFFIRMED. Petitioner UDMC is ordered to pay Cesario, through his beneficiary the persons intended to be benefited and all doubts as to the intent of the
Leonila, optional retirement benefits in the amount of Php 98,252.55 as provided by the law should be resolved in favor of the retiree to achieve its humanitarian
Labor Code. purpose.
4.   UDMC: insists that Cesario would not have been entitled to his ORB as he
RATIO: failed to exercise the option before his untimely death.
1.   Retirement – result of a bilateral act of the parties, a voluntary agreement a.   SC: Retirement encompasses even the concept of death and that the Court
between the employer and the employee whereby the latter, after reaching a has considered death as a form of disability retirement since there is no
certain age, agrees to sever his or her employment with the former. more permanent or total physicaly disability than death.
2.   Contract of Insurance – indemnity; agreement whereby one undertakes for i.   Compulsory retirement and death both involve events beyond the
a consideration to indemnify another against loss, damage or liability employee’s control.
arising from an unknown or contingent event. b.   UDMC also admits that Cesario was already qualified to receive retirement
3.   Retirement plans – Serve to secure loyalty and efficiency on the part of benefits since he was already working there for 23 years, and while the
employees, and to increase continuity of service and decrease the labor choice to retire was within Cesario’s control, his death foreclosed that
turnover, by giving to the employees some assurance of security as they possibility.
approach and reach the age at which earning ability and earnings are c.   UDMC’s optional retirement plan is based on length of service and not of
materially impaired or at an end. reaching a certain age. It rewards loyalty and continued service by granting
4.   Given the above, it is clear that there is a difference between insurance and an employee an earlier age to claim his or her retirement benefits even if
retirement benefits. Hence, grant of insurance proceeds will not necessarily the employee has not reached the twilight years. It would be iniquitous to
bar the grant of retirement benefits; these are 2 separate and distinct withhold it from he just because he died before he applied.
benefits that an employer may provide to its employees. d.   CBA does not mandate that an application must first be filed before
5.   Three types of retirement plans available to employees in the PH: the benefits may vest, hence, this ambiguity should be resolved in favor
a.   Compulsory and Contributory – RA 8282 for private (SSS) and RA 8291 of the retiree.
for government (GSIS). These laws provide mandatory contribution which e.   Retirement benefits are the property interests of the retiree and his or her
shall become a pension fund upon retirement. A mandatory contribution is beneficiaries. CBA doesn’t prohibit the beneficiaries from claiming
deducted from the employee’s monthly income; retirement packages are retirement benefits even if the retiree dies before the proceeds could be
usually crafted as ‘forced savings’. There is a vested right to the benefits released.
here, so the beneficiaries of retired employees are entitled to receive even
after the employee’s death.
                                                                                                                       
4
The last 2 retirement plans are governed by the Labor Code.  
damage through accident for an amount not exceeding P8,000 (Policy No.
02 CCC INSURANCE CORP. v. CA (PLEYTO) MC-1156)
January 30, 1970 | Reyes, J.B.L, J. | Insurance Contract Interpretation 24.   On June 25, 1961, and during the effectivity of the policy, the insured
vehicle, while being driven by the driver of Robes, became involved in a
vehicle collision along Rizal Avenue Extension, Potrero, Malabon, Rizal
PETITIONER: CCC Insurance Corporation
25.   The car was damaged and the repair was estimated to cost P5,300.
RESPONDENTS: Court of Appeals (Fourth Division) and Carlos F. Robes
26.   Robes instituted a civil case in the CFI of Rizal because the insurance
company refused either to pay for the repair or to cause the restoration of
SUMMARY: On Mar. 1, 1961, Robes took an insurance, with the CCC Insurance
the car to its original condition
Corp, on his Dodge Kingsway car against loss or damage through accident for an
27.   The civil case was for recovery not only of the amount necessary for the
amount not exceeding P8000 (policy). On June 25 (while policy was effective),
repair of the insured car but also of actual and moral damages, attys fees
Robes’ driver, while driving the insured car, got into a collision. The cost of damages
and costs
was estimated to cost P5,300. The insurance company refused to either pay for the
28.   Resisting plaintiff’s (Robes) claim, the insurance company disclaimed
repair or the restoration so Robes instituted a civil case for recovery of the amount
liability for payment, alleging that there had been violation of the insurance
necessary for the repair, actual and moral damages, and attys fees. CCC disclaimed
contract because the one driving the car at the time of the incident was not
liability, alleging that there had been a violation of the insurance contract because the
an “authorized driver”
one driving the car was not an authorized driver. The trial court ruled in favor of
29.   After due hearing, judgment was rendered for Robes and CCC was ordered
Robes, granting him the P5,031.28, for the cost of repair, actual and moral damages
to pay the cost of repair in the sum of P5,031.28; the sum of P150, for the
and attorneys fees. CA affirmed this except for actual damages, because it was said
hauling and impounding of the car at the repair shop; P2k as actual
to be too speculative. CCC was not content hence, this petition. The issue is WoN
damages; P1k as attys fees, plus costs
the damage to the insured car was not covered by the insurance policy because at the
30.   CCC went to the CA, raising inter alia the questions of the qualification of
time of the accident it was being driven by one who was not an authorized driver.
Robes’ driver to operate the insured vehicle and the correctness of the trial
CCC claims that the driver is not authorized because does not know how to read and
court’s award to Robes of the cost of repairs, actual and moral damages, and
write; that he was able to secure a driver’s license, without passing any examination
attys fees
thereof, by paying P25 to a certain woman; and that the Cavite agency of the Motor
31.   On Jan. 31, 1966, CA affirmed the ruling of the lower court except the
Vehicles Office has certified not having issued Reyes’ purported driver’s license. CA
award of actual damages in the sum of P2k, for being too speculative
argued that since Reyes' purported driver's license bears all the earmarks of a duly
32.   Not content, CCC filed the present petition of the decision of the CA
issued license, then it is a public document, and CCC then has the burden of
disproving its genuineness, which the latter has failed to do. Also, the fatal flaw in
ISSUE/s:
CCC’s argument is that it ignores the provisions of law existing at the time of the
4.   WoN the damage to the insured car was not covered by the insurance policy
mishap, which says that the issuance of a driving license without previous
because at the time of the accident it was being driven by one who was not
examination does not necessarily imply that the license issued is invalid. Considering
an authorized driver (main) – NO. The Court said that Reyes (the driver) is
that, as pointed out by the Court of Appeals the weight of authority is in favor of a
authorized and his license is genuine, as issued by the MVO. A
liberal interpretation of the insurance policy for the benefit of the party insured, and
5.   WoN the proceedings observed in the trial court were irregular and invalid
strictly against the insurer, SC finds no reason to diverge from the conclusion
– NO. There is nothing wrong with the practice of delegating to a
reached by the Court of Appeals that no breach was committed of the above- quoted
commissioner, usually the clerk of court, who is a duly sworn court officer,
provision of the policy.
the reception of both parties and for him to submit a report thereon to the
court. (not relevant)
DOCTRINE: The weight of authority is in favor of a liberal interpretation of the
insurance policy for the benefit of the party insured, and strictly against the insurer
RULING: the decision of the CA is affirmed, with costs against CCC

FACTS:
22.   This is a petition for review of the decision of the CA, affirming that of the
CFI of Rizal, QC, allowing insurance indemnification of plaintiff for his
damaged car and the payment of atty’s fees
23.   On March 1, 1961, Carlos F. Robes took an insurance, with the CCC
Insurance Corporation (CCC), on his Dodge Kingsway car against loss or
RATIO: license without previous examination does not necessarily imply that the
18.   It is vigorously urged by the insurer that the one driving the insured vehicle license issued is invalid.
at the time of the accident was not an authorized driver thereof within the 25.   As the law stood in 1961, when the claim arose, the examinations could be
5
purview of the provision of the insurance policy dispensed with in the discretion of the Motor Vehicles Office officials.
19.   It has been found as a fact by the CA that Domingo Reyes, the driver, does Whether discretion was abused in issuing the license without examination is
not know how to read and write; that he was able to secure a driver’s not a proper subject of inquiry in these proceedings, though, as a matter of
license, without passing any examination thereof, by paying P25 to a certain legislative policy, the discretion should be eliminated. There is no proof that
woman; and that the Cavite agency of the Motor Vehicles Office has the owner of the automobile knew that the circumstances surrounding such
certified not having issued Reyes’ purported driver’s license issuance showed that it was irregular.
20.   In holding that the damage sustained by the car comes within the coverage 26.   The issuance of the license is proof that the Motor Vehicles Office official
of the insurance policy, the CA argued that since Reyes' purported driver's considered Reyes, the driver of Robes, qualified to operate motor vehicles,
license bears all the earmarks of a duly issued license, then it is a public and the insured was entitled to rely upon such license.
document, and CCC then has the burden of disproving its genuineness, 27.   In this connection, it should be observed that the chauffeur, Reyes, had been
which the latter has failed to do driving since 1957, and without mishap, for all the record shows.
21.   CA ruled: 28.   Considering that, as pointed out by the Court of Appeals the weight of
a. the fact that the Cavite Agency of the Motor Vehicles Office authority is in favor of a liberal interpretation of the insurance policy
states that Drivers License was not issued by that office, does not for the benefit of the party insured, and strictly against the insurer, SC
remove the possibility that said office may have been mistaken or finds no reason to diverge from the conclusion reached by the Court of
that said license was issued by another agency. Appeals that no breach was committed of the above- quoted provision of
b. Indeed a certain Gloria Presa made the notation thereon "no the policy.
license issued" and which notation was the basis of the 1st
Indorsement, signed by the MVO Cavite City Agency's officer-in- 2nd issue
charge. 29.   The next issue is anchored on the fact that the decision of the trial court was
c. Neither Gloria Presa nor the officer-in-charge Marciano A. based on evidence presented to and received by the clerk of court who acted
Monzon was placed on the witness stand to be examined in order as commissioner, although allegedly, there was no written court order
to determine whether said license is indeed void. constituting him as such commissioner, no written request for his
d. thus, the fact remains that Domingo Reyes is in possession of a commission was made by the parties; he did not take an oath prior to
driver's license issued by the Motor Vehicles Office which on its entering into the discharge of his commission; no written report of his
face appears to have been regularly issued. findings was ever submitted to the court; and no notice thereof was sent to
22.   CA then found the license as genuine, that is, one really issued by the MVO the parties, contrary to the provisions of Rule 33 of the Rules of Court.
or its authorized deputy; and this finding of fact is now conclusive and may 30.   There is nothing wrong with the practice of delegating to a commissioner,
not be questioned in this appeal usually the clerk of court, who is a duly sworn court officer, the reception of
23.   Nevertheless, CCC insists that, under the established facts of this case, both parties and for him to submit a report thereon to the court.
Reyes, being admittedly one who cannot read and write, who has never 31.   In fact, this procedure is expressly sanctioned by Revised Rule 33 of the
passed any examination for drivers, and has not applied for a license from Rules of Court. CCC’s objection in this case, however, is directed not
the duly constituted government agency entrusted with the duty of licensing against its referral to the clerk of court but against the alleged non-
drivers, cannot be considered an authorized driver. observance of the prescribed steps in connection with such delegation.
24.   The fatal flaw in CCC’s argument is that it ignores the provisions of law 32.   SC finds no cause sufficient to invalidate the proceedings had in the trial
6 court. SC notes that this issue was brought up by CCC for the first time only
existing at the time of the mishap, which says that the issuance of a driving
in its motion for reconsideration filed in the Court of Appeals.
                                                                                                                                                                                                                                                                                                                                                                                                               
5
 "AUTHORIZED DRIVER: "Any of the following: 6
Under Section 24 of the revised Motor Vehicles Law:
(a)   The insured; “An examination or demonstration to show any applicant's ability to operate motor vehicles may also be
(b)   Any person driving on the Insured's order or with his permission, provided that the person required in the discretion of the Chief, Motor Vehicles Office or his deputies."
driving is permitted in accordance with licensing laws or regulations to drive the motor vehicle And reinforcing Section 24, Section 26 prescribes further:
covered by this Policy, or has been so permitted and is not disqualified by order of a court of "SEC. 26. Issuance of chauffeur's license; professional badge: If, after examination, or without the same,
law or by reason of any enactment or regulation from driving such Motor Vehicle." (Italics the Chief, Motor Vehicles Office or his deputies, believe the applicant to possess the necessary
ours) qualifications and knowledge, they shall issue to such applicant a license to operate as chauffeur . . ."
33.   It was not raised in the trial court, where the defect could still be remedied.
This circumstance precludes ventilation of the issue of validity of the
hearing at this stage; for, if such irregularity is to vitiate the proceeding, the
question should have been seasonably raised, i.e., either before the parties
proceeded with the hearing or before the court handed down its ruling. It is
a procedural point that can be waived by consent of the parties, express or
implied.
34.   For the same reason, appellant cannot insist now on the annulment of the
proceeding on the basis of alleged lack of written consent of the parties to
the commission, or of an order appointing the clerk as commissioner, or of
notice of the submission of his report to the court.
35.   Furthermore, appellant has presented no proof that the clerk of court
committed any mistake or abuse in the performance of the task entrusted to
him, or that the trial court was not able to properly appreciate the evidence
in the case because it was received by another person. If indeed there were
errors at all, they would be non-prejudicial and could not justify the holding
of a new trial, as urged by herein petitioner.
003 Assoc. of Baptists for World Evangelism, Inc. v. Fieldmen’s with Motor No. 032433272555 and Plate No. E-73317 covered by
Insurance (Punsalan) Registration Certificate No. 288141 Rizal, issued by the Davao Motor
Sept. 21, 1983 | Melencio-Herrera, J. | Joyride in Davao; Insurance Contract Vehicles Office Agency No. 20 and owned by Reverend Clinton Bonnel
Interpretation 25.   Association insured said vehicle with Fieldmen’s with Private Car
Comprehensive Policy No. 22 (Comprehensive Policy) against loss or
damage up to P5,000.00.
PETITIONER: Association of Baptists for World Evangelism, Inc.
26.   1961: Association’s representative Dr. Antonio Lim (Dr. Lim) placed the
RESPONDENTS: Fieldmen’s Insurance Co., Inc.
Chevrolet Carry-all at the Jones Monument Mobilgas Service Station at
Davao City, under care of station operator Rene Te (Te) so that the vehicle
SUMMARY: Association has an insurable interest in its Chervrolet Carry-all. It
could be displayed as being for sale and that the station boys would
insured the vehicle with Fieldmen’s with Private Car Comprehensive Policy No.
receive 2% commission should they sell the vehicle.
22 (Comprehensive Policy) against loss or damage up to P5,000. Assoc.’s
27.   1962: One night, Romeo Catiben (station boy in said service station) and a
representative, Dr. Lim, proceeded to Jones Monument Mobilgas Service Station
nephew (unnamed in the case) of the wife of Te, took the vehicle for a
at Davao City where he left the car so that it could be displayed for sale and the
joyrde in Toril, Davao City, without the prior permission, authority, or
station boys would be entitled to 2% commission for it. One night, Romeo
consent of the Association or its rep. Dr. Lim, or of Te, and on its way back
Catiben (one of the station boys) and the nephew of station operator Rene Te,
to Davao City, the vehicle experienced a mechanical defect which led
rode the Chevrolet Carry-all and took it for a joyride to Toril, Davao City, w/o
into an accident, bumping an electric post (actual damages at
the consent of Association, Dr. Lim, or Te. Upon returning, however, the vehicle
P5,518.61)
experienced a mechanical defect and they accidentally bumped into an electric
28.   When the matter was brought up with the Trial Court, it tackled the issue of
post (actual damage from it was P5,518.61). Thus, Association wants to claim
whether the there should be a prior criminal conviction of Catiben for theft
the P5k compensation for the damage sustained by the vehicle but Fieldmen’s
before Association can claim compensation for the actual damage caused to
argue that there was no prior conviction of Catiben for theft of the vehicle, so
its vehicle under the Comprehensive Policy.
they did not give the insurance claim.
29.   The trial court rendered judgment and ordered Fieldmen’s to pay
Association the amount of P5,000.00 as indemnity for the damage sustained
ISSUE: WoN Catiben’s acts constitutes theft under the Comprehensive Policy –
by the vehicle, P2k for atty’s fees, and costs.
YES
30.   Upon appeal, the CA elevated the matter to the SC on a question of law.
It is settled that the act of Catiben in taking the vehicle for a joy ride to Toril,
ISSUE/s:
Davao City, constitutes theft within the meaning of the insurance policy and
5.   WoN the acts of Catiben constitutes theft under the Comprehensive Policy
that recovery for damage to the car is not barred by the illegal use of the car
of Fieldmen’s, thus entitling Association to compensation – YES, because
by one of the station boys. Even without prior conviction, his intent to gain is
theft under the policy does not need a prior conviction; his intent to gain
evident since he derives therefrom utility, satisfaction, enjoyment, and pleasure.
was evident from the joyride. As long as it can be proven by preponderance
Thus, there is no need for prior conviction for the crime of theft to make an
of evidence in a civil action for recovery, the amount on an insurance policy
insurer liable under the theft clause of the policy. (proceed to doctrine)
may be recovered.
DOCTRINE: In a civil action for recovery on an automobile insurance, the
RULING: Judgment in question is affirmed.
question whether a person using a certain automobile at the time of the accident
stole it or not is to be determined by a fair preponderance of evidence and
RATIO:
NOT by the rule of criminal law requiring proof of guilt beyond reasonable
13.   The Comprehensive Policy issued by the insurance company includes loss
doubt.
of or damage to the motor vehicle by "burglary . . . or theft.
14.   It is settled that the act of Catiben in taking the vehicle for a joy ride to
FACTS: Toril, Davao City, constitutes theft within the meaning of the insurance
23.   Association of Baptists for World Evangelism, Inc. (Association) is a policy and that recovery for damage to the car is not barred by the illegal
religious corporation duly organized and registered in PH; Fieldmen’s use of the car by one of the station boys.
Insurance Co., Inc. (Fieldmen’s) is also a domestic corporation duly 15.   Where a car is unlawfully and wrongfully taken by some people, be they
organized and existing under the laws of the PH employees of the car shop or not to whom it had been entrusted, and taken
24.   Association has an insurable interest in a Chevrolet Carry-all, 1955 Model
on a long trip to Montalban without the owner’s consent or knowledge,
constitutes or partakes of the nature of theft as defined in Art. 308 of the
RPC:
(W)ho are liable for theft. — … any person who, with intent to
gain but without violence against or intimidation of persons nor
force upon things, shall take personal property of another without
the latter’s consent
16.   Court sustains that when a person, either with the object of going to a
certain place or learning how to drive or enjoying a free ride, takes
possession of a vehicle belonging to another, w/o the latter’s consent, he is
guilty of theft because his intent to gain is evident since he derives
therefrom utility, satisfaction, enjoyment, and pleasure.
17.   Thus, there is no need for prior conviction for the crime of theft to make an
insurer liable under the theft clause of the policy.
18.   The facts are stipulated where Catiben took the car for a joyride and
thereafter bumped it against an electric post resulting in damages. The act is
theft within the policy of insurance.
19.   In a civil action for recovery on an automobile insurance, the question
whether a person using a certain automobile at the time of the accident stole
it or not is to be determined by a fair preponderance of evidence and
NOT by the rule of criminal law requiring proof of guilt beyond
reasonable doubt. (NOTE: besides, there was no provision in the policy
requiring prior criminal conviction for theft)
004 Mayer Steel Pipe Corporation v. CA (Cristelle) evidenced by Invoice Nos. MSPC-1014, MSPC-1015, MSPC-1025, MSPC-
June 19, 1997 | Puno J. | Insurance Contract Interpretation 1020, MSPC-1017 and MSPC-1022.
3.   Prior to the shipping, petitioner Mayer insured the pipes and fittings against
PETITIONER: MAYER STEEL PIPE CORPORATION and HONGKONG all risks with private respondents South Sea Surety and Insurance Co., Inc.
GOVERNMENT SUPPLIES DEPARTMENT (South Sea) and Charter Insurance Corp. (Charter). The pipes and fittings
RESPONDENTS: COURT OF APPEALS, SOUTH SEA SURETY AND covered by Invoice Nos. MSPC-1014, 1015 and 1025 with a total amount of
INSURANCE CO., INC. and the CHARTER INSURANCE CORPORATION US$212,772.09 were insured with respondent South Sea, while those
SUMMARY: Hong Kong Government Supplies Department contracted Mayer Steel covered by Invoice Nos. 1020, 1017 and 1022 with a total amount of
Pipe Corporation to manufacture and supply various steel pipes and fittings. Prior to US$149,470.00 were insured with respondent Charter.
the shipping, Mayer insured these pipes and fittings against all risks with South Sea 4.   Petitioners Mayer and Hongkong jointly appointed Industrial Inspection
Surety and Insurance Co., Inc. and Charter Insurance Corp., with Industrial Inspection (International) Inc. as third-party inspector to examine whether the pipes
Inc. appointed as third-party inspector. After examining the pipes and fittings, and fittings are manufactured in accordance with the specifications in the
Industrial Inspection certified that they are in good order condition. However, when contract. Industrial Inspection certified all the pipes and fittings to be in
the goods reached Hong Kong, it was discovered that a substantial portion thereof was good order condition before they were loaded in the vessel. Nonetheless,
damaged. The trial court found in favor of the insured. However, when the case was when the goods reached Hongkong, it was discovered that a substantial
elevated to the CA, it set aside the decision of the trial court and dismissed the portion thereof was damaged.
complaint on the ground of prescription. It held that the action was barred under Sec. 5.   Hongkong & Mayer filed a claim against South Sea surety & charter
3(6) of the Carriage of Goods by Sea Act (COGSA) since it was filed only on April insurance corp. for indemnity under the insurance contract. Respondent
17, 1986, more than two years from the time the goods were unloaded from the Charter paid petitioner Hongkong the amount of HK$64,904.75. HK &
vessel. Issue: Whether or not the action is barred by prescription? The cause of Mayer demanded payment of the balance of HK$299,345.30 representing
action had not yet prescribed. Sec. 3(6) of the COGSA states that the carrier and the the cost of repair of the damaged pipes. South Sea & charter refused to pay
ship shall be discharged from all liability for loss or damage to the goods if no suit is because the insurance surveyor's report allegedly showed that the damage is
filed within one year after delivery of the goods or the date when they should have a factory defect.
been delivered. Under this provision, only the carrier’s liability is extinguished if no 6.   On April 17, 1986, petitioners filed an action against South Sea & charter to
suit is brought within one year. But the liability of the insurer is not extinguished recover the sum of HK$299,345.30 South Sea & charter averred that they
because the insurer’s liability is based not on the contract of carriage but on the have no obligation to pay the amount claimed by petitioners because the
contract of insurance. Thus, when private respondents issued the “all risks” policies to damage to the goods is due to factory defects which are not covered by the
Mayer, they bound themselves to indemnify the latter in case of loss or damage to the insurance policies.
goods insured. Such obligation prescribes in ten years, in accordance with Article 7.   TRIAL COURT - ruled in favor of petitioners. It found that the damage to
1144 of the New Civil Code. the goods is not due to manufacturing defects. It also noted that the
DOCTRINE: An insurance contract is a contract whereby one party, for a insurance contracts executed by petitioner Mayer and private respondents
consideration known as the premium, agrees to indemnify another for loss or damage are "all risks" policies which insure against all causes of conceivable loss or
which he may suffer from a specified peril. An “all risks” insurance policy covers all damage. The only exceptions are those excluded in the policy, or those
kinds of loss other than those due to willful and fraudulent act of the insured. sustained due to fraud or intentional misconduct on the part of the insured.
8.   South Sea & charter elevated the case to respondent Court of Appeals.
FACTS: 9.   CA - affirmed the finding of the trial court that the damage is not due to
1.   This is a petition for review on certiorari to annul and set aside the Decision factory defect and that it was covered by the "all risks" insurance policies
of respondent CA dated December 14, 1995 and its Resolution dated issued by South Sea & charter to petitioner Mayer. However, it set aside the
February 22, 1996 in CA-G.R. CV No. 45805 entitled Mayer Steel Pipe decision of the trial court and dismissed the complaint on the ground of
Corporation and Hongkong Government Supplies Department v. South Sea prescription. It held that the action is barred under Section 3(6) of the
Surety Insurance Co., Inc. and The Charter Insurance Corporation. Carriage of Goods by Sea Act since it was filed only on April 17, 1986,
2.   In 1983, petitioner Hongkong Government Supplies Department more than two years from the time the goods were unloaded from the
(Hongkong) contracted petitioner Mayer Steel Pipe Corporation (Mayer) to vessel.
manufacture and supply various steel pipes and fittings. From August to a.   Section 3(6) of the Carriage of Goods by Sea Act provides that
October, 1983, Mayer shipped the pipes and fittings to Hongkong as "the carrier and the ship shall be discharged from all liability in
respect of loss or damage unless suit is brought within one year
after delivery of the goods or the date when the goods should have third-party complaint against the carrier for reimbursement of the amount it
been delivered." Respondent court ruled that this provision applies paid to the shipper. The insurer filed the third-party complaint on January 9,
not only to the carrier but also to the insurer, citing Filipino 1978, more than one year after delivery of the goods on December 17,
Merchants Insurance Co., Inc. vs. Alejandro. 1977. The court held that the Insurer was already barred from filing a claim
10.   Hence this petition for review. against the carrier because under the Carriage of Goods by Sea Act, the suit
a.   CA erred in holding that petitioners' cause of action had already against the carrier must be filed within one year after delivery of the goods
prescribed on the mistaken application of the Carriage of Goods by or the date when the goods should have been delivered. The court said that
Sea Act and the doctrine of Filipino Merchants Co., Inc. v. "the coverage of the Act includes the insurer of the goods."
Alejandro (145 SCRA 42); and 4.   The Filipino Merchants case is different from the case at bar. In Filipino
b.   CA committed an error in dismissing the complaint. Merchants, it was the insurer which filed a claim against the carrier for
reimbursement of the amount it paid to the shipper. In the case at bar, it was
ISSUES: Whether or not the action is barred by prescription? NO. The petition is the shipper which filed a claim against the insurer. The basis of the shipper's
impressed with merit. Respondent court erred in applying Section 3(6) of the claim is the "all risks" insurance policies issued by private respondents to
Carriage of Goods by Sea Act. An insurance contract is a contract whereby one petitioner Mayer.
party, for a consideration known as the premium, agrees to indemnify another for 5.   The ruling in Filipino Merchants should apply only to suits against the
loss or damage which he may suffer from a specified peril. An "all risks" insurance carrier filed either by the shipper, the consignee or the insurer. When
policy covers all kinds of loss other than those due to willful and fraudulent act of the court said in Filipino Merchants that Section 3(6) of the Carriage of
the insured. Thus, when private respondents issued the "all risks" policies to Goods by Sea Act applies to the insurer, it meant that the insurer, like the
petitioner Mayer, they bound themselves to indemnify the latter in case of loss or shipper, may no longer file a claim against the carrier beyond the one-year
damage to the goods insured. Such obligation prescribes in ten years, in accordance period provided in the law. But it does not mean that the shipper may no
with Article 1144 of the New Civil Code. longer file a claim against the insurer because the basis of the insurer's
liability is the insurance contract.
RULING: IN VIEW WHEREOF, the petition is GRANTED. The Decision of
respondent Court of Appeals dated December 14, 1995 and its Resolution dated
February 22, 1996 are hereby SET ASIDE and the Decision of the Regional Trial
Court is hereby REINSTATED. No costs. SO ORDERED.
RATIO:
1.   Section 3(6) of the Carriage of Goods by Sea Act states that the carrier and
the ship shall be discharged from all liability for loss or damage to the
goods if no suit is filed within one year after delivery of the goods or the
date when they should have been delivered.
2.   Under this provision, only the carrier's liability is extinguished if no suit is
brought within one year. But the liability of the insurer is not extinguished
because the insurer's liability is based not on the contract of carriage but on
the contract of insurance. A close reading of the law reveals that the
Carriage of Goods by Sea Act governs the relationship between the carrier
on the one hand and the shipper, the consignee and/or the insurer on the
other hand. It defines the obligations of the carrier under the contract of
carriage. It does not, however, affect the relationship between the shipper
and the insurer. The latter case is governed by the Insurance Code.
3.   Our ruling in Filipino Merchants Insurance Co., Inc. v. Alejandro and the
other cases cited therein does not support respondent court's view that the
insurer's liability prescribes after one year if no action for indemnity is filed
against the carrier or the insurer. In that case, the shipper filed a complaint
against the insurer for recovery of a sum of money as indemnity for the loss
and damage sustained by the insured goods. The insurer, in turn, filed a
005 Great Pacific Life v. CA (Rosales) corresponding form in his own handwriting. Mondragon finally type-wrote
April 30, 1979 | De Castro, J. | Insurance Contract Interpretation the data on the application form which was signed by Ngo Hing.
2.   The Ngo Hing paid the annual premuim the sum of P1,077.75 going over to
PETITIONER: Great Pacific Life Assurance Company; Lapulapu D. the Company, but he retained the amount of P1,317.00 as his commission
Mondragon for being a duly authorized agent of Pacific Life. Upon the payment of the
RESPONDENTS: Court of Appeals; Ngo Hing insurance premuim, the binding deposit receipt was issued to Ngo Hing.
Likewise, Mondragon handwrote at the bottom of the back page of the
SUMMARY: Ngo Hing filed an application with the Great Pacific Life application form his strong recommendation for the approval of the
Assurance for a twenty-year endownment policy in the amount of P50,000.00 on insurance application. Then on April 30, 1957, Mondragon received a letter
the life of his one-year old daughter Helen Go. Pacific Life supplied the essential from Pacific Life disapproving the insurance application. The letter stated
data which Lapulapu D. Mondragon, Branch Manager of the Pacific Life in that the said life insurance application for 20-year endowment plan is not
Cebu City wrote on the corresponding form. Upon the payment of the insurance available for minors below seven years old, but Pacific Life can consider
premuim, the binding deposit receipt was issued to Ngo Hing. Likewise, the same under the Juvenile Triple Action Plan, and advised that if the offer
Mondragon handwrote at the bottom of the back page of the application form his is acceptable, the Juvenile Non-Medical Declaration be sent to the
strong recommendation for the approval of the insurance application. company.
Mondragon received a letter from Pacific Life disapproving the insurance 3.   The non-acceptance of the insurance plan by Pacific Life was allegedly not
application. The non-acceptance of the insurance plan by Pacific Life was communicated by Mondragon to Ngo Hing. Instead, on May 6, 1957,
allegedly not communicated by Mondragon to Ngo Hing. Instead, Mondragon Mondragon wrote back Pacific Life again strongly recommending the
wrote back Pacific Life again strongly recommending the approval of the 20- approval of the 20-year endowment insurance plan to children, pointing out
year endowment insurance plan. Helen Go died of influenza with complication that since 1954 the customers, especially the Chinese, were asking for such
of bronchopneumonia. Thereupon, Ngo Hing sought the payment of the coverage.
proceeds of the insurance, but having failed in his effort, he filed the action for 4.   It was when things were in such state that on May 28, 1957 Helen Go died
the recovery of the same. Issue is WoN the binding deposit receipt constituted a of influenza with complication of bronchopneumonia. Thereupon, Ngo
temporary contract of the life insurance in question? The SC held NO, because Hing sought the payment of the proceeds of the insurance, but having failed
there is an absence of a meeting of the minds between Pacific Life and Ngo Hing in his effort, he filed the action for the recovery of the same before the
and there is and non-compliance of the abovequoted conditions stated in the Court of First Instance of Cebu, which rendered the adverse decision as
disputed binding deposit receipt. As held by this Court, where an agreement is earlier refered to against both Pacific Life and Mondragon.
made between the applicant and the agent, no liability shall attach until the
principal approves the risk and a receipt is given by the agent. The acceptance is ISSUE/s:
merely conditional and is subordinated to the act of the company in approving or 6.   WoN the binding deposit receipt constituted a temporary contract of the life
rejecting the application. Thus, in life insurance, a "binding slip" or "binding insurance in question – NO, because there is an absence of a meeting of the
receipt" does not insure by itself. minds between Pacific Life and Ngo Hing and there is and non-compliance
of the abovequoted conditions stated in the disputed binding deposit receipt.
DOCTRINE: A contract of insurance, like other contracts, must be assented to
by both parties either in person or by their agents ... The contract, to be binding RULING: WHEREFORE, the decision appealed from is hereby set aside, and in
from the date of the application, must have been a completed contract, one that lieu thereof, one is hereby entered absolving petitioners Lapulapu D. Mondragon and
leaves nothing to be dione, nothing to be completed, nothing to be passed upon, Great Pacific Life Assurance Company from their civil liabilities as found by
or determined, before it shall take effect. There can be no contract of insurance respondent Court and ordering the aforesaid insurance company to reimburse the
unless the minds of the parties have met in agreement. amount of P1,077.75, without interest, to private respondent, Ngo Hing. Costs
against private respondent.
FACTS: RATIO:
1.   It appears that on March 14, 1957, Ngo Hing filed an application with the 1.   At the back of the binding receipt are condition precedents required before a
Great Pacific Life Assurance for a twenty-year endownment policy in the deposit is considered a BINDING RECEIPT. The aforequoted provisions
amount of P50,000.00 on the life of his one-year old daughter Helen Go. printed show that the binding deposit receipt is intended to be merely a
Pacific Life supplied the essential data which Lapulapu D. Mondragon,
Branch Manager of the Pacific Life in Cebu City wrote on the
provisional or temporary insurance contract and only upon compliance of application would not have any adverse effect on the allegedly perfected
the following conditions: temporary contract. There was no contract perfected between the parties
(1)   that the company shall be satisfied that the applicant who had no meeting of their minds. Ngo Hing, being an authorized
was insurable on standard rates; insurance agent of Pacific Life at Cebu branch office, is indubitably aware
(2)   that if the company does not accept the application and that said company does not offer the life insurance applied for. When he
offers to issue a policy for a different plan, the filed the insurance application in dispute, Ngo Hing was, therefore, only
insurance contract shall not be binding until the taking the chance that Pacific Life will approve the recommendation of
applicant accepts the policy offered; otherwise, the Mondragon for the acceptance and approval of the application in question
deposit shall be reftmded; and along with his proposal that the insurance company starts to offer the 20-
(3)   that if the applicant is not able according to the standard year endowment insurance plan for children less than seven years.
rates, and the company disapproves the application, the Nonetheless, the record discloses that Pacific Life had rejected the proposal
insurance applied for shall not be in force at any time, and recommendation.
and the premium paid shall be returned to the applicant. 7.   Also, having an insurable interest on the life of his one-year old daughter,
2.   Clearly implied from the aforesaid conditions is that the binding deposit aside from being an insurance agent and an offense associate of
receipt in question is merely an acknowledgment, on behalf of the company, Mondragon, Ngo Hing must have known and followed the progress on the
that the latter's branch office had received from the applicant the insurance processing of such application and could not pretend ignorance of the
premium and had accepted the application subject for processing by the Company's rejection of the 20-year endowment life insurance application.
insurance company; and that the latter will either approve or reject the same 8.   With regard to the issue that Ngo Hing concealed the state of health and
on the basis of whether or not the applicant is "insurable on standard rates." physical condition of Helen Go, which rendered void the aforesaid Binding
Since Pacific Life disapproved the insurance application of Ngo Hing, the Receipt, where he supplied the required essential data for the insurance
binding deposit receipt in question had never become in force at any time. application form, he was fully aware that his one-year old daughter is
3.   Upon this premise, the binding deposit receipt is, manifestly, merely typically a mongoloid child. Such a congenital physical defect could never
conditional and does not insure outright. As held by this Court, where an be ensconced nor disguished. Nonetheless, Ngo Hing, in apparent bad faith,
agreement is made between the applicant and the agent, no liability withheld the fact materal to the risk to be assumed by the insurance
shall attach until the principal approves the risk and a receipt is given compary. As an insurance agent of Pacific Life, he ought to know, as he
by the agent. The acceptance is merely conditional and is subordinated surely must have known his duty and responsibility to such a material fact.
to the act of the company in approving or rejecting the application. 9.   The contract of insurance is one of perfect good faith uberrima fides
Thus, in life insurance, a "binding slip" or "binding receipt" does not meaning good faith, absolute and perfect candor or openness and honesty;
insure by itself. the absence of any concealment or demotion, however slight, not for the
4.   In the absence of a meeting of the minds between Pacific Life and Ngo alone but equally so for the insurer. Concealment is a neglect to
Hing over the 20-year endowment life insurance in the amount of communicate that which a party knows and ought to communicate. Whether
P50,000.00 in favor of the latter's one-year old daughter, and with the non- intentional or unintentional the concealment entitles the insurer to rescind
compliance of the abovequoted conditions stated in the disputed binding the contract of insurance.
deposit receipt, there could have been no insurance contract duly perfected
between the, Accordingly, the deposit paid by Ngo Hing shall have to be
refunded by Pacific Life.
5.   As held in De Lim vs. Sun Life Assurance Company of Canada, supra, "a
contract of insurance, like other contracts, must be assented to by both
parties either in person or by their agents ... The contract, to be binding
from the date of the application, must have been a completed contract,
one that leaves nothing to be dione, nothing to be completed, nothing to
be passed upon, or determined, before it shall take effect. There can be
no contract of insurance unless the minds of the parties have met in
agreement."
6.   We are not impressed with Ngo Hing’s contention that failure of
Mondragon to communicate to him the rejection of the insurance
006 AMERICAN HOME ASSURANCE COMPANY v. TANTUCO
ENTERPRISES (Sabaupan) FACTS:
8 October 2001 | Puno, J. | Insurance Contract Interpretation 31.   Tantuco Enterprises, Inc., engaged in the coconut oil milling and refining
industry, owns two oil mills. Both are located at its factory compound in
Lucena City. Initially, it has only one oil mill. It started operating its second
PETITIONER: American Home Assurance Company
oil mill in 1988 and this oil mill is commonly referred to as the new oil mill.
RESPONDENTS: Tantuco Enterprises, Inc.
32.   The two oil mills were separately covered by fire insurance policies issued
by American Home Assurance Co., Philippine Branch. The first oil mill
SUMMARY: Tantuco Enterprises owns two oil mills, one of which was
was insured for P3 million for the period 1 March 1991 to 1992 while the
commonly referred to as the new oil mill. Both oil mills are covered by
new oil mill is insured for P6 million for the same term.
insurance policy from American Home Assurance for P3 million and P6 million.
33.   In September 1991, a fire broke out which gutted and consumed the new oil
A fire broke which gutted and consumed the new oil mill. Tantuco immediately
mill. Tantuco immediately notified American Home of the incident. After
informed the insurance company of the incident. However, the insurance
inspection, American Home rejected Tantuco’s claim for the insurance
company refused to pay on the ground that the building covered as specified by
proceeds on the ground that no policy was issued by it covering the burned
the boundaries stated in the policy is not the building that was gutted by fire.
oil mill. It stated that the description of the insured establishment referred to
Tantuco filed a claim for specific performance and damages with the RTC. The
another building. The policies allegedly extend insurance coverage to oil
RTC and CA both ruled that American Home Assurance is liable under the
mill under Building No. 5, whilst the affected oil mill is under Building No.
insurance policy. The issue is whether the insurance policy covers the new oil
14.
mill. The Supreme Court ruled in the affirmative. Notwithstanding the
34.   Tantuco instituted a complaint for specific performance and damages with
misdescription in the policy, it is beyond dispute that what the parties manifestly
the RTC. The lower court ruled that American Home is liable on the
intended to insure was the new oil mill. This is obvious from the categorical
insurance policy. On appeal, the CA affirmed. Hence, the instant petition.
statement embodied in the policy, extending its protection the equipment and
35.   American Home’s arguments:
accessories contained in the new oil mill building. If the parties really intended
a.   The burned oil mill is not covered by any insurance policy. The oil
to protect the first oil mill, then there is no need to specify it as new. It would be
mill insured is specifically described in the policy by its boundaries
absurd to assume the Tantuco would protect its first oil mill for different
and the oil mill gutted by fire was not the one described by the
amounts and leave uncovered the new oil mill. The imperfection in the
specific boundaries in the contested policy.
description of the insured oil mill’s boundaries can be attributed to a
b.   Tantuco did not have the supposed wrong description or mistake
misunderstanding between American Home’s general agent, Mr. Borja, and its
corrected despite the fact that the policy was issued 3 years before
policy issuing clerk, who made the error of copying the boundaries of the first oil
the dire and the despite the “Important Notice” in the policy that
mill when typing the policy to be issued for the new one. Mr. Borja testified that
states “Please read and examine the policy and if incorrect, return
the intention of Tantuco is to cover the new oil mill and the policy issuing clerk
it immediately for alteration.”
copied the description of the boundaries from the old policy, but she inserted
c.   Tantuco is barred by the parole evidence rule from presenting
covering the new oil mill, and to Mr. Borja, the important thing is that it covered
evidence other than the policy of its self-serving intention that it
the new oil mill because it is just within one compound and there are only two
intended really to insure the burned oil mill just as it is barred by
oil mills. It is thus clear that the source of the discrepancy happened during the
estoppel from claiming that the description of the insured oil mill
preparation of the written contract.
in the policy was wrong.
d.   Tantuco forfeited the renewal policy for its failure to pay the full
DOCTRINE: In view of the custom of insurance agents to examine buildings
amount of the premium and breach of the Fire Extinguishing
before writing policies upon them, and since a mistake as to the identity and
Appliances Warranty.
character of the building is extremely unlikely, the courts are inclined to consider
that the policy of insurance covers any building which the parties manifestly
ISSUE/s:
intended to insure, however inaccurate the description may be.
7.   Whether insurance policy covers the new oil mill. YES because the intent
was the parties was to cover the new oil mill and the courts are inclined to
In determining what the parties intended, the courts will read and construe the
consider that the policy of insurance covers any building which the parties
policy as a whole and if possible, give effect to all the parts of the contract,
manifestly intended to insure, however inaccurate the description may be.
keeping in mind always, however, the prime rule that in the event of doubt, this
doubt is to be resolved against the insurer.
RULING: Judgment in question is affirmed. Petition dismissed. witness to testify that Tantuco indeed failed to pay the full amount of the
premium.
RATIO: 26.   On the claim of violation of warranty, the SC held that the warranty did not
Insurance Issue require Tantuco to provide for all the fire extinguishing appliances (portable
20.   In construing the words used descriptive of a building insured, the greatest extinguishers, internal and external hydrants, fire pump, and 24-hour
liberality is shown by the courts in giving effect to the insurance. In view of security services) enumerated therein. Neither did it require that the
the custom of insurance agents to examine buildings before writing policies appliances are restricted to those mentioned in the warranty. What the
upon them, and since a mistake as to the identity and character of the warranty mandates is that Tantuco should maintain in efficient working
building is extremely unlikely, the courts are inclined to consider that the condition within the premises of the insured property, firefighting
policy of insurance covers any building which the parties manifestly equipment such as, but not limited to, those identified in the list. Tantuco
intended to insure, however inaccurate the description may be. was compliant with the warranty for within the vicinity of the new oil mill
21.   Notwithstanding the misdescription in the policy, it is beyond dispute that are the following: numerous portable fire extinguishers, two fire hoses, fire
what the parties manifestly intended to insure was the new oil mill. This is hydrant, and an emergency fire engine.
obvious from the categorical statement embodied in the policy, extending 27.   Not only are warranties strictly construed against the insurer, but they
its protection: should by themselves be reasonably interpreted. That reasonableness is to
“On machineries and equipment with complete accessories usual to a be ascertained in light of the factual conditions prevailing in each case. in
coconut oil mill including stocks of copra, copra cake and copra mills whilst this case, there is no more need for an internal hydrant considering that
contained in the new oil mill building x x x” inside the burned building were numerous portable fire extinguishers, an
22.   If the parties really intended to protect the first oil mill, then there is no emergency fire engine, and a fire hose which has a connection to one of the
need to specify it as new. It would be absurd to assume the Tantuco would external hydrants.
protect its first oil mill for different amounts and leave uncovered the new
oil mill. It is unthinkable for Tantuco to obtain the other policy from the Evidence issue
very same company considering that the first oil mill is already insured. The 28.   The present case falls within one of the recognized exceptions to the parole
latter ought to know that a second agreement over the same realty results in evidence rule. Under the Rules of Court, a party may present evidence to
its overinsurance. modify, explain or add to the terms of the written agreement if he puts in
23.   The imperfection in the description of the insured oil mill’s boundaries can issue in his pleading its failure to express the true intent and agreement of
be attributed to a misunderstanding between American Home’s general the parties thereto.
agent, Mr. Borja, and its policy issuing clerk, who made the error of 29.   In this case, the contractual intention of the parties cannot be understood
copying the boundaries of the first oil mill when typing the policy to be from a mere reading of the instrument. While the contract explicitly
issued for the new one. Mr. Borja testified that the intention of Tantuco is to stipulated that it was for the insurance of the new oil mill, the boundary
cover the new oil mill and the policy issuing clerk copied the description of description written on the policy concededly pertains to the first oil mill.
the boundaries from the old policy but she inserted covering the new oil This irreconcilable difference can only be clarified by admitting evidence
mill, and to Mr. Borja, the important thing is that it covered the new oil mill aliunde, which will explain the imperfection and clarify the intent of the
because it is just within one compound and there are only two oil mills. It is parties.
thus clear that the source of the discrepancy happened during the 30.   On the claim of estoppel, the Court ruled that evidence on record reveals
preparation of the written contract. that Tantuco’s operating manager notified Mr. Borja about the inaccurate
24.   The object of the court in construing a contract is to ascertain the intent of description in the policy. However, Mr. Borja assured Tantuco that the use
the parties and to enforce the agreement which the parties have entered into. of the adjective new will distinguish the insured property. The assurance
In determining what the parties intended, the courts will read and construe convinced Tantuco that despite the impreciseness in the specification of the
the policy as a whole and if possible, give effect to all the parts of the boundaries, the insurance will cover the new oil mill.
contract, keeping in mind always, however, the prime rule that in the event
of doubt, this doubt is to be resolved against the insurer.
25.   On the claim of non-payment of premiums, the CA refused to consider such
on the ground that said issue was raised for the first time on appeal, hence,
beyond its jurisdiction to resolve. The SC agrees. Moreover, American
Home fatally neglected to present, during the whole course of the trial, any
RIZAL SURETY v. CA (Saldua) Mar. 13, 1981. The buildings covered by the said fire insurance policy were
July 18, 2000 | Purisima J. | Art. 1377, New Civil Code also insured with New India Assurance Company, Ltd.
PETITIONER: Rizal Surety & Insurance Company 2.   On Jan. 12, 1981, a fire broke out in the compound of Transworld. The fire
RESPONDENTS: Court of Appeals and Transworld Knitting Mills, Inc. razed the middle portion of its four-span building (hereinafter referred to as
“Main Building”). It also partly gutted ( and thus was also destroyed by the
SUMMARY: fire) the left and right sections of the building (behind the said 4-span
Rizal Surety & Insurance Company issued Fire Insurance Policy No. 45727 in building) where fun & amusement machines and spareparts were stored
favor of Transworld Knitting Mills, Inc. initially for ₱1M, and eventually increased (hereinafter reffered to as “Annex or Annex Building”).
to ₱1.5M covering the period from August 14, 1980 to March 13, 1981. On January 3.   Transworld filed its insurance claims with Rizal Surety and New India
12, 1981, a fire broke out in the compound of Transworld razing the middle portion Assurance, but to no avail.
of its Main Building and partly gutting the left and right sections thereof. A 2- 4.   Thus, Transworld brought against them an action for collection of sum of
storey annex building (behind the four-span building) where fun and amusement money and damages praying for judgment ordering Rizal Insurance and
machines and spare parts were stored, was also destroyed by the fire. Transworld New India to pay the amount of ₱2,747,867.00 plus legal interest,
filed its insurance claims with Rizal Surety and New India Assurance Company, ₱400,000.00 as attorney's fees, exemplary damages, expenses of litigation
but to no avail. On May 26, 1982 Transworld brought against the said insurance of ₱50,000.00 and costs of suit.
companies an action for collection of sum of money and damages. Meanwhile, 5.   Meanwhile, Rizal Insurance countered that the subject fire insurance policy
Rizal Insurance countered that the subject fire insurance policy with Transworld with Transworld covered only the ocntents of the 4-span building (which
covered only the ocntents of the 4-span building (which was party burned), and was party burned), and does not cover the damage caused by the fire on the
does not cover the damage caused by the fire on the 2-storey annex building. 2-storey annex building.
Issue is WON the subject fire insurance policy protected only the contents of the 6.   Trial Court = Dismissed the case against New India. Ordered Rizal Surety
main building and did not include those stored in the annex building = NO. The to pay Transworld ₱826,500.00 representing the actual value of the losses
subject fire insurance policy protected both the contents of the main building and suffered by it.
the annex building. In this case, the loer courts found the so-called annex was 7.   CA = New India is ordered to pay ₱1,818,604.19 while Rizal Surety has to
actually an integral and inseparable part of the four-span building described in the pay Transworld ₱470,328.67, based on the actual losses sustained in the
policy. Consequently, the machines and spare parts stored in the annex were fire. Total amount of ₱2,790,376.00 is against the amounts of fire insurance
covered by the stipulations in the subject fire insurance policy. Verily, the 2-storey coverages respectively extended by New India (₱5,800,000.00) and Rizal
annex building involved a permanent structure which adjoins and Surety (₱1,500,000.00).
intercommunicates with the first right span of the lofty story main building, formed 8.   On Aug. 20, 1993, New India appealed the above order on the ground that
part thereof, and meets the requisites for compensability under the fire insurance Transworld could not be compensated for the loss of the Annex Building
policy sued upon. Moreover, the issue of whether or not private respondent has an because Transworld had no insurable interest in the goods/items stored
insurable interest in the fun and amusement machines and spare parts, which therein. However, CA denied this appeal with finality.
entitles it to be indemnified for the loss thereof, had been settled in the case of New
India Assurance Company vs. CA. Accordingly, the assailed decision and ISSUE/s:
resolution of the Court of Appeals were affirmed. 1.   WON the subject fire insurance policy protected only the contents of the
main building and did not include those stored in the annex building. NO.
DOCTRINE: The stipulation as to the coverage of the fire insurance policy under The subject fire insurance policy protected both the contents of the main
controversy has created a doubt regarding the portions of the building insured building and the annex building.
thereby. Article 1377 of the New Civil Code provides: "Art. 1377. The
interpretation of obscure words or stipulations in a contract shall not favor the party RULING: WHEREFORE, the Decision, dated July 15, 1993, and the Resolution,
who caused the obscurity" dated October 22, 1993, of the Court of Appeals in CA-G.R. CV NO. 28779 are
AFFIRMED in toto. No pronouncement as to costs. SO ORDERED.
FACTS: RATIO:
1.   On March 13, 1980, Rizal Insurance issued Fire Insurance Policy No. 1.   The resolution of this case depends upon the interpretation of the stipulation
45727 in favor of Transworld Knitting Mills Inc. (initialy for ₱1M and in the subject fire insurance policy with reference to the coverage reads:
eventually increased to ₱1.5M). It covered the period from Aug. 14, 1980 – ". . . contained and/or stored during the currency of
this Policy in the premises occupied by them
forming part of the buildings situate (sic) within
own Compound . . ."
2.   Thus, it can be understood from the stipulation that the subject fire
insurance policy did not limit its coverage to what were stored in the main
building only. As said by the RTC, 2 requirements must concur in order
that the Annex Building can be said to be protected by the subject fire
insurance policy:
A.   Said properties must be contained and/or stored in the areas
occupied by Transworld.
B.   Said areas must form part of the building described in the
policy.
3.   In this case, both the RTC and CA found that the Annex was actually an
integral and inseparable part of the main building described in the
policy. Consequently, the fun & amusement machines and spare parts
stored in the Annex Building are also covered by the subject fire
insurance policy.7
4.   The annex building (w/c is a 2-storey building) is a permanent structure w/c
adjoins and intercommunicates with the right span of the lofty storey
building formed part of said lofty storey building. Hence, it meets the
requisites for compensability under the subject fire insurance policy.
5.   Moreover, the 2-storey building (annex) was built in 1978, and the subject
fire insurance policy contract was entered into on Jan. 12, 1981. Given this,
Rizal Surety should have specifically excluded the said 2-storey annex
building from the coverage of the fire insurance.
6.   But Rizal Surety did not, and instead, it went on to provide that the said fire
insurance policy covers the products, raw materials and supplied that are
stored within the premises of Transworld which was an integral part of the
4-span main building occupied by Transworld. This is in spite knowing
fully well about the existence of the annex building that is adjoining and
intercommunicating with the right subject of the main building.
7.   Also, Article 1377, New Civil Code provides that the “interpretation of
obscure words or stipulations in a contract shall not favor the party
who caused the obscurity.” Thus, the doubt should be resolved against
Rizal Surety whose lawyer/managers drafted the subject fire insurance
policy.
8.   Considering that Transworld’s insurable interest in, and compensability for
the loss of subject fun and amusement machines and spare parts, had been
adjudicated, settled and sustained by the Court of Appeals (see fact #8) can
no longer be litigated, Rizal Surety is thus bound by this ruling.

                                                                                                                       
7
Annex Building - Two-storey building constructed of partly timber
and partly concrete hollow blocks under g.i. roof which is adjoining
and intercommunicating with the repair of the first right span of the
lofty storey building and thence by property fence wall.
008 PERLA COMPANIA v. CA (EMAR) d.   she sought reimbursement of said amounts from the Perla Compania, which
May 28, 1990 | Fernan, C.J. | Insurance Contract Interpretation notwithstanding the fact that her claim was within its contractual liability
PETITIONER: PERLA COMPANIA DE SEGUROS, INC. under the insurance policy, refused to make such re-imbursement;
RESPONDENTS: CA and MILAGRO CAYAS e.   she suffered moral damages as a consequence of such refusal, and
SUMMARY: Cayas’ bus, insured by Perla Compania, figured in an accident, injuring f.   she was constrained to secure the services of counsel to protect her rights;
people. 3 injured entered into a settlement with Cayas to which Cayas paid P4k/person and praying that judgment be rendered directing Perla Compania to pay:
while Perea filed a case against Cayas for damages in which the court ruled in favor of a.   P50k for compensation of the injured victims, such sum as the
Perea because Cayas failed to appear in the pre-trial and hence, she was declared as in court might approximate as damages, and
default. Cayas filed a complaint for a sum of money and damages against Perla b.   P6k as atty's fees.
Compania (for items in Fact #5a-f). The CFI ordered Perla Compania P50k for the 6.   Because of Cayas' failure to prosecute the case, the court motu propio ordered
injured passengers. CA affirmed. Perla Compania alleges that its liability should be dismissal without prejudice. Cayas filed an MR, alleging:
limited to a total of P12k, and that it should not be ordered to reimburse Cayas’ a.   She did not receive a copy of the answer to the complaint, and
payments to the 3 injured with whom she entered a settlement without Perla b.   "out of sportsmanship", she did not file a motion to hold Perla Compania in
Compania’s written consent. The SC ruled in favor of Perla Compania, on the grounds default.
that the insurance policy explicitly limits Perla Compania’s liability to P12k/person; 7.   March 1982: The MR was granted.
and required Perla Compania’s written consent before any payment in settlement 8.   2 months later: Cayas filed a motion to declare Perla Compania in default for
of any claim could be made. Cayas is precluded from seeking reimbursement of failure to file an answer. The motion was granted and Cayas was allowed to
the payments made to the 3 injured in view of her failure to comply with the adduce evidence ex-parte. The court rendered judgment by default and ordered
condition contained in the insurance policy Perla Compania to pay Cayas
DOCTRINE: The terms of the contract constitute the measure of the insurer's liability a.   P50k as compensation for the injured passengers,
and compliance therewith is a condition precedent to the insured's right of recovery b.   P5k as moral damages and
from the insurer. c.   P5k as atty’s fees.
9.   Said decision was set aside after the Perla Compania filed a motion therefor.
FACTS:
Trial of the case ensued.
1.   Cayas was the registered owner of a Mazda bus which was insured with Perla
a.   Perla Compania’s defense: before it can be made to pay, the liability must
Compania on February 3, 1978.
first be determined in an appropriate court action.
2.   December 1978: the bus figured in an accident in Naic, Cavite injuring several
b.   Despite CFI’s determination of liability, Perla Compania sought escape from
passengers including19-year old Perea who sued Cayas for damages in the CFI;
its obligation by saying that Cayas lost the case due to her negligence
and del Carmen, Magsarili and Antolin who agreed to a settlement of P4k each.
because of which, efforts exerted by Perla Compania’s lawyers in
3.   Cayas failed to appear in the pre-trial and hence, she was declared as in protecting Cayas' rights proved futile and rendered nugatory. It alleges
default. CFI decided in favor of Perea. that had Cayas cooperated fully with Perla Compania’s lawyers, the
4.   When the decision was about to be executed, Cayas filed a complaint against
latter could have won the suit and thus relieved of any obligation to
Perla Compania in the Office of the Insurance Commissioner praying that Perla
Perea. Even accepting Perla Compania’s postulate, it cannot be said, nor
Compania be ordered to pay P40k for all the claims against her arising from the
was it shown positively and convincingly, that if the case had proceeded
vehicular accident plus legal and other expenses. Realizing her procedural
on trial on the merits, a decision favorable to Cayas could have been
mistake, she later withdrew said complaint.
obtained. Nor was it definitely established that if the pre-trial was
5.   November 11, 1981: Cayas filed a complaint for a sum of money and damages
undertaken in that case, defendant's lawyers could have mitigated the claim
against Perla Compania in the CFI, alleging that:
for damages by Perea against Cayas.
a.   to satisfy the judgment in her previous case, her house and lot were sold at
c.   The court: There being proof that Cayas was compelled to engage the
public auction for P38k;
services of counsel to protect her rights under the insurance policy, the
b.   to avoid numerous suits and the "detention" of the insured vehicle, she paid
court granted P5k atty's fees.
P4k to each of the 4 injured passengers; 10.   May 8, 1987: CA affirmed in toto the decision. MR was denied.
c.   she could not have suffered said financial setback had the counsel for Perla
11.   Perla Compania seeks, before the SC, to limit its liability only to the payment
Compania, who also represented her, appeared at the trial and attended to the
made by Cayas to Perea and only up to the amount of P12K. It denies liability
claims of the 3 other victims;
for Cayas’ payments to the other 3 injured passengers for a total of P12k.
ISSUES: 7.   SC ruled as valid and binding upon Cayas the condition requiring her to secure
1.   WON Perla Compania should pay only P12k – YES, as limited by the contract. the written permission of petitioner before effecting any payment in
2.   WON Cayas is precluded from seeking reimbursement of the payments settlement of any claim against her. There is nothing unreasonable, arbitrary
made to the 3 injured– YES because in view of her failure to comply with the or objectionable in this stipulation as would warrant its nullification. The same
condition in the insurance policy to secure Perla Compania’s written consent was obviously designed to safeguard the insurer's interest against collusion
between the insured and the claimants.
RULING: CA decision is modified in that Perla Compania shall pay Cayas P12k 8.   In her cross-examination, Cayas admitted, thus:
plus legal interest from the promulgation of the decision of the lower court until it is Atty. Yabut:
fully paid and P5k atty's fees. No pronouncement as to costs. Q: With respect to the other injured passengers of your bus wherein you made payments you did not
secure the consent of Perla Compania when you made those payments?
A: I informed them about that.
RATIO: Q: But they did not give you the written authority that you were supposed to pay those claims?
1.   There is merit in La Compania’s assertions. A: No, sir.
2.   The insurance policy involved explicitly limits Perla Compania’s liability to 9.   It being specifically required that Perla Compania’s written consent be first
P12k/person and to P50k/accident. Pertinent provisions of the policy state: secured before any payment in settlement of any claim could be made,
SECTION I-Liability to the Public Cayas is precluded from seeking reimbursement of the payments made to
xxx xxx xxx the 3 injured in view of her failure to comply with the condition contained in the
3. The Limit of Liability stated in Schedule A as applicable
insurance policy.
(a) to 3rd PARTY is the limit of the Company's liability for all damages arising out of death, bodily injury
and damage to property combined so sustained as the result of any 1 accident; 10.   The principle that contracts are respected as the law between the contracting
(b) "per person" for PASSENGER liability is the limit of the Company's liability for all damages arising parties finds application in the present case. Thus, it was error on the part of
out of death or bodily injury sustained by one person as the result of any one accident: the TC and CA to disregard the stipulations of the parties and to have substituted
(c) "per accident" for PASSENGER liability is, subject to the above provisions respecting per person, the
their own interpretation of the insurance policy.
total limit of the Company's liability for all such damages arising out of death or bodily injury sustained
by two or more persons as the result of any one accident. 11.   Phil. American General Insurance vs. Mutuc: Contracts, private laws of the
Conditions Applicable to All Sections contracting parties, should be fulfilled according to the literal sense of their
xxx xxx xxx stipulations, if their terms are clear and leave no room for doubt as to the
5. No admission, offer, promise or payment shall be made by or on behalf of the insured without the intention of the contracting parties, for contracts are obligatory, no matter what
written consent of the Company which shall be entitled, if it so desires, to take over and conduct in his
name the defense or settlement of any claim, or to prosecute in his name for its own benefit any claim for form they may be, whenever the essential requisites for their validity are present.
indemnity or damages or otherwise, and shall have full discretion in the conduct of any proceedings in the 12.   Pacific Oxygen & Acetylene Co. vs. Central Bank: The first and fundamental
settlement of any claim, and the insured shall give all such information and assistance as the Company duty of the courts is the application of the law according to its express terms,
may require. If the Company shall make any payment in settlement of any claim, and such payment
interpretation being called for only when such literal application is
includes any amount not covered by this Policy, the Insured shall repay the Company the amount not so
covered. impossible.
3.   Stokes vs. Malayan Insurance Co.: The terms of the contract constitute the 13.   Patent error: Although Cayas was able to prove a total loss of only P44k,
measure of the insurer's liability and compliance therewith is a condition Perla Compania was made liable for the amount of P50k, the maximum
precedent to the insured's right of recovery from the insurer. liability per accident stipulated in the policy. An insurance indemnity, being
4.   Here, the insurance policy clearly and categorically placed Perla Compania’s merely an assistance or restitution insofar as can be fairly ascertained, cannot be
liability for all damages arising out of death/bodily injury sustained by 1 availed of by any accident victim or claimant as an instrument of enrichment by
person as a result of any 1 accident at P12k. reason of an accident.
5.   Said amount complied with the minimum fixed by the law then prevailing, 14.   SC found no reason to disturb the award of atty's fees.
Sec. 377, PD612 (which was retained by P.D. No. 1460, the Insurance Code of
1978), providing that the liability of land transportation vehicle operators for
bodily injuries sustained by a passenger arising out of the use of their
vehicles shall not be less than P12k, which means that the minimum liability is
P12k/passenger.
6.   Perla Compania’s liability under the insurance contract not being less than
P12k, and therefore not contrary to law, morals, good customs, public
order or public policy, said stipulation must be upheld as effective, valid
and binding as between the parties.
009 Zenith Insurance v. CA (Sarmiento) awarded as well.
May 14, 1990 | Medialeda, J. | Insurance—damages

PETITIONER: Zenith Insurance Corporation


FACTS:
RESPONDENTS: Court of Appeals and Lawrence Fernandez
1.   Private respondent Lawrence Fernandez insured his car for “own damage”
SUMMARY: Lawrence Fernandez insured his car for own damages under a under private car Policy No. 50459 with petitioner Zenith Insurance
private car with petitioner Zenith Insurance. The car had an accident and Corporation.
suffered actual damages in the amount of P3,640.00. Afer being given a run
around by Zenith for 2 months, Fernandez filed a complaint for sum of money 2.   The car figured in an accident and suffered actual damages in the amount of
with damages resulting from the refusal of Zenith to pay the amount claimed. A P3,640.00. After allegedly being given a run around by Zenith for two (2)
decision was rendered by the trial court A decision was rendered by the trial months, Fernandez filed a complaint with the Regional Trial Court
court in favor of private respondent Fernandez. Zenith Insurance was ordered to of Cebu for sum of money and damages resulting from the refusal of Zenith
pay Fernandez, the amount of P3,640.00 representing the damage incurred plus to pay the amount claimed.
interest at the rate of twice the prevailing interest rates; P20,000.00 by way of
moral damages; P20,000.00 by way of exemplary damages, amount of 3.   Zenith filed an answer alleging that it offered to pay the claim of Fernandez
P5,000.00 as attorney’s fees; amount of P3,000.00 as litigation expenses. Zenith pursuant to the terms and conditions of the contract which, the private
insurance filed an appeal. Zenith questions the award of moral, exemplary respondent rejected.
damages and attorney’s fees in an amount more than prayed for in the complaint. a.   Zenith wanted to settle the case, but it did not push through. Trial
The award of damages in case of unreasonable delay in the payment of insurance ensued, and Fernandez presented his evidence. Zenith, however
claims is governed by the Philippine Insurance Code. It is clear that under the failed to present its evidence, without any justifiable reason. The
Insurance Code, in case of unreasonable delay in the payment of the proceeds trial court issued an order submitting the case for decision without
of an insurance policy, the damages that may be awarded are: 1) attorney’s fees; Zenith’s evidence.
2) other expenses incurred by the insured person by reason of such unreasonable b.   Zenith filed a petition for certiorari with the Court of Appeals
denial or withholding of payment; 3) interest at twice the ceiling prescribed by assailing the order of the trial court submitting the case for
the Monetary Board of the amount of the claim due the injured; and 4) the decision without petitioner’s evidence. However, the petition was
amount of the claim. As regards the moral and exemplary damages, it governed denied
by the New Civil Code. The act of petitioner Zenith of delaying payment for two
months cannot be considered as so wanton or malevolent to justify an award of 4.   A decision was rendered by the trial court in favor of private respondent
P20,000.00 as moral damages, taking into consideration also the fact that the Fernandez. Zenith Insurance was ordered to pay Fernandez,
actual damage on the car was only P3,640. Moral damages of P10,000 is a.   The amount of P3,640.00 representing the damage incurred plus
equitable. On the other hand, exemplary or corrective damages are imposed, interest at the rate of twice the prevailing interest rates;
by way of example or correction for the public good. The amount of P5,000.00 b.   The amount of P20,000.00 by way of moral damages;
awarded as attorney’s fees is justified under the circumstances. As regards the c.   The amount of P20,000.00 by way of exemplary damages;
actual damages incurred by private respondent, the amount of P3,640.00 had d.   The amount of P5,000.00 as attorney’s fees;
been established. Policy does not also mention the policy, e.   The amount of P3,000.00 as litigation expenses; and
any deductible franchise. f.   Costs.
5.   Upon motion of Fernandez and before the expiration of the period to appeal,
DOCTRINE: It is clear that under the Insurance Code, in case of unreasonable the trial court, ordered the execution of the decision pending appeal.
delay in the payment of the proceeds of an insurance policy, the damages that a.   The order was assailed by petitioner in a petition
may be awarded are: 1) attorney’s fees; 2) other expenses incurred by the for certiorari with the Court of Appeals but which petition was
insured person by reason of such unreasonable denial or also dismissed.
withholding of payment; 3) interest at twice the ceiling prescribed by the b.   Petitioner filed a notice of appeal before the trial court.
Monetary Board of the amount of the claim due the injured; and 4) the amount of
the claim. And under the new civil code, moral and exemplary damages can be
i.   The notice of appeal was granted in the same order a.   “SEC. 244. In case of any litigation for the enforcement of any
granting private respondent’s motion for execution policy or contract of insurance, it shall be the duty of the
pending appeal. Commissioner or the Court, as the case may be, to make a finding
as to whether the payment of the claim of the insured has
ISSUE: Whehter or not the award of moral and exemplary damages is been unreasonably denied or withheld; and in the affirmative case,
proper—NO. Moral damages cannot be awarded because there was no bad faith on the insurance company shall be adjudged to pay damages which
the part of Zenith in delaying the payment of the proceeds. shall consist of attorney’s fees and other expenses incurred by the
insured person by reason of such unreasonable denial or
RULING: ACCORDINGLY, the appealed decision is MODIFIED as above stated. withholding of payment plus interest of twice the ceiling prescribed
by the Monetary Board of the amount of the claim due the insured,
RATIO: from the date following the time prescribed in section two hundred
forty-two or in section two hundred forty-three, as the case may be,
1.   Court of Appeals rendered its decision affirming in toto the decision of the until the claim is fully satisfied; Provided, That the failure to pay
trial court. It also ruled that the matter of the trial court’s denial of any such claim within the time prescribed in said sections shall be
Fernandez’ right to adduce evidence is a closed matter in view of its (CA) considered prima facie evidence of unreasonable delay in
ruling in AC-G.R. 04644 wherein Zenith’s petition questioning the trial payment.”
court’s order submitting the case for decision without Zenith’s evidence,
was dismissed 5.   It is clear that under the Insurance Code, in case of unreasonable delay in
a.   The Motion for Reconsideration of the decision of the Court of the payment of the proceeds of an insurance policy, the damages that may
Appeals was denied for lack of merit. be awarded are: 1) attorney’s fees; 2) other expenses incurred by the
b.   Hence, the instant petition was filed by Zenith on the allegation insured person by reason of such unreasonable denial or
that respondent Court of Appeals’ decision and resolution ran withholding of payment; 3) interest at twice the ceiling prescribed by the
counter to applicable decisions of this Court and that they were Monetary Board of the amount of the claim due the injured; and 4) the
rendered without or in excess of jurisdiction. amount of the claim.

2.   Zenith insurance questions the legal basis of the CA in awarding moral 6.   As regards the award of moral and exemplary damages, the rules under
damages, exemplary damages and attorney’s fees in an amount more than the New Civil Code of the Philippines shall govern.
that prayed for in the complaint. a.   “The purpose of moral damages is essentially indemnity or
a.   The award of actual damages of P3,640.00 instead of only reparation, not punishment or correction. Moral damages are
P1,927.50 which was arrived at after deducting P250.00 emphatically not intended to enrich a complainant at the expense
and P274.00 as deductible franchise and 20% depreciation on parts of a defendant, they are awarded only to enable the injured party to
as agreed upon in the contract of insurance. obtain means, diversions or amusements that will serve to alleviate
b.   Petitioner contends that while the complaint of private respondent the moral suffering he has undergone by reason of the defendant’s
prayed for P10,000.00 moral damages, the lower court awarded culpable action.”
twice the amount, or P20,000.00 without factual or legal basis; b.   While it is true that no proof of pecuniary loss is necessary in order
while private respondent prayed for P5,000.00 exemplary that moral damages may be adjudicated, the assessment of which is
damages, the trial court awarded P20,000.00; and while private left to the discretion of the court according to the circumstances of
respondent prayed for P3,000.00 attorney’s fees, the trial court each case (Art. 2216, New Civil Code), it is equally true that in
awarded P5,000.00. awarding moral damages in case of breach of contract, there must
be a showing that the breach was wanton and deliberately injurious
3.   The propriety of the award of moral damages, exemplary damages and or the one responsible acted fraudulently or in bad faith
attorney’s fees is the main issue raised herein by Zenith.
7.   In the instant case, there was a finding that private respondent was given a
4.   The award of damages in case of unreasonable delay in the payment of “run-around” for two months, which is the basis for the award
insurance claims is governed by the Philippine Insurance Code, which of the damages granted under the Insurance Code for unreasonable delay in
provides: the payment of the claim.
a.   However, the act of petitioner Zenith of delaying payment for two b.   2) P10,000.00 as moral damages;
months cannot be considered as so wanton or malevolent to justify c.   3) P5,000.00 as attorney’s fees;
an award of P20,000.00 as moral damages, taking into d.   4) P3,000.00 as litigation expenses; and
consideration also the fact that the actual damage on the car was e.   4) Costs.
only P3,640.
b.   The reason for Zenith’s failure to indemnify private respondent
Fernandez within the two-month period was that the parties could
not come to an agreement as regards the amount of the actual
damage on the car.
c.   The amount of P10,000.00 prayed for by private respondent as
moral damages is equitable.

8.   On the other hand, exemplary or corrective damages are imposed, by way


of example or correction for the public good (Art. 2229, New Civil Code of
the Philippines).
a.   In the case of Noda v. Cruz-Arnaldo, exemplary damages were not
awarded at the insurance company had not acted in wanton,
oppressive or malevolent manner.
b.   The same is true in the case at bar.

9.   The amount of P5,000.00 awarded as attorney’s fees is justified


under the circumstances of this case considering that there were other
petitions filed and defended by private respondent in connection with this
case.

10.   As regards the actual damages incurred by private respondent, the amount
of P3,640.00 had been established before the trial court and affirmed by the
appellate court.

11.   Respondent appellate court correctly ruled that the deductions of P250.00
and P274.00 as deductible franchise and 20% depreciation on parts,
respectively claimed by petitioners as agreed upon in the contract, had no
basis.
a.   Respondent court CA ruled: “Under its second assigned error,
defendant-appellant puts forward two arguments, both of which are
entirely without merit. It is contented that the amount recoverable
under the insurance policy defendant-appellant issued over the
car of plaintiff-appellee is subject to deductible franchise, and “the
policy, does not mention any deductible franchise.”

12.   Therefore, the award of moral damages is reduced to P10,000.00 and the
award of exemplary damages is hereby deleted. The awards due to private
respondent Fernandez are as follows:
a.   1) P3,640.00 as actual claim plus interest of twice the ceiling
prescribed by the Monetary Board computed from the time of
submission of proof of loss;
010 STRONGHOLD INSURANCE v. INTERPACIFIC CONTAINER 37.   While the policy was in effect, the truck figured into an accident along the
SERVICES (SEE) National highway in Brgy. Palihan, Hermosa, Bataan resulting in 4 deaths
July 1, 2015 | Perez, J. | Insurance Contract Interpretation and 3 serious injuries. 2 vehicles were also heavily damages.
38.   Chong then filed a claim for the recovery of P550,000.
PETITIONER: Stronghold Insurance Company Inc. a.   Comprehensive Third Party Liability (CTPL) --------- ₱50,000.00
b.   Own Damage (OD) --------------------------------------- ₱300,000.00
RESPONDENTS: Interpacific Container Services and Gloria Dee Chong
c.   Excess I Bodily Injury (BI)------------------------------ ₱100,000.00
d.   Third Party Liability (TPL) ----------------------------- ₱100,000.00
SUMMARY: Chong owned a Fuso truck which was insured with Stronghold e.   Total -------------------------------------------------------- ₱550,000.00
Insurance under a comprehensive motor car insurance policy which undertook to 39.   Stronghold denied on the ground that the driver was heavily drunk at the
indemnify the insured against loss or damage to the car and death or injury time of the accident as shown in the Pagpapatunay issued by the Barangay
caused to third persons by reason of accident. While the policy was in effect, the Chairman Rafael Torres and the Medico-Legal Certificate signed by Dr.
truck figured into an accident along the National highway in Brgy. Palihan, Bautista.
Hermosa, Bataan resulting in 4 deaths and 3 serious injuries. 2 vehicles were 40.   Chong and Interpacific Services (I assumed this is Chong’s company) then
also heavily damages. Chong then filed a claim for the recovery of P550,000. sued for recovery of sum of money before the RTC of Caloocan. They
Stronghold denied on the ground that the driver was heavily drunk at the time of argued that there was no sufficient proof to support the claim of the
the accident as shown in the Pagpapatunay issued by the Barangay Chairman Stronghold that the driver was drunk at the time of the incident
and the Medico-Legal Certificate. Chong and Interpacific Services then sued for underscoring the lack of mention of such crucial fact in the police blotter
recovery of sum of money before the RTC of Caloocan. The RTC and the CA report documenting the incident.
ruled for Chong and ordered Stronghold to pay. The issue in this case is WoN it 41.   Stronghold averred hat the intoxication of the driver of the insured vehicle
was proven during the trial that the driver of the insured vehicle was intoxicated legally avoided the liability of the insurance company under the policy.
at the time of the accident thereby precluding Chong and Interpacific from Stronghold further claimed that the insured violated Section 53 of the Land
claiming the proceeds of the insurance policy-NO, the evidences provided by Transportation and Traffic Code which prohibits driving of motor vehicles
Stronghold failed to prove intoxication. The RTC doubted the authenticity of the under the influence of alcohol. Since the driver of the insured vehicle was
Medico Legal Certificate because of the attendant alteration and tampering on found drunk at the time of the accident, the denial of the insurance claim of
the face of the document. The evident tampering of the medico legal certificate Chong is therefore justified under provisions of the insurance contract and
necessitated the presentation by Stronghold of additional evidence to buttress his the existing statutes.
claim but it did not present any other evidence. What further dampens 42.   The RTC ruled for Chong and Interpacific and ordered Stronghold to pay
Stronghold's position is the absence of the crucial fact of intoxication in the them. The CA affirmed but deleted the award for exemplary damages.
blotter report which officially documented the incident.
ISSUE/s:
DOCTRINE: This case involves a contract of insurance, the authenticity and 8.   WoN it was proven during the trial that the driver of the insured vehicle was
validity of which was uncontested. In exempting insurers from liability under intoxicated at the time of the accident thereby precluding Chong and
the contract, proof thereof must be clear, credible and convincing. Interpacific from claiming the proceeds of the insurance policy—NO, the
Fundamental is the rule that the contract is the law between the parties and, that evidences provided by Stronghold failed to prove intoxication.
absent any showing that its provisions are wholly or in part contrary to law,
morals, good customs, public order, or public policy, it shall be enforced to the RULING: WHEREFORE, premises considered, the instant petition is hereby
letter by the courts. DENIED. The assailed Decision of the Court of Appeals in CA-G.R. CV No. 80557
is hereby AFFIRMED.
FACTS:
36.   Gloria Dee Chong owned a Fuso truck with Plate number PWH 512. The RATIO:
truch was insured with Stronghold Insurance under a comprehensive motor 1.   Stronghold claims that the factual findings of the lower courts were fallible.
car insurance policy which undertook to indemnify the insured against loss However, the Supreme Court is not a trier of facs and as a rule, the factual
or damage to the car and death or injury caused to third persons by reason conclusion of the lower courts are rcognized by the Supreme Court, unless
of accident. of some facts and circumstances of weight and substance, having been
overlooked or misinterpreted, might materially affect the disposition of the
case. The exception has not been shown.
2.   Contrary to the claim of Stronghold; it miserably failed to prove the fact of
intoxication during the trial. Aside from the Medico Legal Certificate and
the Pagpapatunay, which were stripped of evidentiary value because of the
dubious circumstances under which they were obtained, the Stronghold did
not adduce other proof to justify the avoidance of the policy.
3.   The RTC doubted the authenticity of the Medico Legal Certificate because
of the attendant alteration and tampering on the face of the document.
In adopting the findings of the trial court, the appellate court reiterated the
evidentiary rule that the party alleging violation of the provision of the
contract bears the burden of proof to prove the same.
4.   The evident tampering of the medico legal certificate necessitated the
presentation by Stronghold of additional evidence to buttress his
claim. For instance, Stronghold could have adduced affidavits of witnesses
who were present at the scene of the accident to attest to the fact that the
driver was intoxicated. It did not. Upon the other hand, Chong and
Interpacific duly established their right to claim the proceeds of a validly
subsisting contract of insurance. Such contract was never denied.
5.   In civil cases, the party having the burden of proof must produce a
preponderance of evidence thereon, with plaintiff having to rely on the
strength of his own evidence and not upon the weakness of the defendant's.
The concept of "preponderance of evidence" refers to evidence which is of
greater weight or more convincing, than that which is offered in opposition
to it; at bottom, it means probability of truth.
6.   What further dampens Stronghold's position is the absence of the
crucial fact of intoxication in the blotter report which officially
documented the incident. Entries in police records made by a police
officer in the performance of the duty especially enjoined by law are prima
facie evidence of the fact therein stated, and their probative value may be
substantiated or nullified by other competent evidence. In this case, the lack
of statement to the effect that the driver was under the influence of alcohol
in the said report is too significant to escape the attention of this Court.
7.   This case involves a contract of insurance, the authenticity and validity of
which was uncontested. In exempting insurers from liability under the
contract, proof thereof must be clear, credible and convincing.
Fundamental is the rule that the contract is the law between the parties and,
that absent any showing that its provisions are wholly or in part contrary to
law, morals, good customs, public order, or public policy, it shall be
enforced to the letter by the courts.
White Gold Marine Services, Inc. vs. Pioneer Insurance and Surety Association (Bermuda) Limited (Steamship Mutual) through Pioneer Insurance
Corporation (Siapno) and Surety Corporation (Pioneer).
July 28, 2005 | Quisumbing, J. | Doing Insurance Business 2.   White Gold was issued a Certificate of Entry and Acceptance. Pioneer also issued
receipts evidencing payments for the coverage. When White Gold failed to fully
PETITIONER: White Gold Marine Services, Inc. pay its accounts, Steamship Mutual refused to renew the coverage.
RESPONDENT: Pioneer Insurance and Surety Corporation and the Steamship 3.   Steamship Mutual thereafter filed a case against White Gold for collection of sum
Mutual Underwriting Association (Bermuda) LTD. of money to recover the latter’s unpaid balance. White Gold on the other hand,
filed a complaint before the Insurance Commission claiming that Steamship
SUMMARY: White Gold Marine Services, Inc. (White Gold) procured a Mutual violated Sections 186 and 187 of the Insurance Code, while Pioneer
protection and indemnity coverage for its vessels from The Steamship Mutual violated Sections 299, 300 and 301 in relation to Sections 302 and 303, thereof
Underwriting Association (Bermuda) Limited (Steamship Mutual) through (see end of the digest, but these provisions talk about the requirements of
Pioneer Insurance and Surety Corporation (Pioneer). When White Gold failed to transacting insurance business in the Philippines)
fully pay its accounts, Steamship Mutual refused to renew the coverage. 4.   Insurance Commission: dismissed the complaint since was no need for Steamship
Steamship Mutual thereafter filed a case against White Gold for collection of Mutual to secure a license because it was not engaged in the insurance business.
sum of money to recover the latter’s unpaid balance. White Gold on the other It explained that Steamship Mutual was a Protection and Indemnity Club (P & I
hand, filed a complaint before the Insurance Commission claiming that Club). Likewise, Pioneer need not obtain another license as insurance agent
Steamship Mutual failed to secure the license needed for transacting insurance and/or a broker for Steamship Mutual because Steamship Mutual was not
businesses in the Philippines. Steamship Mutual claimed to be a Protection and engaged in the insurance business. Moreover, Pioneer was already licensed,
Indemnity Club and Pioneer claimed that it need not obtain another license as hence, a separate license solely as agent/broker of Steamship Mutual was already
insurance agent and/or a broker for Steamship Mutual because Steamship superfluous.
Mutual was not engaged in the insurance business. Issues are: 5.   CA: affirmed the decision of the Insurance Commissioner. In its decision, the
1.   WoN Steamship Mutual, a P & I Club, engaged in the insurance business in appellate court distinguished between P & I Clubs vis--vis conventional
the Philippines – YES The test to determine if a contract is an insurance insurance. The appellate court also held that Pioneer merely acted as a collection
contract or not, depends on the nature of the promise, the act required to be agent of Steamship Mutual.
performed, and the exact nature of the agreement in the light of the
occurrence, contingency, or circumstances under which the performance ISSUE:
becomes requisite. It is not by what it is called. A P & I Club is a form of 1.   WoN Steamship Mutual, a P & I Club, engaged in the insurance business in
insurance against third party liability, where the third party is anyone other the Philippines – YES, A P & I Club is a form of insurance against third party
than the P & I Club and the members. By definition then, Steamship Mutual liability, where the third party is anyone other than the P & I Club and the
as a P & I Club is a mutual insurance association engaged in the marine members. By definition then, Steamship Mutual as a P & I Club is a mutual
insurance business. insurance association engaged in the marine insurance business.
2.   WoN Pioneer needs a license as an insurance agent/broker for Steamship 2.   WoN Pioneer needs a license as an insurance agent/broker for Steamship
Mutual – YES. Although Pioneer is already licensed as an insurance Mutual – YES, Although Pioneer is already licensed as an insurance company, it
company, it needs a separate license to act as insurance agent for Steamship needs a separate license to act as insurance agent for Steamship Mutual as per
Mutual as per Section 299 of the Insurance Code. Section 299 of the Insurance Code

DOCTRINE: The test to determine if a contract is an insurance contract or not, RULING: The Steamship Mutual Underwriting Association (Bermuda) Ltd., and
depends on the nature of the promise, the act required to be performed, and the Pioneer Insurance and Surety Corporation are ORDERED to obtain licenses and to
exact nature of the agreement in the light of the occurrence, contingency, or secure proper authorizations to do business as insurer and insurance agent,
circumstances under which the performance becomes requisite. It is not by what respectively. The petitioners prayer for the revocation of Pioneers Certificate of
it is called. Authority and removal of its directors and officers, is DENIED. Costs against
respondents.
FACTS:
RATIO:
1.   White Gold Marine Services, Inc. (White Gold) procured a protection and
3.   The parties admit that Steamship Mutual is a P & I Club. Steamship Mutual
indemnity coverage for its vessels from The Steamship Mutual Underwriting
admits it does not have a license to do business in the Philippines although
Pioneer is its resident agent. This relationship is reflected in the certifications 13.  A P & I Club is a form of insurance against third party liability, where the
issued by the Insurance Commission. third party is anyone other than the P & I Club and the members. By
4.   Hyopsung Maritime Co., Ltd. v. CA: P & I Club is an association composed of definition then, Steamship Mutual as a P & I Club is a mutual insurance
shipowners in general who band together for the specific purpose of providing association engaged in the marine insurance business.
insurance cover on a mutual basis against liabilities incidental to shipowning that 14.  The records reveal Steamship Mutual is doing business in the country albeit
the members incur in favor of third parties. without the requisite certificate of authority mandated by Section 187 of the
5.   It stresses that as a P & I Club, Steamship Mutuals primary purpose is to solicit Insurance Code. It maintains a resident agent in the Philippines to solicit
and provide protection and indemnity coverage and for this purpose, it has insurance and to collect payments in its behalf. We note that Steamship Mutual
engaged the services of Pioneer to act as its agent. even renewed its P & I Club cover until it was cancelled due to non-payment of
6.   Pioneer and Steamship Mutual contend that although Steamship Mutual is a P & the calls. Thus, to continue doing business here, Steamship Mutual or
I Club, it is not engaged in the insurance business in the Philippines. It is merely through its agent Pioneer, must secure a license from the Insurance
an association of vessel owners who have come together to provide mutual Commission.
protection against liabilities incidental to shipowning. 15.  Since a contract of insurance involves public interest, regulation by the State
7.   Section 2(2) of the Insurance Code enumerates what constitutes doing an is necessary. Thus, no insurer or insurance company is allowed to engage in the
insurance business or transacting an insurance business. These are: insurance business without a license or a certificate of authority from the
(a) making or proposing to make, as insurer, any insurance contract; Insurance Commission.
(b) making, or proposing to make, as surety, any contract of suretyship as a vocation 16.  Pioneer is the resident agent of Steamship Mutual as evidenced by the certificate
and not as merely incidental to any other legitimate business or activity of the of registration issued by the Insurance Commission. It has been licensed to do or
surety; transact insurance business by virtue of the certificate of authority issued by the
(c) doing any kind of business, including a reinsurance business, specifically same agency. However, a Certification from the Commission states that
recognized as constituting the doing of an insurance business within the meaning Pioneer does not have a separate license to be an agent/broker of Steamship
of this Code; Mutual.
(d) doing or proposing to do any business in substance equivalent to any of the 17.  Although Pioneer is already licensed as an insurance company, it needs a
foregoing in a manner designed to evade the provisions of this Code. separate license to act as insurance agent for Steamship Mutual. Section 299
8.   The same provision also provides, the fact that no profit is derived from the of the Insurance Code clearly states:
making of insurance contracts, agreements or transactions, or that no separate or No person shall act as an insurance agent or as an insurance broker in the solicitation
direct consideration is received therefor, shall not preclude the existence of an or procurement of applications for insurance, or receive for services in obtaining
insurance business. insurance, any commission or other compensation from any insurance company
9.   The test to determine if a contract is an insurance contract or not, depends doing business in the Philippines or any agent thereof, without first procuring a
on the nature of the promise, the act required to be performed, and the exact license so to act from the Commissioner, which must be renewed annually on
nature of the agreement in the light of the occurrence, contingency, or the first day of January, or within six months thereafter.
circumstances under which the performance becomes requisite. It is not by 18.  Finally, White Gold seeks revocation of Pioneers certificate of authority and
what it is called. removal of its directors and officers. Regrettably, we are not the forum for these
10.  An insurance contract is a contract of indemnity. In it, one undertakes for a issues.
consideration to indemnify another against loss, damage or liability arising from
an unknown or contingent event. RELATED PROVISIONS:
SEC. 186. No person, partnership, or association of persons shall transact any insurance business in the
11.  In particular, a marine insurance undertakes to indemnify the assured against
Philippines except as agent of a person or corporation authorized to do the business of insurance in the
marine losses, such as the losses incident to a marine adventure. Section 99 of the Philippines, unless possessed of the capital and assets required of an insurance corporation doing the same
Insurance Code enumerates the coverage of marine insurance. kind of business in the Philippines and invested in the same manner; nor unless the Commissioner shall
12.  Relatedly, a mutual insurance company is a cooperative enterprise where the have granted to him or them a certificate to the effect that he or they have complied with all the provisions
of law which an insurance corporation doing business in the Philippines is required to observe.
members are both the insurer and insured. In it, the members all contribute, by a
Every person, partnership, or association receiving any such certificate of authority shall be subject to the
system of premiums or assessments, to the creation of a fund from which all insurance laws of the Philippines and to the jurisdiction and supervision of the Commissioner in the same
losses and liabilities are paid, and where the profits are divided among manner as if an insurance corporation authorized by the laws of the Philippines to engage in the business
themselves, in proportion to their interest. Additionally, mutual insurance of insurance specified in the certificate.
associations, or clubs, provide three types of coverage, namely, protection and SEC. 187. No Insurance Company shall transact any insurance business in the Philippines until after it
shall have obtained a certificate of authority for that purpose from the Commissioner upon application
indemnity, war risks, and defense costs. therefor and payment by the company concerned of the fees hereinafter prescribed.
SEC. 299. No insurance company doing business in the Philippines, nor any agent thereof, shall pay any
commission or other compensation to any person for services in obtaining insurance, unless such person
shall have first procured from the Commissioner a license to act as an insurance agent of such company or
as an insurance broker as hereinafter provided.
No person shall act as an insurance agent or as an insurance broker in the solicitation or procurement of
applications for insurance, or receive for services in obtaining insurance, any commission or other
compensation from any insurance company doing business in the Philippines or any agent thereof, without
first procuring a license so to act from the Commissioner, . . .
SEC. 300. Any person who for compensation solicits or obtains insurance on behalf of any insurance
company or transmits for a person other than himself an application for a policy or contract of insurance to
or from such company or offers or assumes to act in the negotiating of such insurance shall be an
insurance agent within the intent of this section and shall thereby become liable to all the duties,
requirements, liabilities and penalties to which an insurance agent is subject.
SEC. 301. Any person who for any compensation, commission or other thing of value acts or aids in any
manner in soliciting, negotiating or procuring the making of any insurance contract or in placing risk or
taking out insurance, on behalf of an insured other than himself, shall be an insurance broker within the
intent of this Code, and shall thereby become liable to all the duties, requirements, liabilities and penalties
to which an insurance broker is subject.
SEC. 99. Marine insurance includes:
(1) Insurance against loss of or damage to:
(a) Vessels, craft, aircraft, vehicles, goods, freights, cargoes, merchandise, effects, disbursements, profits,
moneys, securities, choses in action, evidences of debt, valuable papers, bottomry, and respondentia
interests and all other kinds of property and interests therein, in respect to, appertaining to or in connection
with any and all risks or perils of navigation, transit or transportation, or while being assembled, packed,
crated, baled, compressed or similarly prepared for shipment or while awaiting shipment, or during any
delays, storage, trasshipment, or reshipment incident thereto, including war risks, marine builders risks,
and all personal property floater risks.
(b) Person or property in connection with or appertaining to a marine, inland marine, transit or
transportation insurance, including liability for loss of or damage arising out of or in connection with the
construction, repair, operation, maintenance or use of the subject matter of such insurance (but not
including life insurance or surety bonds nor insurance against loss by reason of bodily injury to any person
arising out of the ownership, maintenance, or use of automobiles).
(c) Precious stones, jewels, jewelry, precious metals, whether in course of transportation or otherwise.
(d) Bridges, tunnels and other instrumentalities of transportation and communication (excluding buildings,
their furniture and furnishings, fixed contents and supplies held in storage); piers, wharves, docks and
slips, and other aids to navigation and transportation, including dry docks and marine railways, dams and
appurtenant facilities for the control of waterways.
(2) Marine protection and indemnity insurance, meaning insurance against, or against legal liability of the
insured for loss, damage, or expense incident to ownership, operation, chartering, maintenance, use,
repair, or construction of any vessel, craft or instrumentality in use in ocean or inland waterways,
including liability of the insured for personal injury, illness or death or for loss of or damage to the
property of another person.
001 GUINGON vs. DEL MONTE (SOLCO) accident caused by or arising out of the use of the Motor Vehicle/s or in
August 17, 1967 | BENGZON, J.P., J. | No Action Clause in Insurance Contracts connection with the loading or unloading of the Motor Vehicle/s, against all
sums including claimant's costs and expenses which the Insured shall
PETITIONER: DIONISIA GUINGON, et al. become legally liable to pay in respect of: (a.) death of or bodily injury to
RESPONDENTS: ILUMINADO DEL MONTE, et al. any person and (b.) damage to property.”
3.   Iluminado del Monte, one of the drivers of the jeepneys, while driving
SUMMARY: Julio Aguilar owned several Jeepneys which he insured with Capital along the intersection of Juan Luna and Moro streets in Manila, bumped
Insurance & Surety Co,. Inc. (CISC). The policy states that it will reimburse Aguilar, with the jeepney of Gervacio Guingon who had just alighted from another
subject to the limits of his liability, for all accidents caused by the use of the motor jeepney and as a consequence the latter died some days thereafter. A
vehicles covered against all sums including claimant's costs and expenses which corresponding information for homicide thru reckless imprudence was filed
the Insured shall become legally liable to pay in respect of: (a.) death of or bodily against del Monte, who pleaded guilty. A penalty of four months
injury to any person and (b.) damage to property. His jeepney driver, Iluminado imprisonment was imposed on him.
bumped another jeepney while Gervacio, a passenger, was alighting from it, which 4.   Corollary to the criminal action, the heirs of Gervacio Guingon filed an
caused Gervacio to die. So an action for homicide thru reckless imprudence was filed action for damages against Julio Aguilar, and the Capital Insurance &
against him for which he was convicted. Corollary to the action, the heirs filed an Surety Co., Inc. Julio Aguilar did not answer and was thus declared in
action for damages against Aguilar and CISC. CFI held both of them liable. CISC default.
appealed to SC stating that it isn’t liable for the death of Gervacio and also it cannot 5.   Capital Insurance & Surety Co., Inc answered, alleging that the plaintiff has
be impleaded in the case against Aguilar because of a certain “no action” clause in no cause of action against it.
their insurance contract which requires that a final judgement be rendered first 6.   CFI ordered Del Monte and Aguilar jointly and severally to pay P8,572.95
against the insured before they can be sued for the proceeds. SC ruled against CISC to plaintiffs as damages for the death of their father and P1,000 as
and stated that it is clear from the provisions of your contract with Aguilar that you attorney’s fees.
were insuring him against all damages which he may become legally liable for 7.   Capital Insurance and Surety Co., Inc. was also sentenced to pay the
arising out of any accident caused by his jeepneys which causes death or injury to a plaintiffs P5,000.00 and P500.00 as attorney's fees and costs which shall be
person. Secondly, even if the “no action” clause in the insurance contract states that applied in partial satisfaction of the judgment rendered against Iluminado
you cannot be sued or impleaded until a final judgement is rendered against against del Monte and Julio Aguilar in this case.
the insured, such clause cannot prevail against the Rules of Court which allows 8.   Hence this appeal by Capital Insurance & Surety Co., Inc.
permissive joinder of parties and was intended to avoid multiplicity of suits. Sec. 5
of Rule 2 on "Joinder of causes of action" and Sec. 6 of Rule 3 on "Permissive ISSUE/s:
joinder of parties" cannot be superseded, at least with respect to third persons not a 1.   Whether plaintiff can sue Capital Insurance & Surety Co., Inc.? If so, can
party to the contract, as herein, by a "no action" clause in the contract of insurance. plaintiffs sue the insurer jointly with the insured? YES to both, Firstly, the
insurance contract expressly states that it covers said liability.
DOCTRINE: 1. The "no action" clause in the policy of insurance cannot prevail Secondly, the “no action” clause in the contract cannot prevail over the
over the Rules of Court provision aimed at avoiding multiplicity of suits Rules of Court as provided for by jurisprudence.

2.  The right of the person injured to sue the insurer of the party at fault (insured), RULING: Wherefore, the judgment appealed from is affirmed in toto. Costs against
depends on whether the contract of insurance is intended to benefit third persons also appellant. So ordered.
or only the insured. The test to be applied is: Where the contract provides for
indemnity against liability to third persons, then third persons to whom the insured is RATIO:
liable,can sue the insurer. 1.   The right of the person injured to sue the insurer of the party at fault
(insured), depends on whether the contract of insurance is intended to
benefit third persons also or only the insured. The test to be applied is:
FACTS: Where the contract provides for indemnity against liability to third persons,
1.   In 1961, Julio Aguilar owned and operated several jeepneys in Manila. He then third persons to whom the insured is liable, can sue the insurer. Where
entered into a contract with the Capital Insurance &Surety Co., Inc. insuring the contract is for indemnity against actual loss or payment, then third
the operation of his jeepneys against accidents with third-party liability. persons cannot proceed against the insurer, the contract being solely to
2.   The insurance policy provides that “Capital Insurance & Surety Co., Inc. reimburse the insured for liability actually discharged by him thru payment
will, subject to the limits of liability, indemnify the Insured in the event of
to third persons, said third persons' recourse being thus limited to the
insured alone.
2.   The policy in the present case is one whereby the insurer agreed to
indemnify the insured "against all sums which the Insured shall
become legally liable to pay in respect of: a. death of or bodily injury to
any person." Clearly, therefore, it is one for indemnity against liability;
from the fact then that the insured is liable to the third person, such
third person is entitled to sue the insurer
3.   Appellant contends that the "no action" clause in the policy closes the
avenue to any third party which may be injured in an accident wherein the
jeepney of the insured might have been the cause of the injury of third
persons, alleging the freedom of contracts.

Action Against Company Clause:

1.   “No action shall lie against the Company unless, as a condition


precedent thereto, the Insured shall have fully complied with all of the
terms of this Policy, nor until the amount of the Insured's obligation to
pay shall have been finally determined either by judgment against the
Insured after actual trial or by written agreement of the Insured, the
claimant, and the Company.”
2.   “Any person or organization or the legal representative thereof who has
secured such judgment or written agreement shall thereafter be entitled to
recover under this policy to the extent of the insurance afforded by the
Policy. Nothing contained in this policy shall give any person or
organization any right to join the Company as a co-defendant in any
action against the Insured to determine the Insured's liability.”
3.   The "no action" clause in the policy of insurance cannot prevail over the
Rules of Court provision aimed at avoiding multiplicity of suits. In a case
squarely on the point, American Automobile Ins. Co. vs. Struwe, 218 SW
534 (Texas CCA), it was held that a "no action" clause in a policy of
insurance cannot override procedural rules aimed at avoidance of
multiplicity of suits.
4.   Similarly, in the instant suit, Sec. 5 of Rule 2 on "Joinder of causes of
action" and Sec. 6 of Rule 3 on "Permissive joinder of parties" cannot be
superseded, at least with respect to third persons not a party to the contract,
as herein, by a "no action" clause in the contract of insurance.
002 ETERNAL GARDENS MPC. v. PHILAM (STA. MARIA) person is insured as soon as he contracts a loan with Eternal, it cannot be then shown that
April 9, 2008| Velasco, Jr., J. | Covered by Insurance Philam would first need to approve his application for it to be valid; group life insurance
covers all those who contracted with Eternal, with or without the approval.
PETITIONERS: Eternal Gardens Memorial Park Corporation
RESPONDENTS: The Philippine American Life Insurance Company FACTS:
1.   Philamlife entered into an agreement denominated as Creditor Group Life
SUMMARY: Philamlife entered into an agreement denominated as Creditor Group Life Policy with petitioner Eternal Gardens Memorial Park Corporation
Policy with Eternal. Under the policy, the clients of Eternal who purchased burial lots
(Eternal).
from it on installment basis would be insured by Philamlife. The amount of insurance
coverage depended upon the existing balance of the purchased burial lots. The most 2.   Under the policy, the clients of Eternal who purchased burial lots from it on
relevan provision of the policy is EFFECTIVE DATE OF BENEFIT. The insurance of installment basis would be insured by Philamlife. The amount of insurance
any eligible Lot Purchaser shall be effective on the date he contracts a loan with the coverage depended upon the existing balance of the purchased burial lots.
Assured. However, there shall be no insurance if the application of the Lot Purchaser is The policy was to be effective for a period of one year, renewable on a
not approved by the Company.Eternal was required under the policy to submit to yearly basis.
Philamlife a list of all new lot purchasers, together with a copy of the application of each 3.   The relevant provisions8 of the policy are: (MOST RELEVANT)
purchaser, and the amounts of the respective unpaid balances of all insured lot purchasers. EFFECTIVE DATE OF BENEFIT. The insurance of any eligible Lot
Eternal complied by submitting a letter containing a list of insurable balances of its lot Purchaser shall be effective on the date he contracts a loan with the
buyers. One of those included in the list as “new business” John Chuang. His balance of
Assured. However, there shall be no insurance if the application of the Lot
payments was 100K. Later on, Chuang died. Eternal sent a letter dated to Philamlife,
which served as an insurance claim for Chuang’s death. Attached to the claim were Purchaser is not approved by the Company.
certain documents. In reply, Philamlife wrote Eternal a letter requiring Eternal to submit 4.   Eternal was required under the policy to submit to Philamlife a list of all
the additional documents relative to its insurance claim for Chuang’s death. Eternal new lot purchasers, together with a copy of the application of each
transmitted the required documents through a letter which was received by purchaser, and the amounts of the respective unpaid balances of all insured
Philamlife.After more than a year, Philamlife had not furnished Eternal with any reply to lot purchasers.
the latter’s insurance claim. This prompted Eternal to demand from Philamlife the 5.   Eternal complied by submitting a letter containing a list of insurable
payment of the claim for PhP 100,000. Philamlife denied Eternal’s insurance claim as balances of its lot buyers. One of those included in the list as "new
there was no application for Group Insurance was submitted in our office prior to business" was John Chuang. His balance of payments was PhP 100,000. On
Chuang’s death and hence, this was not covered by the insurance. Eternal filed a case
August 2, 1984, Chuang died.
with the RTC for a sum of money against Philamlife, which decided in favor of Eternal,
ordering Philamlife to pay the former 100K representing the proceeds of the policy. CA 6.   Eternal sent a letter to Philamlife, which served as an insurance claim for
reversed. The issue in this case is WON Chuang’s death was covered by the insurance Chuang’s death. Attached to the claim were the following documents: (1)
policy?– Yes. An examination of the provision of the policy under effective date of Chuang’s Certificate of Death; (2) Identification Certificate stating that
benefit, would show ambiguity between its two sentences. The first sentence appears to Chuang is a naturalized Filipino Citizen; (3) Certificate of Claimant; (4)
state that the insurance coverage of the clients of Eternal already became effective upon Certificate of Attending Physician; and (5) Assured’s Certificate.
contracting a loan with Eternal while the second sentence appears to require Philamlife to 7.   Philamlife wrote Eternal a letter requiring Eternal to submit the following
approve the insurance contract before the same can become effective. The SC ruled that documents relative to its insurance claim for Chuang’s death: (1) Certificate
since, an insurance contract is a contract of adhesion this must be construed liberally in of Claimant (with form attached); (2) Assured’s Certificate (with form
favor of the insured and strictly against the insurer. The SC harmonized the sentences to
mean that upon a party’s purchase of a memorial lot on installment from Eternal, an                                                                                                                        
8
insurance contract covering the lot purchaser is created and the same is effective, valid,  ELIGIBILITY.  Any  Lot  Purchaser  of  the  Assured  who  is  at  least  18  but  not  more  than  65  years  of  age,  is  
and binding until terminated by Philamlife by disapproving the insurance application. indebted   to   the   Assured   for   the   unpaid   balance   of   his   loan   with   the   Assured,   and   is   accepted   for   Life  
(Applied to this case, the purchase of the burial lot on installment basis by Chuang made Insurance  coverage  by  the  Company  on  its  effective  date  is  eligible  for  insurance  under  the  Policy.  
his death part of Philamlife’s insurance coverage.) EVIDENCE   OF   INSURABILITY.No   medical   examination   shall   be   required   for   amounts   of   insurance   up   to  
P50,000.00.   However,   a   declaration   of   good   health   shall   be   required   for   all   Lot   Purchasers   as   part   of   the  
application.  The  Company  reserves  the  right  to  require  further  evidence  of  insurability  satisfactory  to  the  
DOCTRINE: (No specific doctrine with respect to the syllabus topic which is what may Company  in  respect  of  the  following:  (1).  Any  amount  of  insurance  in  excess  of  P50,000.00.  (2).  Any  lot  
be insured/insured against? – this can be seen in the application of the facts but the court purchaser  who  is  more  than  55  years  of  age.  
used the rule on interpretation of a contract of adhesion in order to determine the coverage LIFE   INSURANCE   BENEFIT.   The   Life   Insurance   coverage   of   any   Lot   Purchaser   at   any   time   shall   be   the  
of the insurance policy) An insurance contract is a contract of adhesion which must be amount   of   the   unpaid   balance   of   his   loan   (including   arrears   up   to   but   not   exceeding   2   months)   as  
construed liberally in favor of the insured and strictly against the insurer in order to reported  by  the  Assured  to  the  Company  or  the  sum  of  P100,000.00,  whichever  is  smaller.  Such  benefit  
safeguard the latter’s interest. (As an application to the facts) When the contract states the shall  be  paid  to  the  Assured  if  the  Lot  Purchaser  dies  while  insured  under  the  Policy.  
 
attached); (3) Application for Insurance accomplished and signed by the memorial lot on installment from Eternal, an insurance contract covering
insured, Chuang, while still living; and (4) Statement of Account showing the lot purchaser is created and the same is effective, valid, and binding
the unpaid balance of Chuang before his death. until terminated by Philamlife by disapproving the insurance application.
8.   Eternal transmitted the required documents through a letter which was (Applied to this case, the purchase of the burial lot on installment basis by
received by Philamlife. Chuang made his death part of Philamlife’s insurance coverage.)
9.   After more than a year, Philamlife had not furnished Eternal with any reply
to the latter’s insurance claim. This prompted Eternal to demand from RULING: WHEREFORE, we GRANT the petition. The November 26, 2004 CA Decision
Philamlife the payment of the claim for PhP 100,000. in CA-G.R. CV No. 57810 is REVERSED and SET ASIDE. The May 29, 1996 Decision of
10.   In response to Eternal’s demand, Philamlife denied Eternal’s insurance the Makati City RTC, Branch 138 is MODIFIED. Philamlife is hereby ORDERED:
claim in a letter a portion of which reads: (1) To pay Eternal the amount of PhP 100,000 representing the proceeds of the Life Insurance
Policy of Chuang; (2) To pay Eternal legal interest at the rate of six percent (6%) per annum of
PhP 100,000 from the time of extra-judicial demand by Eternal until Philamlife’s receipt of
The deceased was 59 years old when he entered into Contract #9558 and 9529 with Eternal the May 29, 1996 RTC Decision on June 17, 1996; (3) To pay Eternal legal interest at the rate
Gardens Memorial Park in October 1982 for the total maximum insurable amount of of twelve percent (12%) per annum of PhP 100,000 from June 17, 1996 until full payment of
P100,000.00 each. No application for Group Insurance was submitted in our office prior this award; and (4) To pay Eternal attorney’s fees in the amount of PhP 10,000.
to his death. In accordance with our Creditor’s Group Life, under Evidence of Insurability
provision, "a declaration of good health shall be required for all Lot Purchasers as party of the RATIO:
application." We cite further the provision on Effective Date of Coverage under the policy
1.   As a general rule, this Court is not a trier of facts and will not re-examine
which states that "there shall be no insurance if the application is not approved by the
Company." Since no application had been submitted by the Insured/Assured, prior to his factual issues raised before the CA and first level courts, considering their
death, for our approval but was submitted instead after his death, Mr. John Uy Chuang findings of facts are conclusive and binding on this Court. However, such
was not covered under the Policy. We wish to point out that Eternal Gardens being the rule is subject to exceptions. One exception is when the findings of the CA
Assured was a party to the Contract and was therefore aware of these pertinent provisions. xxx are contrary to the trial court which exception is present in the instant case.
Thus, the SC can review the factual findings.
11.   Eternal filed a case before the RTC for a sum of money against Philamlife. 2.   Eternal claims that the evidence that it presented before the trial court
The RTC ruled in favor of Eternal, ordering Philamlife to pay P100,000 supports its contention that it submitted a copy of the insurance application
representing the proceeds of the Policy of John Uy Chuang. of Chuang before his death. In Eternal’s letter, a list of insurable interests of
12.   The RTC found that Eternal submitted Chuang’s application for insurance buyers for October 1982 was attached, including Chuang in the list of new
which he accomplished before his death, as testified to by Eternal’s witness businesses. Eternal added it was noted at the bottom of said letter that the
and evidenced by a lettr. It further ruled that due to Philamlife’s inaction corresponding "Phil-Am Life Insurance Application Forms & Cert." were
from the submission of the requirements of the group insurance to Chuang’s enclosed in the letter that was apparently received by Philamlife. Finally,
death, as well as Philamlife’s acceptance of the premiums during the same Eternal alleged that it provided a copy of the insurance application which
period, Philamlife was deemed to have approved Chuang’s application. The was signed by Chuang himself and executed before his death.
RTC said that since the contract is a group life insurance, once proof of 3.   Philamlife claims that the evidence presented by Eternal is insufficient,
death is submitted, payment must follow. arguing that Eternal must present evidence showing that Philamlife received
13.   Philamlife appealed to the CA which reversed the RTC’s order and the a copy of Chuang’s insurance application.
complaint was dismissed. It based its Decision on the factual finding that 4.   The evidence on record supports Eternal’s position.
Chuang’s application was not enclosed in Eternal’s letter. Thus, the CA 5.   The fact of the matter is, the letter, which Philamlife stamped as received,
concluded, there being no application form, Chuang was not covered by states that the insurance forms for the attached list of burial lot buyers were
Philamlife’s insurance. attached to the letter. Such stamp of receipt has the effect of acknowledging
receipt of the letter together with the attachments. Such receipt is an
ISSUE: admission by Philamlife against its own interest. The burden of evidence
1.   WON Chuang’s death was covered by the insurance policy? – Yes. An has shifted to Philamlife, which must prove that the letter did not contain
examination of the provision of the policy under effective date of benefit, Chuang’s insurance application. However, Philamlife failed to do so; thus,
would show ambiguity between its two sentences. The SC ruled that since, Philamlife is deemed to have received Chuang’s insurance application.
an insurance contract is a contract of adhesion this must be construed 6.   To reiterate, it was Philamlife’s bounden duty to make sure that before a
liberally in favor of the insured and strictly against the insurer. The SC transmittal letter is stamped as received, the contents of the letter are correct
harmonized the sentences to mean that upon a party’s purchase of a and accounted for.
7.   Philamlife’s allegation that Eternal’s witnesses ran out of credibility and it should be construed liberally in favor of the insured and strictly against
reliability due to inconsistencies is groundless. the insurer.
8.   An examination of the testimonies of the witnesses mentioned by 17.   The vague contractual provision, in Creditor Group Life Policy must
Philamlife, however, reveals no overlooked facts of substance and value. be construed in favor of the insured and in favor of the effectivity of the
9.   Philamlife primarily claims that Eternal did not even know where the insurance contract.
original insurance application of Chuang was, as shown by the testimony of 18.   On the other hand, the seemingly conflicting provisions must be
Edilberto Mendoza, when asked where the original was he said: “As far as harmonized to mean that upon a party’s purchase of a memorial lot on
I remember I do not know where the original but when I submitted with that installment from Eternal, an insurance contract covering the lot
payment together with the new clients all the originals I see to it before I purchaser is created and the same is effective, valid, and binding until
sign the transmittal letter the originals are attached therein.” terminated by Philamlife by disapproving the insurance application.
10.   In other words, the witness admitted not knowing where the original 19.   The second sentence of Creditor Group Life Policy on the Effective Date of
insurance application was, but believed that the application was transmitted Benefit is in the nature of a resolutory condition which would lead to the
to Philamlife as an attachment to a transmittal letter. cessation of the insurance contract. Moreover, the mere inaction of the
11.   As to the seeming inconsistencies between the testimony of Manuel Cortez insurer on the insurance application must not work to prejudice the insured;
on whether one or two insurance application forms were accomplished and it cannot be interpreted as a termination of the insurance contract. The
the testimony of Mendoza on who actually filled out the application form, termination of the insurance contract by the insurer must be explicit and
these are minor inconsistencies that do not affect the credibility of the unambiguous.
witnesses. 20.   To characterize the insurer and the insured as contracting parties on equal
12.   In the present case, the number of copies of the insurance application that footing is inaccurate at best. Insurance contracts are wholly prepared by
Chuang executed is not at issue, neither is whether the insurance application the insurer with vast amounts of experience in the industry
presented by Eternal has been falsified. Thus, the inconsistencies pointed purposefully used to its advantage. More often than not, insurance
out by Philamlife are minor and do not affect the credibility of Eternal’s contracts are contracts of adhesion containing technical terms and
witnesses. However, the question arises as to whether Philamlife assumed conditions of the industry, confusing if at all understandable to laypersons,
the risk of loss without approving the application. This question must that are imposed on those who wish to avail of insurance.
be answered in the affirmative. 21.   As such, insurance contracts are imbued with public interest that must be
13.   Philamlife and Eternal entered into an agreement denominated as Creditor considered whenever the rights and obligations of the insurer and the
Group Life Policy. In the policy, it is provided that: EFFECTIVE DATE insured are to be delineated. Hence, in order to protect the interest of
OF BENEFIT.The insurance of any eligible Lot Purchaser shall be insurance applicants, insurance companies must be obligated to act
effective on the date he contracts a loan with the Assured. However, there with haste upon insurance applications, to either deny or approve the
shall be no insurance if the application of the Lot Purchaser is not same, or otherwise be bound to honor the application as a valid,
approved by the Company. binding, and effective insurance contract.
14.   An examination of the above provision would show ambiguity between
its two sentences. The first sentence appears to state that the insurance
coverage of the clients of Eternal already became effective upon contracting
a loan with Eternal while the second sentence appears to require Philamlife
to approve the insurance contract before the same can become effective.
15.   It must be remembered that an insurance contract is a contract of
adhesion which must be construed liberally in favor of the insured and
strictly against the insurer in order to safeguard the latter’s interest.
16.   In Malayan Insurance Corporation v. Court of Appeals, this Court held
that: Indemnity and liability insurance policies are construed in accordance
with the general rule of resolving any ambiguity therein in favor of the
insured, where the contract or policy is prepared by the insurer. A contract
of insurance, being a contract of adhesion, par excellence, any
ambiguity therein should be resolved against the insurer; in other words,
003 Blue Cross Health Care v. Olivares (Soriano) service of limitless consultations for additional p1,000. She paid those
February 12, 2008 | Corona, J. | The Contract of Insurance amounts in full on oct 17, 2002. The application was approved on October
22, 2002.
PETITIONER: Clue Ceoss Health Care Inc. 44.   In the heath care agreement, ailments due to pre-existing conditions were
RESPONDENTS: Neomi and Danilo Olivares excludedfrom the coverage.
45.   On Nov 30, 2002, 38 days from the effectivity of her health insurance,
SUMMARY: Neomi Olivares applied for a health care program with Blue Neomi suffered stroke and was admitted at the Medical City hich was one
Cross. In the health care agreement, ailments due to pre-existing conditions were of the accredited hospitals by Blue Cross. During her confinement, she
excluded from the coverage. Neomi suffered stroke and was admitted to Medical underwent several lab tests.
City, one of the accredited hospitals. She incurred hospital exenses amounting to 46.   She incurred hospital expenses amounting tp P34,217.20. Consequently, she
P34,217.20. She requested from the representative of petitioner at Medical City a requested from the representative of petitioner at Medical City a letter of
letter of authorization in order to settle her medical bills. But petitioner refused authorization in order to settle her medical bills. But petitioner refused to
to issue the letter and suspended payment pending the submission of a issue the letter and suspended payment pending the submission of a
certification from her attending physician that the stroke she suffered was not certification from her attending physician that the stroke she suffered was
caused by a pre-existing condition. She demanded petitioner Blue Cross to pay not caused by a pre-existing condition.
the medical bill. When the petitioner refused, the spouses were constrained to 47.   When she was discharged, she demanded the petitioner to pay her mdical
settle the bill. Thereafter, they filed a complaint in the MeTC. Petitioner said that bill. hen petitioner still refused, she and her husband, respondent Danilo
it had not yet denied the claim, s it was awaiting for the attending physician’s Olivares, were constrained to settle the bill.
report. The Dr. Saniel, the attending physician, stated that as oer the request of 48.   Thereafter, they filed a complaint for collection of sum of money against
Neomi, he willnot release any medical information concerning neomi;s petitioner in the MeTC.
neurologic status to anyone without her approval. MeTC then denied the 49.   Blue Cross maintained that it had not yet denied respondents’ claim as it
complaint. RTC reversed the MeTC ruling and held that it was the burden of was still waiting Dr. Saniel’s report.
petitioner to prove that the stroke of Neomi was excluded. CA affirmed RTC 50.   Dr. Saniel in his letter to the petitioner state that Neomi called and stated
decision. Petitioner argues that respondents prevented Dr. Saniel from that she no longer has any relationship with Blue Cross, and that Dr. Saniel
submitting the medical report. Hence, the presumption that evidence willfully should not release any medical information concerning Neomi’s neurologic
suppressed would be adverse if produced should apply in its favor. Respondents status to anyone without her approval.
counter that the burden was on petitioner to prove that Neomi's stroke was 51.   MeTC dismissed the complaint for lack of cause of action. Stating that the
excluded from the coverage of their agreement because it was due to a pre- evidence (letter) on record reveals that it was no less than [respondent
existing condition. It failed to prove this. Issue: 1. Whther petitioner Blue Neomi] herself who prevented her attending physician from issuing the
Cross was able to prove that Neomi’s stroke was caused by pre-existing required certification, petitioner Blue Cross cannot be faulted from
condition and therefore was excluded from the coverage of the health agreement suspending payment of her claim, for until and unless it can be shown from
–No. Petitioner never presented any evidence to prove that respondent Neomi's the findings made by her attending physician that the stroke she suffered
stroke was due to a pre-existing condition. It merely speculated that Dr. Saniel's was not due to pre-existing conditions could she demand entitlement to the
report would be adverse to Neomi, based on her invocation of the doctor-patient benefits of her policy
privilege. 52.   RTC reversed the decision of the MeTC, ordered Blue Cross to pay the
medical bill, moral, exemplary damages, and attorney’s fees, and held that
DOCTRINE: It is an established rule in insurance contracts that when their that it was the burden of petitioner to prove that the stroke of respondent
terms contain limitations on liability, they should be construed strictly against Neomi was excluded from the coverage of the health care program for being
the insurer. These are contracts of adhesion the terms of which must be caused by a pre-existing condition. It was not able to discharge that burden.
interpreted and enforced stringently against the insurer which prepared the 53.   CA affirned RTC decision. Hence, this petition.
contract. This doctrine is equally applicable to health care agreements. 54.   Petitioner argues that respondents prevented Dr. Saniel from submitting the
medical report. Hence, the presumption that evidence willfully suppressed
would be adverse if produced should apply in its favor.
FACTS: 55.   Respondents counter that the burden was on petitioner to prove that Neomi's
43.   Respondent Neomi Olivares applied for a health care program with Blue stroke was excluded from the coverage of their agreement because it was
Cross, a health maintenance firm. For the period Oct 16, 2002 to Oct 15, due to a pre-existing condition. It failed to prove this.
2003, she paid P11,117. For the same period, she also availed of additional
of the agreement are excluded from its coverage if they become manifest
within one year from its effectivity. Stated otherwise, petitioner is not
ISSUE/s: liable for pre-existing conditions if they occur within one year from the
9.   Whther petitioner Blue Cross was able to prove that Neomi’s stroke was time the agreement takes effect.
caused by pre-existing condition and therefore was excluded from the 3.   The court ruled in a previous case that a health care agreement is in the
coverage of the health agreement –No. Petitioner never presented any nature of a non-life insurance.
evidence to prove that respondent Neomi's stroke was due to a pre-existing 4.   It is an established rule in insurance contracts that when their terms contain
condition. It merely speculated that Dr. Saniel's report would be adverse to limitations on liability, they should be construed strictly against the insurer.
Neomi, based on her invocation of the doctor-patient privilege These are contracts of adhesion the terms of which must be interpreted and
enforced stringently against the insurer which prepared the contract. This
RULING: WHEREFORE, the petition is hereby DENIED. The July 29, 2005 doctrine is equally applicable to health care agreements.
decision and September 21, 2005 resolution of the Court of Appeals in CA-G.R. SP 5.   Petitioner never presented any evidence to prove that respondent Neomi's
No. 84163 are AFFIRMED. stroke was due to a pre-existing condition. It merely speculated that Dr.
RATIO: Saniel's report would be adverse to Neomi, based on her invocation of the
1.   The court agrees with the respondents. doctor-patient privilege. This was a disputable presumption at best.
2.   The health care agreement defined a pre-existing condition as: 6.   Disputable presumption does not apply if (a) the evidence is at the disposal
a.   Disability which existed before the commencement date of membership of both parties; (b) the suppression was not willful; (c) it is merely
whose natural history can be clinically determined, whether the Member corroborative or cumulative and (d) the suppression is an exercise of a
was aware of such illness or condition. Such conditions also include privilege.
disabilities existing prior to reinstatement date in the case of lapse of an 7.   Here, respondents' refusal to present or allow the presentation of Dr.
Agreement. Notwithstanding, the following disabilities but not to the Saniel's report was justified. It was privileged communication between
exclusion of others are considered pre-existing conditions including their physician and patient.
complications when occurring during the first year of a Members 8.   Furthermore, as already stated, limitations of liability on the part of the
coverage: insurer or health care provider must be construed in such a way as to
I. Tumor of Internal Organs
preclude it from evading its obligations. Accordingly, they should be
II. Hemorrhoids/Anal Fistula scrutinized by the courts with extreme jealousy and care and with
III. Diseased tonsils and sinus conditions requiring surgery a jaundiced eye.
IV. Cataract/Glaucoma 9.   Since petitioner had the burden of proving exception to liability, it should
V. Pathological Abnormalities of nasal septum or turbinates
VI. Goiter and other thyroid disorders
have made its own assessment of whether respondent Neomi had a pre-
VII. Hernia/Benign prostatic hypertrophy existing condition when it failed to obtain the attending physician's report. It
VIII. Endometriosis could not just passively wait for Dr. Saniel's report to bail it out. The mere
IX. Asthma/Chronic Obstructive Lung disease reliance on a disputable presumption does not meet the strict standard
X. Epilepsy
XI. Scholiosis/Herniated disc and other Spinal column
required under our jurisprudence.
abnormalities 10.   On the issue on the grant of exemplary and moral damages: The RTC and
XII. Tuberculosis CA found that there was a factual basis for the damages adjudged against
XIII. Cholecysitis
petitioner. They found that it was guilty of bad faith in denying a claim
XIV. Gastric or Duodenal ulcer
XV. Hallux valgus
based merely on its own perception that there was a pre-existing condition
XVI. Hypertension and other Cardiovascular diseases
XVII. Calculi
XVIII. Tumors of skin, muscular tissue, bone or any form of
blood dyscracias
XIX. Diabetes Mellitus
XX. Collagen/Auto-Immune disease

After the Member has been continuously covered for 12


months, this pre-existing provision shall no longer be
applicable except for illnesses specifically excluded by an
endorsement and made part of this Agreement
b.   Under this provision, disabilities which existed before the commencement
004 MALAYAN INSURANCE v. REGIS BROKERS (Paul) 4.   When the shipment arrived at ABB Koppels warehouse, it was discovered
Nov. 23, ’07 | Tinga, J. | What may be insured – has to be proven that only 65 of the 120 pieces of motors were actually delivered and that
the remaining 55 motors, valued at US$2,374.35, could not be accounted
for.
PETITIONER: Malayan Insurance Co., Inc. 5.   The shipment was insured by Malayan. Malayan paid ABB Koppel Php
RESPONDENTS: Regis Brokerage Corp. 156,549.55 and got subrogated to the right of ABB Koppel v. Regis and
Paircargo.
SUMMARY: On Feb, 1, ’95, Fasco Motors Group loaded 120 pieces of motors 6.   Malayan then filed a complaint for damages against Regis and Paircargo
on board board China Airlines Flight 621 bound for Manila from the United with the Manila MeTC. In the course of the trial, Malayan presented
States. The cargo was to be delivered to consignee ABB Koppel, Inc. (ABB Marine Risk Note No. RN-0001-19832 (Marine Risk Note) dated 21
Koppel). When the cargo arrived at NAIA, it was cleared and forwarded to March 1995 as proof that the cargo was insured by Malayan.
Peoples Aircargo & Warehousing Corp’s (Paircargos) warehouse for temporary 7.   MeTC ruled that Regis alone was solely liable. The RTC affirmed this.
storage pending release by the Bureau of Customs. Paircargo remained in 8.   Regis brought it to the CA which reversed the RTC judgment and dismissed
possession of the cargo until 7 March 1995, at which point respondent Regis Malayan’s complaint. It held that the Marine Risk Note presented as proof
Brokerage Corp. (Regis) withdrew the cargo and delivered the same to ABB that the cargo was insured was invalid.
Koppel at its warehouse. When the shipment arrived at ABB Koppels warehouse, 9.   It was observed that the Marine Risk Note was procured from Malayan only
it was discovered that only 65 of the 120 pieces of motors were actually on 21 March 1995, when in fact the insured, ABB Koppel, had learned of
delivered. Malayan paid and filed a complaint for damages against Regis the partial loss of the motors as early as 7 March 1995.
and Paircargo with the MeTCMalayan presented the Marine Risk Note 10.   The CA noted that under Section 3 of the Insurance Code, the past event
dated 21 March 1995 as proof that the cargo was insured by Malayan. The which may be insured against must be unknown to the parties and so for
MeTC and the RTC ruled in Malayan’s favor. However, the CA reversed it, that reason the insurance contract in this case violated Section 3.
saying that the Marine Risk Note was procured from Malayan only on 21 11.   Also, the CA held that due execution and authenticity of the subrogation
March 1995, but the loss was discovered on March 7. The SC ruled that receipt presented before the trial court by Malayan were not duly proven
Malayan failed to establish its cause of action because it never presented the since the signatories thereto were not presented by Malayan before the trial
Marine Insurance Policy. Because Malayan's right to recovery derives from court to identify their signatures thereon, and neither was evidence
contractual subrogation as an incident to an insurance relationship, and not presented to establish the genuineness of such signatures
from any proximate injury to it inflicted by the Regis, it is critical that 12.   Malayan filed an MR and said that the Marine Risk Note is an open policy
Malayan establish the legal basis of such right to subrogation by presenting the per Marine Open Cargo Policy No. OPEN POLICY-0001-00410
contract constitutive of the insurance relationship between it and ABB Koppel. issued before February 1, 1995.
ISSUES:
DOCTRINE: The particular date as to when such insurance contract was 1.   W/N Malayan was able to establish its cause of action against Regis? No
constituted cannot be established with certainty without the contract itself, because it wasn’t able to conclusively prove the existence of an insurance
and that point is crucial since there can be no insurance on a risk that had contract.
already occurred by the time the contract was executed.
RULING: WHEREFORE, the petition is DENIED. Costs against petitioner.
FACTS:
1.   On Feb, 1, ’95, Fasco Motors Group loaded 120 pieces of motors on board RATIO:
board China Airlines Flight 621 bound for Manila from the United States. 1.   Malayan was arguing that the lost cargo was covered by not only the Marine
The cargo was to be delivered to consignee ABB Koppel, Inc. (ABB Risk Note but also by the Marine Insurance Policy (Fact 12) which was
Koppel). issued days before the cargo arrived in Manila.
2.   When the cargo arrived at NAIA, it was cleared and forwarded to Peoples 2.   The Marine Policy was attached to the petition of the SC, but it was not
Aircargo & Warehousing Corp’s (Paircargos) warehouse for temporary presented in trial.
storage pending release by the Bureau of Customs 3.   Malayan tried to showed that it had an “open policy” with ABB Koppel
3.   Paircargo remained in possession of the cargo until 7 March 1995, at which covered by Section 60 of the Insurance Code, wherein the value of the thing
point respondent Regis Brokerage Corp. (Regis) withdrew the cargo and insured is not agreed upon but left to be ascertained in case of loss, and that
delivered the same to ABB Koppel at its warehouse. the Marine Risk Note was nothing but a determination of the value of the
thing insured pursuant to the open policy as established by the Marine 13.   What the Marine Risk Note bears, as a matter of evidence, is that it is not
Insurance Policy. apparently the contract of insurance by itself, but merely a complementary
4.   Unfortunately for Malayan, the Court could not attribute any evidentiary or supplementary document to the contract of insurance that may have
weight to the Marine Insurance Policy. existed as between Malayan and ABB Koppel.
5.   The SC is not a trier of facts. Therefore, it relied on CA’s conclusion that at 14.   But again, Malayan never introduced the Marine Insurance Policy as the
the very instance the Marine Risk Note was offered in evidence, Regis main insurance contract nor even referred to it in the original complaint.
already posed its objection to the admission of said document on the ground Therefore, it failed to establish its cause of action.
that such was "immaterial, impertinent and irrelevant to this case because 15.   Because Malayan's right to recovery derives from contractual subrogation
the same was issued on March 21, 1995 which is after the occurrence of as an incident to an insurance relationship, and not from any proximate
the loss on February 1, 1995." injury to it inflicted by the Regis, it is critical that Malayan establish the
6.   Since no insurance policy was presented at the trial by Malayan, or even legal basis of such right to subrogation by presenting the contract
before the CA, there is no basis for the SC to consider it, notwithstanding constitutive of the insurance relationship between it and ABB Koppel.
Malayans attempt to submit such document to us along with its present 16.   his should have been accomplished from the moment it filed the complaint.
petition, even it may have reflected the existence of an insurance contract Since the Marine Insurance Policy was constitutive of the insurer-insured
between Malayan and ABB Koppel prior to the loss of the motors. relationship from which Malayan draws its right to subrogation, such
7.   Malayan’s theory of the case it pursued before the trial court was that the document should have been attached to the complaint itself.
perfected insurance contract which it relied upon as basis for its right to 17.   It is an actionable document and Sec. 710, Rule 9 of the Rules of Civil
subrogation was not the Marine Insurance Policy but the Marine Risk Procedure governs it.
Note which, unlike the former, was actually presented at the trial and 18.   It was incumbent on Malayan, whose right of subrogation derived from the
offered in evidence. Marine Insurance Policy, to set forth the substance of such contract in its
8.   The Claims Processor of Malayan who testified in court in behalf of his complaint and to attach an original or a copy of such contract in the
employer actually acknowledged that the "proof that ABB Koppel insured complaint.
the shipment to Malayan" was the Marine Risk Note, and not the Marine 19.   Not doing so actually violates Regis’ right to due proves because the latter
Insurance Policy. cannot object to it.
9.   Even the very complaint filed by Malayan before the MeTC stated that "the 20.   It cannot be denied from the only established facts that Malayan and ABB
subject shipment was insured by Malayan under Risk Note No. 0001- Koppel comported as if there was an insurance relationship between them
19832," and not by the Marine Insurance Policy, which was not adverted to and documents exist that evince the presence of such legal relationship.
at all in the complaint. 21.   But under these premises, the very insurance contract emerges as the white
10.   Therefore, the SC can only consider the Marine Risk Note in determining elephant in the room — an obdurate presence which everybody reacts to,
the existence of an insurance contract. However, again, it is dated Mar. 21, yet legally invisible as a matter of evidence since no attempt had been made
1995 – after the loss9. However, the motors insured were already to prove its corporeal existence in the court of law.
compromised by then. 22.   Malayan would have us effectuate an insurance contract without having to
11.   The Court, in Aboitiz c. Philam Gen. has already held that a Marine Risk consider its particular terms and conditions, and on a blind leap of faith that
Note is not an insurance policy on its own. It is only an acknowledgment or such contract is indeed valid and subsisting.
declaration of the private respondent confirming the specific shipment 23.   The conclusion further works to the utter prejudice of Regis since it would
covered by its Marine Open Policy, the evaluation of the cargo, and the be deprived the opportunity to examine the document that gives rise to the
chargeable premium, a description that is reflective as well of the present Malayan’s right to recover against them, or to raise arguments or objections
Marine Risk Note, if not of marine risk notes in this country in general. against the validity or admissibility of such document.
12.   Malayan correctly points out that the Marine Risk Note itself adverts to
"Marine Cargo Policy Number Open Policy-0001-00410" as well as to "the                                                                                                                        
10
standard Marine Cargo Policy and the Company's Marine Open Policy."  SEC.  7.  Action  or  defense  based  on  document.   —  Whenever  an  action  or  defense  is  based  
                                                                                                                        upon  a  written  instrument  or  document,  the  substance  of  such  instrument  or  document  shall  
9
 "Had  this  day  noted  the  above-­‐mentioned  risk  in  your  favor  and  hereby  guarantees  that  this   be   set   forth   in   the   pleading,   and   the   original   or   a   copy   thereof   shall   be   attached   to   the  
document   has   all   the   force   and   effect   of   the   terms   and   conditions   in   the   Corporation's   pleading  as  an  exhibit,  which  shall  be  deemed  to  be  a  part  of  the  pleading,  or  said  copy  may  
printed  form  of  the  standard  Marine  Cargo  Policy  and  the  Company's  Marine  Open  Policy."     with  like  effect  be  set  forth  in  the  pleading.    
   
24.   If a legal claim is irrefragably sourced from an actionable document, the
defendants cannot be deprived of the right to examine or utilize such
document in order to intelligently raise a defense.
25.   The inability or refusal of the plaintiff to submit such document into
evidence constitutes an effective denial of that right of the defendant which
is ultimately rooted in due process of law.
26.   The particular date as to when such insurance contract was constituted
cannot be established with certainty without the contract itself, and that
point is crucial since there can be no insurance on a risk that had
already occurred by the time the contract was executed.
27.   Since the documents in evidence and testimonies allude to "marine
insurance" or "marine risk note," it also is a legitimate question whether
the particular marine insurance relationship between Malayan and ABB
Koppel also covers cargo delivered not by ships at sea but by airplane
flights, as had occurred in this case. Only the actual policy itself could
definitively settle such a question.
005 EASTERN SHIPPING v. PRUDENTIAL (TIMBOL)
September 11, 2009 | Del Castillo, J. | What can be insured/insured against FACTS:
33.   56 cases of completely knock-down auto parts of Nissan motor vehicles
PETITIONER: Eastern Shipping Lines, Inc. (cargoes) were loaded on board M/V Apollo Tujuh (carrier) at Nagoya,
RESPONDENTS: Prudential Guarantee and Assurance, Inc. Japan, to be shipped to Manila
1.   The shipment was consigned to Nissan Motor Philippines, Inc.
SUMMARY: 56 cases of completely knock-down auto parts of Nissan Motor (Nissan) and was covered by Bill of Lading No. NMA-1
vehicles were loaded on board M/V Apollo Tujuh, owned by Eastern Shipping, at 2.   The carrier was owned and operated by Eastern Shipping Lines,
Japan to be shipped to Manila. When it arrived to Manila, it was discharged to ATI Inc.
in good condition, except for the four cases. From ATI, they were then brought to 34.   The carrier arrived at the port of Manila. The shipment was then discharged
Nissan’s warehouse, and after conducting a survey, the four cases were found to from the vessel onto the custody of the arrastre operator, Asian Terminals,
have shortage and damage due to the pilferage and improper handling of the vessel Inc. (ATI), complete and in good condition, except for four cases
and/or the Arrastre contractors. Thus, Nissan demanded for reimbursement from 35.   The shipment was withdrawn by Saefront Customs and Brokerage from the
Eastern Shipping and ATI, but failed. After which, Nissan claimed reimbursement pier and delivered to the warehouse of Nissan in Quezon City
from Prudential as the insurer based on all risks per Marine Open Policy and Marine 36.   A survey of the shipment was then conducted by Tan-Gaute Adjustment
Cargo Risk Notice. Upon payment of Prudential to Nissan, the former filed a claim Company, Inc. (surveyor) at Nissan’s warehouse
against Eastern and ATI. The RTC ruled in favor of Prudential finding Eastern and 1.   The surveyor submitted its report with a finding that there were
ATI solidarily liable. However, on appeal, the CA exonerated ATI and held Eastern “short (missing)” items in Cases Nos. 10/A26/T3K and 10/A26/7K
Shipping solely responsible. And that the presentation of insurance policy is and “broken/scratched” and “broken” items in Case No.
dispensible. 10/A26/70K”;
2.   And that in its opinion, the shortage and darnage sustained by the
Hence the issue before the Court being whether the Marine Risk Note and the shipment were due to pilferage and improper handling,
Subrogation Receipt, without the presentation of the Marine Insurance Policy, are respectively while in the custody of the vessel and/or Arrastre
sufficient to prove Prudential’s right to subrogation. Contractors
37.   Nissan demanded the sum of P1,047,298.34 representing the cost of the
The SC held that NO, presentation in this case is required for Prudential to claim the damages sustained by the shipment from Eastern Shipping, the owner of the
right of subrogration. It must first be noted that a Marine Risk Note is not an vessel, and ATI, the arrastre operated
insurance policy. Rather, it is an acknowledgement or declaration of the insurer 1.   However, the demands were not heeded
confirming the specific shipment covered by its marine open policy, the evaluation 38.   As insurer of the shipment against all risks per Marine Open Policy No. 86-
of the cargo and the chargeable premium. Furthermore, the Marine Risk Note was 168 and Marine Cargo Risk Note No. 3921/95, Prudential Guarantee and
questionable since the date of the such note is the same as the time the goods arrived Assurance Inc. paid Nissan the sum of P1,047,298.34
the port. Meaning that the goods were not specifically covered by any particular 39.   Prudential sued Eastern and ATI for reimbursement of the amount it paid to
insurance at the time of transit. And without the presentation of the Marine Nissan before the RTC of Makati, and claimed that it was subrogated to the
Insurance Policy, it would be impossible to know the specifics of the Policy as rights of Nissan by virtue of said payment
written in the Note, as well as the terms and conditions were complied before 40.   RTC rendered a decision in favor of Prudential and held that Eastern and
Prudential paid Nissan’s claim. Moreover, the reading of the transcript of records ATI are solidarily liable
show that Eastern Shipping already objected to the non-presentation of the Marine 41.   On appeal, the CA exonerated ATI and ruled that Eastern was solely
Insurance Policy. And since the claim of insurance is based on an actionable responsible for the damages caused to the cargoes.
document, it was incumbent upon Prudential to present the policy based on Sec 7, 1.   Moreover, the CA relying on Delsan Transport Lines, Inc. v. CA,
Rule 9 of the Rules of Civil Procedure. ruled that the right of subrogation accrues upon payment by the
insurance company of the insurance claim and that the presentation
DOCTRINE: It is significant that the date when the alleged insurance contract was of the insurance policy is not indispensable before the appellee
constituted cannot be established with certainty without the contract itself. Said point may recover in the exercise of its subrogatory right
is crucial because there can be no insurance on a risk that had already occurred by 42.   As such, Eastern argues that Prudential was not properly subrograted
the time the contract was executed. Surely, the Marine Risk Note on its face does not because of the non-presentation of the marine insurance policy. In the
specify when the insurance was constituted
case at bar, in order to prove its claim, Prudential presented a marine cargo guarantees that this document has all the force and effect of the
risk note and a subrogation receipt. terms and conditions in the Corporation’s printed form of the
standard Marine Cargo Policy and the Company’s Marine Open
ISSUE/s: Policy “
6.   WoN the Marine Risk Note and the Subrogation Receipt, without the 41.   Likewise, the date of the issuance of the Marine Risk Note also caught the
Marine Insurance Policy, are sufficient to prove Prudential’s right of attention of Eastern Shipping, stating in it’s comment that:
subrogration – NO, because a Mairne Risk Note is only an 1.   Exh. B, Marine Cargo Risk Note No. 39821 dated Nov. 16, 1995 is
acknowledgement or declaration of the insurer confirming the specific being objected to for being irrelevant and immaterial as it was
shipment covered by its marine open policy. The presentation of the marine executed on Nov.16, 1995. The cargoes arrived in Manila on Nov.
open policy indicating therein the extent of the note, as well as the terms 16, 1995.
and conditions needed to be complied with. 2.   This means that the cargoes are not specifically covered by any
particular insurance at the time of transit
RULING: WHEREFORE, premises considered, the petition is GRANTED. The 3.   The alleged Marine Open Policy was not presented. Marine Open
April 26, 2006 Decision and August 15, 2006 Resolution of the Court of Appeals in Policy may be subject to Institute Cargo Clauses which may
CA G.R. CV No. 68165 are hereby REVERSED and SET ASIDE. The Complaint require arbitration prior to the filing of an action in court.
in Civil Case No. 96 1665 is DISMISSED. 42.   The Marine Risk Note relied upon by Prudential as the basis for its
claim for subrogation is insufficient to prove said claim
RATIO: 1.   Hence, without a copy of the marine insurance policy, it would be
36.   Before anything else, it must be emphasized that a marine risk note is not impossible and simply guesswork to know whether the cargo was
an insurance policy. It is only an acknowledgement or declaration of the inusred during the voyage which started on Nov. 8, 1995
insurer confirming the specific shipment covered by its marine open policy, 2.   Again, without the marine insurance policy, it would be
the evaluation of the cargo and the chargeable premim impossible for this Court to know the following:
37.   The nature of a marine cargo risk note was explained in ICTSI v. FGU 1.   The specifics of the Institute Cargo Clauses A and
Insurance Corporation: other terms and conditions per Marine Open Policy-
1.   It is the marine open polic which is the main insurance contract. In 86-168 as alluded to in the Marine Risk Note;
other words, the marine open policy is the blanket insurance ot be 2.   If the said terms and conditions were actually
undertaken by FGU on all goods to be shipped by RAGC during complied with before Prudential paid Nissan’s claim
the existence of the contract, while the marine risk note specifies 43.   Furthermore, a reading of the transcript of the records clearly show that, at
the particular goods/shipment insured by FGU on that specific the RTC, Eastern Shipping had already objected to the non-presentation of
transaction, including the sum insured, the shipment the marine insurance policy
particulars as well as the premium paid for such shipment 1.   Clearly, Eastern Shipping was not remis when it openly objected to
38.   It is undisputed that the cargoes were already on board the carrier as early as the non-presentation oft eh Marine Insurance Policy
Nov. 8, 1995 and that the same arrived at the port of Manila on Nov. 16, 44.   In addition, it was Prudential’s burden to present the evidence necessary to
1995 substantiate its claim
39.   It is, however, very apparent that the Marine Cargo Risk Note was issued 1.   In its Complaint, Prudential alleged: “That the above-described
only on November 16, 1995. The same, therefore, should have raised a red shipment was insured for P14,173,042.91 against all risks under
flag, as it would be impossible to know whether said goods were actually Marine Cargo Risk Note No. 39821/Marine Open Policy No. 86-
insured while the same were in transit from Japan to Manila. 168.”
40.   Malayan Insurance Co., Inc. v. Regis Brokerage Corp: 2.   Therefore, other than the marine cargo risk note, Prudential should
1.   Thus, we can only consider the Marine Risk Note in determining have also presented the marine insurance policy, as the same also
whether there existed a contract of insurance between ABB Koppel served as the basis for its complaint
and Malayan at the time of the loss of the motors 45.   Sec. 7, Rule 9 of the 1997 Rules of Civil Procedure, provide:
2.   However, the very terms of the Marine Risk Note itself are 1.   Action or defense based on document – Whenever an action or
quite damning. It is dated 21 Marhc 1995, or after the defense is based upon a written instrument or document, the
occurrence of the loss, and specifically states that Malayan “had substance of such instrument or document shall be set forth in
this day noted the above-mentioned risk in your favor and hereby the pleading, and the original or a copy thereof shall be
attached to the pleading as an exhibit, which shall be deemed vessel, unlike in Home Insurance in which the cargo passed
to be a part of the pleading, or said copy may, with like effect, through several statges with different parties and it could not be
be set forth in the pleading. determined when the damage to the cargo occurred, such that the
46.   Malayan Insurance Co., Inc. v. Regis Brokerage Corp is instructive: insurer should be liable for it
1.   xxx Our procedural rules make plain how easily Malayan could 51.   Although the CA may have ruled that the damage to the cargo
have adduced the Marine Insurance Policy. Ideally, this should occurred while the same was in Eastern Shipping’s custody, this Court
have been accomplished from the momen tit filed the coplaint. cannot apply the ruling in ICTSI to the case at bar
Since the Marine Insurance Policy was constitute of the 1.   In contrast, unlike in ICTSI where there was no issue as
insurer-insured relationship from which Malayan draws its regrads the provisions of the marine insurance policy, such
right to subrogation, such document should have been attached that the presentation of the contract itself is necessary for
to the complaint itself, as provided for in Section 7, Rule 9 of the perusal, herein Eastern Shipping had repeatedly objected to
1997 Ruels of Civil Procedure x x x the non-presentation of the marine insurance policy and had
47.   Therefore, since Prudential alluded to the actionable document in its manifested its desire to know the specific provisions thereof
complaint, the contract of insurance between it and Nissan, as integral to its 2.   Moreover, and the same is critical, the marine risk note in the
cause of action against Eastern Shipping, the Marine Insurance Policy case at bar is questionable, because:
should have been attached to the Complaint 1.   It is dated on the same day the cargoes arrived at the
48.   It is significant that the date when the alleged insurance contract was port of Manila and not during the duration of the
constituted cannot be established with certainty without the contract voyage
itself. Said point is crucial because there can be no insurance on a risk 2.   Wihtout the marine insurance policy to elucidate on
that had already occurred by the timte the contract was executed. the specifics of the terms and conditions allueded to in
Surely, the Marine Risk Note on its face does not specify when the the marine risk note, it would be simply guesswork to
insurance was constituted know if the same were complied with
49.   The importance of the presentation of the Marine Insurance Policy was also 52.   The Court further recognizes the danger as precedent should we
emphasized in Wallemn Philippines Shipping v. Prudential Guarantee & sustain Malayan’s position, and not only because such a ruling would
Assurance: formally violate the rule on actionable documents
1.   The contract of insurance must be presented in evidence to indicate 53.   Malayan would have us effectuate an insurance contract without
the extent of its coverage. As there was no determination of rights having to consider its particular terms and conditions, and on a blind
under the insurance contract, this Court’s ruling in Home Insurance leap of faith that such contract is indeed valid and subsisting
Corporation v. CA is applicable: 54.   The conclusion further works to the utter prejudice of defendants such
1.   The insurance contract has not been presented. It may be as Regis or Paircargo since they would be deprived the opportunity to
assumed for the sake of argument that the subrogration examine the document that gives rise to the plaintiff’s right to recover
receipt may nevertheless be used to establish the against them, or to raise arguments or objections against the validity or
relationship between Home Insurance and Nestle Phil admissibility of such document
(consignee) and teha moutn paid to settle the claim. But 55.   If a legal claim is irrefragably sourced from an actionable document,
this is all the document can do. By itself alone, the the defendant cannot be deprived of the right to examine or utilize such
subrogation receipt is not sufficient to prove the document in order to intelligently raise a defense
petitioner’s (Home Insurance) claim holding the 56.   The inability or refusal of the plaintiff (insurer) to submit such
respondent (Mabuhay Brokerage) liable for the damage to document into evidence constitutes an effective denial of that right of
the engine the defendant which is ultimately rooted in due process of law, to say
50.   There are however, cases where the Court ruled that the non-presentation of nothing on how such failure fatally diminishes the plaintiff’s
the marine insurance policy is not fatal, as can be gleaned in the case of substantiation of its own cause of action
ICTSI: 57.   In conclusion, this Court rules that based on the applicable
1.   However, as in every general rule, there are admitted exceptions. jurisprudence, because of the inadequacy of the Marine Cargo Risk
In Delsan Transport Lines, Inc. v. CA, the Court stated that the Note for the reasons already stated, it was incumbent on respondent to
presentation of the insurance policy was not fatal because the loss present in evidence the Marine Insurance Policy, and having failed in
of the cargo undoubtedly occurred while on board the petitioner’s doing so, its claim of subrogation must necessarily fail
006 PARAMOUNT v. SPOUSES REMONDEULAZ (CHIQUI) improvements thereon, however, Sales failed to return the subject vehicle
November 28, 2012 | Peralta, J. | What May Be Insured within the agreed three-day period.
4.   As a result, Spouses notified Paramount Insurance to claim for the
PETITIONER: Paramount Insurance Corporation reimbursement of their lost vehicle. However, Paramount Insurance refused
RESPONDENTS: Spouses Yves and Maria Teresa Remondeulaz to pay.
5.   The Spouse lodged a complaint for a sum of money against Paramount
Insurance before the RTC praying for the payment of the insured value of
SUMMARY: Spouses Remondeulaz insured their car under Paramount Insurance
their car plus damages. After presentation of the Spouses’ evidence,
for one year for own damage, theft, third-party property damage, and third-party
Paramount Insurance filed a Demurrer to Evidence.
personal injury. The car was unlawfully taken by Sales who was supposed to add
6.   RTC dismissed the complaint filed by the Spouses.
accessories and improvements on it but failed to return the vehicle within the agreed
a.   Paramount Insurance had successfully prosecuted and had been
period. The Spouses sought for reimbursement from Paramount Insurance but the
awarded the amount claimed in this action, in another action which
latter refused to pay. Spouses then filed a case for a sum of money in the RTC. RTC
involved the loss of the same vehicle under the same circumstances
sided with Paramount Insurance stating that the Spouses already had reimbursement
although under a different policy and insurance company (Standard
for the same subject vehicle but from another insurance company. On the other
Insurance Company, Inc). This, considered with the principle that
hand, CA reversed the decision saying that that car is different from the one insured
an insured may not recover more than its interest in any property
under Paramount Insurance. W/N Paramount Insurance is liable under the insurance
subject of an insurance, leads the court to dismiss this action.
policy for the loss of the Spouses’ vehicle – YES, the Spouses’ policy clearly
7.   CA reversed and set aside the Order issued by the trial court the subject car
undertook to indemnify the insured against loss of or damage to the scheduled
is different from the one insured with another insurance company.
vehicle when caused by theft. (see doctrine) Sales did not have juridical possession
over the vehicle. Records would show that the Spouses entrusted possession of their
vehicle only to the extent that Sales will introduce repairs and improvements ISSUE/s:
thereon, and not to permanently deprive them of possession thereof. Since, theft can 1.   W/N Paramount Insurance is liable under the insurance policy for the loss
also be committed through misappropriation, the fact that Sales failed to return the of the Spouses’ vehicle – YES, the Spouses’ policy clearly undertook to
subject vehicle to the Spouses constitutes Qualified Theft. Hence, since the indemnify the insured against loss of or damage to the scheduled vehicle
Spouses’ car is undeniably covered by a Comprehensive Motor Vehicle Insurance when caused by theft.
Policy that allows for recovery in cases of theft, Paramount Insurance is liable under
the policy for the loss of the Spouses’ vehicle under the "theft clause."
RULING: WHEREFORE, the instant petition is DENIED. The Decision dated April
DOCTRINE: In interpreting the theft clause, SC explained that when one takes the 12, 2005 and Resolution dated July 20, 2006 of the Court of Appeals are hereby
motor vehicle of another without the latter's consent even if the motor vehicle is AFFIRMED in toto. SO ORDERED.
later returned, there is theft — there being intent to gain as the use of the thing
unlawfully taken constitutes gain. RATIO:
58.   Paramount Insurance argues that the loss of the Spouses vehicle is not a
FACTS: peril covered by the policy. It maintains that it is not liable for the loss,
since the car cannot be classified as stolen as the Spouses entrusted the
possession thereof to another person.
1.   Spouses Remondeulaz insured with Paramount Insurance their 1994 Toyota 59.   Adverse to Paramount Insurance’s claim, the Spouses’ policy clearly
Corolla Sedan under a Comprehensive Motor Vehicle Insurance Policy for
undertook to indemnify the insured against loss of or damage to the
one year for Own Damage, Theft, Third-Party Property Damage and Third-
scheduled vehicle when caused by theft.
Party Personal Injury, for the period commencing 26 May 1994 to 26 May
a.   SECTION III LOSS OR DAMAGE: The Company will, subject to
1995.
the Limits of Liability, indemnify the insured against loss of or
2.   During the effectivity of said insurance, Spouses’ car was unlawfully taken.
damage to the Scheduled Vehicle and its accessories and spare
Hence, they immediately reported the theft to the Traffic Management
parts whilst thereon: —
Command of the PNP who made them accomplish a complaint sheet.
b.   xxx (b) by fire, external explosion, self-ignition or lightning or
3.   In said complaint sheet, the Spouses alleged that a certain Ricardo Sales burglary, housebreaking or theft;
(Sales) took possession of the subject vehicle to add accessories and
60.   In People v. Bustinera, this Court had the occasion to interpret the "theft
clause" of an insurance policy. In this case, the Court explained that when
one takes the motor vehicle of another without the latter's consent even if
the motor vehicle is later returned, there is theft — there being intent to gain
as the use of the thing unlawfully taken constitutes gain.
61.   Also, in Malayan Insurance Co., Inc. v. Court of Appeals, this Court held
that the taking of a vehicle by another person without the permission or
authority from the owner thereof is sufficient to place it within the
ambit of the word theft as contemplated in the policy, and is therefore,
compensable.
62.   Moreover, the case of Santos v. People is worthy of note. Similarly in
Santos, the owner of a car entrusted his vehicle to therein petitioner Lauro
Santos who owns a repair shop for carburetor repair and repainting.
However, when the owner tried to retrieve her car, she was not able to do so
since Santos had abandoned his shop. In the said case, the crime that was
actually committed was Qualified Theft. However, the Court held that
because of the fact that it was not alleged in the information that the object
of the crime was a car, which is a qualifying circumstance, the Court found
that Santos was only guilty of the crime of Theft and merely considered the
qualifying circumstance as an aggravating circumstance in the imposition of
the appropriate penalty.
63.   In the instant case, Sales did not have juridical possession over the vehicle.
Here, it is apparent that the taking of the Spouses’ vehicle by Sales is
without any consent or authority from the former.
64.   Records would show that the Spouses entrusted possession of their
vehicle only to the extent that Sales will introduce repairs and
improvements thereon, and not to permanently deprive them of
possession thereof. Since, theft can also be committed through
misappropriation, the fact that Sales failed to return the subject vehicle
to the Spouses constitutes Qualified Theft.
65.   Hence, since the Spouses’ car is undeniably covered by a Comprehensive
Motor Vehicle Insurance Policy that allows for recovery in cases of theft,
Paramount Insurance is liable under the policy for the loss of the Spouses’
vehicle under the "theft clause."
66.   All told, Sales' act of depriving the Spouses of their motor vehicle at, or
soon after the transfer of physical possession of the movable property,
constitutes theft under the insurance policy, which is compensable.
007 United Merchants v. Country Bankers (Valle)
11 july 2012 | Carpio, J. | What may be insured 58.   UMC and Country executed Endorsement F/96-154 and Fire Invoice No.
16583A to form part of the Insurance Policy. Endorsement F/96-154
PETITIONER: United Merchants Corporation provides that UMCs stocks in trade were insured against additional
RESPONDENTS: Country Bankers Insurance Corporation perils, to wit: typhoon, flood, ext. cover, and full earthquake. The sum
insured was also increased to P50,000,000.00 effective 7 May 1996 to 10
SUMMARY: January 1997. On 9 May 1996, CBIC issued Endorsement F/96-157 where
UMC owns a warehouse where UMC stored its products (Christmas lights). the name of the assured was changed from Alfredo Tan to UMC.
UMC insured the Christmas lights with Country. This was later changed into 59.   A fire gutted the warehouse. Country designated CRM Adjustment
insure stocks in trade. A fire gutted the warehouse. UMC is claiming Php 50M Corporation (CRM) to investigate and evaluate the loss. Country’s insurer,
from Country but Country refused to pay, alleging that the fire was done through Central Surety, also requested the NBI to conduct an investigation.
arson and that UMC committed fraud. Country claimed that UMC was declaring 60.   UMC submitted to Country its Sworn Statement of Formal claim with
loss and claiming for loss that wasn’t covered under the policy. proofs of the loss. UMC demanded for at least 50% payment of its claim.
The issue in this case is whether or not UMC can validly claim for the Php 50M Country rejected the claim due to breach of the Insurance Policy which
it allegedly lost due to the fire. states:
a.   Condition No 15. If the claim be in any respect fraudulent, or if
The SC held that UMC cannot claim from Country. The Insurance Policy any false declaration be made or used in support thereof, or if any
provides that Country agreed to insure UMCs stocks in trade. UMC defined fraudulent means or devices are used by the Insured or anyone
stock in trade as tangible personal property kept for sale or traffic. UMC was acting in his behalf to obtain any benefit under this Policy; or if the
showing invoices that were suspicious, the Certificate from the Bureau of Fire loss or damage be occasioned by the willful act, or with the
Protection showed that the 50M it was claiming from Country included both the connivance of the Insured, all the benefits under this Policy shall
building and the contents of the building. But the Insurance Policy covered the be forfeited
stocks in trade. Basically, UMC was trying to claim 50M from Country by 61.   UMC filed a complaint in the RTC against Country. Country raised the
presenting imvoices and documents which were fraudulent. The Supreme Court following defenses:
refused the claim of UMC since submisisons of false invoices is a clear case of a.   No cause of action
fraud and fraud voids the policy. b.   Already prescribed
c.   UMC’s fire claim is tainted with fraud because the Statement of
DOCTRINE: Inventory showed that it had no stocks in trade and that UMC’s
The submission of false invoices to the adjusters establishes a clear case of fraud suspicious purchases for the year didn’t amount to 25M. The GIS
and misrepresentation which voids the insurer’s liability as per condition of the and Financial reports of UMC revealed that it had insufficient
policy. capital, which meant that UMC couldn’t have afforded the alleged
50M worth of stocks in trade.
What can be insured is only what is covered by the Policy. 62.   UMC presented five witnesses. The first witness was Josie Ebora (Ebora),
UMCs disbursing officer. Ebora testified that UMCs stocks in trade, at the
time of the fire, consisted of:
FACTS: a.   raw materials for its Christmas lights;
56.   United Merchants Corporation (UMC) is engaged in buying, selling, and b.   Christmas lights already assembled; and
manufacturing Christmas lights. It leased a warehouse where UMC c.   Christmas lights purchased from local suppliers.
assembled and stored its products.
57.   UMC’s general manager, Alfredo Tan insured UMC’s stocks in trade                                                                                                                                                                                                                                                                                        
Of Christmas lights against fire with Country Bankers Inusrance (Country) responsible  in  the  event  of  loss  and/or  damage  during  the  currency  of  this  
for P 15M. The policy was valid until Sept 6 1996.11
policy,   whilst   contained   in   the   building   of   one   lofty   storey   in   height,  
                                                                                                                        constructed   of   concrete   and/or   hollow   blocks   with   portion   of   galvanized  
11
  PROPERTY   INSURED:   On   stocks   in   trade   only,   consisting   of   Christmas   iron   sheets,   under   galvanized   iron   rood,   occupied   as   Christmas   lights  
Lights,   the   properties   of   the   Assured   or   held   by   them   in   trust,   on   storage.  
commissions,   or   on   joint   account   with   others   and/or   for   which   they   are    
d.   These stocks in trade were delivered from August 1995 to May shifts to the insurer to controvert the insureds prima facie case.
1996. She stated that Straight Cargo Commercial Forwarders 32.   Here, UMC established prima facie case against Country. Country doesn’t
delivered the imported materials to the warehouse, evidenced by dispute that UMC’s stocks were insured against fire and that the warehouse
delivery receipts. However, for the year 1996, UMC had no was gutted with fire. But since Country alleged an excepted risk, the burden
importations and only bought from its local suppliers. is shifted to Country to prove such exception.
e.   Ebora identified the suppliers as Fiber Technology Corporation 33.   Country failed to discharge its primordial burden of establishing that the
from which UMC bought stocks worth P1,800,000.00 on 20 May damage or loss was caused by arson, a limitation in the policy. Country’s
1996; Fuze Industries Manufacturer Philippines from which evidence did not prove that the fire was intentionally caused by UMC.
UMC bought stocks worth P19,500,000.00 from 20 January 1996 34.   First, the findings of Country’s witnesses, Cabrera and Lazaro, were based
to 23 February 1996; and Tomco Commercial Press from which on an investigation conducted more than four months after the fire. The
UMC bought several Christmas boxes. testimonies of Cabrera and Lazaro, as to the boxes doused with kerosene as
f.   Ebora testified that all these deliveries were not yet paid. Ebora told to them by barangay officials, are hearsay because the barangay
also presented UMCs Balance Sheet, Income Statement and officials were not presented in court.
Statement of Cash Flow. Per her testimony, UMCs purchases 35.   Cabrera and Lazaro even admitted that they did not conduct a forensic
amounted to P608,986.00 in 1994; P827,670.00 in 1995; and investigation of the warehouse nor did they file a case for arson.
P20,000,000.00 in 1996. Ebora also claimed that UMC had sales 36.   Second, the Sworn Statement of Formal Claim submitted by UMC, through
only from its fruits business but no sales from its Christmas lights CRM, states that the cause of the fire was faulty electrical wiring/accidental
for the year 1995. in nature. Country is bound by this evidence because in its Answer, it
63.   Annie Pabustan testified that the company provided 25 workers to assemble admitted that it designated CRM to evaluate UMCs loss.
and pack. 37.   Third, the Certification by the Bureau of Fire Protection states that the fire
64.   The third witness is Cesar Martinez, Officer of Metrobank, who said that was accidental in origin. This Certification enjoys the presumption of
UMC opened letters of credit with it for 1995 only regularity, which Country failed to rebut.
65.   The fourth witness presented was Ernesto Luna (Luna), the delivery 38.   In Uy Hu & Co. v. The Prudential Assurance Co., Ltd.,
checker of Straight Commercial Cargo Forwarders. Luna affirmed the a.   the Court held that where a fire insurance policy provides that if
delivery of UMCs goods to its warehouse the claim be in any respect fraudulent, or if any false declaration be
66.   . Lastly, CRMs adjuster Dominador Victorio testified that he inspected made or used in support thereof, or if any fraudulent means or
UMCs warehouse and prepared preliminary reports in this connection. devices are used by the Insured or anyone acting on his behalf
67.   Country rpesented Cabrera and Lazar who testified that they were hired to to obtain any benefit under this Policy, and the evidence is
investigate and found that arson was committed based from the interview conclusive that the proof of claim which the insured submitted
with barangay officials and pictures that showed blackened surfaces were was false and fraudulent both as to the kind, quality and
present at different parts of the warehouse. amount of the goods and their value destroyed by the fire, such
68.   RTC ruled in favor of UMC. The CA ruled in favor of Country. The CA a proof of claim is a bar against the insured from recovering on
said that the claim under the policy was void and that the fire was the policy even for the amount of his actual loss.
intentional in origin and that the claim was overvalued through fraudulent 39.   In this case, proof of its loss of stocks in trade amounting to
transactions. P50,000,000.00, UMC submitted its Sworn Statement of Formal Claim
together with the following documents:
ISSUE/s: a.   letters of credit and invoices for raw materials, Christmas lights
10.   WoN UMC can claim payment from Country– NO, because it committed and cartons purchased;
fraud which is a breach of the Policy. Commission of fraud voids the policy. b.   charges for assembling the Christmas lights; and
c.   delivery receipts of the raw materials.
RULING: Judgment in question is affirmed. 40.   However, the charges for assembling the Christmas lights and delivery
receipts could not support its insurance claim. The Insurance Policy
RATIO: provides that Country agreed to insure UMCs stocks in trade. UMC
31.   The party who asserts affirmative of the ossue has the burden of proof to defined stock in trade as tangible personal property kept for sale or
obtain a favorable judgment. Particularly, in insurance cases, once an traffic.Applying UMCs definition, only the letters of credit and invoices
insured makes out a prima facie case in its favor, the burden of evidence for raw materials, Christmas lights and cartons may be considered.
41.   The invoices, however, cannot be taken as genuine. The invoices reveal that
the stocks in trade purchased for 1996 amounts to P20,000,000.00 which
were purchased in one month. Thus, UMC needs to prove purchases
amounting to P30,000,000.00 worth of stocks in trade for 1995 and prior
years.
42.   However, in the Statement of Inventory it submitted to the BIR, UMC
stated that it had no stocks in trade as of 31 December 1995. In its
defense, UMC alleged that it did not include as stocks in trade the raw
materials to be assembled as Christmas lights, which it had on 31
December 1995. However, as proof of its loss, UMC submitted invoices
for raw materials, knowing that the insurance covers only stocks in
trade.
43.   Plus, the invoices from Fuze Industries were suspicious. The purchases
amounted to Php 19, 550 400 worth of Christmas lights. But the testimony
of Cabrera revealed that there was no Fuze Industries located at the business
address appearing in the invoices.
44.   In UMC’s Income Statement, it says that UMC has 1050862.71as assets
and 167, 058.47 as liabilities. It is claiming for more.
45.   Plus, in the certification from the Bureau of Fire Protection, it says that it
estimated a damage of Php 55M to the building and its contents.
008 VALENZUELA v. CA (VARGAS) commission of 32%. However, Valenzuela did not receive his commission
October 19, 1990 | Gutierrez, Jr., J. | Parties to the Contract amounting to P1.6M. During 1976-1978, premium payments were paid
directly to Philamgen.
PETITIONER: Arturo P. Valenzuela and Hospitalita N. Valenzuela 4.   In 1977, Philamgen started to become interested in and expressed its intent
RESPONDENTS: Hon. Court of Appeals, Bienvenido M. Aragon, Robert E. to share in the commission due Valenzuela on a fifty-fifty basis.
Parnell, Carlos K. Catolico, and the Philippine American General Insurance Valenzuela refused. Philamgen insisted on the sharing of commission with
Company, Inc. Valenzuela. Valenzuela firmly reiterated his objection to the proposals of
SUMMARY: Valenzuela is the General Agent for Philamgen (insurance Philamgen.
company). Valenzuela solicited marine insurance from Delta Motors from which 5.   Because of the refusal of Valenzuela, Philamgen took drastic action against
he was entitled to a commission of 32.5% from Philamgen. In 1977, Philamgen Valenzuela:
started to become interested in and expressed its intent to share in the a.   Reversed the commission due him by not crediting in his account the
commission due Valenzuela on a 50-50 basis thus giving Valenzuela an agent’s commission earned from the Delta Motors, Inc.
commission of 16.25%. Despite several attempts to convince Valenzuela, he still b.   Placed agency transactions on a cash and carry basis
refused to accept such proposals to share commission with Philamgen. c.   Threatened the cancellation of policies issued by his agency
Consequently, Philamgen did some acts of harassment against Valenzuela which d.   Started to leak out news that Valenzuela has a substantial account with
led to the decline of Valenzuela’s business as insurance agent. Eventually,
Philamgen.
Philamgen terminated the General Agency Agreement with Valenzuela. The
issues in this case are All of these acts resulted in the decline of his business as insurance agent.
1.   WON Philamgen and/or its officers can be held liable for damages Then on December 27, 1978, Philamgen terminated the General Agency
due to the termination of the General Agency Agreement it entered Agreement of Valenzuela.
into with Valenzuela – YES. The agency involving Valenzuela and 6.   Vaenzuela filed a complaint against Philamgen.
Philamgen is one "coupled with an interest," and, therefore, should 7.   Trial Court ruled in favor of Valenzuela stating that the principal cause of
not be freely revocable at the unilateral will of the latter. The principal the termination of Valenzuela as General Agent of Philamgen was based on
cause of the termination of Valenzuela as General Agent of Philamgen
his refusal to share his Delta Commission. The acts of harassment done by
arose from his refusal to share his Delta Commission. Such termination,
as held by the Trial Court, is tainted with bad faith. As such, Philamgen to force Valenzuela to agree to the sharing of his Delata
2.   WON Valenzuela is liable to Philamgen for the unpaid and commission do not justify the termination of the General Agency
uncollected premiums – NO. The remedy for the non-payment of Agreement entered into by Philamgen and Valenzuela.
premiums is to put an end to and render the insurance policy not 8.   CA ruled in favor of Philamgen stating that the principal’s power to revoke
binding. an agency at will is so pervasive that termination may be effected even if
DOCTRINE: Valenzuela, being a mere agent of Philamgen, is not a party to the
the principal acts in bad faith, subject only to the principal’s liability for
contract of insurance. The contract is essentially between Philamgen and Delta
Motors. Consequently, Valenzuela is under no obligation to pay Philamgen for damages.
the unpaid and uncollected premiums. The remedy for the non-payment of
premiums is to put an end to and render the insurance policy not binding. ISSUE:
FACTS: 1.   WON Philamgen and/or its officers can be held liable for damages due
1.   Characters: to the termination of the General Agency Agreement it entered into
Valenzuela – General Agent of Philamgen with Valenzuela – YES. The agency involving Valenzuela and Philamgen
Philamgen – Insurance Company is one "coupled with an interest," and, therefore, should not be freely
Delta Motors – Insured revocable at the unilateral will of the latter. The principal cause of the
2.   Valenzuela is a General Agent of Philamgen since 1965. As such, he was termination of Valenzuela as General Agent of Philamgen arose from his
authorized to solicit and sell in behalf of Philamgen all kinds of non-life refusal to share his Delta Commission. Such termination, as held by the
insurance, and in sconsideration of services rendered was entitled to receive Trial Court, is tainted with bad faith.
the full agent’s commission of 32.5% from Philamgen. 2.   WON Valenzuela is liable to Philamgen for the unpaid and uncollected
3.   From 1973-1975, Valenzuela solicited marine insurance from one of his premiums – NO. The remedy for the non-payment of premiums is to put an
clients Delta Motors amounting to P4.4M from which he was entitled to a end to and render the insurance policy not binding.
embarrassment, difficulties, and oftentimes frustrations in the solicitation
RULING: ACCORDINGLY, the petition is GRANTED. The impugned decision of and procurement of the insurance policies. To sell policies, an agent exerts
January 29, 1988 and resolution of April 27, 1988 of respondent court are hereby great effort, patience, perseverance, ingenuity, tact, imagination, time and
SET ASIDE. The decision of the trial court dated January 23, 1986 in Civil Case No.
money.
121126 is REINSTATED with the MODIFICATIONS that the amount of FIVE
HUNDRED TWENTY ONE THOUSAND NINE HUNDRED SIXTY-FOUR AND 51.   In the case of Valenzuela, he was able to build up an Agency from scratch
16/100 PESOS (P521,964.16) representing the petitioners Delta commission shall in 1965 to a highly productive enterprise. The records sustain the finding
earn only legal interests without any adjustments under Article 1250 of the Civil that the Philamgen started to covet a share of the insurance business that
Code and that the contractual relationship between Arturo P. Valenzuela and Valenzuela had built up, developed and nurtured to profitability through
Philippine American General Insurance Company shall be deemed terminated upon over thirteen (13) years of patient work and perseverance. When Valenzuela
the satisfaction of the judgment as modified. refused to share his commission in the Delta account, the boom suddenly
fell on him.
RATIO:
46.   The principal cause of the termination of Valenzuela as General Agent of 52.   With the termination of the General Agency Agreement, Valenzuela would
Philamgen arose from his refusal to share his Delta commission. The no longer be entitled to commission on the renewal of insurance policies of
records sustain the conclusions of the trial court on the apparent bad faith of clients sourced from his agency. Worse, despite the termination of the
the Philamgen in terminating the General Agency Agreement of agency, Philamgen continued to hold Valenzuela jointly and severally
Valenzuela. In the case at bar, the records show that the findings and liable with the insured for unpaid premiums. Under these circumstances,
conclusions of the trial court are supported by substantial evidence and it is clear that Valenzuela had an interest in the continuation of the
there appears to be no cogent reason to disturb them. agency when it was unceremoniously terminated not only because of the
47.   As early as September 30,1977, Philamgen told the petitioners of its desire commissions he should continue to receive from the insurance business he
to share the Delta Commission with them. It stated that should Delta back has solicited and procured but also for the fact that by the very acts of the
out from the agreement, Valenzuela would be charged interests through a respondents, he was made liable to Philamgen in the event the insured fail
reduced commission after full payment by Delta. to pay the premiums due. They are estopped by their own positive
48.   On January 23, 1978 Philamgen proposed reducing the Valenzuela’s averments and claims for damages. Therefore, the respondents cannot state
commissions by 50% thus giving them an agent's commission of 16.25%. that the agency relationship between Valenzuela and Philamgen is not
On February 8, 1978, Philamgen insisted on the reduction scheme. There coupled with interest.
were other pressures:
a.   The petitioners were told that the Delta commissions would not be As to Valenzuela’s liability to pay Philamgen for the unpaid and uncollected
credited to their account premiums
b.   They were informed that the Valenzuela agency would be placed on a 53.   We rule that the respondent court erred in holding Valenzuela liable. We
cash and carry basis thus removing the 60-day credit for premiums due find no factual and legal basis for the award. Under Section 7712 of the
c.   Existing policies were threatened to be cancelled Insurance Code, the remedy for the non-payment of premiums is to put an
d.   The Valenzuela business was threatened with diversion to other agencies end to and render the insurance policy not binding.
e.   Rumors were also spread about alleged accounts of the Valenzuela 54.   Philippine Phoenix Surety and Insurance, Inc. v. Woodworks, Inc.: Non-
agency payment of premium does not merely suspend but puts an end to an
49.   It is also evident from the records that the agency involving Valenzuela and
                                                                                                                       
Philamgen is one "coupled with an interest," and, therefore, should not be 12
Sec. 77 ... [N]otwithstanding any agreement to the contrary, no policy or contract of insurance is valid
freely revocable at the unilateral will of the latter. and binding unless and until the premiums thereof have been paid except in the case of a life or industrial
50.   In the insurance business in the Philippines, the most difficult and life policy whenever the grace period provision applies (P.D. 612, as amended otherwise known as the
Insurance Code of 1974)
frustrating period is the solicitation and persuasion of the prospective clients
to buy insurance policies. Normally, agents would encounter much
 
insurance contract since the time of the payment is peculiarly of the essence
of the contract.
55.   Since admittedly the premiums have not been paid, the policies issued have
lapsed. The insurance coverage did not go into effect or did not continue
and the obligation of Philamgen as insurer ceased. Hence, for Philamgen
which had no more liability under the lapsed and inexistent policies to
demand, much less sue Valenzuela for the unpaid premiums would be the
height of injustice and unfair dealing. In this instance, with the lapsing of
the policies through the nonpayment of premiums by the insured there
were no more insurance contracts to speak of.
009 RCBC v. CA (VICENCIO) FACTS:
April 20, 1998 | Melo, J. | Parties to a Contract 43.   Parties: Insured: GOYU; Insurer: MICO thru its agent Alchester;
Mortgagor: GOYU; Mortgagee: RCBC
PETITIONER: Rizal Commercial Banking Corporation (RCBC), Uy Chun Bing, Eli D. Law 44.   Goyu & Sons (GOYU) applied for credit facilities and accomodations with
RESPONDENTS: Court of Appeals and Goyu & Sons Rizal Commercial Banking Corporation (RCBC) at its Binondo Branch.
45.   After due evaluation, RCBC, through its key officers, petitioner Uy Chin
SUMMARY: Goyu and Sons (GOYU) applied for credit facilities with RCBC. A credit Bing and Eli D. Lao, recommended GOYU’s application for approval with
amount of P30M was granted, which was incrased to P117M. As security for its credit, GOYU RCBC’s executive committee.
executed two real estate mortagages and two chattel mortgages, in favor of RCBC. Under each 46.   A credit facilitiy in the amount of P30 million was initially granted. Upon
of these contracts, GOYU committed itself to insure the mortgages property with an insurance GOYU’s application and Uy’s and Lao’s recommendation, RCBCs
company approved by RCBC. GOYU obtained a total of 10 insurance policies from Malayan
Insurance Compan (MICO). Alchester Insurance Company (Alchester), MICO’s insurance
executive committee increased GOYUs credit facility to P50 million, then
agent, issued nine endorsements in favor of RCBC. On April 27, 1992, one of GOYU’s to P90 million, and finally to P117 million.
factory buildings in Valenzuela was gutted by fire. GOYU submitted its claim for indemnity 47.   As security for its credit facilities with RCBC, GOYU executed two real
against MICO. MICO however denied the claim on the ground that the insurance policies estate mortgages and two chattel mortgages in favor of RCBC.
were already subject to attachments by other courts because the insurance proceeds were also 48.   Under each of these four mortgage contracts, GOYU committed itself to
claimed by other creditors of GOYU. insure the mortgaged property with an insurance company approved by
RCBC, and subsequently, to endorse and deliver the insurance policies to
RCBC, one of GOYU’s creditors, also filed with MICO its formal claim over the proceeds of RCBC.
the insurance policies. RCBC here is claiming that since they are the mortgagee of security 49.   GOYU obtained in its name a total of ten insurance policies from Malayan
provided by GOYU, the insured, they also have the right to claim the insurance proceeds from
MICO, the insurer. Its claim was also denied by MICO, however. The RTC ruled in favor of
Insurance Company (MICO). Alchester Insurance Agency, Inc. (Alchester),
GOYU, ordering MICO to pay GOYU its fire loss claims, but that GOYU should also pay the insurance agent where GOYU obtained the MICO policies, issued nine
RCBC for its loan obligations with the latter. CA partly affirmed. endorsements in favor of RCBC seemingly upon instructions of GOYU.
50.   On April 27, 1992, one of GOYU’s factory buildings in Valenzuela was
The issue before the SC is WoN RCBC, as mortgagee, has any right over the insurance gutted by fire.
policies by GOYU, as mortgagor, in case of occurrence of loss. The SC held that YES, but 51.   Consequently, GOYU submitted its claim for indemnity on account of the
only because of the peculiarity of the circumstances obtaining in the instant case which loss insured against MICO.
presented a justification to take exception to the strict application of Sec. 53 of the Insurance 52.   MICO denied the claim on the ground that the insurance policies were
Code. GOYU voluntarily procured insurance policies to cover the mortgaged property from
either attached pursuant to writs of attachments/garnishments issued by
MICO, no less than a sister company of RCBC. Endorsement documents were prepared by
MICO’s underwriter, Alchester Insurance Agency, Inc., and copies thereof were sent to
various courts or that the insurance proceeds were also claimed by other
GOYU, MICO, and RCBC. GOYU did not assail, until of late, the validity of said creditors of GOYU alleging better rights to the proceeds than the insured.
endorsements. GOYU continued until the occurrence of the fire, to enjoy the benefits of the 53.   GOYU filed a complaint for specific performance and damages with the
credit facilities extended by RCBC. GOYU is bound by the principle of estoppel. The RTC.
intention of the parties was also plain: RCBC is truly the person or entity for whose benefit the 54.   RCBC, one of GOYU’s creditors, also filed with MICO its formal claim
policies were clearly intended. over the proceeds of the insurance policies, but said claims were also denied
for the same reasons that MICO denied GOYU’s claims.
Being exclusively payable to RCBC by reason of the endorsement by Alchester to RCBC, 1.   Personal note: RCBC here is claiming that since they are the
these 8 policies (of 10) can not be attached by GOYU’s other creditors up to the extent of the mortgagee of security provided by GOYU, the insured, they also
GOYUs outstanding obligation in RCBC’s favor. The other 2 policies, since they have not
been proven to be endorsed and delivered by Alchester, can be subject of attachments by other
have the right to claim the insurance proceeds from MICO, the
creditors. insurer
55.   The RTC confirmed that GOYU’s other creditors, namely, Urban Bank,
DOCTRINE: Section 53 of the Insurance Code ordains that the insurance proceeds of the Alfredo Sebastian, and Philippine Trust Company obtained their respective
endorsed policies shall be applied exclusively to the proper interest of the person for whose writs of attachments form various cours, covering an aggregate ammoung of
benefit it was made. P14,938,080, and ordered that the proceeds of the ten insurance policies be
deposited with the said court minus the aforementioned 14M.
56.   Accordingly, MICO deposited the amoun gog P50,505,594 with the RTC.
57.   The RTC rendered judgment in favor of GOYU:
1.   For defendant MICO to pay GOYU its fire loss claims in the total of P74,040,518, 69.   In the present case, although it appears that GOYU obtained the subject
less the amount of P50M which is deposited with this Court.
insurance policies naming itself as the sole payee, the intentions of the
2.   To pay MICO damages by way of interest for the delay
3.   For RCBC to pay GOYU actual damages of 2PM parties as shown by their contemporaneous acts, must be given due
4.   For the both MICO and RCBC to pay GOYU exemplary and attorney’s fees. consideration in order to better serve the interest of justice and equity.
5.   And on the counterclaim of RCBC: ordering GOYU to pay its loan obligations with 70.   It is to be noted that nine endorsement documents were prepared by
RCBC in the amoung of P68,785,069, with interest.
Alchester in favor of RCBC. The Court is in a quandary how Alchester
58.   All parties appealed. GOYU was unsatisfied with the amounts awarded in could arrive at the idea of endorsing any specific insurance policy in
its favor. MICO and RCBC disputed the trial courts findings of liability on favor of any particular beneficiary or payee (RCBC) other than the
their part. insured (GOYU) had not such named payee or beneficiary been
59.   The CA partly granted GOYUs appeal (it additionally ordered MICO to pay specifically disclosed by the insured itself.
GOYU damages by way of interest until GOYU receives the said amount), 71.   It is also significant that GOYU voluntarily and purposely took the
but sustained the findings of the trial court with respect to MICO and insurance policies from MICO, a sister company of RCBC, and not just
RCBCs liabilities. from any other insurance company.
60.   Hence this appeal by RCBC and MICO. 72.   Had it not been for GOYU, Alchester would not have known of GOYU’s
intention of obtaining insurance coverage in compliance with its
ISSUE/s: undertaking in the mortgage contracts with RCBC, and verily, Alchester
7.   WoN RCBC, as mortgagee, has any right over the insurance policies taken would not have endorsed the policies to RCBC had it not been so directed
by GOYU, the mortgagor, in case of the occurrence of loss – YES, but only by GOYU.
because of the peculiarity of the circumstances obtaining in the instant case 73.   On equitable principles, particularly on the ground of estoppel, the Court is
which presented a justification to take exception to the strict application of constrained to rule in favor of mortgagor RCBC.
Sec. 53 of the Insurance Code, it having been sufficiently established that it 74.   Evelyn Lozada of Alchester testified that upon instructions of Mr. Go,
was the intention of the parties to designate RCBC as the party for whose through a certain Mr. Yam, she prepared in quadruplicate on February 11,
benefit the insurance policies were taken out. 1992 the nine endorsement documents for GOYUs nine insurance policies
in favor of RCBC.
RULING: Petitions are hereby GRANTED and the decision and resolution of 75.   RCBC, in good faith, relied upon the endorsement documents sent to it
December 16, 1996 and April 3, 1997 in CA-G.R. CV No. 46162 are hereby as this was only pursuant to the stipulation in the mortgage contracts.
REVERSED and SET ASIDE. A new one entered: The Court finds such reliance to be justified under the circumstances of
1.   Dismissing the Complaint of private respondent GOYU in Civil Case No. 93-65442 before
Branch 3 of the Manila Regional Trial Court for lack of merit; the case.
2.   Ordering Malayan Insurance Company, Inc. to deliver to Rizal Commercial Banking 76.   GOYU failed to seasonably repudiate the authority of the person or
Corporation the proceeds of the insurance policies in the amount of P51,862,390.94 (per report persons who prepared such endorsements.
of adjuster Toplis & Harding (Far East), Inc., Exhibits 2 and 2-1), less the amount of 77.   Over and above this, GOYU continued, in the meantime, to enjoy the
P50,505,594.60 (per O.R. No. 3649285);
3.   Ordering the Clerk of Court to release the amount of P50,505,594.60 including the interests benefits of the credit facilities extended to it by RCBC.
earned to Rizal Commercial Banking Corporation; 78.   After the occurrence of the loss insured against, it was too late for
4.   Ordering Goyu & Sons, Inc. to pay its loan obligation with Rizal Commercial Banking GOYU to disown the endorsements for any imagined or contrived lack
Corporation in the principal amount of P107,246,887.90, with interest at the respective rates
of authority of Alchester to prepare and issue said endorsements.
stipulated in each promissory note from January 21, 1993 until finality of this judgment, and
surcharges at 2% and penalties at 3% from January 21, 1993 to March 9, 1993, minus 79.   If there had not been actually an implied ratification of said endorsements
payments made by Malayan Insurance Company, Inc. and the proceeds of the amount by virtue of GOYUs inaction in this case, GOYU is at the very least
deposited with the trial court and its earned interest. The total amount due RCBC at the time of estopped from assailing their operative effects.
the finality of this judgment shall earn interest at the legal rate of 12% in lieu of all other
80.   To permit GOYU to capitalize on its non-confirmation of these
stipulated interests and charges until fully paid
endorsements while it continued to enjoy the benefits of the credit
RATIO: facilities of RCBC which believed in good faith that there was due
67.   It is settled that a mortgagor and a mortgagee have separate and distinct endorsement pursuant to their mortgage contracts, is to countenance
insurable interests in the same mortgaged property, such that each one of grave contravention of public policy, fair dealing, good faith, and
them may insure the same property for his own sole benefit. justice.
68.   There is no question that GOYU could insure the mortgaged property for its 81.   Under the peculiar circumstances obtaining in this case, the Court is bound
own exclusive benefit. to recognize RCBC’s right to the proceeds of the insurance policies if not
for the actual endorsement of the policies, at least on the basis of the use, with the declarations, amplifications and limitations established by law,
whether the estate remains in the possession of the mortgagor, or it passes into the
equitable principle of estoppel.
hands of a third person.
82.   GOYU cannot seek relief under Section 53 of the Insurance Code which 88.   Significantly, the Court notes that out of the 10 insurance policies subject of
provides that the proceeds of insurance shall exclusively apply to the this case, only 8 of them appear to have been subject of the endorsements
interest of the person in whose name or for whose benefit it is made. The prepared and delivered by Alchester for and upon instructions of GOYU.
peculiarity of the circumstances obtaining in the instant case presents a 1.   Policy Number F-114-07795 has not been endorsed. This fact was
justification to take exception to the strict application of said provision, it admitted by MICOs witness, Atty. Farolan.
having been sufficiently established that it was the intention of the parties to 2.   Likewise, the record shows no endorsement for Policy Number
designate RCBC as the party for whose benefit the insurance policies were CI/F-128-03341.
taken out. 89.   Being exclusively payable to RCBC by reason of the endorsement by
83.   The Court considered the following: Alchester to RCBC, which we already ruled to have the force and effect
1.   It is undisputed that the insured pieces of property were the subject of
of an endorsement by GOYU itself, these 8 policies can not be attached
mortgage contracts entered into between RCBC and GOYU in
consideration of and for securing GOYUs credit facilities from by GOYU’s other creditors up to the extent of the GOYUs outstanding
RCBC. The mortgage contracts contained common provisions whereby obligation in RCBC’s favor.
GOYU, as mortgagor, undertook to have the mortgaged property properly 90.   Section 53 of the Insurance Code ordains that the insurance proceeds of
covered against any loss by an insurance company acceptable to RCBC. the endorsed policies shall be applied exclusively to the proper interest
2.   GOYU voluntarily procured insurance policies to cover the mortgaged of the person for whose benefit it was made.
property from MICO, no less than a sister company of RCBC and 91.   In this case, to the extent of GOYUs obligation with RCBC, the interest of
definitely an acceptable insurance company to RCBC. GOYU in the subject policies had been transferred to RCBC effective
3.   Endorsement documents were prepared by MICOs underwriter, Alchester as of the time of the endorsement.
Insurance Agency, Inc., and copies thereof were sent to GOYU, MICO,
92.   These policies may no longer be attached by the other creditors of GOYU,
and RCBC. GOYU did not assail, until of late, the validity of said
endorsements. like Alfredo Sebastian, which may nonetheless forthwith be dismissed for
4.   GOYU continued until the occurrence of the fire, to enjoy the benefits of being moot and academic in view of the results reached herein.
the credit facilities extended by RCBC which was conditioned upon the 93.   Only the two other policies may be validly attached, garnished, and levied
endorsement of the insurance policies to be taken by GOYU to cover the upon by GOYUs other creditors.
mortgaged properties. 94.   To the extent of GOYUs outstanding obligation with RCBC, all the rest of
84.   This Court can not over stress the fact that upon receiving its copies of the the other insurance policies above-listed which were endorsed to RCBC,
endorsement documents prepared by Alchester, GOYU, despite the are, therefore, to be released from attachment, garnishment, and levy by the
absence of its written conformity thereto, obviously considered said other creditors of GOYU.
endorsement to be sufficient compliance with its obligation under the 95.   Given the factual milieu spread hereover, the Court also rulse that it was
mortgage contracts since RCBC accordingly continued to extend the error to hold MICO liable in damages for denying or withholding the
benefits of its credit facilities and GOYU continued to benefit proceeds of the insurance claim to GOYU:
therefrom. 1.   Firstly, by virtue of the mortgage contracts as well as the endorsements of
85.   Just as plain too is the intention of the parties to constitute RCBC as the the insurance policies, RCBC has the right to claim the insurance
beneficiary of the various insurance policies obtained by GOYU. proceeds, in substitution of the property lost in the fire. Having assigned
86.   The intention of the parties will have to be given full force and effect in this its rights, GOYU lost its standing as the beneficiary of the said insurance
particular case. The insurance proceeds may, therefore, be exclusively policies.
applied to RCBC, which under the factual circumstances of the case, is 2.   Secondly, for an insurance company to be held liable for unreasonably
delaying and withholding payment of insurance proceeds, the delay must
truly the person or entity for whose benefit the policies were clearly be wanton, oppressive, or malevolent. The case at bar does not show that
intended. MICO wantonly and in bad faith delayed the release of the proceeds. The
87.   Moreover, the laws evident intention to protect the interests of the problem in the determination of who is the actual beneficiary of the
mortgagee upon the mortgaged property is expressed in Article 2127 of the insurance policies, aggravated by the claim of various creditors who
Civil Code which states: wanted to partake of the insurance proceeds, not to mention the
1.   ART. 2127. The mortgage extends to the natural accessions, to the improvements, importance of the endorsement to RCBC, to our mind, and as now borne
growing fruits, and the rents or income not yet received when the obligation out by the outcome herein, justified MICO in withholding payment to
becomes due, and to the amount of the indemnity granted or owing to the proprietor GOYU.
from the insurers of the property mortgaged, or in virtue of expropriation for public
010 PALILEO v. COSIO (VILLAVICENCIO) 4.   This document did not express the true intention of the parties which was merely
November 28, 1955 | Bautista Angelo, J. | Parties to the contract to place said property as security for the payment of the loan.
5.   Cosio insured the building against fire with the Associated Insurance & Surety
PETITIONER: Cherie Palileo Co., Inc. for the sum of P15,000, the insurance policy having been issued in the
RESPONDENTS: Beatriz Cosio name of Cosio.
6.   The building was partly destroyed by fire and, after proper demand, Cosio
SUMMARY: Palileo obtained a loan from Cosio. Cosio required Palileo to sign a collected from the insurance company an indemnity of P13,107.00.
“Conditional sale of residential building” to secure the payment of the loan. This 7.   Palileo demanded from Cosio that she be credited with the necessary amount to
document did not express the true intention of the parties that was merely to place pay her obligation out of the insurance proceeds but Cosio refused to do so.
said property as security for the payment of the loan. Cosio insured the building with 8.   Trial court rendered a decision ordering Cosio to credit the 13,107 to Palileo’s
Associated Insurance & Surety Co., Inc. The building was party destroyed by fire. payment to her.
Cosio then collected 13,107 from Associated Insurance. Palileo demanded from
Cosio that she be credited with the necessary amount to pay her obligation out of the ISSUE/s:
insurance proceeds but Cosio refused to do so. Trial court rendered a decision 1.   WoN the obligation of Palileo can be fully compensated by the insurance
ordering Cosio to credit the 13,107 to Palileo’s payment to her. WoN the obligation amount – NO, because it runs counter to the rule governing an insurance taken
of Palileo can be fully compensated by the insurance amount – NO, because it runs by a mortgagee independently of the mortgagor.
counter to the rule governing an insurance taken by a mortgagee independently of the
mortgagor. In that event, upon the destruction of the property the insurance money RULING: Consistent with the foregoing pronouncement, we therefore modify the
paid to the mortgagee will not inure to the benefit of the mortgagor, and the amount judgment of the lower court as follows:(1) the transaction had between the plaintiff
due under the mortgage debt remains unchanged. The mortgagee, however, is not and defendant as shown in Exhibit A is merely an equitable mortgage intended to
allowed to retain his claim against the mortgagor, but it passes by subrogation to the secure the payment of the loan of P12,000;(2) that the proceeds of the insurance
insurer, to the extent of the insurance money paid. amounting to P13,107.00 was properly collected by defendant who is not required to
account for it to the plaintiff; (3) that the collection of said insurance proceeds shall
DOCTRINE: The rule is that "where a mortgagee, independently of the mortgagor, not be deemed to have compensated the obligation of the plaintiff to the defendant,
insures the mortgaged property in his own name and for his own interest, he is but bars the latter from claiming its payment from the former; and (4) defendant shall
entitled to the insurance proceeds in case of loss, but in such case, he is not allowed pay to the plaintiff the sum of P810.00 representing the overpayment made by
to retain his claim against the mortgagor, but is passed by subrogation to the insurer plaintiff by way of interest on the loan. No pronouncement as to costs.
to the extent of the money paid." Or, stated in another way, "the mortgagee may
insure his interest in the property independently of the mortgagor. RATIO:
96.   The rule is that "where a mortgagee, independently of the mortgagor, insures the
mortgaged property in his own name and for his own interest, he is entitled to
the insurance proceeds in case of loss, but in such case, he is not allowed to
FACTS: retain his claim against the mortgagor, but is passed by subrogation to the
1.   Palileo obtained from Cosio a loan in the sum of P12,000 subject to the
insurer to the extent of the money paid."
following conditions: (a) that Palileo shall pay to Cosio an interest in the amount
97.   Or, stated in another way, "the mortgagee may insure his interest in the property
of P250 a month; (b) that Cosio shall deduct from the loan certain obligations of
independently of the mortgagor.
Palileo to third persons amounting to P4,550, plus the sum of P250 as interest
98.   In that event, upon the destruction of the property the insurance money paid to
for the first month; and (c) that after making the above deductions, Cosio shall
the mortgagee will not inure to the benefit of the mortgagor, and the amount due
deliver to Palileo only the balance of the loan of P12,000.
under the mortgage debt remains unchanged.
2.   Pursuant to their agreement, Palileo paid to Cosio as interest on the loan a total
99.   The mortgagee, however, is not allowed to retain his claim against the
of P2,250.00 corresponding to nine months from December 18, 1951, on the
mortgagor, but it passes by subrogation to the insurer, to the extent of the
basis of P250.00 a month, which is more than the maximum interest authorized
insurance money paid."
by law.
100.  This is the same rule upheld by this Court in a case that arose in this jurisdiction.
3.   To secure the payment of the aforesaid loan, Cosio required Palileo to sign a In the case mentioned, an insurance contract was taken out by the mortgagee
document known as "Conditional Sale of Residential Building", purporting to
upon his own interest, it being stipulated that the proceeds would be paid to him
convey to Cosio, with right to repurchase, a two-story building of strong
only and when the case came up for decision, this Court held that the mortgagee,
materials belonging to Palileo.
in case of loss, may only recover upon the policy to the extent of his credit at the
time of the loss. It was declared that the mortgaged had no right of action against
the mortgagee on the policy. (San Miguel Brewery vs. Law Union, 40 Phil.,
674.)
101.  The general rule and the weight of authority is, that the insurer is thereupon
subrogated to the rights of the mortgagee under the mortgage.
102.  the correct solution should be that the proceeds of the insurance should be
delivered to Cosio but that her claim against Palileo should be considered
assigned to the insurance company who is deemed subrogated to the rights of
Cosio to the extent of the money paid as indemnity.
011 GREAT PACIFIC LIFE ASSURANCE v. CA (Yap) (Grepalife).
October 13, 1999 | Quisumbing, J. | Parties to the contract

PETITIONER: Great Pacific Life Assurance Corp. (Grepalife) FACTS:


RESPONDENT: Court of Appeals and Medarda V. Leuterio 69.   Great Pacific Life Assurance (Petitioner / Grepalife) and the Development
Bank of the Philippines (DBP) entered into a Group Life Insurance to
SUMMARY: Grepalife and DBP entered into a group life insurance to insure insure the lives of eligible housing loan mortgagors of DBP (a.k.a.,
the lives of eligivle housing loan mortgagors of DBP (a.k.a., mortgage Mortgage Redemption Insurance).
redemption insurance). When Dr. Wilfredo Leuterio applied, he indicated that he 70.   Dr. Wilfredo Leuterio, a physicial and a housing debtor of DBP, applied for
had never consulted a physician for a heart condition, high blood pressure, etc. membership in the group life insurance.
and that to the best of his knowledge, he is in good health. Thereafter, he died a.   In his application, he answered that (1) “He had NEVER consulted
due to massive cerebral hemorrhage. When DBP submitted its claim, from the a physician for a heart condition, high blood pressure, cancer,
insurance, to Grepalife, the latter denied on the ground that it is entitled to diabetes, lung, kidney or stomach disorder or any other physical
rescind the contract since Wilfredo disclosed his hypertension, as attested by Dr. impairment;” and (2) “He was, to the best of his knowledge, in
Hernando Mejia that “the cerebral hemorrhage was probably secondary to good health.”
hypertension.” Because of the denial, DBP proceeded to claim from Wilfredo b.   Grepalife granted Wilfredo an insurance coverage of PHP86,200
and even foreclosed on the mortgaged property. Medarda Leuterio, the widow, over his DBP mortgage indebtedness.
filed a complaint for specific performance and damages against Grepalife to 71.   Wilfredo died due to massive cerebral hemorrhage.
fulfill its obligation under the insurance contract. Grepalife now argues that 72.   DBP submitted a death claim to Grepalife.
Medarda is not the real party-in-interest, which should be DBP the mortgagee. a.   Grepalife denied the claim on the ground that Wilfredo was not
physically healthy when he applied for an insurance coverage, and
The pertinent issue is whether Medarda is the real party-in-interest. that he concealed the fact that he ws suffering from hypertension,
which caused his death.
The SC ruled in the affirmative. In Gonzales v. Yek Tong Lin, the SC ruled that b.   According to Grepalife, this was attested to the attending
the insured may be regarded as the real party-in-interest, ALTHOUGH he has physician, Dr. Hernando Mejia, when he said that Wilfredo “died
assigned the policy for the purpose of collection, or has assigned as collateral of cerebral hemorrhage, PROBABLY secondary to hypertension.”
security any judgment he may obtain. In this case, since a policy of insurance c.   Thus, DBP collected the debt from the mortgagor (Wilfredo) and
upon life or health MAY PASS by transfer, will or succession to any person, took the necessary action of foreclosure on the residential lot of the
whether he has an insurable interest or not, and such person may recover it Leuterios.
whatever the insured might have recovered, the widow (Medarda) of the 73.   Medarda Leuterio, Wilfredo’s widow, filed this complaint against Grepalife
decedent may file the suit against the insurer (Grepalife). (To simplify, Wilfredo for specific performance with damages, alleging that Grepalife’s claim
remained as the real party-in-interest even though he assigned the DBP based on Dr. Mejia’s testimony was not a conclusive inference because
mortgage indebtedness to Grepalife. By virtue of Wilfredo’s death, his heirs Wilfredo was not autopsied, hence other causes could not be ruled out.
succeeded to his rights in the insurance contract). Hence, Medarda has the a.   Trial court ruled in favor of Medarda. CA sustained.
proper standing to claim from Grepalife, which erred in rescinding the insurance 74.   Grepalife now alleges that this complaint was not filed by the real party-in-
contract on the ground of Wilfredo’s alleged concealment – there was no interest which is DBP.
concealment since the testimony of Dr. Mejia, as to the “probability” that the
cerebral hemorrhage was secondary to hypertension, was not conclusive proof ISSUE/s:
that Wilfredo was not in good health when he applied for the insurance policy. 11.   Whether Medarda is a real party-in-interest – YES, because Wilfredo
There was no fraudulent intent established. Now, the rightful owners of the remained the real party-in-interest despite the mortgage, and when he died,
proceeds are Wilfredo’s heirs. his heirs succeeded thereto.
12.   Whether Grepalife may rescind the group life insurance of Wilfredo – NO,
DOCTRINE: Since a policy of insurance upon life or health MAY PASS by because there is no conclusive proof of fraudulent intent since the testimony
transfer, will or succession to any person, whether he has an insurable interest or of Dr. Mejia merely mentioned that his cerebral hemorrhage was
not, and such person may recover it whatever the insured might have recovered, “probably” secaondary to hypertension.
the widow (Medarda) of the decedent may file the suit against the insurer
RULING: Petition DENIED. CA decision AFFIRMED with MODIFICATION that c.   Misrepresentation as a defense of the insurer to avoid liability is an
Grepalife is ORDERED to pay PHP86,200 insurance proceeds to the heirs of the affirmative defense.
insured. i.   The duty to establish such defense by satisfactory and
RATIO: convincing evidence rests upon the insurer.
56.   Medarda is the real party-in-interest. 58.   Since there was no evidence on record as to the amount of Wilfredo’s
a.   Nature of a Mortgage Redemption Insurance contract: outstanding indebtedness to DBP at the time of his death, the measure of
i.   In favor of the mortgagee (DBP) – in the event of the indemnity under a policy of insurance upon life or health is the sum fixed in the
unexpected demise of the mortgagor during the substinence of policy (here, it’s PHP86,200).
the mortgage contract, the proceeds from such insurance will 59.   Considering that DBP already foreclosed on the residential lot subject of the
be applied to the payment of the mortgage debt, thereby mortgage, it cannot unjustly enrich itself by collecting the insurance proceeds.
relieving the heirs of the mortgagor from paying the a.   The proceeds now rightly belong to Wilfredo’s heirs.
obligation.
ii.   In favor of the mortgagor (Wilfredo) – in the event of death,
the mortgage obligation will be extinguished by the
application of the insurance proceeds to the mortgage
indebtedness.
iii.   Where the mortgagor pays the insurance premium under the
group insurance policy, making the loss payable to the
mortgagee, the insurance is on the mortgagor’s interest, and
the mortgagor continues to be a party to the contract. Here the
mortgagee is simply an appointee of the insurance fund,
such loss-payable clause does not make the mortgagee a
party to the contract.
b.   §8, Insurance Code – In the event of the debtor’s (Wilfredo’s) death
before his indebtedness with the creditor (DBP) shall have been fully
paid, an amount to pay the outstanding indebtedness shall first be paid
to the creditor and the balance of sum assured, if there is any, shall then
be paid to the beneficiary/ies designated by the debtor.
c.   Gonzales La O v. Yek Tong Lin – the insured may be regarded as the
real party-in-interest, ALTHOUGH he has assigned the policy for the
purpose of collection, or has assigned as collateral security any
judgment he may obtain.
i.   Since a policy of insurance upon life or health MAY PASS by
transfer, will or succession to any person, whether he has an
insurable interest or not, and such person may recover it
whatever the insured might have recovered, the widow
(Medarda) of the decedent may file the suit against the insurer
(Grepalife).
57.   Grepalife may not rescind the group life insurance since Wilfredo did not
conceal the status of his health.
a.   The certification of Dr. Mejia was that Wilfredo “died of cerebral
hemorrhage, PROBABLY secondary to hypertension.” Hence, contrary
to Grepalife’s allegations, there was no sufficient proof that the
Wilfredo had suffered from hypertension.
b.   To entitle Grepalife to rescind the contract, the fraudulent intent on the
part of Wilfredo must be established.
001 EL ORIENTE FABRICA DE TABACOS, INC. v. POSADAS (Adrias) 3.   El Oriente, in order to protect itself against the loss that it might suffer by
September 21, 1931 | Malcolm, J. | Life Insurance Proceeds – Taxable or Not reason of the death of its manager A. Velhagen, who had more than 35
years of experience in the manufacture of cigars in the Philippines and
PLAINTIFF-APPELLANT: El Oriente Fabrica De Tabacos, Inc. whose death would be a serious loss to El Oriente, procured from
DEFENDANT-APPELLEE: Juan Posadas, Collector of Internal Revenue Manufacturers Life Insurance Co. of Toronto, Canada through its local
agent R.R. Elser (Manulife) an insurance policy on the life of A. Velhagen
Insurer: Manufacturers Life Insurance Co., of Toronto, Canada, thru its local agent for the sum of 50,000 USD.
E.E. Elser 4.   El Oriente designated itself as the sole beneficiary. The insured has no
Insured: A. Velhagen (Manager of El Oriente) interest or participation in the proceeds of said life insurance policy.
Beneficiary: El Oriente Fabrica de Tabacos, Inc. 5.   El Oriente charged as expenses of its business all the said premiums and
deducted the same from its gross incomes as reported in its annual income
SUMMARY: El Oriente, in order to protect itself against the loss that it might tax returns, which deductions were allowed by Posadas upon showing by El
suffer by reason of the death of its manager A. Velhagen, who had more than 35 Oriente that such premiums were legitimate expenses of the business.
years of experience in the manufacture of cigars in the Philippines and whose death 6.   Upon the death of Manager A. Velhagen, El Oriente received all the
would be a serious loss to El Oriente, procured from Manufacturers Life Insurance proceeds of the life insurance policy together with the interest and the
Co. of Toronto, Canada through its local agent R.R. Elser (Manulife) an insurance dividends accruing thereon, aggregating P104,957.88.
policy on the life of A. Velhagen for the sum of 50,000 USD. El Oriente designated 7.   Posadas assessed and levied the sum of P3,148.74 as income tax on the
itself as the sole beneficiary. El Oriente charged as expenses of its business all the proceeds of the insurance policy, which was paid by El Oriente under
said premiums and deducted the same from its gross incomes as reported in its protest.
annual income tax returns. Upon the death of Manager A. Velhagen, El Oriente 8.   RTC Decision: Ruled in favor of Posadas
received all the proceeds of the life insurance policy. Posadas assessed and levied the 9.   El Oriente’s contentions:
sum of P3,148.74 as income tax on the proceeds of the insurance policy, which was a.   That trial court erred in holding that section 4 of the Income Tax
paid by El Oriente under protest. The issue in this case is WoN the proceeds of Law (Act No. 2833) is not applicable to the present case.
insurance taken by a corporation on the life of an important official to indemnify it b.   The trial court erred in reading into the law certain exceptions and
against loss in case of his death, are taxable as income under the Philippine Income distinctions not warranted by its clear and unequivocal provisions.
Tax Law? The Court ruled in the negative. Act No. 2833 “Income Tax Law for the c.   The trial court erred in assuming that the proceeds of the life
Philippines” provides in Section 4, Chapter I on Individuals that life insurance insurance policy in question represented a net profit to El Oriente
proceeds are exempt from Income Tax. However, no provisions in Chapter II on when, as a matter of fact, it merely represented an indemnity, for
Corporations explicitly provide for such exemption. Considering, however, the the loss suffered by it thru the death of its manager, the insured.
purport of the stipulated facts, considering the uncertainty of Philippine law, and d.   The trial court erred in refusing to hold that the proceeds of the life
considering the lack of express legislative intention to tax the proceeds of life insurance policy in question is not taxable income.
insurance policies paid to corporate beneficiaries, particularly when in the exemption
in favor of individual beneficiaries in the chapter on this subject, the clause is ISSUE/s:
inserted "exempt from the provisions of this law," we deem it reasonable to hold 1.   WoN the proceeds of insurance taken by a corporation on the life of an
the proceeds of the life insurance policy in question as representing an important official to indemnify it against loss in case of his death, are
indemnity and not taxable income. taxable as income under the Philippine Income Tax Law? – NO.
Considering, therefore, the purport of the stipulated facts, considering
DOCTRINE: El Oriente, as a corporation, can purchase a life insurance policy on an the uncertainty of Philippine law, and considering the lack of express
employee and it can designate itself as the sole beneficiary. The corporation will not legislative intention to tax the proceeds of life insurance policies paid to
be taxed on such proceeds, since it is considered as an indemnity and not income. corporate beneficiaries, particularly when in the exemption in favor of
individual beneficiaries in the chapter on this subject, the clause is
inserted "exempt from the provisions of this law," we deem it
FACTS:
reasonable to hold the proceeds of the life insurance policy in
1.   El Oriente is a domestic corporation and existing under and by virtue of the
question as representing an indemnity and not taxable income
laws of the Philippine Islands.
2.   Juan Posadas (Posadas) is the duly appointed, qualified and acting Collector
of Internal Revenue of the Philippine Islands.
RULING: WHEREFORE, The foregoing pronouncement will result in the judgment 9.   In reality, what El Oriente received was in the nature of an indemnity for
being reversed and in another judgment being rendered in favor of El Oriente and the loss, which it actually suffered because of the death of its manager A.
against Posadas for the sum of P3,148.74. So ordered, without costs in either Velhagen.
instance. 10.   Considering, therefore, the purport of the stipulated facts, considering the
uncertainty of Philippine law, and considering the lack of express legislative
RATIO: intention to tax the proceeds of life insurance policies paid to corporate
1.   The Income Tax Law for the Philippines is Act No. 2833, as amended. beneficiaries, particularly when in the exemption in favor of individual
a.   In Chapter I On Individuals, is to be found Section 4 which beneficiaries in the chapter on this subject, the clause is inserted "exempt
provides that, from the provisions of this law," we deem it reasonable to hold the
"The following incomes shall be exempt from the proceeds of the life insurance policy in question as representing an
provisions of this law: (a) The proceeds of life insurance indemnity and not taxable income.
policies paid to beneficiaries upon the death of the
insured ... ."  
b.   In Chapter II on Corporations does not provide as above.
i.   Section 10, as amended, in Chapter II On Corporations,
provides that, There shall be levied, assessed, collected,
and paid annually upon the total net income received in
the preceding calendar year from all sources by every
corporation ... a tax of three per centum upon such
income ... ."
ii.   Section 11 in the same chapter, provides the exemptions
under the law, but neither here nor in any other section is
reference made to the provisions of section 4 in Chapter I.
2.   It is certain that the proceeds of life insurance policies are exempt. It is
not so certain that the proceeds of life insurance policies paid to corporate
beneficiaries upon the death of the insured are likewise exempt.
3.   The situation will be better elucidated by a brief reference to laws on the
same subject in the United States.
4.   The Income Tax Law of 1916 extended to the Philippine Legislature, when
it came to enact Act No. 2833, to copy the American statute.
5.   Subsequently, the Congress of the United States enacted its Income Tax
Law of 1919, in which certain doubtful subjects were clarified.
6.   Thus, as to the point before us, it was made clear, when not only in the part
of the law concerning individuals were exemptions provided for
beneficiaries, but also in the part concerning corporations, specific reference
was made to the exemptions in favor of individuals, thereby making the
same applicable to corporations.
7.   This was authoritatively pointed out and decided by the United States
Supreme Court in the case of United States vs. Supplee-Biddle Hardware
Co., which involved facts quite similar to those before us.
8.   To quote the exact words in the cited case of Chief Justice Taft delivering
the opinion of the court:
“It is earnestly pressed upon us that proceeds of life insurance paid
on the death of the insured are in fact capital, and cannot be taxed
as income … that proceeds of a life insurance policy paid on the
death of the insured are not usually classed as income.”
002 INSULAR LIFE V. EBRADO (ALCAZAR) FACTS:
October 28,1977 | Martin, J. | Designation of Beneficiary 1.   Buenaventura Ebrado was issued by The Life Assurance Co., Ltd. a
policy on a whole-life insurance for P5,882.00 with a, rider for
PETITIONER: The Insular Life Assurance Company, Ltd. Accidental Death for the same amount. He designated Carponia T.
RESPONDENTS: Carponia T. Ebrado and Pascuala Vda. De Ebrado Ebrado as the revocable beneficiary in his policy. He referred to her
as his wife.
SUMMARY: Buenaventura Ebrado was insured by The Life Assurance Co., 2.   Ebrado diedwhen he was hit by a falling branch of a tree.
Ltd. a policy on a whole-life insurance with a rider on accidental death with 3.   As the policy was in force, The Insular Life Assurance Co., Ltd.
Carponia Ebrada as his beneficiary. He refered to her as his wife. He died liable to pay the coverage in the total amount of P11,745.73,
when a falling branch of a tree fell on him. Carponia (Common law wife) representing the face value of the policy in the amount of P5,882.00
filed a claim for the proceeds of the insurance as the designated beneficiary. plus the additional benefits for accidental death also in the amount of
While Pascual Vda. De Ebrado (Legal wife) also filed her claim as the P5,882.00 and the refund of P18.00 paid for the premium minus the
widow of the deceased insured. The Insurer filed an interpleader before the unpaid premiums and interest thereon.
CFI to determine who to give the insurance proceeds to. The CFI rendered a 4.   Carponia T. Ebrado filed with the insurer a claim for the proceeds of
decision in favor Pascuala. The CA affirmed this decision and ruled that the Policy as the designated beneficiary therein, although she admits
Carponia was disqualified from becoming a beneficiary of Ebrado. that she and the insured Buenaventura C. Ebrado were merely living
The issue is: Whether a common-law wife named as beneficiary in the life as husband and wife without the benefit of marriage.
insurance policy of a legally married man may claim the proceeds thereof in 5.   Pascuala Vda. de Ebrado also filed her claim as the widow of the
case of death of the latter. NO. Common-law spouses are barred from deceased insured. She asserts that she is the one entitled to the
receiving donations from each other. insurance proceeds, not the common-law wife, Carponia T. Ebrado.
In essence, a life insurance policy is no different from a civil donation insofar 6.   In doubt as to whom the insurance proceeds shall be paid, the
as the beneficiary is concerned. Both are founded upon the same insurer, The Insular Life Assurance Co., Ltd. commenced an action
consideration: liberality. A beneficiary is like a donee, because from the for Interpleader before the Court of First Instance
premiums of the policy, which the insured pays out of liberality, the 7.   CFI: Found that Pascuala had 6 legitimate children with Ebrado and
beneficiary will receive the proceeds or profits of said insurance. Article 739 Carponia had 2 illegitimate children with Ebrado. The court ruled
of the new Civil Code provides: The following donations shall be void: (1) that Carponia T. Ebrado disqualified from becoming beneficiary of
Those made between persons who were guilty of adultery or concubinage at the insured Buenaventura Ebrado and directing the payment of the
the time of donation. As a consequence, the proscription in Article 739 of the insurance proceeds to the estate of the deceased insured.
new Civil Code should equally operate in life insurance contracts. The 8.   CA: Affirmed the judgment of the lower court. It ruled that:
mandate of Article 2012 should be followed: any person who cannot receive •   Section 50 of the Insurance Act which provides that "(t)he
a donation cannot be named as beneficiary in the life insurance policy of the insurance shall be applied exclusively to the proper interest of
person who cannot make the donation. Therefore, Carponia Ebrada as the the person in whose name it is made".
common law wife is disqualified to be a beneficiary. As a consequence, the •   The word "interest" highly suggests that the provision refers
proceeds of the policy are hereby held payable to the estate of the deceased only to the "insured" and not to the beneficiary, since a contract
insured. of insurance is personal in character. Otherwise, the prohibitory
laws against illicit relationships especially on property and
DOCTRINE: Any person who is forbidden from receiving any donation descent will be rendered nugatory, as the same could easily be
under Article 739 cannot be named a beneficiary of a life insurance policy by circumvented by modes of insurance.
the person who cannot make a donation to him. Common-law spouses are, •   Rather, the general rules of civil law should be applied to resolve
definitely, barred from receiving donations from each other. this void in the Insurance Law. Article 2011 of the New Civil
Code states: "The contract of insurance is governed by special
laws. Matters not expressly provided for in such special laws
shall be regulated by this Code." When not otherwise specifically donation cannot be named as beneficiary in the life insurance policy
provided for by the Insurance Law, the contract of life insurance of the person who cannot make the donation.
is governed by the general rules of the civil law regulating 3.   Family laws, reason and morality dictate that the impediments
contracts. imposed upon married couple should likewise be imposed upon
•   Article 739 of the new Civil Code provides: The following extra-marital relationship. If a legitimate relationship is
donations shall be void: (1) Those made between persons who circumscribed by these legal disabilities, with more reason should an
were guilty of adultery or concubinage at the time of donation; illicit relationship be restricted by these disabilities.
x x x 4.   A conviction for adultery or concubinage is not required before the
In the case referred to in No. 1, the action for declaration of disabilities mentioned in Article 739 may effectuate. The article says
nullity may be brought by the spouse of the donor or donee; and that in the case referred to in No. 1, the action for declaration of
the guilt of the donee may be proved by preponderance of nullity may be brought by the spouse of the donor or donee; and the
evidence in the same action. guilty of the donee may be proved by preponderance of evidence in
•   Under Article 2012 of the same Code, "any person who is the same action.
forbidden from receiving any donation under Article 739 5.   The underscored clause neatly conveys that no criminal conviction
cannot be named beneficiary of a life insurance policy by the for the offense is a condition precedent. The law plainly states that
person who cannot make a donation to him. Common-law the guilt of the party may be proved “in the same acting for
spouses are, definitely, barred from receiving donations from declaration of nullity of donation.” And, it would be sufficient if
each other. evidence preponderates.
6.   The requisite proof of common-law relationship between the insured
ISSUE/s: and the beneficiary has been conveniently supplied by the
1.   Whether a common-law wife named as beneficiary in the life stipulations between the parties in the pre-trial conference of the
insurance policy of a legally married man may claim the case. The judicial admissions are sufficient and are considered valid.
proceeds thereof in case of death of the latter. NO. Common-law 7.   Carponia Ebrada as the common law wife is disqualified to be a
spouses are barred from receiving donations from each other. beneficiary. As a consequence, the proceeds of the policy are hereby
held payable to the estate of the deceased insured.
RULING: ACCORDINGLY, the appealed judgment of the lower court is
hereby affirmed. Carponia T. Ebrado is hereby declared disqualified to be the  
beneficiary of the late Buenaventura C. Ebrado in his life insurance policy.
As a consequence, the proceeds of the policy are hereby held payable to the
estate of the deceased insured.

RATIO:
1.   In essence, a life insurance policy is no different from a civil
donation insofar as the beneficiary is concerned. Both are founded
upon the same consideration: liberality. A beneficiary is like a
donee, because from the premiums of the policy, which the
insured pays out of liberality, the beneficiary will receive the
proceeds or profits of said insurance.
2.   As a consequence, the proscription in Article 739 of the new Civil
Code should equally operate in life insurance contracts. The mandate
of Article 2012 should be followed: any person who cannot receive a
003 PHILAM v. PINEDA (APASAN)
July, 19, 1989 | Paras, J. | Designation of beneficiary ISSUE/s:
8.   WoN the designation of the irrevocable beneficiaries could be changed or
PETITIONER: Philippine American Insurance Company amended without the consent of all the irrevocable beneficiaries – No, this
RESPONDENTS: Honorable Gregorio G. Pineda and Rodolfo C. Dimayuga was expressly prohibited by the Insurance Act, which was the law
applicable at that time.
SUMMARY: Rodolfo Dimayuga (Dimayuga) procured an ordinary life insurance 9.   WoN the irrevocable beneficiaries herein, who are all minors (NOTE: wife,
policy from Philippine American Insurance Company (Philam) designating his wife who was also a beneficiary, predeceased Dimayuga), could validly give
and children as irrevocable beneficiaries. Subsequently, Dimayuga filed a petition consent to the change or amendment in the designation of the irrevocable
with the CFI to amend the designation in the policy from irrevocable to recovable. beneficiaries – No, because being minors, this would not be an effective
Philam opposed, but the lower court denied such opposition and granted the petition. ratification.
Issues are: 1.) WoN the designation of irrevocable beneficiaries could be
amended without the consent of all the beneficiaries – No, under the Insurance RULING: Order of the lower court was nullified and set aside.
Act, which was the law applicable at that time, the beneficiary designated in a life
insurance contract cannot be changed without the consent of the beneficiary because RATIO:
he has a vested interest in the policy. Also, the provisions of the Insurance Contract First Issue
did not provide for any exception regarding its irrevocability. Both the law and the 1.   Needless to say, the applicable law in the instant case is the Insurance Act,
policy do not provide for any other exception, thus, abrogating the contention of the otherwise known as Act No. 2427 as amended, the policy having been
Dimayuga that said designation can be amended if the Court finds a just, reasonable procured in 1968.
ground to do so 2.) WoN the irrevocable beneficiaries herein who are all minors a.   Under the said law, the beneficiary designated in a life insurance
(NOTE: wife, who was also a beneficiary, predeceased Dimayuga), could validly contract cannot be changed without the consent of the
give consent to the change or amendment in the designation of the irrevocable beneficiary because he has a vested interest in the policy.
beneficiaries – No. First, being minors, their ratification would be ineffective. 2.   In this regard, it is worth noting that the Beneficiary Designation
Second, neither could they act through their father insured since their interests are Indorsement in the policy which forms part of Insurance Policy in the name
quite divergent from one another. Lastly, parent-insured (Dimayuga) cannot exercise of Rodolfo Dimayuga states that the designation of the beneficiaries is
rights and/or privileges pertaining to the insurance contract, for otherwise, the vested irrevocable, to wit:
rights of the irrevocable beneficiaries would be rendered inconsequential. a.   It is hereby understood and agreed that, notwithstanding the
provisions of this policy to the contrary, inasmuch as the
DOCTRINE: Under the Insurance Act, otherwise known as Act No. 2427, the designation of the primary/contingent beneficiary/beneficiaries
beneficiary designated in a life insurance contract cannot be changed without the in this Policy has been made without reserving the right to
consent of the beneficiary because he has a vested interest in the policy. Further, change said beneficiary/ beneficiaries, such designation may not
minor-beneficiaries cannot validly give their consent to such amendment. be surrendered to the Company, released or assigned; and no right
or privilege under the Policy may be exercised, or agreement
made with the Company to any change in or amendment to the
FACTS: Policy, without the consent of the said
1.   On January 15, 1968, Rodolfo Dimayuga (Dimayuga) procured an ordinary beneficiary/beneficiaries.
life insurance policy from Philippine American Insurance Company 3.   Be it noted that the foregoing is a fact which the private respondent did not
(Philam) and designated his wife and children as irrevocable beneficiaries bother to disprove.
of said policy. 4.   Inevitably therefore, based on the aforequoted provision of the contract,
2.   On February 22, 1980 Dimayuga filed a petition before the Court of First it is only with the consent of all the beneficiaries that any change or
Instance of Rizal to amend the designation of the beneficiaries in his life amendment in the policy concerning the irrevocable beneficiaries may
policy from irrevocable to revocable. be legally and validly effected. Both the law and the policy do NOT
3.   Philam filed its Comment and/or Opposition to Petition. provide for any other exception, thus, abrogating the contention of the
a.   Respondent Judge Gregorio G. Pineda (Judge Pineda) denied this Dimayuga that said designation can be amended if the Court finds a
opposition, and the consequence of which was the issuance of the just, reasonable ground to do so.
questioned Order granting the petition.
4.   Hence, this petition.
Second Issue 10.   Finally, the fact that the contract of insurance does not contain a
5.   Similarly, the alleged acquiescence of the six (6) children beneficiaries of contingency when the change in the designation of beneficiaries could
the policy (the beneficiary-wife predeceased the insured) CANNOT be be validly effected means that it was never within the contemplation of
considered an effective ratification to the change of the beneficiaries the parties.
from irrevocable to revocable. a.   The lower court, in gratuitously providing for such contingency,
6.   Indubitable is the fact that all the six (6) children named as beneficiaries made a new contract for them, a proceeding which the Supreme
were minors at the time, for which reason, they could NOT validly give Court cannot tolerate.
their consent. Neither could they act through their father insured since b.   Ergo, the Supreme Court cannot help but conclude that the lower
their interests are quite divergent from one another. In point is an court acted in excess of its authority when it issued the Order dated
excerpt from the Notes and Cases on Insurance Law by Campos and March 19, 1980 amending the designation of the beneficiaries from
Campos, 1960, reading- "irrevocable" to "revocable" over the disapprobation of the
a.   The insured ... can do nothing to divest the beneficiary of his rights petitioner insurance company.
without the latter’s consent. He cannot assign his policy, nor even
take its cash surrender value without the consent of the beneficiary.  
Neither can the insured's creditors seize the policy or any right
thereunder. The insured may not even add another beneficiary
because by doing so, he diminishes the amount which the
beneficiary may recover and this he cannot do without the
beneficiary's consent.
7.   Therefore, the parent-insured cannot exercise rights and/or privileges
pertaining to the insurance contract, for otherwise, the vested rights of
the irrevocable beneficiaries would be rendered inconsequential.
8.   Of equal importance is the well-settled rule that the contract between the
parties is the law binding on both of them and for so many times, this court
has consistently issued pronouncements upholding the validity and
effectivity of contracts.
a.   Where there is nothing in the contract which is contrary to law,
good morals, good customs, public policy or public order the
validity of the contract must be sustained.
b.   Likewise, contracts which are the private laws of the contracting
parties should be fulfilled according to the literal sense of their
stipulations, if their terms are clear and leave no room for doubt as
to the intention of the contracting parties, for contracts are
obligatory, no matter in what form they may be, whenever the
essential requisites for their validity are present.
9.   In the recent case of Francisco Herrera vs. Petrophil Corporation, 146
SCRA 385, this Court ruled that: ... it is settled that the parties may
establish such stipulations, clauses, terms, and conditions as they may want
to include; and as long as such agreements are not contrary to law, good
morals, good customs, public policy or public order, they shall have the
force of law between them.
a.   Undeniably, the contract in the case at bar, contains the
indispensable elements for its validity and does not in any way
violate the law, morals, customs, orders, etc. leaving no reason for
the Court to deny sanction thereto.
004 HEIRS OF LORETO MARAMAG v. MARAMAG (Arcenas) 1.   Vicenta Maramag (legal wife), representing the heirs of Loreto, filed a case
June 5, 2009| Nachura, C. J. | Designation of beneficiary against Eva Maramag et al (kabit and anak ng kabit) with the RTC for
revocation and/or reduction of insurance proceeds for being void and/or
PETITIONERS: Heirs Of Loreto C. Maramag, Represented By Surviving Spouse inofficious, with prayer for a temporary restraining order (TRO) and a writ
Vicenta Pangilinan Maramag of preliminary injunction, alleging that
RESPONDENTS: Eva Verna De Guzman Maramag, Odessa De Guzman Maramag, a.   Heirs were the legitimate wife and children of Loreto Maramag,
Karl Brian De Guzman Maramag, Trisha Angelie Maramag, The Insular Life while Eva Maramag et al were Loretos illegitimate family;
Assurance Company, Ltd., And Great Pacific Life Assurance Corporation b.   Eva de Guzman Maramag (Eva) was a concubine of Loreto and a
suspect in the killing of the latter, and thus disqualified to receive
SUMMARY: Petitioners in this case are the legitimate heirs of Loreto, represented any proceeds from his insurance policies from Insular Life
by the legitimate wife, Vienta. They were not named as beneficiaries in the insurance Assurance Company, Ltd. (Insular) and Great Pacific Life
policies issued by Insular and Grepalife, instead, Loreto designated his concubine, Assurance Corporation (Grepalife);
Eva, and their illegitimate children as his beneficiaries under policies with Insular c.   the illegitimate children of Loreto: Odessa, Karl Brian, and Trisha
Life and Grepalife. Vicenta et al are seeking the revocation and reduction of Angelie were entitled only to one-half of the legitime of the
insurance proceeds in their favor. The basis of the claim is that Eva, being a legitimate children, thus, the proceeds released to Odessa and those
concubine of Loreto and a suspect in Loreto’s murder, is disqualified from being to be released to Karl Brian and Trisha Angelie were inofficious
designated as beneficiary of the insurance policies, and that Eva’s children with and should be reduced; and
Loreto, being illegitimate children, are entitled to a lesser share of the proceeds of d.   Heirs could not be deprived of their legitimes, which should be
the policies. They also argued that pursuant to Section 12 of the Insurance Code, satisfied first.
Evas share in the proceeds should be forfeited in their favor, the former having e.   That part of the insurance proceeds had already been released in
brought about the death of Loreto. Thus, they prayed that the share of Eva and favor of Odessa, while the rest of the proceeds are to be released in
portions of the shares of Loretos illegitimate children should be awarded to them, favor of Karl Brian and Trisha Angelie, both minors, upon the
being the legitimate heirs of Loreto entitled to their respective legitimes. RTC appointment of their legal guardian.
initially ruled that the claim be dismissed against the heirs and sustained the action f.   Prayer for the total amount of P320,000.00 as actual litigation
against Eva, Insular and Grepalife. However, reversed itself by granting the motion expenses and attorneys fees.
for reconsideration of the same parties. On appeal, the same was denied for lack of 2.   INSULAR’S ANSWER: Insular admitted that Loreto misrepresented Eva
jurisdiction and was filed beyond reglementary period. Hence, this petition. The as his legitimate wife and Odessa, Karl Brian, and Trisha Angelie as his
issue is whether or not the legitimate children can claim Eva’s share since she was legitimate children, and that they filed their claims for the insurance
disqualified as beneficiary under the insurance policy of the deceased, and that the proceeds of the insurance policies;
same should not have been distributed among the illegitimate children. SC held that a.   that when it ascertained that Eva was not the legal wife of Loreto,
they cannot since the legitimate children are third parties to the insurance contracts, it disqualified her as a beneficiary and divided the proceeds among
and not entitled to the proceeds despite disqualification of Eva as beneficiary illegitimate children, as the remaining designated beneficiaries;
because the designation of the illegitimate children as beneficiaries remained valid. and
Also, Eva’s share whether forfeited by the court in view of the prohibition on b.   that it released Odessas share as she was of age, but withheld the
donations under Article 739 of the Civil Code or by the insurers themselves for release of the shares of minors Karl Brian and Trisha Angelie
reasons based on the insurance contracts, must be awarded to the said illegitimate pending submission of letters of guardianship.
children, the designated beneficiaries, to the exclusion of the legitimate children. c.   Insular alleged that the complaint or petition failed to state a cause
of action: when it sought to declare as void the designation of Eva
DOCTRINE: The only persons entitled to claim the insurance proceeds are either as beneficiary, because Loreto already revoked Eva’s designation
the insured, if still alive; or the beneficiary, if the insured is already deceased, upon as such in one policy and it disqualified her in another and when it
the maturation of the policy. It is only in cases where the insured has not designated sought to declare as inofficious the shares of Odessa, Karl Brian,
any beneficiary, or when the designated beneficiary is disqualified by law to receive and Trisha Angelie, considering that no settlement of Loretos
the proceeds, that the insurance policy proceeds shall redound to the benefit of the estate had been filed nor had the respective shares of the heirs been
estate of the insured. determined.

FACTS:
d.   Insular further claimed that it was bound to honor the insurance 8.   MR GRANTED: dismissed prosecution order against eva, insular and
policies designating the children of Loreto with Eva as grepalife.
beneficiaries pursuant to Section 53 of the Insurance Code. a.   RTC ruled it is only in cases where there are no beneficiaries
3.   GREPALIFE’s ANSWER: with compulsory counterclaim designated, or when the only designated beneficiary is disqualified,
a.   Grepalife alleged that Eva was not designated as an insurance that the proceeds should be paid to the estate of the insured.
policy beneficiary; b.   As to the claim that the proceeds to be paid to Loreto’s illegitimate
b.   that the claims filed by Odessa, Karl Brian, and Trisha Angelie children should be reduced based on the rules on legitime, the trial
were denied because Loreto was ineligible for insurance due to a court held that the distribution of the insurance proceeds is
misrepresentation in his application form that he was born on governed primarily by the Insurance Code, and the provisions of
December 10, 1936 and, thus, not more than 65 years old when he the Civil Code are irrelevant and inapplicable.
signed it in September 2001; c.   Grepalife policy: Eva was never designated as a beneficiary, but
c.   that the case was premature, there being no claim filed by the only the illegitimate children; thus, it upheld the dismissal of the
legitimate family of Loreto; and case as to them, matter of Loretos misrepresentation was premature
d.   that the law on succession does not apply where the designation of 9.   CA RULING: dismissed Vicenta’s appeal for lack of jurisdiction, holding
insurance beneficiaries is clear that the decision of the trial court dismissing the complaint for failure to
4.   As the whereabouts of Eva et al were not known to Vicenta et al, summons state a cause of action involved a pure question of law, and that the appeal
by publication was resorted to. Eva et al failed to answer, and declared in was filed beyond the reglementary period. Hence, this petition
default by the trial court. ISSUE/s:
5.   VICENTA ET AL’s comment: alleged that the issue raised by Insular and 1.   W/N the legitimate children can claim Eva’s share since she was
Grepalife was purely legal whether the complaint itself was proper or not disqualified as beneficiary under the insurance policy of the deceased, and
and that the designation of a beneficiary is an act of liberality or a donation that the same should not have been distributed among the illegitimate
and, therefore, subject to the provisions of Articles 752 and 772 of the Civil children – NO, legitimate children are third parties to the insurance
Code. contracts, and not entitled to the proceeds despite disqualification of Eva as
6.   REPLY: Both insurance companies countered that the insurance proceeds beneficiary because the designation of the illegitimate children as
belong exclusively to the designated beneficiaries in the policies, not to the beneficiaries remained valid. Also, Eva’s share whether forfeited by the
estate or to the heirs of the insured. court in view of the prohibition on donations under Article 739 of the Civil
a.   Grepalife also reiterated that it had disqualified Eva as a Code or by the insurers themselves for reasons based on the insurance
beneficiary when it ascertained that Loreto was legally married to contracts, must be awarded to the said illegitimate children, the designated
Vicenta Pangilinan Maramag. beneficiaries, to the exclusion of the legitimate children.
7.   RTC RULING: dismissed the complaint illegitimate children for the
reduction and/or declaration of inofficiousness of donation as primary RULING: WHEREFORE, the petition is DENIED for lack of merit. Costs against
beneficiary in the insurances of the late Loreto C. Maramag, but it proceeds petitioners.
against Eva, Insular and Grepalife.
a.   Can proceed against eva because she is the concubine of the RATIO:
deceased: any person who is forbidden from receiving any 1.   In this case, it is clear from the petition filed before the trial court that,
donation under Article 739 cannot be named beneficiary of a life although Vicenta et al are the legitimate heirs of Loreto, they were not
insurance policy of the person who cannot make any donation to named as beneficiaries in the insurance policies issued by Insular and
him, according to said article (Art. 2012, Civil Code). If a Grepalife.
concubine is made the beneficiary, it is believed that the insurance a.   The basis of the claim is that Eva, being a concubine of Loreto and
contract will still remain valid, but the indemnity must go to the a suspect in his murder, is disqualified from being designated as
legal heirs and not to the concubine, for evidently, what is beneficiary of the insurance policies, and that Evas children with
prohibited under Art. 2012 is the naming of the improper Loreto, being illegitimate children, are entitled to a lesser share of
beneficiary the proceeds of the policies.
b.   Since designation to Eva is void, the insurance indemnity paid to b.   They also argued that pursuant to Section 12 of the Insurance
her must go to the legal heirs Code, Evas share in the proceeds should be forfeited in their favor,
the former having brought about the death of Loreto.
c.   Thus, they prayed that the share of Eva and portions of the shares
of Loretos illegitimate children should be awarded to them, being
the legitimate heirs of Loreto entitled to their respective legitimes.
2.   It is evident from the face of the complaint that Vicente et al are not entitled
to a favorable judgment in light of Article 2011 of the Civil Code which
expressly provides that insurance contracts shall be governed by special
laws, i.e., the Insurance Code, specifically Section 53.13
a.   Pursuant thereto, it is obvious that the only persons entitled to
claim the insurance proceeds are either the insured, if still
alive; or the beneficiary, if the insured is already deceased,
upon the maturation of the policy.
b.   The exception to this rule is a situation where the insurance
contract was intended to benefit third persons who are not parties
to the same in the form of favorable stipulations or indemnity. In
such a case, third parties may directly sue and claim from the
insurer.
3.   Vicenta et al are third parties to the insurance contracts with Insular
and Grepalife and, thus, are not entitled to the proceeds.
a.   Accordingly, Insular and Grepalife have no legal obligation to turn
over the insurance proceeds to Vicenta et al.
b.   The revocation of Eva as a beneficiary in one policy and her
disqualification as such in another are of no moment considering
that the designation of the illegitimate children as beneficiaries in
Loretos insurance policies remains valid.
c.   Because no legal proscription exists in naming as beneficiaries
the children of illicit relationships by the insured, the shares of
Eva in the insurance proceeds, whether forfeited by the court in
view of the prohibition on donations under Article 739 of the Civil
Code or by the insurers themselves for reasons based on the
insurance contracts, must be awarded to the said illegitimate
children, the designated beneficiaries, to the exclusion of Vicenta
et al.
d.   It is only in cases where the insured has not designated any
beneficiary, or when the designated beneficiary is disqualified by
law to receive the proceeds, that the insurance policy proceeds
shall redound to the benefit of the estate of the insured.

                                                                                                                       
13  
SECTION 53. The insurance proceeds shall be applied exclusively to the proper interest of the person in
whose name or for whose benefit it is made unless otherwise specified in the policy.
 
Lampano v. Jose, et. al. (Lindon) sum of P2,000, evidenced by a promissory note, and Placida A. Jose still
March 30, 1915 | Trent, J. | Insurable Interest owed Mariano R. Barretto on the cost of the construction the sum of
P2,000.
PETITIONER: Antonina Lampano 5.   After the completion of the house and sometime before it was destroyed,
RESPONDENTS: Placida Jose Mariano R. Barretto took out an insurance policy upon it in his own name,
with the consent of Placida A. Jose, for the sum of P4,000.
SUMMARY: Barretto built a house for Jose in Manila. Then, Jose sold the same 6.   After its destruction, he collected P3,600 from the insurance company,
to Lampano for 6k. A few months later, house was gutted by fire. At that time, having paid in premiums the sum of P301.50.
Lampano still owed Jose 2k, evidenced by a promissory note, and Jose still owed
Barretto the cost of construction (2k as well). After the completion of the house Arguments before the lower courts
and before it was destroyed, Barretto took out an insurance policy in his name Petitioner’s Side
with consent of Jose for 4k. Barretto was able to collect 3.6k after paying 300 as 7.   Antonina Lampano alleged in her complaint that there was a verbal
premiums. agreement between her and Placida A. Jose, at the time of the purchase and
sale of the house, to the effect that the latter14 agreed to deliver to her the
Lampano sued Jose and Barretto for the amount of the insurance collected. Jose insurance policy on the building;
denied the transfer of the policy to Lampano since it was Barretto’s. Jose filed a 8.   She15 did not learn that the policy was in the name of Barretto16 until after
counterclaim against Lamoano for the balance of the purchase price. Trial court the fire; and the neither Placida A. Jose17 nor Mariano R. Barretto18 has any
ruled in favor of Jose against Barretto for the difference between insurance right to the insurance or to the money received therefrom.
proceeds and the amount yet due him. TC did not rule on Barretto v. Lampano 9.   She19 prayed for judgment against each20 of them for the sum of P3,600, the
for lack of privity. Hence, review before SC as Barretto and Lampano appealed. amount of the insurance collected.

In the case at bar Barretto assumed the responsibility for the insurance. Respondents’ Side
The premiums, as we have indicated, were paid by him without any 10.   To this complaint the defendant, Placida A. Jose,21 answered, denying that
agreement or right to recoup the amount paid therefor should no loss result she agreed to transfer the policy of insurance to the plaintiff22 and alleging:
to the property. It would not, therefore, be in accordance with the law and his a.   that the insurance was taken out and paid for by Barretto before the
contractual obligations to compel him to account for the insurance money, or any sale of the house to the plaintiff;
par thereof, to Antonina Lampano, who assumed no risk whatever. b.   that Barretto did this because he had constructed the house and she
was owing him therefor; and
DOCTRINE: Where different persons have different interests in the same c.   that the insurance was entirely for the personal account and in the
property, the insurance taken by one in his own right and in his own interest does exclusive interest of Barretto.
not in any way insure to the benefit of another. This is the general rule prevailing 23
in the United States and we find nothing different in this jurisdiction. 11.   In her cross-complaint she asked for judgment against Antonina

                                                                                                                       
14
N.B. The alleged oral transfer of the policy was not proven in the court below.  PLACIDA  A  JOSE  
15
 ANTONINA  LAMPANO  
  16
 THE  CO-­‐DEFENDANT  WHO  CONSTRUCTED  THE  HOUSE  
FACTS: 17
 THE  DEFENDANT  WHOM  BARRETTO  BUILT  THE  HOUSE  FOR  
1.   Mariano R. Barretto constructed a house for the other defendant, Placida 18
A. Jose on land described as No. 72, plot F. Estate of Nagtahan, district of  THE  CO-­‐DEFENDANT  WHO  CONSTRUCTED  THE  HOUSE  
19
Sampaloc, city of Manila, for the agreed price of P6,000.  PLACIDA  A  JOSE  
20
2.   Subsequent thereto and on November 12, 1912, Placida A. Jose sold the  PLACIDA  A  JOSE  and  MARIANO  R.  BARRETTO  
house to the plaintiff, Antonina Lampano, for the sum of P6,000. 21
 THE  PERSON  WHOM  THE  HOUSE  WAS  CONSTRUCTED  FOR  
3.   On March 22, 1913, the house was destroyed by fire. 22
 ANTONINA  LAMPANO  
4.   At the time of the fire Antonina Lampano still owed Placida A. Jose the 23
 I  think  this  should  have  been  a  counter-­‐claim  
Lampano for the sum of P2,000, the balance due on the purchase price. 2.   If Barretto had an insurable interest in the house, he could insure this
Barretto answered, reciting the facts giving rise to his taking out the interest for his sole protection. The policy was in the name of Barretto
insurance on the house and denying any obligation to Antonina Lampano in alone. It was, therefore, a personal contract between him and the
connection therewith. company and not a contract which ran with the property. According to
this personal contract the insurance policy was payable to the insured
Judgment of the lower courts without regard to the nature and extent of his interest in the property,
12.   Judgment was entered against Barretto and in favor of Placida A. Jose for provided that he had, as we have said, an insurable interest at the time of the
the sum of P1,298.50, being the difference between the amount collected by making of the contract, and also at the time of the fire. Where different
Barretto on the insurance and the amount yet due him for the construction persons have different interests in the same property, the insurance
of the house, including the premiums paid. taken by one in his own right and in his own interest does not in any
13.   Judgment was also entered in favor of the defendant, Placida A. Jose, way insure to the benefit of another. This is the general rule prevailing
against the plaintiff for the sum of P2,000, being the balance of the purchase in the United States and we find nothing different in this jurisdiction.
price of the house. 3.   In the case of Shadgett vs. Phillips and Crew Co., reported in 56 L. R. A.,
14.   The plaintiff was authorized to offset this judgment against her for P2,000 461, Mrs. Shagett received a piano as a gift from her husband and insured it.
by the P2,000 which the court declared had been paid the defendant, Placida She knew that it was the obligation of her husband to insure he piano for the
A. Jose, by Barretto out of the insurance money. A final judgment was benefit of the vendor. The court held, however, that the vendor (mortgagee)
entered in favor of the plaintiff against the defendant, Placida A. Jose, for was not entitled to the proceeds of the insurance as "there was no
the sum of P1,298.50, being the amount of the judgment against Barretto. undertaking on the part of Mrs. Shadgett to either insure for complainant's
15.   Barretto appealed the judgment in the CROSS-CLAIM. benefit, or to assume her husband's obligation to so insure, and mere
16.   No judgment was rendered in favor of the Antonina Lampano since they knowledge of that obligation did not impose it upon her."
have no juridical tie. Antonina Lampano appealed as well. a.   The contract of insurance was wholly between the defendant and
the insurance company, and was personal, in the sense that the
ISSUE/s: money agreed to be paid in case of loss was not to stand in the
1.   Has Antonina Lampano any right to recover from Barretto any portion of place of the piano itself, but was a mere indemnity against the loss
the insurance money? (This was the wording in the case) – No. Barretto of defendant's interest therein. If her interest was small, on account
alone assumed the responsibility for the insurance. of incumbrances existing in favor of the complainant, that fact was
RULING: For the foregoing reasons the judgment appealed from, in so far as it for the consideration only of the insurer and defendant, for
affects the appellant, is reversed and he is absolved. Without costs. So ordered. complaint has no concern with the adjustment of the loss between
them. We know of no principle, either of law or equity, which
RATIO: would bind defendant to carry out her donor's contract to insure, in
the absence of any agreement on her part to do so, even though the
Complications property in her hands was subject to complainant's rights therein as
1.   The plaintiff does not contend that Barretto participated in this sale, or even a conditional vendor.
had any knowledge of it, until sometime after it was consummated. Placida b.   A contract of insurance made for the insurer's (insured) indemnity
A. Jose denies that she agreed to transfer the policy to the plaintiff, and the only, as where there is no agreement, express or implied, that it
deed of purchase and sale makes no mention of such an agreement. The shall be for the benefit of a third person, does not attach to or run
policy is not mentioned in this document, although it was agreed that the with the title to the insured property on a transfer thereof personal
vendor would transfer to the vendee all of the former's right, title, and as between the insurer and the insured. In such case strangers to
interest in the leasehold to the land upon which the house was built. It the contract cannot require in their own right any interest in the
would seem that if the vendor agreed to transfer the policy, this agreement insurance money, except through an assignment or some contract
would have been inserted in the document of purchase and sale, the same as with which they are connected.
that with reference to the lease. The trial court did not find that such an 4.   In Vandergraf vs. Medlock (3 Porter, 389; 29 Am. Dec., 256), it was held
agreement existed and we think the plaintiff has failed to establish this that the mortgage is not entitled to the proceeds of an insurance policy
verbal agreement. procured by the mortgages, there being no agreement that such insurance
should be effected by the latter for the benefit of the former. The court says:
Ruling Proper "It is well settled that a policy of insurance is a distinct independent contract
between the insured and insurers, and third person have no right either in a
court of equity, or in a court of law, to the proceeds of it, unless there be
some contract or trust, expressed or implied, between the insured and third
persons."
5.   In Burlingane vs. Goodspeed (10 L. R. A., 495), the court says that where a
mortgage at his own expense and without any agreement or understanding
with he mortgagor obtains insurance upon his interest as a mortgage and
collects the money from the insurer after a loss, he is not bound to account
for it to the mortgagor.
6.   In the case at bar Barretto assumed the responsibility for the
insurance. The premiums, as we have indicated, were paid by him
without any agreement or right to recoup the amount paid therefor
should no loss result to the property. It would not, therefore, be in
accordance with t he law and his contractual obligations to compel him to
account for the insurance money, or any par thereof, to Antonina Lampano,
who assumed no risk whatever.
7.   That Barretto had an insurable interest in the house, we think there can be
no question. He construed the building, furnishing all the materials and
supplies, and insured it after it had been completed

 
006 TAI TONG CHUACHE & CO. v. INSURANCE COMMISSION Building 50,000 24,918.79
(BALISONG) SSS Building 25,000 5,938.50
29 Feb. 1988 | Gancayco, J. | Insurable Interest; Fire Insurance Accredited
Traveller’s Contents 70,000 16,628.00
PETITIONER: Tai Tong Chuache & Co. Building 30,000 14,467.31
RESPONDENTS: Insurance Commission and Traveller’s Multi-Indemnity Zenith, Phil. British, and SSS paid the whole or a portion of the claim. However,
Corporation Traveller’s disclaimed liability, prompting Palomo to claim from the other insurers
Traveller’s share based on the adjusted computation:
SUMMARY: Tai Tong loaned P100,000 to Palomo. The loan was secured by a
mortgage over a land and a building owned by Palomo. The building and its contents COMPANY RISK INSURES PAYS
were insured by Palomo with SSS, Zenith, and Phil. British. Tai Tong insured its Zenith Building P50,000 17,610.93
interest over the building and its contents with Traveller’s through Arsenio Chua. Phil. British Household 70,000 24,655.31
The building was gutted by fire. Claims were made by Palomo. Traveller’s refused to Building 50,000 39,186.10
pay on the ground that Palomo was not the beneficiary of the Policy, but Arsenio SSS Building 25,000 8,805.47
Chua. It also denied Tai Tong’s claim, saying that the loan and mortgage was Accredited
between Palomo and Chua. The issue is whether Tai tong has an insurable interest
over the building and the contents thereof. The Court held in the affirmative.
Arsenio, in contracting with the parties here, was acting as an agent of Tai Tong,
4.   The Zenith and Phil. British denied the claim on the ground that the claim
being its managing partner. Further, Palomo admitted that the mortgage remained
effective at the time of the fire as she has yet to pay Tai Tong. The Insurance had already been waived, extinguished, or paid. SSS paid the balance in
Commission was reversed. full.

DOCTRINE: The mortgagee has insurable interest over the subject of the mortgage 5.   Traveller’s reason for non-payment was that the Fire Insurance Policy over
is it can be shown that the loan secured by the same has not yet been paid at the time the building and the furniture inside was taken by Arsenio Chua as
the risk happened. mortgage creditor, and that it was Arsenio Chua who paid the premiums,
and that Palomo endorsed the Policy to Chua to protect the latter’s interest
FACTS: therein. Tai Tong filed a complaint in intervention, but was opposed by
1.   Palomo acquired a parcel of land and a building under an assumption of Traveller’s arguing that Tai Tong did not have insurable interest as it is not
mortgage arrangement. The building was insured with S.S.S. Accredited a party to the contract, the real party being Arsenio Chua.
Group of Insurers for P25,000.00.
6.   The Insurance Commission ruled in favor of Traveller’s. It held that Palomo
2.   Palomo obtained a P100,000-loan from Tai Tong Chuache, Inc. (Tai Tong), had no interest on the proceeds of the policy because it was taken out by the
secured by a mortgage over the land and the building. Aresenio Chua, in mortgagee. It also denied Tai Tong’s intervention on the strength of a
representation of Tai Tong, insured the partnership’s interest with certification from the CFI of Davao that a case between Arsenio Chua and
Traveller’s Multi-Indemnity Corporation for P100,000. Palomo secured a Palomo is pending before said court, which case, the Commission surmised,
Fire Insurance Policy over the building for P50,000 with Zenith was grounded on the mortgage contract between Arsenio Chua and Palomo,
Corporation, and for P50,000 (building) and P70,000 (contents) with NOT Tai Tong and Palomo, hence Tai Tong cannot claim under the policy
Philippine British Assurance Corporation. The building was razed by fire. as it had no insurable interest on the building, its contents, and the land over
which it stands.
3.   Palomo filed claims based on the following:
ISSUE/s:
COMPANY RISK INSURES PAYS
1.   Whether Tai Tong had insurable interest on the building and its contents.
Zenith Building P50,000 11,877.14
YES — The fact that Palomo admitted that she was indebted to Tai Tong
Phil. British Household 70,000 16,628.00
should have settled the matter. Further to this, Arsenio Chua is the
managing partner of Tai Tong, and was acting in representation of the
partnership in the above actions.

RULING: IN VIEW OF THE FOREGOING, the decision appealed from is hereby


SET ASIDE and ANOTHER judgment is rendered ordering private respondent
Travellers Multi-Indemnity Corporation to pay petitioner the face value of Insurance
Policy No. 599-DV in the amount of P100,000.00. Costs against said private
respondent.

RATIO:
1.   Traveller’s did not assail the validity of the insurance policy taken out by
petitioner over the mortgaged property. Neither did it deny that the said
property was totally razed by fire within the period covered by the
insurance.

2.   The record of the case shows that Tai Tong, to support its claim for the
insurance proceeds, offered as evidence the contract of mortgage which has
not been cancelled nor released. It has been held in a long line of cases that
when the creditor is in possession of the document of credit, he need not
prove non-payment for it is presumed.

3.   Tai Tong being a partnership may sue and be sued in its name or by its duly
authorized representative. The fact that Arsenio Lopez Chua is the
representative of Tai Tong is not questioned. Tai Tong’s declaration that
Arsenio Lopez Chua acts as the managing partner of the partnership was
corroborated by Traveller’s.

 
007 SPOUSES CHA v. CA (Buenaventura) FACTS:
August 18, 1997 | Padilla, J. | Lack of insurable interest
1.   Petitioner-spouses Nilo Cha and Stella Uy-Cha, as lessees, entered into a
lease contract with private respondent CKS Development Corporation
PETITIONER: Spouses Nilo and Stella Cha, and United Insurance Co., Inc
(hereinafter CKS), as lessor, on 5 October 1988.
RESPONDENTS: CA and CKS Development Corporation
2.   One of the stipulations of the one (1) year lease contract states:
SUMMARY:
a.   18. x x x. The LESSEE shall not insure against fire the chattels,
Spouses Nilo Cha and Stella Uy-Cha and CKS Development Corporation
merchandise, textiles, goods and effects placed at any stall or store
entered a 1 year lease contract with a stipulation not to insure against fire the
or space in the leased premises without first obtaining the written
chattels, merchandise, textiles, goods and effects placed at any stall or store or
consent and approval of the LESSOR. If the LESSEE obtain(s) the
space in the leased premises without first obtaining the written consent and
insurance thereof without the consent of the LESSOR then the
approval of the lessor. But it insured against loss by fire their merchandise
policy is deemed assigned and transferred to the LESSOR for its
inside the leased premises for P500,000 with the United Insurance Co., Inc.
own benefit; x x x
without the written consent of CKS
3.   Notwithstanding the above stipulation in the lease contract, the Cha spouses
On the day the lease contract was to expire, fire broke out inside the leased
insured against loss by fire their merchandise inside the leased premises for
premises and CKS learning that the spouses procured an insurance wrote to
Five Hundred Thousand (P500,000.00) with the United Insurance Co., Inc.
United to have the proceeds be paid directly to them. But United refused so CKS
(hereinafter United) without the written consent of private respondents
filed against Spouses Cha and United.
CKS.
RTC: United to pay CKS the amount of P335,063.11 and Spouses Cha to
4.   On the day that the lease contract was to expire, fire broke out inside the
pay P50,000 as exemplary damages, P20,000 as attorney’s fees and costs of suit
leased premises.
CA: deleted exemplary damages and attorney’s fees
5.   When CKS learned of the insurance earlier procured by the Cha spouses
The issue is whether or not the paragraph 18 of the lease contract entered
without its consent, it wrote the insurer (United) a demand letter asking that
into between CKS and the Cha spouses is valid insofar as it provides that any
the proceeds of the insurance contract (between the Cha spouses and
fire insurance policy obtained by the lessee Cha spouses over their merchandise
United) be paid directly to CKS, based on its lease contract with Cha
inside the leased premises is deemed assigned or transferred to the lessor CKS.
spouses.
SC held that NO, the provision is void, as against public policy. CKS has
6.   United refused to pay CKS. Hence, the latter filed a complaint against the
no insurable interest over the merchandise.
Cha spouses and United.
A non-life insurance policy such as the fire insurance policy taken by
7.   RTC Manila, rendered a decision* ordering therein defendant United to pay
petitioner-spouses over their merchandise is primarily a contract of
CKS the amount of P335,063.11 and defendant Cha spouses to
indemnity. Insurable interest in the property insured must exist at the time the
pay P50,000.00 as exemplary damages, P20,000.00 as attorney’s fees and
insurance takes effect and at the time the loss occurs.
costs of suit.
The automatic assignment of the policy to CKS under the provision of the
8.   CA rendered a decision affirming the trial court decision, deleting however
lease contract previously quoted is void for being contrary to law and/or public
the awards for exemplary damages and attorneys fees. A motion for
policy. The proceeds of the fire insurance policy thus rightfully belong to the
reconsideration by United was denied on 29 March 1996.
spouses.
ISSUE:
DOCTRINE:
1.   Whether or not the paragraph 18 of the lease contract entered into between
The basis of such requirement of insurable interest in property insured is based
CKS and the Cha spouses is valid insofar as it provides that any fire
on sound public policy: to prevent a person from taking out an insurance policy
insurance policy obtained by the lessee Cha spouses over their merchandise
on property upon which he has no insurable interest and collecting the proceeds
inside the leased premises is deemed assigned or transferred to the lessor
of said policy in case of loss of the property.
CKS – NO, the provision is void, as against public policy. CKS has no
insurable interest over the merchandise.

RULING: WHEREFORE, the decision of the Court of Appeals in CA-G.R. CV


No. 39328 is SET ASIDE and a new decision is hereby entered, awarding the
proceeds of the fire insurance policy to petitioners Nilo Cha and Stella Uy-Cha. SO merchandise, without the consent of CKS, is a separate and distinct issue
ORDERED. which we do not resolve in this case.

RATIO:
 
1.   It is, of course, basic in the law on contracts that the stipulations contained
in a contract cannot be contrary to law, morals, good customs, public order
or public policy.
2.   Sec. 18 of the Insurance Code provides:
Sec. 18. No contract or policy of insurance on property shall be
enforceable except for the benefit of some person having an insurable
interest in the property insured.
3.   A non-life insurance policy such as the fire insurance policy taken by
petitioner-spouses over their merchandise is primarily a contract of
indemnity. Insurable interest in the property insured must exist at the time
the insurance takes effect and at the time the loss occurs. The basis of such
requirement of insurable interest in property insured is based on sound
public policy: to prevent a person from taking out an insurance policy
on property upon which he has no insurable interest and collecting the
proceeds of said policy in case of loss of the property.
4.   In such a case, the contract of insurance is a mere wager which is void
under Section 25 of the Insurance Code, which provides:
SECTION 25. Every stipulation in a policy of Insurance for the
payment of loss, whether the person insured has or has not any interest
in the property insured, or that the policy shall be received as proof of
such interest, and every policy executed by way of gaming or
wagering, is void.
5.   In the present case, it cannot be denied that CKS has no insurable interest in
the goods and merchandise inside the leased premises under the provisions
of Section 17 of the Insurance Code which provide.
Section 17. The measure of an insurable interest in property is the
extent to which the insured might be damnified by loss of injury
thereof."
6.   Therefore, respondent CKS cannot, under the Insurance Code a special law
be validly a beneficiary of the fire insurance policy taken by the petitioner-
spouses over their merchandise. This insurable interest over said
merchandise remains with the insured, the Cha spouses. The automatic
assignment of the policy to CKS under the provision of the lease
contract previously quoted is void for being contrary to law and/or
public policy. The proceeds of the fire insurance policy thus rightfully
belong to the spouses. The insurer (United) cannot be compelled to pay the
proceeds of the fire insurance policy to a person (CKS) who has no
insurable interest in the property insured.
7.   The liability of the Cha spouses to CKS for violating their lease contract in
that Cha spouses obtained a fire insurance policy over their own
008 DELSAN TRANSPORT LINES, INC. v. CA (CABUSORA) 61.   Caltex Philippines (Caltex) entered into a contract of affreightment with
November 15, 2001 | De Leon, Jr., J. | Measure of Indemnity the petitioner Delsan Transport Lines, Inc. (common carrier), for a one
year where Delsan would transport Caltex’s industrial fuel oil from the
PETITIONER: Delsan Transport Lines, Inc. Batangas-Bataan Refinery to different parts of the country.
RESPONDENTS: The Hon. Court Of Appeals and American Home Assurance 62.   Under the contract, Delsan took on board its vessel (MT Maysun)
Corporation 2,277.314 kiloliters of said oil to be transported to Caltex Oil Terminal in
Zamboanga City. Said shipment was insured with private respondent
SUMMARY: Caltex and Delsan entered into a contract of affreightment where American Home Assurance Corporation (AHAC).
Delsan would transport industrial fuel oil from the Batangas-Bataan Refinery to 63.   (14 Aug 1986) MT Maysun set sail to Zamboanga City.
Caltex Oil Terminal in Zamboanga City. Said shipment was insured by Caltex with 64.   (16 Aug 1986) MT Maysun sank near Panay Gulf and Cuyo East Pass in
AHAC. While in transit, Delsan’s ship (MT Maysun) sunk allegedly due to bad Visayas.
weather. AHAC paid Caltex for the lost shipment. When AHAC went after Delsan 65.   AHAC subsequently paid Caltex P5,096,635.57 (amount of insured cargo).
for reimbursement, the latter refused. Thus AHAC filed a complaint for complaint 66.   AHAC later demanded Delsan the same amount as an exercise of its right
for sum of money. The TC dismissed the case holding that Delsan was liable due to of subrogation under Art. 2207, New Civil Code.
force majeure. On appeal, the CA reversed the decision stating that there was no 67.   Delsan refused to pay, thus AHAC filed a complaint for collection of sum
force majeure. of money with the RTC, Makati.
The issues were (1) WON payment of AHAC to Caltex amounted to a and admission 68.   Trial Court: Dismissed the complaint reasoning that MT Maysun was
that MT Maysun was seaworthy and thus precluding any action AHAC may have seaworthy as determined by the Philippine Coast Guard per Survey
against Delsan (NO) and (2) WON the marine insurance policy in this case was Certificate Report No. M5-016-MH upon inspection during its annual
necessary (NO). dry-docking and that the incident was caused by unexpected inclement
Payment by AHAC to Caltex was only a waiver of AHAC’s right to enforce the weather condition or force majeure, thus exempting the common carrier
term of the implied warranty against Caltex under the marine insurance policy but (Delsan) from liability for the loss of its cargo.
does not mean admission of the vessel’s seaworthiness by AHAC as to foreclose 69.   CA: Reversed the TC Decision. CA gave credence to the weather report
recourse against Delsan for any liability under its contractual obligation as a issued by PAGASA which which showed that from 2-8 a.m. on 16 Aug
common carrier. The marine insurance policy is not necessary in this case and that 1986, the wind speed remained at 10-20 knots per hour while the waves
the subrogation receipt was already sufficient having showed both (1) relationship measured from 0.7-2 meters in height only in contrast to herein Delsan’s
of AHAC and Caltex, as insurer and assured, respectively and (2) amount paid to allegation that the waves were 20 ft. high.
settle the insurance claim. 70.   Delsan filed an MR which was however denied and hence this petition.
71.   Delsan invokes/alleges:
1.   Sec. 113, Insurance Code states that in every marine insurance upon a
DOCTRINE24: Payment by insurer to insured under a marine insurance policy ship or freight, or freightage, or upon any thing which is the subject of
means the waiver of the insurer’s right to enforce the term of the implied warranty marine insurance there is an implied warranty by the shipper that the
against the insured but does not mean waiver of the insurer’s right to subrogation. ship is seaworthy. Consequently, the insurer will not be liable to the
The presentation in evidence of the insurance policy is not indispensable before the assured for any loss under the policy in case the vessel would later on be
insurer may recover. The subrogation receipt, by itself, is sufficient to establish not found as not seaworthy at the inception of the insurance. The act of
only the relationship of the insurer and the insured, but also the amount paid to settle AHAC of paying Caltex the value of the lost cargo amounted to a tacit
the insurance. recognition that MT Maysun was seaworthy, otherwise, AHAC was not
legally liable to Caltex due to the latter’s breach of implied warranty under
the marine insurance policy that the vessel was seaworthy.
FACTS: 2.   CA erred in finding that MT Maysun was not seaworthy on the ground
                                                                                                                        that Francisco Berina, chief mate of the vessel, was not qualified. Under
24
 Contracts  of  marine  insurance  are  contracts  of  indemnity  which  means  its  purpose  is,  in  case  of  loss,  to   Sec. 116, Insurance Code, the implied warranty of seaworthiness of the
place   the   insured   in   the   same   situation   in   which   he   was   before   the   loss   subject   to   the   terms   and   vessel, which AHAC admitted as having been fulfilled by its payment of
conditions  of  the  policy.  Amount  of  indemnity  may  be  determined  after  the  loss  or  is  previously  fixed  in   the insurance proceeds to Caltex of its lost cargo, extends to the vessel’s
the   contract.   The   amount   of   insurance   fixed   in   the   policy   of   a   marine   insurance   is   not   the   exact   measure   complement. Despite Berina having merely a 2nd officer’s license, he was
of  indemnity  to  which  the  insured  is  entitled,  but  the  maximum  indemnity  which  he  might  obtain.  The   qualified to act as the vessel’s chief officer under Chapter IV(403),
insured  cannot  recover  in  excess  of  his  actual  loss.  (The  case  makes  no  mention  of  the  topic  this  case  was   Category III(a)(3)(ii)(aa) of the Philippine Merchant Marine Rules and
assigned   –   Measure   of   Indemnity.   This   doctrine   is   from   De   Leon,   and   it’s   the   closest   I   can   relate   the   case   Regulations. In fact, all the crew and officers of MT Maysun were
to.  Apologies.)  
exonerated in the administrative investigation conducted by the Board of company of the insurance claim. Consequently, the payment made by the
Marine Inquiry after the subject accident. AHAC (insurer) to Caltex (assured) operates as an equitable assignment to
3.   The doctrine in Home Insurance Corporation v. CA (i.e. failure of the the former of all the remedies which the latter may have against Delsan.
private respondent to present the insurance policy in evidence is allegedly 105.  From the nature of their business and for reasons of public policy, common
fatal to its claim inasmuch as there is no way to determine the rights of the
carriers are bound to observe extraordinary diligence in the vigilance over
parties thereto), should have been applied herein, since AHAC failed to
present the subject marine cargo insurance policy as evidence. the goods and for the safety of passengers transported by them, according to
all the circumstances of each case.
ISSUE/s: 1.   GR: In the event of loss, destruction or deterioration of the insured
10.   WON the payment made by the AHAC to Caltex for the insured value of goods, common carriers shall be responsible.
the lost cargo amounted to an admission that the vessel was seaworthy, thus Exception: If the same is brought about, among others, by flood,
precluding any action for recovery against the petitioner. – NO. Payment by storm, earthquake, lightning or other natural disaster or calamity.
AHAC to Caltex was only a waiver of AHAC’s right to enforce the term 2.   GR: If the goods are lost, destroyed or deteriorated, common
of the implied warranty against Caltex under the marine insurance policy carriers are presumed to have been at fault or to have acted
but does not mean admission of the vessel’s seaworthiness by AHAC as negligently.
to foreclose recourse against Delsan for any liability under its contractual Exception: If the common carrier proves that they observed
obligation as a common carrier. extraordinary diligence.
11.   WON the non-presentation of the marine insurance policy bars the 106.  Jaime Jarabe, captain of MT Maysun, and Berina testified that at around
complaint for recovery of sum of money for lack of cause of action. – NO. 3:15 a.m. a squall ("unos") carrying strong winds with an approximate
In this case, said insurance policy is not necessary. The subrogation receipt velocity of 30 knots per hour and big waves averaging 18-20 ft. high caused
was already sufficient and it showed both (1) relationship of AHAC and the vessel to sink. However, this tale was effectively rebutted PAGASA’s
Caltex, as insurer and assured, respectively and (2) amount paid to settle weather report. (See Fact #9) Thus, Delsan’s defense of force majeure
the insurance claim. cannot be sustained. Also, said witnesses as employees of Delsan, cannot be
expected to testify against the latter. (See Fact #11.2) Exoneration of MT
RULING: WHEREFORE, the instant petition is DENIED. The Decision dated June Maysun’s officers and crew to liability is only with respect to administrative
17, 1996 of the Court of Appeals in CA-G.R. CV No. 39836 is AFFIRMED. Costs liabilities and does not operate to absolve Delsan’s civil liabilities arising
against the petitioner. SO ORDERED. from its negligence as a common carrier. (See Fact #11.2)
107.  Evidence certificates that MT Maysun was fit for voyage at the time of
RATIO: dry-docking and inspection by the Philippine Coast Guard, was also not
103.  (First Issue) The payment made by the AHAC for the insured value of the credited by the court to prove the vessel’s fitness for voyage at the time of
lost cargo operates as waiver of its (AHAC) right to enforce the term of the the commencement of the voyage. “Seaworthiness relates to a vessel's
implied warranty against Caltex under the marine insurance policy. actual condition. Neither the granting of classification or the issuance of
However, the same cannot be validly interpreted as an automatic certificates establishes seaworthiness.”
admission of the vessel's seaworthiness by the AHAC as to foreclose 108.  (Second Issue) The marine insurance policy is not necessary in this case.
recourse against the Delsan for any liability under its contractual obligation The subrogation receipt, by itself, is sufficient to establish (1) the
as a common carrier. Said payment grants AHAC subrogatory right to relationship of AHAC, as insurer, and Caltex, as the assured shipper of
exercise legal remedies available to Caltex against Delsan, the common the lost cargo and (2) the amount paid to settle the insurance claim. The
carrier by virtue of Art. 2207, Civil Code25. right of subrogation accrues simply upon payment by the insurance
104.  The right of subrogation has its roots in equity. It is not dependent upon, company of the insu rance claim.
nor does it grow out of, any privity of contract or upon written 109.  The presentation of the insurance policy was necessary in the case of Home
assignment of claim. It accrues simply upon payment by the insurance Insurance Corporation v. CA because the shipment therein passed through
several stages with different parties involved in each stage. In that case, in
                                                                                                                        the absence of proof of stipulations to the contrary, the hauler can be liable
25
  Art.   2207.   If   the   plaintiff's   property   has   been   insured,   and   he   has   received   indemnity   from   the   only for any damage that occurred from the time it received the cargo until
insurance  company  for  the  injury  or  loss  arising  out  of  the  wrong  or  breach  of  contract  complained  of,  
the   insurance   company   shall   be   subrogated   to   the   rights   of   the   insured   against   the   wrongdoer   or   the  
it finally delivered it to the consignee. It cannot be held responsible for the
person  who  has  violated  the  contract.  If  the  amount  paid  by  the  insurance  company  does  not  fully  cover   handling of the cargo before it actually received it. The insurance contract,
the  injury  or  loss,  the  aggrieved  party   shall  be  entitled  to  recover  the  deficiency  from  the  person  causing   which was not presented in evidence in that case would have indicated the
the  loss  or  injury.  
scope of the insurer's liability, if any, since no evidence was adduced
indicating at what stage in the handling process the damage to the
cargo was sustained.
110.  In this case, there is no doubt that the cargo of oil belonging to Caltex was
lost while on board MT Maysun, which was owned by Delsan. Thus,
Delsan cannot deny its liability.
 
009 GAISANO CAGAYAN, INC. v. INSURANCE COMPANY OF Insurance Company failed to present sufficient evidence to prove its cause of
NORTH AMERICA (Callueng) action. Hence, the claim of subrogation to Gaisano Cagayan's case for recovery
June 8, 2006 | Austria-Martinez, J. | Measure of Indemnity of the amount of P535,613.00 for LSPI is fatal.

PETITIONER: Gaisano Cagayan, Inc. DOCTRINE: A vendor or seller retains an insurable interest in the property sold
RESPONDENTS: Insurance Company of North America so long as he has any interest therein, in other words, so long as he would suffer
by its destruction, as where he has a vendor's lien.
SUMMARY: IMC is the maker of Wrangler Blue Jeans. Levi Strauss (Phils.)  
Inc. LSPI is the local distributor of products bearing trademarks owned by Levi FACTS:
Strauss & Co.. IMC and LSPI separately obtained from Insurance Company of 1.   Intercapitol Marketing Corporation (IMC) is the maker of Wrangler Blue
North America fire insurance policies with book debt endorsements. The Jeans. Levi Strauss (Phils.) Inc. LSPI is the local distributor of products
insurance policies provide for coverage on "book debts in connection with bearing trademarks owned by Levi Strauss & Co.. IMC and LSPI separately
ready-made clothing materials which have been sold or delivered to various
obtained from Insurance Company of North America (Insurance Company)
customers and dealers of the Insured anywhere in the Philippines." The policies
fire insurance policies with book debt endorsements.
defined book debts as the "unpaid account still appearing in the Book of Account
2.   The insurance policies provide for coverage on "book debts in connection
of the Insured 45 days after the time of the loss covered under this Policy." with ready-made clothing materials which have been sold or delivered to
Gaisano Cagayan is a customer and dealer of the products of IMC and LSPI. The various customers and dealers of the Insured anywhere in the Philippines."
Gaisano Superstore Complex in Cagayan de Oro City, owned by Gaisano, was 3.   The policies defined book debts as the "unpaid account still appearing in the
consumed by fire. Included in the items lost or destroyed in the fire were stocks Book of Account of the Insured 45 days after the time of the loss covered
of ready-made clothing materials sold and delivered by IMC and under this Policy."
LSPI. Insurance Company filed a complaint for damages against Gaisano. It 4.   The policies also provide for the following conditions:
alleges that IMC and LSPI filed with Insurance Company their claims under 1.   Warranted that the Company shall not be liable for any unpaid account in respect of
their respective fire insurance policies with book debt endorsements; that there the merchandise sold and delivered by the Insured which are outstanding at the date
are unpaid accounts of Gaisano with IMC for P2,119,205.00 while with LSPI of loss for a period in excess of six (6) months from the date of the covering invoice
for P535,613.00; Insurance Company paid the claims of IMC and LSPI and, by or actual delivery of the merchandise whichever shall first occur.
2.   Warranted that the Insured shall submit to the Company within twelve (12) days
virtue thereof, , Insurance Company was subrogated to their rights against after the close of every calendar month all amount shown in their books of accounts
Gaisano. Gaisano Cagayan contends that it could not be held liable because the as unpaid and thus become receivable item from their customers and dealers.
property covered by the insurance policies were destroyed due to fortuities event 5.   Gaisano Cagayan, Inc. (Gaisano Cagayan) is a customer and dealer of the
or force majeure. The RTC dismissed Insurance Company’s complaint. But, the products of IMC and LSPI. On February 25, 1991, the Gaisano Superstore
CA rendered its decision setting aside the decision of the RTC. ISSUE: WoN Complex in Cagayan de Oro City, owned by Gaisano, was consumed by
Gaisano Cagayan is liable for the unpaid accounts. YES, because the insurance fire. Included in the items lost or destroyed in the fire were stocks of ready-
in this case is not for loss of goods by fire but for Gaisano Cagayan's accounts made clothing materials sold and delivered by IMC and LSPI.
with IMC and LSPI that remained unpaid 45 days after the fire. Gaisano 6.   On February 4, 1992, Insurance Company filed a complaint for damages
Cagayan's argument that it is not liable because the fire is a fortuitous event against Gaisano. It alleges that IMC and LSPI filed with , Insurance
under Article 1174 of the Civil Code is misplaced. As held earlier, Gaisano Company their claims under their respective fire insurance policies with
Cagayan bears the loss under Article 1504 (1) of the Civil Code. Moreover, it book debt endorsements; that as of February 25, 1991, the unpaid accounts
must be stressed that the insurance in this case is not for loss of goods by fire but of Gaisano on the sale and delivery of ready-made clothing materials with
for Gaisano Cagayan's accounts with IMC and LSPI that remained unpaid 45 IMC was P2,119,205.00 while with LSPI it was P535,613.00; that ,
days after the fire. Accordingly, Gaisano Cagayan's obligation is for the payment Insurance Company paid the claims of IMC and LSPI and, by virtue
of money. As correctly stated by the CA, where the obligation consists in the thereof, , Insurance Company was subrogated to their rights against
payment of money, the failure of the debtor to make the payment even by reason Gaisano; that , Insurance Company made several demands for payment
of a fortuitous event shall not relieve him of his liability. Thus, whether fire is a upon Gaisano but these went unheeded.
fortuitous event or Gaisano Cagayan was negligent are matters immaterial to this 7.   In its Answer with Counter Claim, Gaisano Cagayan contends that it could
case. What is relevant here is whether it has been established that Gaisano not be held liable because the property covered by the insurance policies
Cagayan has outstanding accounts with IMC and LSPI. With respect to IMC, the were destroyed due to fortuities event or force majeure; that , Insurance
Insurance Company has adequately established its claim. However, as to LSPI, Company right of subrogation has no basis inasmuch as there was no breach
of contract committed by it since the loss was due to fire which it could not deemed to be over credit since an insurance "on credit" belies not only the
prevent or foresee; that IMC and LSPI never communicated to it that they nature of fire insurance but the express terms of the policies; that it was not
insured their properties; that it never consented to paying the claim of the credit that was insured since Insurance Company paid on the occasion of
insured. the loss of the insured goods to fire and not because of the non-payment by
8.   The parties failed to arrive at an amicable settlement. Gaisano Cagayan of any obligation; that, even if the insurance is deemed as
9.   The RTC rendered its decision dismissing, Insurance Company’s one over credit, there was no loss as the accounts were not yet due since no
complaint. It held that the fire was purely accidental; that the cause of the prior demands were made by IMC and LSPI against Gaisano Cagayan for
fire was not attributable to the negligence of the Gaisano Cagayan; that it payment of the debt and such demands came from Insurance Company only
has not been established that Gaisano Cagayan is the debtor of IMC and after it had already paid IMC and LSPI under the fire insurance policies.
LSPI; that since the sales invoices state that "it is further agreed that merely 2.   Gaisano Cagayan avers that despite delivery of the goods, Gaisano
for purpose of securing the payment of purchase price, the above-described Cagayan-buyer IMC and LSPI assumed the risk of loss when they secured
merchandise remains the property of the vendor until the purchase price is fire insurance policies over the goods.
fully paid", IMC and LSPI retained ownership of the delivered goods and 3.   Gaisano Cagayan submits that there is no subrogation in favor of Insurance
must bear the loss. Company as no valid insurance could be maintained thereon by IMC and
10.   Dissatisfied, Gaisano Cagayan appealed to the CA. The CA rendered its LSPI since all risk had transferred to Gaisano Cagayan upon delivery of the
decision setting aside the decision of the RTC. goods; that Gaisano Cagayan was not privy to the insurance contract or the
11.   The CA held that the sales invoices are proofs of sale, being detailed payment between Insurance Company and its insured nor was its consent or
statements of the nature, quantity and cost of the thing sold; that loss of the approval ever secured; that this lack of privity forecloses any real interest on
goods in the fire must be borne by Gaisano Cagayan since the part of Insurance Company in the obligation to pay, limiting its interest
the proviso contained in the sales invoices is an exception under Article to keeping the insured goods safe from fire.
1504 (1) of the Civil Code, to the general rule that if the thing is lost by a 4.   Insurance Company counters that while ownership over the ready- made
fortuitous event, the risk is borne by the owner of the thing at the time the clothing materials was transferred upon delivery to Gaisano Cagayan, IMC
loss under the principle of res perit domino; that Gaisano Cagayan's and LSPI have insurable interest over said goods as creditors who stand to
obligation to IMC and LSPI is not the delivery of the lost goods but the suffer direct pecuniary loss from its destruction by fire; that Gaisano
payment of its unpaid account and as such the obligation to pay is not Cagayan is liable for loss of the ready-made clothing materials since it
extinguished, even if the fire is considered a fortuitous event; that by failed to overcome the presumption of liability under Article 1265 of the
subrogation, the insurer has the right to go against Gaisano Cagayan; that, Civil Code; that the fire was caused through Gaisano Cagayan's negligence
being a fire insurance with book debt endorsements, what was insured was in failing to provide stringent measures of caution, care and maintenance on
the vendor's interest as a creditor. its property because electric wires do not usually short circuit unless there
12.   Gaisano Cagayan filed a motion for reconsideration but it was denied by the are defects in their installation or when there is lack of proper maintenance
CA. and supervision of the property; that Gaisano Cagayan is guilty of gross and
evident bad faith in refusing to pay Insurance Company's valid claim and
ISSUE/s: should be liable to Insurance Company for contracted lawyer's fees,
1.   WoN Gaisano Cagayan is liable for the unpaid accounts. YES, because the litigation expenses and cost of suit.
insurance in this case is not for loss of goods by fire but for Gaisano 5.   At issue is the proper interpretation of the questioned insurance policy.
Cagayan's accounts with IMC and LSPI that remained unpaid 45 days Gaisano Cagayan claims that the CA erred in construing a fire insurance
after the fire. policy on book debts as one covering the unpaid accounts of IMC and LSPI
since such insurance applies to loss of the ready-made clothing materials
sold and delivered to Gaisano Cagayan.
RULING: WHEREFORE, the petition is partly GRANTED. The assailed Decision
dated October 11, 2000 and Resolution dated April 11, 2001 of the Court of Appeals 6.   Indeed, when the terms of the agreement are clear and explicit that they do
in CA-G.R. CV No. 61848 are AFFIRMED with the MODIFICATION that the not justify an attempt to read into it any alleged intention of the parties, the
terms are to be understood literally just as they appear on the face of the
order to pay the amount of P535,613.00 to Insurance Company is DELETED for
contract. Thus, what were insured against were the accounts of IMC and
lack of factual basis.
LSPI with Gaisano Cagayan which remained unpaid 45 days after the loss
through fire, and not the loss or destruction of the goods delivered.
RATIO:
1.   Gaisano Cagayan contends that the insurance in the present case cannot be
7.   Gaisano Cagayan argues that IMC bears the risk of loss because it expressly where the obligation consists in the payment of money, the failure of the
reserved ownership of the goods by stipulating in the sales invoices that debtor to make the payment even by reason of a fortuitous event shall not
"[i]t is further agreed that merely for purpose of securing the payment of the relieve him of his liability. The rationale for this is that the rule that an
purchase price the above described merchandise remains the property of the obligor should be held exempt from liability when the loss occurs thru a
vendor until the purchase price thereof is fully paid." fortuitous event only holds true when the obligation consists in the delivery
8.   The Court is not persuaded. of a determinate thing and there is no stipulation holding him liable even in
9.   The present case clearly falls under paragraph (1), Article 1504 of the Civil case of fortuitous event. It does not apply when the obligation is pecuniary
Code: in nature.
(1)   ART. 1504. Unless otherwise agreed, the goods remain at the seller's risk until the 16.   Thus, whether fire is a fortuitous event or Gaisano Cagayan was negligent
ownership therein is transferred to the buyer, but when the ownership therein is are matters immaterial to this case. What is relevant here is whether it has
transferred to the buyer the goods are at the buyer's risk whether actual delivery has
been made or not, except that: been established that Gaisano Cagayan has outstanding accounts with IMC
i.   Where delivery of the goods has been made to the buyer or to a bailee and LSPI.
for the buyer, in pursuance of the contract and the ownership in the 17.   With respect to IMC, the Insurance Company has adequately established its
goods has been retained by the seller merely to secure performance by
claim. Exhibits "C" to "C-22"show that Gaisano Cagayan has an
the buyer of his obligations under the contract, the goods are at the
buyer's risk from the time of such delivery; outstanding account with IMC in the amount of P2,119,205.00. Exhibit
10.   Thus, when the seller retains ownership only to insure that the buyer will "E"is the check voucher evidencing payment to IMC. Exhibit "F"is the
pay its debt, the risk of loss is borne by the buyer. Accordingly, Gaisano subrogation receipt executed by IMC in favor of Insurance Company upon
Cagayan bears the risk of loss of the goods delivered. receipt of the insurance proceeds. All these documents have been properly
11.   IMC and LSPI did not lose complete interest over the goods. They have an identified, presented and marked as exhibits in court. The subrogation
insurable interest until full payment of the value of the delivered goods. receipt, by itself, is sufficient to establish not only the relationship of
Unlike the civil law concept of res perit domino, where ownership is the Insurance Company as insurer and IMC as the insured, but also the amount
basis for consideration of who bears the risk of loss, in property insurance, paid to settle the insurance claim. The right of subrogation accrues simply
one's interest is not determined by concept of title, but whether insured has upon payment by the insurance company of the insurance claim. Insurance
substantial economic interest in the property. Company's action against Gaisano Cagayan is squarely sanctioned by
12.   Section 13 of our Insurance Code defines insurable interest as "every Article 2207 of the Civil Code which provides:
interest in property, whether real or personal, or any relation thereto, or (1)   Art. 2207. If the plaintiff's property has been insured, and he has received indemnity
from the insurance company for the injury or loss arising out of the wrong or breach
liability in respect thereof, of such nature that a contemplated peril might of contract complained of, the insurance company shall be subrogated to the rights
directly damnify the insured." Parenthetically, under Section 14 of the same of the insured against the wrongdoer or the person who has violated the contract.
Code, an insurable interest in property may consist in: (a) an existing 18.   Gaisano Cagayan failed to refute Insurance Company's evidence.
interest; (b) an inchoate interest founded on existing interest; or (c) an 19.   As to LSPI, Insurance Company failed to present sufficient evidence to
expectancy, coupled with an existing interest in that out of which the prove its cause of action. No evidentiary weight can be given to Exhibit "F
expectancy arises. Levi Strauss", a letter dated April 23, 1991 from Gaisano Cagayan's
13.   Indeed, a vendor or seller retains an insurable interest in the property sold General Manager, Stephen S. Gaisano, Jr., since it is not an admission of
so long as he has any interest therein, in other words, so long as he would Gaisano Cagayan's unpaid account with LSPI. It only confirms the loss of
suffer by its destruction, as where he has a vendor's lien. In this case, the Levi's products in the amount of P535,613.00 in the fire that razed Gaisano
insurable interest of IMC and LSPI pertain to the unpaid accounts appearing Cagayan's building on February 25, 1991.
in their Books of Account 45 days after the time of the loss covered by the 20.   Moreover, there is no proof of full settlement of the insurance claim of
policies. LSPI; no subrogation receipt was offered in evidence. Thus, there is no
14.   Gaisano Cagayan's argument that it is not liable because the fire is a evidence that Insurance Company has been subrogated to any right which
fortuitous event under Article 1174 of the Civil Code is misplaced. As held LSPI may have against Gaisano Cagayan. Failure to substantiate the claim
earlier, Gaisano Cagayan bears the loss under Article 1504 (1) of the Civil of subrogation is fatal to Gaisano Cagayan's case for recovery of the amount
Code. of P535,613.00.
15.   Moreover, it must be stressed that the insurance in this case is not for loss of
goods by fire but for Gaisano Cagayan's accounts with IMC and LSPI that
remained unpaid 45 days after the fire. Accordingly, Gaisano Cagayan's
obligation is for the payment of money. As correctly stated by the CA,
010 GLARAGA V. SUN LIFE ASSURANCE (CANDELARIA) 74.   Glaraga alleges that defendant O.O. Hanson, the insurance agent (Hanson)
December 14, 1926 | J. Johns | Premium Payment who insured Juares, was the same person who collected the first premiums.
75.   Juares, during his life, defaulted in paying his second premium, upon the
PETITIONER: SUSANA GLARAGA allegation that Hanson allegedly made written instructions not to send the
RESPONDENTS: SUN LIFE ASSURANCE COMPANY OF CANADA and O.O. premium because Hanson would pay the premium, subject to Juares’
HANSON reimbursement.
76.   Sunlife alleges that the terms of the policy provided that the premiums
SUMMARY: Sunlife executed a life insurance policy in favor of Juares. The terms should be paid on the 1st of June and December of each year, or within a
of the insurance policy provided specific powers and limitations which shall be grace period of not more than 30 days. However, Juares defaulted with the
complied with in order to bind Sunlife (see FACTS, par. 6). Juares paid the first second premium because no payment was made on the due date and
premium, but defaulted in the second premium because Hanson, the insurance agent, within the grace period. By reason of the default, the policy elapsed and
sent a letter informing Juares that Hanson will pay the second premium, subject to was of no legal force or effect at the time of death of Juares.
77.   * RELEVANT PROVISION: “No persons, except the President,
reimbursement from Juares. However, no payment was made. When Juares died,
Managing-Director or Secretary has power to alter this contract, to extend
Glaraga, as the administratrix, made a demand for the proceeds of the policy. Sunlife
the time for paying a premium, to bind the Company by making any
refused, alleging that the policy was no longer in effect because of the default in the
promise or by receiving any representation or information not contained
second premium. The trial court ruled in favor of Glaraga.
in the application for this policy. No payment made to any person, except
in exchange for the Company’s official receipt, will be recognized by the
The issue is whether or not the policy was still in effect upon Glaraga’s demand.
Company. This policy does not take effect until the first premium has been
actually paid, during the life and good health of the insured.”
The Supreme Court stated that the policy was no longer in effect, and ruled in favor
78.   Sunlife alleges that Hanson was not authorized or empowered to modify the
of Sunlife. The Court emphasized that when an insurance policy specifically defines
terms of the policy, and that Hanson is not an official of Sunlife. Neither did
and limits the powers and duties of an agent, it shall bind the parties. Non-
the company know that a promise was made by Hanson, and it never
compliance, as stated in this case, shall render the policy ineffective. When Hanson
received from him any payment for the second premium.
sent the letter to Juares assuming the obligation to pay the premium was contrary to
79.   The trial court ruled in favor of Glaraga. Hence, this appeal by Sunlife.
the terms of the policy. Adding to the fact that no one paid the second premium, the
policy officially lapsed after the 30-day grace period. Hanson’s promise bound only
ISSUE/s:
him and the insured party. Sunlife is absolved from liability, as agreed in the
1.   W/N the insurance policy was still in effect upon Glaraga’s demand – NO.
contract, and Hanson will be held liable to Glaraga.
When an insurance policy specifically defines and limits the powers and
duties of an agent, it shall bind the parties. Non-compliance, as stated in
DOCTRINE: Where a life insurance company issued and delivered one of its
this case, shall render the policy ineffective. Under the relevant provisions
policies to the insured in and by which the powers of its pecial agents were limited
of the contract, payment made to any person, except in exchange for the
and defined, and the policy specified how and in what manner the premiums were to
company’s official receipt would not be recognized by the company, and
be paid and to whom they were to paid, in the absence of allegation and proof of an
that no person except those provided, had any power to alter the contract or
established custom or another rule of conduct ratified and approved by the company
extend the time for the premium, or bind the company itself by making of
the premiums on the policy must be paid at the time and in the way and manner
any promise.
specified in the policy and be forfeited by its own terms.
RULING: The SC reversed the trial court’s decision, but affirmed against Hanson.
FACTS: There was a written contract, in the form of the policy, duly signed and accepted by
72.   Sun Life Assurance (Sunlife), a domestic corporation, issued a life both parties, which expressed the manner the premiums are to be paid, and the terms
insurance policy on the life of Juares by which it promised to pay Php 5K to which specifies and limits the powers and dutiesof the agent to the delivery of the
the legal representatives of the insured, upon the condition that the policy official receipt of the company upon the payment to him of the amount of the
was in legal force and effect at the time of death of the insured. Juares died. premium. Without compliance with such terms, the subsequent acts after the
73.   Glaraga, the insured, was appointed as the administratrix of his estate, and perfection of the contract will have no effect; and non-compliance with the grace
she made a demand against Sunlife. Sunlife refused to pay, which led to period, as expressly stated, extinguished the conract.
Glaraga filing for collection and damages.
method by which Hanston collects premiums which would be binding upon
RATIO: the company.  
1.   It is conclusive that the second premium was never paid to Sunlife.   10.   Failure to comply with the contract after the default, the insurance policy
2.   The written promise by Hanson stated that Hanson assumed payment of the was no longer binding and effective against the Sunlife. However, Hanson
second premium, subject to the reimbursement from Juares.   is bound because he promised and agreed to premium. It is an established in
3.   On the other hand, the insurance policy, in writing and issued to Juares, the law on agency that when an agent acts beyond the authority given to
provided for 30 days grace period. If the insured party defaults and fails to him by the principal, it shall be binding on him as if he had entered into the
pay within the grace period, or if the balance due at the end of the term of contract by himself. Therefore, damage resulting to a third party, such as
the policy exceeds the coverage of the premium, then the policy shall lapse Glaraga, must be enforceable against him.  
and become void.    
4.   By its express terms, the non-payment of any premium when due or within  
the 30-day grace period, ipso facto causes the policy to lapse, and relieves    
the insurance company from all liability.  
5.   Under the relevant provision (see FACTS, par. 6), a payment made to any
person, except in exchange for the company’s official receipt would not be
recognized by the company, and that no person except those provided, had
any power to alter the contract or extend the time for the premium, or bind
the company itself by making of any promise.  
6.   In this case, there is no claim that in the writing of Hanson, he claimed that
he was acting for, or representing, the company, either as its agent or
otherwise. Juares relied on Hanson’s promise to pay the premium. But that
promise of Hanson, and of Hanson only, and was made by him to Juares,
and was made by him only.  
7.   There is no evidence that Sunlife ever ratified or approved Hanson’s letter,
or that it ever knew that it was written. It was not proven that Hanson or
anyone else paid the premium, or that Sunlife extended the time for its
payment. The letter was not the act of company or even that of its agent,
and even if it purported to be the act of the agent of the company, it
might well be doubted whether it would be binding upon the company,
in the absence of ratification or approval.  
8.   There is a marked distinction between the legal force and effect of the
powers and duties of a life insurance agent in soliciting insurance, and what
he says and does before the policy is issued, and what he says and does after
the policy is issued.  
a.   No written contract between the insured and insurer – insurance is
stopped to deny the authority of its agent as to acts previous to and
upon issuance of the policy; or  
b.   With written contract – specifically defined and limiting the
powers the duties of the agent, agent must comply to bind the
insurer  
9.   If Hansons was vested with all the powers of a general agent of the
company, such contract made by him would then be binding upon the
company, but there is no claim or pretense that Hanson was anything more
than a special agent with limited powers and dutes as the receipt of
premiums which are specifically defined by the express provisions of
the policy. Neither is there proof of an established custom as to manner or
011 ARCE v. CAPITAL INSURANCE (Castillo) 3.   November 27, 1965 - COMPANY sent to the INSURED Renewal
September 30, 1982 | Abad Santos, J. | Payment of premiums necessary for Certificate No. 47302 to cover the period December 5, 1965 to December 5,
effectivity of policy 1966. The COMPANY also requested payment of the corresponding
premium in the amount of P38.10.
4.   Anticipating that the premium could not be paid on time, the INSURED,
PLAINTIFF-APPELLEE: Pedro Arce thru his wife, promised to pay it on January 4, 1966. The COMPANY
DEFENDANT-APPELLANT: The Capital Insurance & Surety Co., Inc. accepted the promise but the premium was not paid on January 4, 1966. On
January 8, 1966, the house of the INSURED was totally destroyed by fire.
SUMMARY: The appellee owned a residential house which was insured with 5.   January 10, 1966 - NSURED's wife presented a claim for indemnity to the
the appellant COMPANY since 1961. In November 1965, the COMPANY sent COMPANY. She was told that no indemnity was due because the premium
to the INSURED a Renewal Certificate to cover the period from December 5, on the policy was not paid.
1965 to December 5,1966, and requested payment of the corresponding 6.   Nonetheless the COMPANY tendered a check for P300.00 as financial aid
premium. Anticipating that the premium could not be paid on time, the which was received by the INSURED's daughter, Evelina R. Arce. The
INSURED asked for an extension which was granted by the COMPANY. After voucher for the check which Evelina signed stated that it was "in full
the lapse of the requested extension, INSURED still failed to pay the premium. settlement (ex gratia) of the fire loss under Claim No. F-554 Policy No. F-
Thereafter, the house of the INSURED was totally destroyed by fire. Upon 24202.”
INSURED's presentation of claim for indemnity, he was told that no indemnity 7.   INSURED and his wife went to the office of the COMPANY to have his
was due because the premium was not paid. The INSURED sued the signature on the check identified preparatory to encashment.
COMPANY for indemnity. The trial court held the COMPANY liable to 8.   At that time the COMPANY reiterated that the check was given "not as
indemnify the INSURED on the ground that since the COMPANY could have an obligation, but as a concession" because the renewal premium had
demanded payment of the premium, mutuality of obligation required that it not been paid. The INSURED cashed the check but then sued the
should be liable on the policy. Issue is WoN the Company is liable to indemnify COMPANY on the policy.
the Insured even if no premium was paid. The Court ruled in the negative 9.   The court a quo held that since the COMPANY could have demanded
stating that Section 72 of the Insurance Act as amended by R.A.. 3540 states payment of the premium, mutuality of obligation requires that it should
that "no policy issued by an insurance company is valid and binding unless and also be liable on its policy. The court a quo also held that the INSURED
until the premium thereof has been paid.” Because of this amendment, the legal was not bound by the signature of Evelina on the check voucher because he
regime changed in that unless the premium is paid there is no insurance. did not authorize her to sign the waiver.
DOCTRINE: It is obvious from both the Insurance Act, as amended, and the
ISSUES:
stipulation of the parties that time is of the essence in respect to the payment of
1.   WoN the Company is liable to indemnify the Insured even if no premium
the insurance premium so that if it is not paid the contract does not take effect
was paid – NO because Section 72 of the Insurance Act as amended by
unless there is still another stipulation to the contrary. In the instant case, the
R.A.. 3540 states that "no policy issued by an insurance company is valid
INSURED was given a grace period to pay the premium but the period
and binding unless and until the premium thereof has been paid."
having expired with no payment made, he cannot insist that the
COMPANY is nonetheless obligated to him.  
RULING: WHEREFORE, the decision of the court a quo is reversed; the appellee's
complaint is dismissed. No special pronouncement as to costs.
 
FACTS:
1.   In Civil Case No. 66466 of the Court of First Instance of Manila, the RATIO:
Capital Insurance and Surety Co., Inc., (COMPANY) was ordered to pay 1.   Capital Insurance and Surety Co., Inc. vs. Delgado: appellants failed to pay
Pedro Arce (INSURED) the proceeds of a fire insurance policy. Not a balance of P583.95 on the premium charges due, notwithstanding
satisfied with the decision, the company appealed to this Court on questions demands made upon them. As with the issuance of the policy to appellants
of law. the same became effective and binding upon the contracting parties, the
2.   The INSURED was the owner of a residential house in Tondo, Manila, latter can not avoid the obligation of paying the premiums agreed upon. It is
which had been insured with the COMPANY since 1961 under Fire Policy clear from the foregoing that appellants are under obligation to pay the
No. 24204 amount sued upon.
2.   Sec. 72 of the Insurance Act, as amended by R.A. No. 3540: SEC. 72. An
insurer is entitled to payment of premium as soon as the thing insured is
exposed to the perils insured against, unless there is clear agreement to
grant credit extension for the premium due. No policy issued by an
insurance company is valid and binding unless and until the premium
thereof has been paid.
3.   Parties in this case had stipulated: notwithstanding anything to the
contrary contained in the within policy, this insurance will be deemed valid
and binding upon the Company only when the premium and documentary
stamps therefor have actually been paid in full and duly acknowledged in an
official receipt signed by an authorized official/representative of the
Company.
4.   It is obvious from both the Insurance Act, as amended, and the stipulation
of the parties that time is of the essence in respect of the payment of the
insurance premium so that if it is not paid the contract does not take effect
unless there is still another stipulation to the contrary.
5.   In the instant case, the INSURED was given a grace period to pay the
premium but the period having expired with no payment made, he cannot
insist that the COMPANY is nonetheless obligated to him.
6.   Delgado was decided in the light of the Insurance Act before Sec. 72 was
amended. Prior to the amendment, an insurance contract was effective even
if the premium had not been paid so that an insurer was obligated to pay
indemnity in case of loss and correlatively he had also the right to sue for
payment of the premium.
7.   But the amendment to Sec. 72 has radically changed the legal regime in
that unless the premium is paid there is no insurance.
8.   With the foregoing, it is not necessary to dwell at length on the trial court's
second proposition that the INSURED had not authorized his daughter
Evelina to make a waiver because the INSURED had nothing to waive; his
policy ceased to have effect when he failed to pay the premium.
   
012 MAKATI TUSCANY CONDOMINIUM v. CA (CASTRO) March 1983, with a total premium of P466,103.05.
November 6, 1992 | Bellosillo, J. | Insurance Premium
2.   The premium was paid on installments on 12 March 1982, 20 May 1982, 21
PETITIONER: MAKATI TUSCANY CONDOMINIUM CORPORATION June 1982 and 16 November 1982, all of which were accepted by AHAC
RESPONDENTS: THE COURT OF APPEALS, AMERICAN HOME
ASSURANCE CO., represented by American International Underwriters 3.   On 10 February 1983, AHAC issued to Makati Tuscany an insurance policy
(Phils.), Inc., which replaced and renewed the previous policy, for a term covering
INSURANCE POLICY 2: 1 March 1983 to 1 March 1984. The premium
SUMMARY: Basically, this case involves 3 insurance policies (1982-1983, in the amount of P466,103.05 was again paid on installments on 13 April
1983-1984, and the source of the controversy 1984-1985). In these insurance 1983, 13 July 1983, 3 August 1983, 9 September 1983, and 21 November
policies, Makati Tuscany pays its premiums by installments which were 1983. All payments were likewise accepted
accepted by American Home Assuance. For the 3rd insurance policy, Makati
Tuscany refused to complete its payment of premium by claiming that because 4.   On 20 January 1984, the policy was again renewed for the period
of the arrangement of paying in installments, then the insurance policy was no INSURANCE POLICY 3: 1 March 1984 to 1 March 1985. On this
valid and binding because the said arrangement is proscribed by the insurance renewed policy, Makati Tuscany made two installment payments
code. Hence, American Home filed a collection case against Makati Tuscany for (amounting to 152,000), both accepted by AHAC. After the two
the payment of balance while Makati filed for a counterclaim refunding the said installments, Makati Tuscany refused to pay the balance of the premium
premiums paid. RTC ruled that he insurance policy is valid and that Makati
Tuscany should pay the balance which was affirmed by the CA with some 5.   Consequently, AHAC filed an action to recover the unpaid balance
modifications. CA further stated that Makati Tuscany cannot deny liability on amounting 314,103.05 (arrived by deducting the installments paid which is
the payment of other premiums while AHAC cannot also deny liability on the 152,000 from the amount of the policy which is 466,103.05)
insurance because it agreed with the set-up and it accepted the installment 6.   In its answer, Makati Tuscany admitted the issuance of the insurance policy.
payments. The issue now before the SC is W/N an arrangement to pay the It explained that it discontinued the payment premiums because the policy
premiums in installment will invalidate the contract of insurance and SC said did not contain a credit clause in its favor and the receipts of the installment
NO, because what merely the law precludes is the stipulation making the payments covering the policy for 1984-1985, as well as the two previous
contract valid despite non-payment of premiums at all. The installment policies, stated the following reservations:
arrangement is not contrary to law, morals, good customs, or public policy. a.   Acceptance of this payment shall not waive any of the company
rights to deny liability on any claim under the policy arising
DOCTRINE: Section 77 merely precludes the parties from stipulating that the before such payments or after the expiration of the credit clause of
policy is valid even if premiums are not paid, but does not expressly prohibit an the policy; and
agreement granting credit extension, and such an agreement is not contrary to b.   Subject to no loss prior to premium payment. If there be any
morals, good customs, public order or public policy). So is an understanding to loss such is not covered.
allow insured to pay premiums in installments not so proscribed. At the very
least, both parties should be deemed in estoppel to question the arrangement they 7.   Makati Tuscany claimed that the policy was never binding and valid, and no
have voluntarily accepted. risk attached to the policy. It then pleaded a counterclaim for 152,000
refund for the premiums already paid for 1984-1985. Later on it amended
  the counterclaim, sought the refund of 924,206 representing the premium
FACTS: payments for 1982-1985.
INSURED: (Petitioner in the SC but the Defendant in the lower court case):
MANILA TUSCANY CONDOMINIUM 8.   After some incidents, Makati Tuscany and AHAC moved for summary
INSURER: AHAC through American Underwriters (Plaintiff in the lower judgment
court)
1.   Sometime in early 1982, private respondent American Home Assurance Co. 9.   RTC dismissed the complaint and the counterclaim upon the following
(AHAC), represented by American International Underwriters (Phils.), Inc., fiindings:
issued in favor of petitioner Makati Tuscany Condominium Corporation a.   While it is true that the receipts issued to Makati Tuscany
(TUSCANY) an insurance policy on the latter's building and premises, for a contained the aforementioned reservations, it is equally true that
period beginning INSURANCE POLICY 1: 1 March 1982 and ending 1
payment of the premiums of the three aforementioned policies
(being sought to be refunded) were made during the lifetime or 12.   Makati Tuscany now asserts that its payment by installment of the
term of said policies, hence, it could not be said, inspite of the premiums for the insurance policies for 1982, 1983 and 1984 invalidated
reservations, that no risk attached under the policies. said policies because of the provisions of Sec. 77 of the Insurance Code26,
Consequently, Makati Tuscany's counterclaim for refund is not as amended, and by the conditions stipulated by the insurer in its receipts,
justified. disclaiming liability for loss for occurring before payment of premiums.
b.   As regards the unpaid premiums, in view of the reservation in the
receipts ordinarily issued by AHAC on premium payments the 13.   Hence this petition
only plausible conclusion is that AHAC has no right to demand
their payment after the lapse of the term of said policy on
March 1, 1985. Therefore, the defendant was justified in refusing ISSUE/s:
to pay the same. 1.   W/N the payment by installments of the premiums due on an insurance
policy invalidates a contract of insurance in view of Sec. 77 of PD 612 –
10.   Both parties appealed from the judgment of the trial court. Thereafter, the No,, Sec 77 merely precludes the parties from stipulating that the policy is
Court of Appeals rendered a decision modifying that of the trial court by valid even if premiums are not paid, but does not expreslly prohibit
ordering Makati Tuscany to pay the balance of the premiums due on granting credit extension in a form of payment by installments.
policy plus legal interest until fully paid, and affirming the denial of the
counterclaim. RULING: WHEREFORE, finding no reversible error in the judgment appealed
from, the same is AFFIRMED. Costs against petitioner.
11.   CA explained the decision by saying:
a.   The obligation to pay premiums when due is ordinarily as SO ORDERED.
indivisible obligation to pay the entire premium. Here, the parties
herein agreed to make the premiums payable in installments, and
there is no pretense that the parties never envisioned to make the RATIO:
insurance contract binding between them. It was renewed for two SUBJECT POLICIES ARE VALID DESPITE INSTALLMENT PAYMENTS
succeeding years, the second and third policies being a 1.   The records clearly show that Makati Tuscany and AHAC intended subject
renewal/replacement for the previous one. And the insured never insurance policies to be binding and effective notwithstanding the staggered
informed the insurer that it was terminating the policy because the payment of the premiums. The initial insurance contract entered into in
terms were unacceptable. 1982 was renewed in 1983, then in 1984. In those three (3) years, the
insurer accepted all the installment payments. Such acceptance of
b.   While it may be true that under Section 77 of the Insurance Code, payments speaks loudly of the insurer's intention to honor the policies
the parties may not agree to make the insurance contract valid it issued to Makati Tuscany. Certainly, basic principles of equity and
and binding without payment of premiums, there is nothing in fairness would not allow the insurer to continue collecting and
said section which suggests that the parties may not agree to accepting the premiums, although paid on installments, and later deny
allow payment of the premiums in installment, or to consider liability on the lame excuse that the premiums were not prepared in
the contract as valid and binding upon payment of the first full.
premium. Otherwise, the court would allow the insurer to renege
on its liability under the contract, had a loss incurred (sic) before                                                                                                                        
completion of payment of the entire premium, despite its voluntary 26
  Sec.   77.   An   insurer   is   entitled   to   the   payment   of   the   premium   as  
acceptance of partial payments, a result eschewed by a basic
considerations of fairness and equity.
soon   as   the   thing   is   exposed   to   the   peril   insured   against.  
Notwithstanding  any  agreement  to  the  contrary,  no  policy  or  contract  
c.   the insurance contract became valid and binding upon payment of of   insurance   issued   by   an   insurance   company   is   valid   and   binding  
the first premium, and AHAC could not have denied liability on unless   and   until   the   premium   thereof   has   been   paid,   except   in   the  
the ground that payment was not made in full, for the reason
that it agreed to accept installment payment. case   of   a   life   or   an   industrial   life   policy   whenever   the   grace   period  
provision  applies  
2.   While the import of Section 77 is that prepayment of premiums is strictly
required as a condition to the validity of the contract, The court is not
prepared to rule that the request to make installment payments duly
approved by the insurer, would prevent the entire contract of insurance
from going into effect despite payment and acceptance of the initial
premium or first installment. Section 78 of the Insurance Code in effect
allows waiver by the insurer of the condition of prepayment by making an
acknowledgment in the insurance policy of receipt of premium as
conclusive evidence of payment so far as to make the policy binding despite
the fact that premium is actually unpaid.
3.   Section 77 merely precludes the parties from stipulating that the policy
is valid even if premiums are not paid, but does not expressly prohibit
an agreement granting credit extension, and such an agreement is not
contrary to morals, good customs, public order or public policy). So is
an understanding to allow insured to pay premiums in installments not
so proscribed. At the very least, both parties should be deemed in
estoppel to question the arrangement they have voluntarily accepted.
4.   Lastly, Makati Tuscany’s reliance on Arce v. Capital Surety is unavailing,
for the facts therein are substantially different because there was no
premium paid at all in Arce. While in this case Makati Tuscany paid the
initial installment and thereafter made staggered payments.
5.   It appearing from the peculiar circumstances that the parties actually
intended to make three (3) insurance contracts valid, effective and
binding, Makati Tuscany may not be allowed to renege on its obligation
to pay the balance of the premium after the expiration of the whole
term of the third policy (No. AH-CPP-9210651) in March 1985.
6.   Moreover, as correctly observed by the appellate court, where the risk
is entire and the contract is indivisible, the insured is not entitled to a
refund of the premiums paid if the insurer was exposed to the risk
insured for any period, however brief or momentary.
   
013 Areola v. CA (COSCOLLUELA) same credited “a return premium of P1,609.65 plus documentary stamps
September 22, 1994 | Romero, J. | Premium Payment and premium tax” to the account of Areola.
84.   Shocked, Areola confronted the representative of Prudential, Carlito Ang.
PETITIONER: Santos B. Areola and Lydia D. Areola Areola still failed to receive any official receipt from Prudential
RESPONDENTS: Court of Appeals, Prudential Guarantee and Assurance, Inc. 85.   Areola sent a letter demanding from Prudential that he be insured under the
same terms and conditions of his previous policy commencing upon the
receipt of his letter, or that the current commercial rate of increase on the
SUMMARY: 7 months after the issuance of Santos Areola’s Personal Accident payment he had made under the provisional receipt be return within 5 days.
Insurance Policy, Prudential unilaterally cancelled the same since company records 86.   Areola received a letter from production manager Malapit, informing him of
revealed that Areola failed to pay his premiums evidenced by the lack of Official the “partial payment” of P1,000 he had made on the policy had been
Receipt which was supposed to be issued within 7 days after the Provisional Receipt. exhausted pursuant to the provisions of the Short Period Rate Scale. He was
Prudential offered to reinstate same policy it had previously cancelled and even also warned that failure to pay the balance, the company’s liability would
proposed to extend its lifetime, upon a finding that the cancellation was erroneous cease to operate.
and that the premiums were paid in full by Areola but were not remitted by Teofilo 87.   Prudential, in response to Areola’s letter, wrote a letter to the latter stating
M. Malapit, Prudential’s Baguio City branch manager. Areola and his wife filed a that the company was verifying whether the payment had in fact been
complaint with the trial court which rendered a decision in favor of them but this was remitted to the company and why no official receipt had been issued
reversed by the CA. therefor. Asst. Vice-President Mariano Ampil III emphasized that the OR
should have been issued 7 days from the issuance of the provisional receipt,
The issue is WoN Prudential should be held liable for the fraudulent act of Malapit. and because there was no such OR, there was reason to believe that no
payment had been made.
Yes, because a corporation acts solely thru its employees. The latter’s acts are 1.   Ampil apologized for the inconvenience and said they will hold Areola
considered as its own for which it can be held to account. It is beyond doubt that covered under the terms of the policy until such time that the matter is
Malapit represented the interests of Prudential and acted in its behalf. His act of cleared.
receiving the premiums collected is well within the provice of his authority. The fact 2.   Ampil wrote another letter confirming the total amount was received and
that Prudential was itself defrauded due to the anomalies that took place in its Baguio that Prudential was amenable to extending the policy up to December 17,
branch office, such as the non-accrual of said premiums to its account, does not free 1985.
the same from its obligation to Areola. Consequently, Prudential is liable by way of 88.   Unfortunately, as early as August 6, 1985 Areola and his wife already filed
damages for the fraudulent acts committed by Malapit that gave occasion to the a breach of contract with damages.
erroneous cancellation of subject insurance policy. Its earlier act of reinstating the 89.   Prudential admitted that the cancellation was due to Malapit’s failure to turn
insurance policy also can not obliterate the injury. over the premiums. However, it argued that by acknowledging the
inconvenience and taking steps to rectify its omission, Prudential had
DOCTRINE: A corporation acts solely thru its employees. The latter’s acts are complied with its obligation. Hence, Areola has no longer a cause of action.
considered as its own for which it can be held to account. 90.   Trial court ruled in favor of Areola ruling that Prudential was in bad faith in
unilaterally cancelling the insurance policy. Had Areola met an accident,
Prudential would certainly have disclaimed liability. It held that Prudential
FACTS:
breached its contract with Areola.
80.   Santos Areola (Areola), a lawyer from Dagupan bought from Prudential
91.   CA reversed the trial court. It held that Prudential was not motivated by bad
Guarantee and Assurance, Inc. (Prudential) a personal accident insurance
faith or malice and that the cancellation was based on what the existing
policy covering the one-year period between noon of November 28, 1984
records merely showed. In fact, it’s subsequent acts of rectifying the wrong
and noon of November 28, 1985.
indicate that Prudential did not ct precipitately or willfully to inflict a wrong
81.   Areola was allegedly not able to pay the total amount of P1,609.65 which
on Areola.
included the premium, documentary stamp and premium tax.
92.   Hence, this appeal.
82.   On December 17, 1984, Prudential issued a collector’s provisional receipt
93.   Areola argues that it was Malapit’s misappropriation of the premiums
to Areola for the total amount. It noted thereon that the collector’s
which was the proximate cause of the cancellation of the insurance policy.
provisional receipt will be confirmed by their official receipt.
Malapit’s act of signing and even sending the notice of cancellation himself,
83.   The Branch Manager of Baguio City sent Areola an Endorsement which
notwithstanding his personal knowledge of Areola’s full payment of
“cancelled flat” his insurance policy for non-payment of premium. The
premiums, further reinforces the allegation of bad faith. Subsequent and a creditor of the other, such that the obligation of one is dependent upon
reinstatement could not possibly absolve Prudential from liability, there the obligation of the other.
being an obvious breach of contract. 118.  Under the law governing reciprocal obligations the injured party is given a
94.   Prudential however argues that where reinstatement, the equitable relief choice between fulfillment or rescission of the obligation. The law also
sought by Areola was granted at an opportune moment i.e. prior to the filing entitles Areola to payment of damages, regardless of whether he demands
of the complaint, he was left without a cause of action fulfillment or rescission.
119.  The nature of the damages is in the form of nominal damages contrary to
ISSUE/s: what was awarded by the trial court because o actual or substantial damage
111.  WoN Prudential should be held liable for its agent’s fraudulent act of or injury was inflicted on Areola at the time the insurance policy was
misappropriating the premiums paid by Areola – YES, because a cancelled. (Areola was awarded actual, moral, exemplary, and atty’s fees)
corporation acts solely thru its employees. The latter’s acts are considered
as its own for which it can be held to account.

RULING: WHEREFORE, the petition for review on certiorari is


hereby GRANTED and the decision of the Court of Appeals in CA-G.R. No. 16902  
on May 31, 1990, REVERSED. The decision of Branch 40, RTC Dagupan City, in    
Civil Case No. D-7972 rendered on June 30, 1987 is hereby REINSTATED subject
to the following modifications: (a) that nominal damages amounting to P30,000.00
be awarded petitioner in lieu of the damages adjudicated by court a quo; and (b) that
in the satisfaction of the damages awarded therein, respondent insurance company is
ORDERED to pay the legal rate of interest computed from date of filing of
complaint until final payment thereof.

SO ORDERED.

RATIO:
112.  Malapit’s fraudulent act of misappropriating the premiums paid by Areola
is beyond doubt directly imputable to Prudential. A corporation acts solely
thru its employees. The latter’s acts are considered as its own for which it
can be held to account.
113.  It is beyond doubt that Malapit represented the interests of Prudential and
acted in its behalf. His act of receiving the premiums collected is well
within the provice of his authority.
114.  Malapit’s failure to remit the premiums he received cannot constitute as a
defense for Prudential.
115.  The fact that Prudential was itself defrauded due to the anomalies that took
place in its Baguio branch office, such as the non-accrual of said premiums
to its account, does not free the same from its obligation to Areola.
Consequently, Prudential is liable by way of damages for the fraudulent acts
committed by Malapit that gave occasion to the erroneous cancellation of
subject insurance policy.
116.  Its earlier act of reinstating the insurance policy also can not obliterate the
injury.
117.  Prudential should be reminded that a contract of insurance creates reciprocal
obligations for both insurer and insured. Reciprocal obligations are those
which arise from the same cause and in which each party is both a debtor
014 TIBAY v. CA (Cruz) FACTS:
May 24, 1996 | Bellosillo, J. | Partial Payment of premiums 75.   Fortune Life and General Insurance Co., Inc. (FORTUNE) issued
Petitioners: Sps. Antonio A. Tibay & Violeta R. Tibay and Ofelia Roraldo, Fire Insurance Policy No. 136171 in favor of Violeta R. Tibay (Violeta)
Victorina Roraldo, Virgilio Roraldo, Myrna Roraldo and Rosabella Roraldo and/or Nicolas Roraldo (Nicolas) on their two-storey residential building
Respondent: CA and Fortune Life & General Insurance Co., Inc., located at 5855 Zobel Street, Makati City, together with all their personal
effects therein. The insurance was for P600,000.00 covering the period from
SUMMARY: FORTUNE issued Fire Insurance Policy in favor of Violeta 23 January 1987 to 23 January 1988. On 23 January 1987, of the total
and/or Nicolas on their two-storey residential building. The insurance was for premium of P2,983.50, Violeta Tibay only paid P600.00 thus leaving a
P600,000.00. Violeta only paid P600.00 out of the total premium of P2,983.50, considerable balance unpaid.
thus leaving a considerable balance unpaid. The insured building was completely 76.   The insured building was completely destroyed by fire. Two days later
destroyed by fire. Two days later Violeta paid the balance of the premium. On Violeta Tibay paid the balance of the premium. On the same day, she filed
the same day, she filed with FORTUNE a claim on the fire insurance policy In a with FORTUNE a claim on the fire insurance policy.
letter FORTUNE denied the claim of Violeta for violation of Policy Condition a.   Her claim was accordingly referred to its adjuster, Goodwill
No. 2 and of Sec. 77 of the Insurance Code. Violeta et al sued FORTUNE for Adjustment Services, Inc. (GASI), which immediately wrote
damages. The trial court ruled for Violeta et al and adjudged FORTUNE liable Violeta requesting her to furnish it with the necessary documents
for the total value of the insured building and personal properties The CA for the investigation and processing of her claim. To which Violeta
reversed this. The issue in this case is WoN fire insurance policy is valid, forthwith complied.
binding and enforceable upon mere partial payment of premium. The SC 77.   On 28 March 1987 she signed a non-waiver agreement with GASI to the
ruled in the negative. The Fire Insurance Policy provides for payment of effect that any action taken by the companies or their representatives in
premium in full. Accordingly, where the premium has only been partially paid investigating the claim made by the claimant for his loss which occurred at
and the balance paid only after the peril insured against has occurred, the 5855 Zobel Roxas, Makati on March 8, 1987, or in the investigating or
insurance contract did not take effect and the insured cannot collect at all on the ascertainment of the amount of actual cash value and loss, shall not waive
policy. The stipulations in the policy explicitly worded and taken in conjunction or invalidate any condition of the policies of such companies held by said
with Sec. 77 of the Insurance Code the payment of partial premium by the claimant, nor the rights of either or any of the parties to this agreement,
assured in this particular instance should not be considered the payment required and such action shall not be, or be claimed to be, an admission of liability
by the law and the stipulation of the parties. Rather, it must be taken in the on the part of said companies or any of them.
concept of a deposit to be held in trust by the insurer until such time that the full 78.   In a letter FORTUNE denied the claim of Violeta for violation of Policy
amount has been tendered and duly receipted for. In other words, as expressly Condition No. 2 and of Sec. 77 of the Insurance Code. Efforts to settle the
agreed upon in the contract, full payment must be made before the risk occurs case before the Insurance Commission proved futile.
for the policy to be considered effective and in force. Moreover, The case 79.   Violeta and the other petitioners sued FORTUNE for damages in the
of South Sea Surety and Insurance Company, Inc. v. Court of Appeals, speaks amount of P600,000.00 representing the total coverage of the fire insurance
only of two (2) statutory exceptions to the requirement of payment of the entire policy plus 12% interest per annum, P 100,000.00 moral damages, and
premium as a prerequisite to the validity of the insurance contract. These attorneys fees equivalent to 20% of the total claim.
exceptions are: a) in case the insurance coverage relates to life or industrial life 80.   The trial court ruled for Sps Tibay and adjudged FORTUNE liable for the
(health) insurance when a grace period applies, and b) when the insurer makes a total value of the insured building and personal properties in the amount of
written acknowledgment of the receipt of premium, this acknowledgment being P600,000.00 plus interest at the legal rate of 6% per annum from the filing
declared by law to, be then conclusive evidence of the premium payment. under of the complaint until full payment, and attorneys fees equivalent to 20% of
Sec. 77, as well as Sec. 78, until the premium is paid, and the law has not the total amount claimed plus costs of suit.
expressly excepted partial payments, there is no valid and binding 81.   The CA reversed the court a quo by declaring FORTUNE not to be liable
contract. Hence, in the absence of clear waiver of prepayment in full by the therein but ordering the return of the premium of P2,983.50 plus 12%
insurer, the insured cannot collect on the proceeds of the policy interest from when Violeta paid the balance of the premium (2 days after
DOCTRINE: For as long as the current Insurance Code remains unchanged and the fire) until full payment.
partial payment of premiums is not mentioned at all as among the exceptions 82.   Hence this petition for review.
provided in Secs. 77 and 78, no policy of insurance can ever pretend to be
efficacious or effective until premium has been fully paid. ISSUE/s:
13.   WoN fire insurance policy is valid, binding and enforceable upon mere
 
partial payment of premium – NO where the premium has only been an insurance contract with the insured. In the case before
partially paid and the balance paid only after the peril insured against has us, there is, quite unlike in Phoenix, a specific stipulation
occurred, the insurance contract did not take effect and the insured cannot that (t)his policy xxx is not in force until the premium has
collect at all on the policy been fully paid and duly receipted by the Company x x x.
ii.   It is correct to say that in Phoenix a contract was
RULING: WHEREFORE, the petition is DENIED and the assailed Decision of the perfected upon partial payment of the premium since the
Court of Appeals dated 24 March 1995 is AFFIRMED. parties had not otherwise stipulated that prepayment of
SO ORDERED. the premium in full was a condition precedent to the
existence of a contract.
RATIO: b.   In Phoenix, by accepting the initial payment of P3,000.00 and then
1.   Insurance is a contract whereby one undertakes for a consideration to later demanding the remainder of the premium without any other
indemnify another against loss, damage or liability arising from an precondition to its enforceability as in the instant case, the insurer
unknown or contingent event. in effect had shown its intention to continue with the existing
a.   The consideration is the premium, which must be paid at the time contract of insurance, as in fact it was enforcing its right to collect
and in the way and manner specified in the policy, and if not so premium, or exact specific performance from the insured. This is
paid, the policy will lapse and be forfeited by its own terms. not so here. By express agreement of the parties, no vinculum
2.   the Policy provides for payment of premium in full. Accordingly, where the juris or bond of law was to be established until full payment was
premium has only been partially paid and the balance paid only after the effected prior to the occurrence of the risk insured against.
peril insured against has occurred, the insurance contract did not take effect 5.   In Makati Tuscany Condominium Corp. v. Court of Appeals the parties
and the insured cannot collect at all on the policy. This is fully supported by mutually agreed that the premiums could be paid in installments, which in
Sec. 7727 of the Insurance Code fact they did for three (3) years, hence, this Court refused to invalidate the
3.   Apparently the crux of the controversy lies in the phrase unless and until the insurance policy.
premium thereof has been paid. For whatever judicial construction may be 6.   While it maybe true that under Section 77 of the Insurance Code, the parties
accorded the disputed phrase must ultimately yield to the clear mandate of may not agree to make the insurance contract valid and binding without
the law. The principle that where the law does not distinguish the court payment of premiums, there is nothing in said section which suggests that
should neither distinguish assumes that the legislature made no qualification the parties may not agree to allow payment of the premiums in installment,
on the use of a general word or expression. or to consider the contract as valid and binding upon payment of the first
4.   Violeta et al insists that FORTUNE is liable on the policy despite partial premium. Otherwise we would allow the insurer to renege on its liability
payment of the premium due and the express stipulation thereof to the under the contract, had a loss incurred (sic) before completion of payment
contrary. They rely heavily on the 1967 case of Philippine Phoenix and of the entire premium, despite its voluntary acceptance of partial payments,
Insurance Co., Inc. v. Woodworks, Inc. where the Court sustained the ruling a result eschewed by basic considerations of fairness and equity.
of the trial court that partial payment of the premium made the policy 7.   These two (2) cases, Phoenix and Tuscany, adequately demonstrate the
effective during the whole period of the policy. In that case, the insurance waiver, either express or implied, of prepayment in full by the insurer:
company commenced action against the insured for the unpaid balance on a impliedly, by suing for the balance of the premium as inPhoenix, and
fire insurance policy. In its defense the insured claimed that nonpayment of expressly, by agreeing to make premiums payable in installments as
premium produced the cancellation of the insurance contract. in Tuscany.
a.   The 1967 Phoenix case is not persuasive; neither is it decisive of 8.   But contrary to the stance taken by Violeta et al, there is no waiver express
the instant dispute. or implied in the case at bench. Precisely, the insurer and the insured
i.   The factual scenario is different.In Phoenix it was the expressly stipulated that (t)his policy including any renewal thereof and/or
insurance company that sued for the balance of the any indorsement thereon is not in force until the premium has been fully
premium, i.e., it recognized and admitted the existence of paid to and duly receipted by the Company x x x and that this policy shall
be deemed effective, valid and binding upon the Company only when the
                                                                                                                        premiums therefor have actually been paid in full and duly acknowledged.
27
 SEC.  77.  An  insurer  is  entitled  to  payment  of  the  premium  as  soon  as  the  thing  insured  is  exposed  to  
the   peril   insured   against.Notwithstanding   any   agreement   to   the   contrary,   no   policy   or   contract   of  
9.   Conformably with the aforesaid stipulations explicitly worded and taken in
insurance  issued  by  an  insurance  company  is  valid  and  binding  unless  and  until  the  premium  thereof  has   conjunction with Sec. 77 of the Insurance Code the payment of partial
been   paid,  except   in   the   case   of   a   life   or   an   industrial   life   policy   whenever   the   grace   period   provision   premium by the assured in this particular instance should not be considered
applies  (Italics  supplied).  
the payment required by the law and the stipulation of the parties. Rather, it 15.   In the desire to safeguard the interest of the assured, it must not be ignored
must be taken in the concept of a deposit to be held in trust by the insurer that the contract of insurance is primarily a risk-distributing device, a
until such time that the full amount has been tendered and duly receipted mechanism by which all members of a group exposed to a particular risk
for. In other words, as expressly agreed upon in the contract, full payment contribute premiums to an insurer. To ensure payment for losses, the law
must be made before the risk occurs for the policy to be considered mandates all insurance companies to maintain a legal reserve fund in favor
effective and in force. of those claiming under their policies.
10.   Thus, no vinculum juris whereby the insurer bound itself to indemnify the a.   It should be understood that the integrity of this fund cannot be
assured according to law ever resulted from the fractional payment of secured and maintained if by judicial fiat partial offerings of
premium. The insurance contract itself expressly provided that the policy premiums were to be construed as a legal nexus between the
would be effective only when the premium was paid in full. It would have applicant and the insurer despite an express agreement to the
been altogether different were it not so stipulated. Ergo, petitioners had contrary.
absolute freedom of choice whether or not to be insured by FORTUNE b.   What could prevent the insurance applicant from deliberately or
under the terms of its policy and they freely opted to adhere thereto. wilfully holding back full premium payment and wait for the risk
11.   In the construction of an insurance contract is the intention of the parties as insured against to transpire and then conveniently pass on the
expressed in the policy. Courts have no other function but to enforce the balance of the premium to be deducted from the proceeds of the
same. The rule that contracts of insurance will be construed in favor of the insurance?
insured and most strongly against the insurer should not be permitted to 16.   For as long as the current Insurance Code remains unchanged and
have the effect of making a plain agreement ambiguous and then construe it partial payment of premiums is not mentioned at all as among the
in favor of the insured. exceptions provided in Secs. 77 and 78, no policy of insurance can ever
a.   Verily, it is elemental law that the payment of premium is requisite pretend to be efficacious or effective until premium has been fully paid.
to keep the policy of insurance in force. If the premium is not paid 17.   Premium is the elixir vitae of the insurance business because by law the
in the manner prescribed in the policy as intended by the parties insurer must maintain a legal reserve fund to meet its contingent obligations
the policy is ineffective. Partial payment even when accepted as a to the public, hence, the imperative need for its prompt payment and full
partial payment will not keep the policy alive even for such satisfaction
fractional part of the year as the part payment bears to the whole 18.   Once payment of premium is left to the whim and caprice of the insured, as
payment. when the courts tolerate the payment of a mere P600.00 as partial
12.   Applying further the rules of statutory construction, the position maintained undertaking out of the stipulated total premium of P2,983.50 and the
by Violeta et al becomes even more untenable. balance to be paid even after the risk insured against has occurred, as
13.   The case of South Sea Surety and Insurance Company, Inc. v. Court of Violeta have done in this case, on the principle that the strength of
Appeals, speaks only of two (2) statutory exceptions to the requirement the vinculumjuris is not measured by any specific amount of premium
of payment of the entire premium as a prerequisite to the validity of the payment, we will surely wreak havoc on the business and set to naught what
insurance contract. These exceptions are: has taken actuarians centuries to devise to arrive at a fair and equitable
a.   in case the insurance coverage relates to life or industrial life distribution of risks and benefits between the insurer and the insured.
(health) insurance when a grace period applies, and 19.   The terms of the insurance policy constitute the measure of the insurers
b.   when the insurer makes a written acknowledgment of the receipt of liability. In the absence of statutory prohibition to the contrary, insurance
premium, this acknowledgment being declared by law to, be then companies have the same rights as individuals to limit their liability and to
conclusive evidence of the premium payment. impose whatever conditions they deem best upon their obligations not
14.   A maxim of recognized practicality is the rule that the expressed exception inconsistent with public policy. The validity of these limitations is by law
or exemption excludes others. Exceptio firm at regulim in casibus non passed upon by the Insurance Commissioner who is empowered to approve
exceptis. all forms of policies, certificates or contracts of insurance which insurers
a.   Thus, under Sec. 77, as well as Sec. 78, until the premium is paid, intend to issue or deliver. That the policy contract in the case at bench was
and the law has not expressly excepted partial payments, there is approved and allowed issuance simply reaffirms the validity of such policy,
no valid and binding contract. Hence, in the absence of clear particularly the provision in question.
waiver of prepayment in full by the insurer, the insured cannot
collect on the proceeds of the policy.    
015 American Home Assurance Co. v. Chua (Daguman) behalf payment of any premium which is due on such policy or contract of
June 28,1999 | Davide,Jr, J. | Premiums insurance at the time of its issuance or delivery or which becomes due thereon.
Section 78 of the Insurance Code explicitly provides: “An acknowledgment in a
PETITIONER: American Home Assurance Company policy or contract of insurance of the receipt of premium is conclusive evidence
RESPONDENT: Antonio Chua of its payment, so far as to make the policy binding, notwithstanding any
stipulation therein that it shall not be binding until the premium is actually paid.”
SUMMARY: On April 5, 1990, Antonio Chua renewed the fire insurance for
its stock-in-trade of his business, Moonlight Enterprises with American Home  
Assurance Company by issuing a check to its agent James Uy who delivered the FACTS:
Renewal Certificate to him. On April 6, 1990, Moonlight Enterprises was 83.   American Home Assurance Company (AHAC) is a domestic corporation
completely razed by fire. On April 10, 1990, an official receipt was issued and engaged in the insurance business. In 1990, Chua obtained a fire insurance
subsequently, a policy was issued covering March 25 1990 to March 25 1991 from AHAC covering the stock-in trade of his business, Moonlight
Antonio Chua filed an insurance claim with American Home and 4 other co- Enterprises. The insurance was to expire on 25 March 1990.
insurers (Pioneer Insurance and Surety Corporation, Prudential Guarantee and 84.   5 April 1990, Chua issued a PCIBank Check to AHAC’s agent, for the
Assurance, Inc. and Filipino Merchants Insurance Co) American Home refused renewal of the policy. The check was drawn and deposited in AHAC’s bank
alleging the no premium was paid RTC: favored Antonio Chua for paying by account.
way of check a day before the fire occurred CA: Affirmed in toto. Won there 85.   A corresponding official receipt was issued on April 10.
was a valid payment of premium, considering that Chua’s check was cashed 86.   Subsequently, a new insurance policy was issued, whereby AHAC
after the occurrence of the fire? YES. the renewal certificate issued to Chua undertook t indemnify Chua for any damage or loss arising from fire for the
contained the acknowledgment that premium had been paid. It is not disputed period March 25 1990- March 25 1991
that the check drawn by Chua in favor of AHAC and delivered to its agent was 87.   6 April 1990, Moonlight Enterprises was completely razed by fire. Chua
honored when presented and AHAC forthwith issued its official receipt to Chua filed an insurance claim with AHAC and 4 other co-insurers (Pioneer,
on 10 April 1990. Section 306 of the Insurance Code provides that any insurance Prudential, Filipino Merchants and Domestic Insurance- Insurance
company which delivers a policy or contract of insurance to an insurance agent Companies)
or insurance broker shall be deemed to have authorized such agent or broker to 88.   AHAC refused to honor the insurance despite demands. Chua filed an
receive on its behalf payment of any premium which is due on such policy or action before the trial court against AHAC.
contract of insurance at the time of its issuance or delivery or which becomes 89.   AHAC argued that there was no existing insurance contract when the fire
due thereon. In the instant case, the best evidence of such authority is the fact occurred since Chua did not pay the premium. And even assuming that
that AHAC accepted the check and issued the official receipt for the payment. It there is a contract, Chua violated several conditions: 1. submission of
is, as well, bound by its agent’s acknowledgment of receipt of payment. WoN fraudulent income tax returns and financial statements. 2. Failure to
Chua violated the policy by his submission of fraudulent documents and non- establish actual loss 3. Failure to notify AHAC of any insurance already
disclosure of the other existing insurance contracts; and finally? NO. The effected to cover the insured goods.
submission of the alleged fraudulent documents pertained to Chua’s income tax 90.   The trial court ruled in favor of Chua. It found Chua paid by way of check a
returns for 1987 to 1989. Chua, however, presented a BIR certification that he day before the fire occurred, which was deposited in AHAC’s account.
had paid the proper taxes for the said years. AHAC’s loss adjuster had known all There was even an acknowledged certificate issued by AHAC’s agent. The
along of the other existing insurance contracts, yet, he did not use that as basis fraudulent documents were limited to a disparity of official receipts inssued
for his recommendation of denial. The loss adjuster, being an employee of by the BIR and the income tax returns for years 1987 to 1989. As to Chua’s
AHAC, is deemed a representative of the latter whose awareness of the other failure to notify AHAC of other insurance contracts, it held that AHAC
insurance contracts binds AHAC failed to show such fraudulent omission was intentional and fraudulent.
91.   CA affirmed in toto, MR was denied.
DOCTRINE: The general rule in insurance laws is that unless the premium is 92.   [AHAC’s Argument]
paid the insurance policy is not valid and binding. The only exceptions are life 93.   AHAC cites the case of Arce v. Capital Insurance & Surety Co., Inc., where
and industrial life insurance we ruled that unless and until the premium is paid there is no insurance.
Section 306 of the Insurance Code provides that any insurance company which AHAC emphasizes that when the fire occurred on 6 April 1990 the
delivers a policy or contract of insurance to an insurance agent or insurance insurance contract was not yet subsisting pursuant to Article 1249 of the
broker shall be deemed to have authorized such agent or broker to receive on its Civil Code, which recognizes that a check can only effect payment once it
has been cashed. Although Chua testified that he gave the check on 5 April AHAC’s loss adjuster had known all along of the other existing insurance
to a certain James Uy, the check, drawn against a Manila bank and contracts, yet, he did not use that as basis for his recommendation of denial.
deposited in a Cagayan de Oro City bank, could not have been cleared by 6 The loss adjuster, being an employee of AHAC, is deemed a representative
April, the date of the fire. In fact, the official receipt issued for Chua’s of the latter whose awareness of the other insurance contracts binds AHAC
check payment was dated 10 April 1990, four days after the fire occurred.
94.   Citing jurisprudence, AHAC also contends that Chua’s non-disclosure of RULING: Whereby, petition is denied.
the other insurance contracts rendered the policy void. It underscores the
trial court’s neglect in considering the Commission on Audit’s certification RATIO:
that the BIR receipts submitted by Chua were, in effect, fake since they 1.   The general rule in insurance laws is that unless the premium is paid the
were issued to other persons insurance policy is not valid and binding. The only exceptions are life
95.   [Chua’s Argument] and industrial life insurance. Whether payment was indeed made is a
96.   Chua refutes the reason for AHAC’s denial of his claim. As found by the question of fact which is best determined by the trial court
trial court, AHAC’s loss adjuster admitted prior knowledge of Chua’s 2.   According to the trial court the renewal certificate issued to Chua contained
existing insurance contracts with the other insurance companies. the acknowledgment that premium had been paid. It is not disputed that the
Nonetheless, the loss adjuster recommended the denial of the claim, not check drawn by Chua in favor of AHAC and delivered to its agent was
because of the said contracts, but because he was suspicious of the honored when presented and AHAC forthwith issued its official receipt to
authenticity of certain documents which Chua submitted in filing his claim. Chua on 10 April 1990. Section 306 of the Insurance Code provides that
97.   To bolster his argument, Chua cites Section 66 of the Insurance Code, any insurance company which delivers a policy or contract of insurance
which requires the insurer to give a notice to the insured of its intention to to an insurance agent or insurance broker shall be deemed to have
terminate the policy fortyfive days before the policy period ends. In the authorized such agent or broker to receive on its behalf payment of any
instant case, AHAC opted not to terminate the policy. Instead, it renewed premium which is due on such policy or contract of insurance at the
the policy by sending its agent to Chua, who was issued a renewal time of its issuance or delivery or which becomes due thereon.
certificate upon delivery of his check payment for the renewal of premium. 3.   In the instant case, the best evidence of such authority is the fact that
At this precise moment the contract of insurance was executed and already AHAC accepted the check and issued the official receipt for the
in effect. He also claims that it is standard operating procedure in the payment. It is, as well, bound by its agent’s acknowledgment of receipt
provinces to pay insurance premiums by check when collected by insurance of payment. Section 78 of the Insurance Code explicitly provides:
agents.
98.   Hence, these issues. “An acknowledgment in a policy or contract of insurance of the
receipt of premium is conclusive evidence of its payment, so far as
ISSUE/s: to make the policy binding, notwithstanding any stipulation
1.   Won there was a valid payment of premium, considering that Chua’s check therein that it shall not be binding until the premium is actually
was cashed after the occurrence of the fire? YES. Section 306 of the paid.”
Insurance Code provides that any insurance company which delivers a
policy or contract of insurance to an insurance agent or insurance broker 4.   This Section establishes a legal fiction of payment and should be interpreted
shall be deemed to have authorized such agent or broker to receive on its as an exception to Section 77. Is Chua guilty of the policy violations
behalf payment of any premium which is due on such policy or contract of imputed against him? We are not convinced by AHAC’s arguments. The
insurance at the time of its issuance or delivery or which becomes due submission of the alleged fraudulent documents pertained to Chua’s income
thereon. In the instant case, the best evidence of such authority is the fact tax returns for 1987 to 1989. Chua, however, presented a BIR certification
that AHAC accepted the check and issued the official receipt for the that he had paid the proper taxes for the said years. The trial court and the
payment. It is, as well, bound by its agent’s acknowledgment of receipt of Court of Appeals gave credence to the certification and it being a question
payment. of fact, we hold that said finding is conclusive.
2.   WoN Chua violated the policy by his submission of fraudulent documents 5.   Ordinarily, where the insurance policy specifies as a condition the
and non-disclosure of the other existing insurance contracts; and finally? disclosure of existing co-insurers, nondisclosure thereof is a violation
NO. The submission of the alleged fraudulent documents pertained to that entitles the insurer to avoid the policy. This condition is common in
Chua’s income tax returns for 1987 to 1989. Chua, however, presented a fire insurance policies and is known as the “other insurance clause.”
BIR certification that he had paid the proper taxes for the said years. The purpose for the inclusion of this clause is to prevent an increase in the
moral hazard. We have ruled on its validity and the case of Geagonia v.    
Court of Appeals clearly illustrates such principle. However, we see an
exception in the instant case.
6.   Citing Section 29 of the Insurance Code, the trial court reasoned that Chua’s
failure to disclose was not intentional and fraudulent. The application of
Section 29 is misplaced. Section 29 concerns concealment which is
intentional. The relevant provision is Section 75, which provides that:

“A policy may declare that a violation of specified provisions


thereof shall avoid it, otherwise the breach of an immaterial
provision does not avoid the policy. contracts must be upon the
same subject matter and with the same interest and risk.”

7.   Indeed, Chua acquired several co-insurers and he failed to disclose this


information to AHAC. Nonetheless, AHAC is estopped from invoking
this argument. The trial court cited the testimony of AHAC’s loss
adjuster who admitted previous knowledge of the coinsurers.

Thus, COURT:
Q The matter of additional insurance of other companies,
was that ever discussed in your investigation?
A Yes, sir.
Q In other words, from the start, you were aware the
insured was insured with other companies like Pioneer
and so on?
A Yes, Your Honor.
Q But in your report you never recommended the denial of
the claim simply because of the non-disclosure of other
insurance? [sic]
A Yes, Your Honor.
Q In other words, to be emphatic about this, the only
reason you recommended the denial of the claim, you
found three documents to be spurious. That is your only
basis?
A Yes, Your Honor. [Emphasis supplied]

8.   Indubitably, it cannot be said that AHAC was deceived by Chua by the


latter’s non-disclosure of the other insurance contracts when AHAC
actually had prior knowledge thereof. AHAC’s loss adjuster had known
all along of the other existing insurance contracts, yet, he did not use
that as basis for his recommendation of denial. The loss adjuster, being
an employee of AHAC, is deemed a representative of the latter whose
awareness of the other insurance contracts binds AHAC. We, therefore,
hold that there was no violation of the “other insurance” clause by Chua.

 
016 UCPB GENERAL INSURANCE v. MASAGANA TELAMART (Dim) policies. Written notice was given to the addresses stated in the policies.
June 15, 1999 | J. Pardo | Actual payment of premium is needed 101.  On June 13, 1992, a fire razed Masagana's property covered by three of
the insurance policies UCPB issued. On the same day, Masagana
PETITIONER: UCPB General Insurance Co Inc. presented 5 manager’s checks to the cashier of UCPB worth
RESPONDENT: Masagana Telamart, Inc. P225,753.95, representing the premium for the renewal of the policies.
No notice of loss was filed by Masagana under the policies at this time
SUMMARY: UCPB issued 5 insurance policies covering Masagana Telamart’s 102.  On July 14, 1992, Masagana filed with UCPB its formal claim for
properties against fire from May 22, 1991 to May 22, 1992. UCPB decided not indemnification of the insured property. On the same day, UCPB returned
to renew the policies upon their expiration and sent notice to Masagana’s to Masagana the five (5) manager's checks that it tendered, and at the
insurance broker. On June 13, 1992, a fire razed the properties of Masagana same time rejected Masagana's claim for the reasons (a) that the
covered by those insurance policies. On the same day, Masagana presented 5 policies had expired and were not renewed, and (b) that the fire
checks to UCPB representing premiums for the renewal of the policies. A month occurred on June 13, 1992, before Masagana's tender of premium
later, Masagana filed a formal claim for indemnification for the properties razed payment.
by the fire. UCPB returned the 5 checks and rejected the claim of Masagana
103.  Masagana filed with the RTC of Makati a civil complaint against UCPB for
because the policies had expired and were not renewed when the fire occurred.
recovery of P18,645,000.00, representing the face value of the policies
Masagana filed a civil complaint against UCPB for the recovery of the value of
covering Masagana's insured property razed by fire, and for attorney's fees
the razed properties. The RTC and CA ruled in favor of Masagana, first stating
that in previous practice, Masagana was allowed a 60-90 day credit term for the 104.  UCPB filed an answer, stating that the complaint fails to state a cause of
renewal of its policies, second, that the acceptance of the late premium payment action, and that UCPB was not liable for insurance proceeds under the
suggested an understanding that payment could be made later, and third, that policies because at the time of the loss of the property due to fire, the
notice to the broker was not notice to Masagana.The issue before the SC is WoN policies had long expired and were not renewed.
the fire insurance polcies had been renewed upon payment of premiums on a
date later than the expiration of the policies. The Court ruled in the negative. 105.  The RTC ruled in favor of Masagana. Upon appeal, the CA affirmed the
Any insurance policy, other than life, issued originally or on renewal, is not valid ruling, holding that following previous practise, Masagana was allowed
and binding until actual payment of the premium. Any agreement to the contrary a sixty (60) to ninety (90) day credit term for the renewal of its policies,
is void. The parties may not agree expressly or impliedly on the extension of and that the acceptance of the late premium payment suggested an
credit or time to pay the premium and consider the policy binding before actual understanding that payment could be made later. Hence the appeal to the
payment. SC.

DOCTRINE: Notwithstanding any agreement to the contrary, no policy or 106.  Masagana submits that the CA correctly ruled that no timely notice of non-
contract of insurance issued by an insurance company is valid and binding unless renewal was sent. The notice of non-renewal sent to broker Zuellig which
and until the premium thereof has been paid, except in the case of a life or an claimed that it verbally notified the insurance agency but not Masagana
industrial life policy. itself did not suffice.

FACTS: [this case is shorter than my attention span] ISSUE/s:


WoN the fire insurance policies had been extended or renewed by an implied
99.   UCPB General Insurance Company Inc. issued five (5) insurance policies credit arrangement though actual payment of premium was tendered on a later
covering Masagana Telamart Inc.'s various property described therein date after the occurrence of the risk insured against. – NO. This is against
against fire, for the period from May 22, 1991 to May 22, 1992. Section 7728 of the Insurance Code.
100.  UCPB evaluated the policies and decided not to renew them upon                                                                                                                        
expiration of their terms on May 22, 1992. UCPB advised Masagana's 28
    Section   77.   An   insurer   is   entitled   to   payment   of   the   premium   as  
broker, Zuellig Insurance Brokers, Inc. of its intention not to renew the
soon   as   the   thing   insured   is   exposed   to   the   peril   insured   against.  
RULING: WHEREFORE, the Court hereby REVERSES and SETS ASIDE the
decision of the Court of Appeals. In lieu thereof, the Court renders judgment
dismissing respondent's complaint and petitioner's counterclaims thereto filed with
the Regional Trial Court, Branch 58, Makati City. Without costs.

RATIO:
60.   The answer is easily found in the Insurance Code. Any insurance policy,
other than life, issued originally or on renewal, is not valid and binding
until actual payment of the premium. Any agreement to the contrary is
void. The parties may not agree expressly or impliedly on the extension of
credit or time to pay the premium and consider the policy binding before
actual payment.

61.   The case of Malayan Insurance Co., Inc. vs. Cruz-Arnaldo, cited by the CA,
is not applicable. In that case, payment of the premium was in fact actually
made on December 24, 1981, and the fire occurred on January 18, 1982.

62.   In this case, the payment of the premium for renewal of the policies was
tendered on July 13, 1992, a month after the fire occurred on June 13,
1992. The assured did not even give the insurer a notice of loss within a
reasonable time after occurrence of the fire.

 
   

                                                                                                                                                                                                                                                                                       
Notwithstanding  any  agreement  to  the  contrary,  no  policy  or  contract  
of   insurance   issued   by   an   insurance   company   is   valid   and   binding  
unless   and   until   the   premium   thereof   has   been   paid,   except   in   the  
case   of   a   life   or   an   industrial   life   policy   whenever   the   grace   period  
provision   applies,   or   whenever   under   the   broker   and   agency  
agreements  with  duly  licensed  intermediaries,  a  ninety  (90)-­‐day  credit  
extension  is  given.  No  credit  extension  to  a  duly  licensed  intermediary  
should  exceed  ninety  (90)  days  from  date  of  issuance  of  the  policy.  
017 UCPB GENERAL INSURANCE vs. MASAGANA TELEMART (Eleazar) notwithstanding any stipulation therein that it shall not be binding until premium
April 4, 2001 | Davide, C.J. | Premium Payment is actually paid. Makati Tuscany v Court of Appeals- Section 77 may not apply
if the parties have agreed to the payment in installments of the premium and
PETITIONERS: UCPB GENERAL INSURANCE CO. INC. partial payment has been made at the time of loss. Section 78 allows waiver by
RESPONDENTS: MASAGANA TELAMART, INC. the insurer of the condition of prepayment and makes the policy binding despite
the fact that premium is actually unpaid.
SUMMARY: Masagana obtained from UCPB five (5) insurance policies on its Section 77 does not expressly prohibit an agreement granting credit extension.
Manila properties. The policies were effective from May 22, 1991 to May 22, At the very least, both parties should be deemed in estoppel to question the
1992. On June 13, 1992, Masagana’s properties were razed by fire. On July 13, arrangement they have voluntarily accepted.
1992, Masagana tendered five checks for as renewal premium payments. A The Tuscany case has provided another exception to Section 77 that the insurer
receipt was issued. On July 14, 1992, Masagana made its formal demand for may grant credit extension for the payment of the premium. If the insurer has
indemnification for the burned insured properties. UCPB then rejected granted the insured a credit term for the payment of the premium and loss occurs
Masagana’s claims under the argument that the fire took place before the tender before the expiration of the term, recovery on the policy should be allowed even
of payment. Hence Masagana filed a case. The Court of Appeals disagreed with though the premium is paid after the loss but within the credit term.
UCPB’s argument that Masagana’s tender of payment of the premiums on 13 Moreover, there is nothing in Section 77 which prohibits the parties in an
July 1992 did not result in the renewal of the policies, having been made within insurance contract to provide a credit term within which to pay the premiums.
the effective date of renewal as provided under Policy Condition No. 26, which That agreement is not against the law, morals, good customs, public order or
states: 26. Renewal Clause. -- Unless the company at least 45 days in advance of public policy. The agreement binds the parties.
the end of the policy period mails or delivers to the assured at the address shown It would be unjust if recovery on the policy would not be permitted against
in the policy notice of its intention not to renew the policy or to condition its UCPB, which had consistently granted a 60- to 90-day credit term for the
renewal upon reduction of limits or elimination of coverages, the assured shall payment of premiums. Estoppel bars it from taking refuge since Masagana
be entitled to renew the policy upon payment of the premium due on the relied in good faith on such practice. Estoppel then is the fifth exception.
effective date of renewal.
DOCTRINE: There are exceptions to IC Sec. 77.
Both the Court of Appeals and the trial court found that sufficient proof exists 1.   In case of a life or industrial life policy whenever the grace period
that Masagana, which had procured insurance coverage from UCPB for a provision applies [Sec. 77]
number of years, had been granted a 60 to 90-day credit term for the renewal of 2.   Any acknowledgment of the receipt of premiumis conclusive evidence
the policies. of payment [Sec. 78]
3.   If the parties have agreed to the payment ininstallments of the premium
The Supreme Court ruled against UCPB in the first case on the issue of whether and partial payment has been made at the time of loss [Makati Tuscany
the fire insurance policies issued by UCPB to the Masagana covering the period Condominium v. CA]
from May 22, 1991 to May 22, 1992 had been extended or renewed by an 4.   The insurer may grant credit extension for the payment of the premium
implied credit arrangement though actual payment of premium was tendered on [Makati Tuscany Condominium]
a later date and after the occurrence of the risk insured against. UCPB filed a 5.   Estoppel
motion for reconsideration. The Supreme Court, upon observing the facts,
affirmed that there was no valid notice of non-renewal of the policies in Sec. 77 merely precludes the parties from stipulating that the policy is valid even
question, as there is no proof at all that the notice sent by ordinary mail was if premiums are not paid, but does not expressly prohibit an agreement granting
received by Masagana. Also, the premiums were paid within the grace period. credit extension, and such an agreement is not contrary to morals, good customs,
The issue in this case is whether Section 77 of the Insurance Code of 1978 must public order or public policy.
be strictly applied to UCPB’s advantage despite its practice of granting a 60- to
90-day credit term for the payment of premiums? The SC said no. Section 77 of FACTS:
the Insurance Code provides: No policy or contract of insurance issued by an 1.   In the SC’s decision of 15 June 1999 in this case, they reversed and set
insurance company is valid and binding unless and until the premium thereof has aside the assailed decision of the Court of Appeals, which affirmed with
been paid. An exception to this section is Section 78 which provides: Any modification the judgment of the trial court
acknowledgment in a policy or contract of insurance of the receipt of premium is a.   (a) allowing Masagana to consign the sum of P225,753.95 as full
conclusive evidence of its payment, so far as to make the policy binding, payment of the premiums for the renewal of the five insurance
policies on Respondents properties; 5.   Both the Court of Appeals and the trial court found that sufficient proof
b.   (b) declaring the replacement-renewal policies effective and exists that Masagana, which had procured insurance coverage from UCPB
binding from 22 May 1992 until 22 May 1993; and for a number of years, had been granted a 60 to 90-day credit term for the
c.   (c) ordering Petitioner to pay Respondent P18,645,000.00 as renewal of the policies. Such a practice had existed up to the time the claims
indemnity for the burned properties covered by the renewal- were filed.
replacement policies. The modification consisted in the (1) 6.   In the decision of 15 June 1999, the SC defined the main issue to be
deletion of the trial courts declaration that three of the policies whether the fire insurance policies issued by UCPB to the Masagana
were in force from August 1991 to August 1992; and (2) reduction covering the period from May 22, 1991 to May 22, 1992 had been extended
of the award of the attorneys fees from 25% to 10% of the total or renewed by an implied credit arrangement though actual payment of
amount due the Respondent. premium was tendered on a later date and after the occurrence of the (fire)
2.   The material operative facts upon which the appealed judgment was based risk insured against.
are summarized by the Court of Appeals in its assailed decision as follows. 7.   The Court resolved this issue in the negative in view of Section 77 of the
a.   Masagana obtained from UCPB five (5) insurance policies on its Insurance Code and the decisions in Valenzuela v. Court of Appeals; South
properties in Pasay City and Manila. Sea Surety and Insurance Co., Inc. v. Court of Appeals; and Tibay v. Court
b.   All 5 policies reflect on their face the effectivity term: "from 4:00 of Appeals. Accordingly, we reversed and set aside the decision of the Court
P.M. of 22 May 1991 to 4:00 P.M. of 22 May 1992." of Appeals.
c.   On June 13, 1992, Masagana's properties located at 2410-2432 and 8.   Masagana seasonably filed a motion for the reconsideration of the adverse
2442-2450 Taft Avenue, Pasay City were razed by fire. verdict. It alleges in the motion that the Court had made in the decision their
d.   On July 13, 1992, Masagana tendered, and UCPB accepted, five own findings of facts, which are not in accord with those of the trial court
(5) Equitable Bank Manager's Checks in the total amount of and the Court of Appeals.
P225,753.45 as renewal premium payments for which Official 9.   The courts below correctly found that no notice of non-renewal was made
Receipt Direct Premium No. 62926 was issued by UCPB. within 45 days before 22 May 1992, or before the expiration date of the fire
e.   On July 14, 1992, Masagana made its formal demand for insurance policies. Thus, the policies in question were renewed by operation
indemnification for the burned insured properties. On the same of law and were effective and valid on 30 June 1992 when the fire
day, UCPB returned the five (5) manager's checks stating in its occurred, since the premiums were paid within the 60- to 90-day credit
letter that it was rejecting Masagana's claim on the following term.
grounds: 10.   Masagana likewise disagrees with the Court’s ruling that parties may
i.   "a) Said policies expired last May 22, 1992 and were not neither agree expressly or impliedly on the extension of credit or time to pay
renewed for another term; the premium nor consider a policy binding before actual payment. It urges
ii.   b) Defendant had put plaintiff and its alleged broker on the Court to take judicial notice of the fact that despite the express provision
notice of non-renewal earlier; and of Section 77 of the Insurance Code, extension of credit terms in premium
iii.   c) The properties covered by the said policies were payment has been the prevalent practice in the insurance industry.
burned in a fire that took place last June 13, 1992, or 11.   Most insurance companies, including UCPB, extend credit terms because
before tender of premium payment.” Section 77 of the Insurance Code is not a prohibitive injunction but is
3.   Hence Masagana filed this case. merely designed for the protection of the parties to an insurance
4.   The Court of Appeals disagreed with UCPB’s stand that Masagana’s tender contract. The Code itself, in Section 78, authorizes the validity of a policy
of payment of the premiums on 13 July 1992 did not result in the renewal of notwithstanding non-payment of premiums.
the policies, having been made beyond the effective date of renewal as 12.   Masagana also asserts that the principle of estoppel applies to UCPB.
provided under Policy Condition No. 26, which states: Despite its awareness of Section 77 UCPB persuaded and induced
a.   26. Renewal Clause. -- Unless the company at least forty five days Masagana to believe that payment of premium on the 60- to 90-day credit
in advance of the end of the policy period mails or delivers to the term was perfectly alright; in fact it accepted payments within 60 to 90 days
assured at the address shown in the policy notice of its intention after the due dates.
not to renew the policy or to condition its renewal upon reduction 13.   By extending credit and habitually accepting payments 60 to 90 days from
of limits or elimination of coverages, the assured shall be entitled the effective dates of the policies, UCPB has implicitly agreed to modify the
to renew the policy upon payment of the premium due on the tenor of the insurance policy and in effect waived the provision therein that
effective date of renewal. it would pay only for the loss or damage in case the same occurred after
payment of the premium. except in the case of a life or an industrial life policy whenever the
14.   UCPB filed an opposition to Masagana’s motion for reconsideration. It grace period provision applies.
argues that both the trial court and the Court of Appeals overlooked the fact 2.   This Section is a reproduction of Section 77 of P.D. No. 612 (The Insurance
that on 6 April 1992 UCPB sent by ordinary mail to Masagana a notice of Code) promulgated on 18 December 1974. In turn, this Section has its
non-renewal and sent by personal delivery a copy thereof to Masagana’s source in Section 72 of Act No. 2427 otherwise known as the Insurance Act
broker, Zuellig. Both courts likewise ignored the fact that Masagana was as amended by R.A. No. 3540, approved on 21 June 1963, which reads:
fully aware of the notice of non-renewal. A reading of Section 66 of the a.   SEC. 72. An insurer is entitled to payment of premium as soon as
Insurance Code readily shows that in order for an insured to be entitled to a the thing insured is exposed to the peril insured against, unless
renewal of a non-life policy, payment of the premium due on the effective there is clear agreement to grant the insured credit extension of the
date of renewal should first be made. Masagana’s argument that Section 77 premium due. No policy issued by an insurance company is valid
is not a prohibitive provision finds no authoritative support. and binding unless and until the premium thereof has been paid.
15.   Upon a meticulous review of the records and reevaluation of the issues (Underscoring supplied)
raised in the motion for reconsideration and the pleadings filed thereafter by 3.   It can be seen at once that Section 77 does not restate the portion of Section
the parties, the Court resolved to grant the motion for reconsideration. 72 expressly permitting an agreement to extend the period to pay the
16.   The following facts, as found by the trial court and the Court of Appeals, premium. But there exceptions to Sec. 77.
are indeed duly established: 4.   The first exception is provided by Section 77 itself, and that is, in case of a
a.   For years, UCPB had been issuing fire policies to the Masagana, life or industrial life policy whenever the grace period provision applies.
and these policies were annually renewed. 5.   The second is that covered by Section 78 of the Insurance Code, which
b.   UCPB had been granting Masagana a 60- to 90-day credit term provides:
within which to pay the premiums on the renewed policies. a.   SEC. 78. Any acknowledgment in a policy or contract of insurance
c.   There was no valid notice of non-renewal of the policies in of the receipt of premium is conclusive evidence of its payment, so
question, as there is no proof at all that the notice sent by ordinary far as to make the policy binding, notwithstanding any stipulation
mail was received by Masagana, and the copy thereof allegedly therein that it shall not be binding until premium is actually paid.
sent to Zuellig was ever transmitted to Masagana. 6.   A third exception was laid down in Makati Tuscany
d.   The premiums for the policies in question in the aggregate amount Condominium Corporation vs. Court of Appeals, wherein the Court ruled
of P225,753.95 were paid by Masagana within the 60- to 90-day that Section 77 may not apply if the parties have agreed to the payment in
credit term and were duly accepted and received by UCPB cashier. installments of the premium and partial payment has been made at the time
of loss. Thus:
ISSUE/s a.   We hold that the subject policies are valid even if the premiums
WoN Section 77 of the Insurance Code must be strictly applied to UCPB’s were paid on installments. The records clearly show that the
advantage despite its practice of granting a 60- to 90-day credit term for the payment petitioners and private respondent intended subject insurance
of premiums. policies to be binding and effective notwithstanding the staggered
payment of the premiums. The initial insurance contract entered
RULING: WHEREFORE, the Decision in this case of 15 June 1999 is into in 1982 was renewed in 1983, then in 1984. In those three
RECONSIDERED and SET ASIDE, and a new one is hereby entered DENYING the years, the insurer accepted all the installment payments. Such
instant petition for failure of UCPB to sufficiently show that a reversible error was acceptance of payments speaks loudly of the insurers intention to
committed by the Court of Appeals in its challenged decision, which is hereby honor the policies it issued to petitioner. Certainly, basic principles
AFFIRMED in toto. of equity and fairness would not allow the insurer to continue
collecting and accepting the premiums, although paid on
RATIO: installments, and later deny liability on the lame excuse that the
1.   Section 77 of the Insurance Code of 1978 provides: premiums were not prepaid in full.
a.   SEC. 77. An insurer is entitled to payment of the premium as soon 7.   Not only that. In Tuscany, the Court also quoted with approval the
as the thing insured is exposed to the peril insured following pronouncement of the Court of Appeals in its Resolution denying
against. Notwithstanding any agreement to the contrary, no policy the motion for reconsideration of its decision:
or contract of insurance issued by an insurance company is valid a.   While the import of Section 77 is that prepayment of premiums is
and binding unless and until the premium thereof has been paid, strictly required as a condition to the validity of the contract, We
are not prepared to rule that the request to make installment
payments duly approved by the insurer would prevent the entire
contract of insurance from going into effect despite payment and
acceptance of the initial premium or first installment. Section 78 of
the Insurance Code in effect allows waiver by the insurer of the
condition of prepayment by making an acknowledgment in the
insurance policy of receipt of premium as conclusive evidence of
payment so far as to make the policy binding despite the fact that
premium is actually unpaid. Section 77 merely precludes the
parties from stipulating that the policy is valid even if premiums
are not paid, but does not expressly prohibit an agreement granting
credit extension, and such an agreement is not contrary to morals,
good customs, public order or public policy. So is an
understanding to allow insured to pay premiums in installments not
so prescribed. At the very least, both parties should be deemed in
estoppel to question the arrangement they have voluntarily
accepted.
8.   By the approval of the aforequoted findings and conclusion of the Court of
Appeals, Tuscany has provided a fourth exception to Section 77, namely,
that the insurer may grant credit extension for the payment of the premium.
9.   This simply means that if the insurer has granted the insured a credit term
for the payment of the premium and loss occurs before the expiration of the
term, recovery on the policy should be allowed even though the premium is
paid after the loss but within the credit term.
10.   Moreover, there is nothing in Section 77 which prohibits the parties in an
insurance contract to provide a credit term within which to pay the
premiums. That agreement is not against the law, morals, good customs,
public order or public policy. The agreement binds the parties. Article 1306
of the Civil Code provides:
a.   ART. 1306. The contracting parties may establish such stipulations
clauses, terms and conditions as they may deem convenient,
provided they are not contrary to law, morals, good customs,
public order, or public policy.
11.   Finally in the instant case, it would be unjust and inequitable if recovery on
the policy would not be permitted against UCPB, which had consistently
granted a 60- to 90-day credit term for the payment of premiums despite its
full awareness of Section 77. Estoppel bars it from taking refuge under said
Section, since Masagana relied in good faith on such practice. Estoppel then
is the fifth exception to Section 77.

   
18 SOUTH SEA SURETY AND INSURANCE COMPANY INC. v. CA insurance policy for the plaintiffs logs, he is deemed to have been authorized by
(HORTALEZA) the South Sea Surety and Insurance Co., Inc. to receive the premium which is
June, 02, 1995 | VITUG, J. | PREMIUM PAYMENT due on its behalf. When therefore the insured logs were lost, the insured had
already paid the premium to an agent of the South Sea Surety and Insurance
PETITIONER: South Sea Surety and Insurance Company Inc. Co., Inc., which is consequently liable to pay the insurance proceeds under the
RESPONDENTS: Hon.Court of Appeals and Valenzuela Hardwood and Industrial policy it issued to the insured.
Supply Inc. DOCTRINE: Sec. 306. . . . Any insurance company which delivers to an insurance
SUMMARY: (take note of the dates!) Valenzuela Hardwood and Industrial Supply agent or insurance broker a policy or contract of insurance shall be deemed to have
Inc. (Hardwood) undertook to board lauan round logs numbering 940 on January 16 authorized such agent or broker to receive on its behalf payment of any
1984 with Seven Brothers. On January 20 1984 the logs were then insured with premium which is due on such policy of contract of insurance at the time of its
South Sea Surety and Insurance Company Inc. (South Sea) for 2 million pesos. The issuance or delivery or which becomes due thereon.
logs were then boarded into vessel M/V Seven Ambassador owned by Seven (the doctrine in my own words) When an insurer delivers the insurance policy to
Brothers Shipping Corporation. On January 24 1984, Hardwood gave a check to an individual or a person he/she is deemed to have been authorized by the
Victorio Chua who works as an administrative assistant for the past ten years and an insurer to receive the premium which is due on its behalf. Thus when the
agent for less than ten years of the Columbia Insurance Brokers, Ltd. The M/V payment is made to the agent it is deemed as if made to the insurer himself.
Seven Ambassador ultimately sank on January 25 1984, and led to Hardwood
claiming upon the obligations of South Sea to pay the value of the loss logs. South FACTS:
Sea denies the claim on the premise that they only received on January 30 1984 a 1.   Two issues on the subject of insurance are raised in this petition, that assails
check for P5,625.00 serving as the Payment for the Premium agreed upon. The the decision, that assails the decision of the Court of Appeals. (in CA G.R.
refusal of South Sea to indemnify Hardwood led to the filing of the case. Hardwood NO. CV- 20156), the first dealing on the requirement of premium
and South Sea argue on whether Chua was acting as an agent for the insured payment and the second relating to the agency relationship of parties
(Hardwood) or the insurer (South Sea). South Sea claims that since Chua received under that contract.
the insurance premiums as an agent of the Columbia Insurance Broker, acted as an 2.   It appears that on 16 January 1984, Valenzuela Hardwood and Industrial
agent of the insured under Section 301 of the insurance code: Sec. 301.”Any person Supply, Inc. entered into an agreement with the defendant Seven Brothers
who for any compensation, commission or other thing of value, acts, or aids in whereby the latter undertook to load on board its vessel M/V Seven
soliciting, negotiating or procuring the making of any insurance contract or in Ambassador the former's lauan round logs numbering 940 at the port of
placing risk or taking out insurance, on behalf of an insured other than himself, shall Maconacon, Isabela for shipment to Manila.
be an insurance broker within the intent of this Code, and shall thereby become liable 3.   On 20 January 1984, plaintiff insured the logs, against loss and/or,
to all the duties requirements, liabilities and penalties to which an insurance broker is damage with defendant South Sea Surety and Insurance Co., Inc. for
subject.” while Hardwood claims under the second paragraph of Sec 306 of the P2,000,000.00 end the latter issued its Marine Cargo Insurance Policy
insurance code Sec. 306. “. . . Any insurance company which delivers to an No. 84/24229 for P2,000,000.00 on said date.
insurance agent or insurance broker a policy or contract of insurance shall be 4.   On 24 January 1984, the plaintiff gave the check in payment of the
deemed to have authorized such agent or broker to receive on its behalf premium on the insurance policy to Mr. Victorio Chua.
payment of any premium which is due on such policy of contract of insurance at 5.   In the meantime, the said vessel M/V Seven Ambassador sank on 25
the time of its issuance or delivery or which becomes due thereon”. In this January 1984 resulting in the loss of the plaintiffs insured logs.
petition for review on certiorari brought by South Sea Surety and Insurance Co., Inc 6.   On 30 January 1984, a check for P5,625.00 to cover payment of the
it argues that it likewise should have been freed from any liability to Hardwood. The premium and documentary stamps due on the policy was tendered to the
Issue was WoN the appellate court erred (a) for having Supposedly disregarded insurer but was not accepted. Instead, the South Sea Surety and Insurance
Section 7729 of the insurance Code and (b) for holding Victorio Chua to have been an Co., Inc. cancelled the insurance policy it issued as of the date of inception
authorized representative of the insurer.? NO to both,When the appellant South for non payment of the premium due in accordance with Section 77 of the
Sea Surety and Insurance Co., Inc. delivered to Mr. Chua the marine cargo Insurance Code.
7.   On 2 February 1984, plaintiff demanded from defendant South Sea Surety
                                                                                                                       
29 and Insurance Co., Inc. the payment of the proceeds of the policy but the
 Sec. 77. An insurer is entitled to payment of the premium as soon as the thing insured is exposed to the latter denied liability under the policy. Plaintiff likewise filed a formal
peril insured against. Notwithstanding any agreement to the contrary, no policy or contract of insurance
issued by an insurance company is valid and binding unless and until the premium thereof has been paid,
claim with defendant Seven Brothers Shipping Corporation for the value of
except in the case of a life or an industrial life policy whenever the grace period provision applies.  
the lost logs but the latter denied the claim.
8.   In its decision, dated 11 May 1988, the trial court rendered judgment in insurance, on behalf of an insured other than himself,
favor of plaintiff Hardwood. shall be an insurance broker within the intent of this
9.   In the decision the court dismissed the claims against the Seven Brothers Code, and shall thereby become liable to all the duties
Shipping corporation because there was a stipulation exempting them from requirements, liabilities and penalties to which an
liability and they were deemed here to be a private carrier (only relevant insurance broker is subject.
information about the shipping corporation is this) c.   The appellees, upon the other hand, claim that the second
10.   In this petition for review on certiorari brought by South Sea Surety and paragraph of Section 306 of the Insurance Code provide as
Insurance Co., Inc., argues that it likewise should have been freed from any follows:
liability to Hardwood. It faults the appellate court (a) for having Supposedly i.   Sec. 306. . . . Any insurance company which
disregarded Section 77 of the insurance Code and (b) for holding Victorio delivers to an insurance agent or insurance broker a policy
Chua to have been an authorized representative of the insurer. or contract of insurance shall be deemed to have
a.   Sec. 77. An insurer is entitled to payment of the premium as soon authorized such agent or broker to receive on its behalf
as the thing insured is exposed to the peril insured against. payment of any premium which is due on such policy of
Notwithstanding any agreement to the contrary, no policy or contract of insurance at the time of its issuance or delivery
contract of insurance issued by an insurance company is valid and or which becomes due thereon.
binding unless and until the premium thereof has been paid, except
in the case of a life or an industrial life policy whenever the grace ISSUE/s:
period provision applies. 1.   WoN the appellate court erred (a) for having Supposedly disregarded
11.   No attempt becloud the issues can disguise the fact that the sole question Section 77 of the insurance Code and (b) for holding Victorio Chua to have
raised in the instant petition is really evidentiary in nature, i.e., whether been an authorized representative of the insurer.? No to both,When the
or not Victorio Chua, in receiving the check for the insurance premium appellant South Sea Surety and Insurance Co., Inc. delivered to Mr.
prior to the occurrence of the risk insured against has so acted as an Chua the marine cargo insurance policy for the plaintiffs logs, he is
agent of South Sea. The appellate court, like the trial court, has found in deemed to have been authorized by the South Sea Surety and Insurance
the affirmative. Said the appellate court: Co., Inc. to receive the premium which is due on its behalf. When
a.   In the instant case, the Marine Cargo Insurance Policy No. therefore the insured logs were lost, the insured had already paid the
84/24229 was issued by defendant insurance company on 20 premium to an agent of the South Sea Surety and Insurance Co., Inc.,
January 1984. At the time the vessel sank on 25 January 1984 which is consequently liable to pay the insurance proceeds under the
resulting in the loss of the insured logs, the insured had already policy it issued to the insured.
delivered to Victorio Chua the check in payment of premium.
But, as Victorio Chua testified, it was only in the morning of 30 RULING: WHEREFORE, the resolution, dated 01 February 1993, granting due
January 1984 or 5 days after the vessel sank when his course to the petition is RECALLED, and the petition is DENIED. Costs against
messenger tendered the check to defendant South Sea Surety petitioner.
and Insurance Co., Inc.
b.   Appellant surety company insists that Mr. Chua is an RATIO:
administrative assistant for the past ten years and an agent for less 1.   Undoubtedly, the payment of the premium is a condition precedent to, and
than ten years of the Columbia Insurance Brokers, Ltd. He is paid a essential for, the efficaciousness of the contract. The only two statutorily
salary as a administrative assistant and a commission as agent provided exceptions are (a) in case the insurance coverage relates to life or
based on the premiums he turns over to the broker. Appellant industrial life (health) insurance when a grace period applies and (b) when
therefore argues that Mr. Chua, having received the insurance the insurer makes a written acknowledgment of the receipt of premium, this
premiums as an agent of the Columbia Insurance Broker, acted as acknowledgment being declared by law to be then conclusive evidence of
an agent of the insured under Section 301 of the Insurance Code the premium payment (Secs. 77-78, Insurance Code). The appellate court,
which provides as follows: contrary to what the petition suggests, did not make any pronouncement to
i.   Sec. 301. Any person who for any compensation, the contrary. Indeed, it has said:
commission or other thing of value, acts, or aids in a.   Concerning the issue as to whether there is a valid contract of
soliciting, negotiating or procuring the making of any insurance between plaintiff-appellee and defendant-appellant
insurance contract or in placing risk or taking out South Sea Surety and Insurance Co., Inc., Section 77 of the
Insurance Code explicitly provides that notwithstanding any
agreement to the contrary, no policy issued by an insurance
company is valid and binding unless and until premium thereof has
been paid. It is therefore important to determine whether at the
time of the loss, the premium was already paid
2.   On cross-examination in behalf of South Sea Surety and Insurance Co., Inc.
Mr. Chua testified that the marine cargo insurance policy for the plaintiff's
logs was delivered to him on 21 January 1984 at his office to be
delivered to the plaintiff.
3.   When the appellant South Sea Surety and Insurance Co., Inc. delivered to
Mr. Chua the marine cargo insurance policy for the plaintiffs logs, he is
deemed to have been authorized by the South Sea Surety and Insurance
Co., Inc. to receive the premium which is due on its behalf.
4.   When therefore the insured logs were lost, the insured had already paid
the premium to an agent of the South Sea Surety and Insurance Co.,
Inc., which is consequently liable to pay the insurance proceeds under
the policy it issued to the insured
 
   
019 GAISANO v. DEVELOPMENT INSURANCE (Fordan) at the time of loss on Sept. 27. It does not also falls in the 4th and 5th exceptions
Feb. 27, 2017 | Jardeleza, J. | Payment of insurance premiums since Gaisano failed to establish the fact of a grant by Development Insurance of
a credit term in his favor, or that the grant has been consistent. While there was
PETITIONER: Jaime T. Gaisano mention of a credit agreement between Trans-Pacific and Development
RESPONDENTS: Development Insurance and Surety Corporation Insurance, such arrangement was not proven and was internal between agent and
principal. Under the principle of relativity of contracts, contracts bind the parties
SUMMARY: 2. On Sept. 27, 1996, Development Insurance issued a who entered into it. It cannot favor or prejudice a third person, even if he is
comprehensive commercial vehicle policy to Gaisano of P1.5M over his vehicle, aware of the contract and has acted with knowledge. Moreover, the policy itself
Mitsubishi Montero for 1 year commencing on said date. It also issued 2 other states that the insured's application for the insurance is subject to the payment of
commercial vehicle policies to Gaisano covering 2 other motor vehicles for the the premium. There is no waiver of pre-payment, in full or in installment, of the
same period. A statement of account was sent to Noah’s Ark, Gaisano’s premiums under the policy. Consequently, Development Insurance cannot be
company, wherein it immediately processed and issued a check for the payment placed in estoppel. Thus, SC find that Gaisano is not entitled to the insurance
of the premiums. However, Trans-Pacific, agent of Development Insurance, proceeds because no insurance policy became effective for lack of premium
informed Noah’s Ark that it would pick-up the check the following day since payment.
there was a celebration in the company. Meanwhile, on the evening of Sept. 27,
while under the custody of Pacquing, Noah’s Ark Marketing Manager, the DOCTRINE: The general rule in insurance laws is that unless the premium is
vehicle was stolen in SM Megamall. The incident was reported to the PNP and paid, the insurance policy is not valid and binding pursuant to Sec. 77 of the
despite the search and retrieval efforts, the vehicle was not recovered. Oblivious Insurance Code. However, UCPB General Insurance Co., Inc. v. Masagana
of the incident, Trans-Pacific did picked-up the check on Sept. 28, issued an OR, Telamart, Inc., summarized the following exceptions:
and deposited it to Metrobank on Oct. 1. Upon notice from Pacquing, Gaisano 1.  in case of life or industrial life policy, whenever the grace period provision
reported the loss and and filed a claim with Development Insurance for the applies, as expressly provided by Sec. 77 itself;
insurance proceeds of P1.5M. However, Development Insurance denied the 2.  where the insurer acknowledged in the policy or contract of insurance itself
claim on the ground that there was no insurance contract since non-payment of the receipt of premium, even if premium has not been actually paid, as
the premium rendered the policy ineffective. The premium was received only on expressly provided by Sec. 78 itself;
Oct. 2, 1996, and there was no known loss covered by the policy to which the 3.  where the parties agreed that premium payment shall be in installments and
payment could be applied.. Gaisano then filed a complaint for collection of sum partial payment has been made at the time of loss, as held in Makati
of money and damages with the RTC. RTC ruled in favor of Gaisano. However, Tuscany Condominium Corp. v. Court of Appeals;
the CA reversed the decision. Hence, the current petition. 4.  where the insurer granted the insured a credit term for the payment of the
premium, and loss occurs before the expiration of the term, as held in
The issue is whether or not there is a binding insurance contract between Makati Tuscany Condominium Corp.; and
Gaisano and Development Insurance. NO. The SC held that just like any other 5.  where the insurer is in estoppel as when it has consistently granted a 60-to
contract, insurance contract requires a cause or consideration. The consideration 90-day credit term for the payment of premiums.
is the premium, which must be paid at the time and in the way and manner
specified in the policy. If not so paid, the policy will lapse and be forfeited by its FACTS:
own terms. The general rule in insurance laws is that unless the premium is paid, 107.  Petitioner Jaime T. Gaisano (Gaisano) was the registered owner of a 1992
the insurance policy is not valid and binding pursuant to Sec. 77 of the Insurance Mitsubishi Montero with plate number GTJ 777 (vehicle) while
Code. 4. In this case, there is no dispute that the check was delivered to and was respondent Development Insurance and Surety Corporation (Development
accepted by Trans-Pacific, only on Sept. 28. No payment of premium had thus Insurance) is a domestic corporation engaged in the insurance business.
been made at the time of the loss of the vehicle on Sept. 27. However, there are 108.  On Sept. 27, 1996, Development Insurance issued a comprehensive
exceptions to the rule that no insurance contract takes effect unless premium is commercial vehicle policy to Gaisano of P1.5M over the vehicle for 1 year
paid as summarized in UCPB General Insurance Co., Inc. v. Masagana commencing on Sept. 27, 1996. It also issued 2 other commercial vehicle
Telamart, Inc. (kindly see doctrine). In this case, the insurance policy in question policies to Gaisano covering 2 other motor vehicles for the same period.
does not fall under the 1st to 3rd exceptions laid out in UCPB General Insurance 109.  To collect the premiums, Trans-Pacific Underwriters Agency (Trans-
Co., Inc.: (1) the policy is not a life or industrial life policy; (2) the policy does Pacific), agent of Development Insurance, issued a statement of account to
not contain an acknowledgment of the receipt of premium but merely a Gaisano’s company, Noah’s Ark Merchandising (Noah’s Ark).
statement of account on its face; and (3) no payment of an installment was made 110.  Noah's Ark immediately processed the payments and issued a Far East Bank
check (of P140,893.50) dated Sept. 27, 1996 payable to Trans Pacific on 120.  Hence, the current petition. Gaisano argues that:
the same day. The check represents payment for the 3 insurance policies, a.   there was a valid and binding insurance contract between him and
with P55,620.60 for the premium and other charges over the vehicle. Development Insurance;
111.  However, nobody from Trans Pacific picked up the check that day (Sept. b.   it comes within the exceptions to the rule in Sec. 77 of the Insurance
27) because its president and general manager, Rolando Herradura, was Code that no contract of insurance becomes binding unless and until
celebrating his birthday. Trans Pacific informed Noah's Ark that its the premium thereof has been paid;
messenger would get the check the next day, Sept. 28. c.   the prohibitive tenor of Sec. 77 does not apply because the parties
112.  Meanwhile, in the evening of Sept. 27, while under the official custody of stipulated for the payment of premiums wherein the parties intended
Noah's Ark marketing manager Achilles Pacquing (Pacquing) as a service the contract of insurance to be immediately effective upon issuance,
company vehicle, the vehicle was stolen in the vicinity of SM Megamall at despite nonpayment of the premium, because Development Insurance
Ortigas, Mandaluyong City. trusted him;
113.  Pacquing reported the loss to the PNP Traffic Management Command at d.   Development Insurance waived its right to a prepayment in full of the
Camp Crame in Quezon City. Despite search and retrieval efforts, the terms of the policy, and is in estoppel;
vehicle was not recovered. e.   assuming he is not entitled to recover insurance proceeds but only to
114.  Oblivious of the incident, Trans-Pacific picked up the check the next day the return of the premiums paid, then he should be able to recover the
(Sept. 28). It issued an OR (dated Sept. 28) acknowledging the receipt of full P140,893.50 and not merely P55,620.60 since the insurance policy
the premium and other charges over the vehicle. The check was then covered 3 vehicles yet Development Insurance's intention was merely
deposited with Metrobank for encashment on Oct. 1 which was the same to disregard the contract for only the lost vehicle. Thus, the principle
day that Pacquing informed Gaisano of the vehicle's loss. of mutuality of contracts is violated, at his expense.
115.  Thereafter, Gaisano reported the loss and filed a claim with Development
Insurance for the insurance proceeds of P1.5M. However, Development ISSUE: Whether or not there is a binding insurance contract between Gaisano
Insurance denied the claim on the ground that there was no insurance and Development Insurance would be binding. – NO, since there was no payment
contract. of premium that has been made at the time of the loss of the vehicle on Sept. 27 and
116.  Gaisano, through counsel, sent a final demand but Development Insurance the circumstances does not fall into any of the exceptions noted in the case of UCPB
still refused to pay the insurance proceeds or return the premium paid on the General Insurance Co., Inc.
vehicle. Thus, Gaisano filed a complaint for collection of sum of money and
damages with the RTC. RULING: The petition is denied. The assailed decision of the CA and the
117.  In its Answer, Development Insurance asserted that the nonpayment of the Resolution are affirmed with the modification that Development Insurance should
premium rendered the policy ineffective. The premium was received only return the P55,620.60 with the legal interest computed at the rate of 6% per annum
on Oct. 2, 1996, and there was no known loss covered by the policy to reckoned from July 7, 1997 until finality of this judgment. Thereafter, the total
which the payment could be applied. amount shall earn interest at the rate of 6% per annum from the finality of this
118.  The RTC ruled in favor of Gaisano It considered the premium paid as of judgment until its full satisfaction.
Sept. 27, even if the check was received only on Sept. 28 because (1) Trans-
Pacific, agent, acknowledged payment of the premium on that date (Sept. RATIO:
27), and (2) the check issued was honored in acknowledgment of the 63.   Insurance is a contract whereby one undertakes for a consideration to
authority of the agent to receive it. indemnify another against loss, damage or liability arising from an
119.  On appeal, the CA reversed the decision of the RTC. It upheld unknown or contingent event. Just like any other contract, it requires a
Development Insurance’s position that an insurance contract becomes valid cause or consideration. The consideration is the premium, which must
and binding only after the premium is paid pursuant to Sec. 77 of the be paid at the time and in the way and manner specified in the policy. If
Insurance Code. It found that the premium was not yet paid at the time of not so paid, the policy will lapse and be forfeited by its own terms.
the loss on Sept. 27, but only a day after, when the check was picked up by 64.   The law, however, limits the parties' autonomy as to when payment of
TransPacific. It also found that none of the exceptions to Sec. 77 obtains in premium may be made for the contract to take effect. The general rule in
this case. Nevertheless, the CA ordered Development Insurance to return insurance laws is that unless the premium is paid, the insurance policy
the premium it received in of P55,620.60, with interest at the rate of 6% per is not valid and binding. Section 77 of the Insurance Code, applicable at
annum from the date of the denial of the claim on Oct. 9, 1996 until the time of the issuance of the policy, provides:
payment. Sec. 77. An insurer is entitled to payment of the premium as soon as the
thing insured is exposed to the peril insured against. Notwithstanding any
agreement to the contrary, no policy or contract of insurance issued by an 60-to 90-day credit term for the payment of premiums.
insurance company is valid and binding unless and until the premium 69.   In this case, the insurance policy in question does not fall under the 1st
thereof has been paid, except in the case of a life or an industrial life to 3rd exceptions laid out in UCPB General Insurance Co., Inc.: (1) the
policy whenever the grace period provision applies. policy is not a life or industrial life policy; (2) the policy does not
65.   In Tibay v. Court of Appeals, we explained that in an insurance contract, contain an acknowledgment of the receipt of premium but merely a
both the insured and insurer undertake risks. On one hand, there is the statement of account on its face; and (3) no payment of an installment
insured, a member of a group exposed to a particular peril, who contributes was made at the time of loss on Sept. 27.
premiums under the risk of receiving nothing in return in case the 70.   Furthermore, the SC does not agree with Gaisano that his case falls under
contingency does not happen; on the other, there is the insurer, who the 4th and 5th exceptions.
undertakes to pay the entire sum agreed upon in case the contingency 71.   The 4th and 5th exceptions to Sec. 77 operate under the facts obtaining in
happens. This risk-distributing mechanism operates under a system where, Makati Tuscany Condominium Corp. and UCPB General Insurance Co.,
by prompt payment of the premiums, the insurer is able to meet its legal Inc. Both contemplate situations where the insurers have consistently
obligation to maintain a legal reserve fund needed to meet its contingent granted the insured a credit extension or term for the payment of the
obligations to the public. The premium, therefore, is the elixir vitae or premium.
source of life of the insurance business. 72.   Here, Gaisano failed to establish the fact of a grant by Development
66.   In this case, there is no dispute that the check was delivered to and was Insurance of a credit term in his favor, or that the grant has been
accepted by Trans-Pacific, only on Sept. 28. No payment of premium consistent. While there was mention of a credit agreement between
had thus been made at the time of the loss of the vehicle on Sept. 27. Trans-Pacific and Development Insurance, such arrangement was not
67.   While Gaisano claims that Trans-Pacific was informed that the check was proven and was internal between agent and principal. Under the
ready for pick-up on Sept. 27, the notice of the availability of the check, by principle of relativity of contracts, contracts bind the parties who
itself, does not produce the effect of payment of the premium. Trans-Pacific entered into it. It cannot favor or prejudice a third person, even if he is
could not be considered in delay in accepting the check because when it aware of the contract and has acted with knowledge.
informed Gaisano that it will only be able to pick-up the check the next day, 73.   Furthermore, the SC cannot sustain Gaisano’s claim that the parties agreed
the latter did not protest to this, but instead allowed it to do so. Thus, at the that the insurance contract is immediately effective upon issuance despite
time of loss, there was no payment of premium yet to make the insurance non-payment of the premiums.
policy effective. 74.   Even if there is a waiver of prepayment of premiums, that in itself does not
68.   There are, of course, exceptions to the rule that no insurance contract become an exception to Sec. 77, unless the insured clearly gave a credit
takes effect unless premium is paid. In UCPB General Insurance Co., term or extension. This is the clear import of the 4th exception in the UCPB
Inc. v. Masagana Telamart, Inc., we summarized the exceptions as General Insurance Co., Inc. To rule otherwise would render nugatory the
follows: requirement in Sec. 77 that "[n]otwithstanding any agreement to the
a.   in case of life or industrial life policy, whenever the grace period contrary, no policy or contract of insurance issued by an insurance company
provision applies, as expressly provided by Sec. 77 itself; is valid and binding unless and until the premium thereof has been paid, x x
b.   where the insurer acknowledged in the policy or contract of x."
insurance itself the receipt of premium, even if premium has not 75.   Moreover, the policy itself states:
been actually paid, as expressly provided by Sec. 7830 itself; WHEREAS THE INSURED, by his corresponding proposal and
c.   where the parties agreed that premium payment shall be in declaration, and which shall be the basis of this Contract and deemed
installments and partial payment has been made at the time of loss, incorporated herein, has applied to the company for the insurance
as held in Makati Tuscany Condominium Corp. v. Court of Appeals; hereinafter contained, subject to the payment of the Premium as
d.   where the insurer granted the insured a credit term for the payment consideration for such insurance. (Emphasis supplied.)
of the premium, and loss occurs before the expiration of the term, as 76.   The policy states that the insured's application for the insurance is
held in Makati Tuscany Condominium Corp.; and subject to the payment of the premium. There is no waiver of pre-
e.   where the insurer is in estoppel as when it has consistently granted a payment, in full or in installment, of the premiums under the policy.
Consequently, Development Insurance cannot be placed in estoppel.
                                                                                                                        77.   Thus, SC find that Gaisano is not entitled to the insurance proceeds
30
SEC. 78. Any acknowledgment in a policy or contract of insurance of the receipt of because no insurance policy became effective for lack of premium
premium is conclusive evidence of its payment, so far as to make the policy binding, payment.
notwithstanding any stipulation therein that it shall not be binding until premium is actually
78.   The consequence of this declaration is that Gaisano is entitled to a return of
paid.
the premium paid for the vehicle of P55,620.60 under the principle of unjust
enrichment.
79.   There is unjust enrichment when a person unjustly retains a benefit to the
loss of another, or when a person retains money or property of another
against the fundamental principles of justice, equity and good conscience.
80.   Gaisano cannot claim the full P140,893.50, which includes the payment of
premiums for the 2 other vehicles. These two policies are not affected by
the ruling on the policy subject of this case because they were issued as
separate and independent contracts of insurance.
 
   
020 PHILIPPINE PHOENIX SURETY & INSURANCE v. b.   Phoenix Surety issued to Woodworks on April 1, 1960 Fire Policy
WOODWORKS (GALINDEZ) No. 9652 for P300,000 under the terms and conditions set forth in
August 31, 1967 | Dizon, J. | Return of Premium the policy
c.   The premiums of the policy amounted to P6,051.95, the margin fee
amounted to P363.72 and documentary stamp tax, P96.42
PETITIONER: Philippine Phoenix Surety & Insurance, Inc.
d.   Woodworks paid P3,000 on September 22, 1960
RESPONDENTS: Woodworks, Inc. e.   Phoenix Surety made several demands on Woodworks to pay
P3,522.09.
SUMMARY: Phoenix Surety issued to Woodworks on April 1, 1960 Fire
Policy No. 9652 for P300,000 under the terms and conditions set forth in the ISSUE/s:
policy. The premiums of the policy amounted to P6,051.95, the margin fee 1.   WoN the nonpayment of premiums produced the cancellation of the
amounted to P363.72 and documentary stamp tax, P96.42. Woodworks paid
insurance contract – NO. The correct view would seem to be this: as the
P3,000 on September 22, 1960. Phoenix Surety made several demands on
contract had become perfected, the parties could demand from each other
Woodworks to pay P3,522.09.
the performance of whatever obligations they had assumed.
Phoenix Surety instituted an action to recover from Woodworks the balance of
RULING: Wherefore, the appealed decision being in accordance with law and the
P3,522.09 for the premium of the fire policy. Woodworks refused to pay and the
evidence, the same is hereby affirmed, with costs.
case reached the SC. Woodworks’ theory: non-payment by it of the premium
due produced the cancellation of the contract of insurance. However, such RATIO:
theory would place exclusively in the hands of one of the contracting parties the 1.   There is no doubt that between the insurer and insured, there was not only a
right to decide whether the contract should stand or not. perfected contract of insurance but a partially performed one as far as the
payment of the agreed premium was concerned.
ISSUE: WoN the nonpayment of premiums produced the cancellation of the
2.   After, the obligation of the insurer to pay the insured the amount for which
insurance contract – NO. (See Doctrine)
the policy was issued in case the conditions had been complied with, arose
and became binding upon it, while the obligation of the insured to pay the
In the case of the insurer, it is obvious that it had the right to demand from the
remainder of the total amount of the premium became due and demandable.
insured the completion of the payment of the premium due or sue for the
3.   The Court cannot agree with Woodworks’ theory that non-payment by it of
rescission of the contract. As it chose to demand specific performance of the
the premium due produced the cancellation of the contract of insurance.
insured's obligation to pay the balance of the premium, the latter's duty to pay is
Such theory would place exclusively in the hands of one of the contracting
indeed indubitable. parties the right to decide whether the contract should stand or not.
4.   Rather the correct view would seem to be this: as the contract had become
DOCTRINE: The correct view would seem to be this: as the contract had perfected, the parties could demand from each other the performance of
become perfected, the parties could demand from each other the performance of whatever obligations they had assumed.
whatever obligations they had assumed.
5.   In the case of the insurer, it is obvious that it had the right to demand from
the insured the completion of the payment of the premium due or sue for the
This is more or less the whole case J rescission of the contract.
6.   As it chose to demand specific performance of the insured's obligation to
FACTS: pay the balance of the premium, the latter's duty to pay is indeed
1.   Appellee Phoenix Surety commenced this action in the Municipal Court of indubitable.
Manila to recover from appellant Woodworks, Inc. P3,522.09 representing    
the unpaid balance of the premiums on a fire insurance policy issued by
Phoenix Surety in favor of Woodworks for a term of one year, from April 1
1960 to 1961.
2.   From an adverse decision of said court, Woodworks appealed to the CFI of
Manila where the parties submitted the following stipulation of facts:
a.   Both parties are corporations duly organized and existing by virtue
of Philippine laws
021 CAPITAL INSURANCE v. PLASTIC ERA (Gonzales) 3.   When the policy was delivered, Plastic Era failed to pay the corresponding
July 18, 1975 | Martin, J. | Return of Premium insurance premium. However, through its duly authorized representative, it
executed acknowledgment receipt.31
PETITIONER: Capital Insurance & Surety Co., Inc. 4.   In partial payment, Plastic Era delivered to Capital Insurance, a check for
RESPONDENT: Plastic Era Co., Inc. and Court of Appeals the amount of P1,000.00 postdated January 16, 1961.
5.   However, Capital Insurance tried to deposit the check only on February 20,
SUMMARY: When Capital Insurance delivered to Plastic Era its open fire policy 1961 and the same was dishonored for lack of funds. The records show that
whereby the former undertook to insure the latter's property, the latter did not pay the as of January 19, 1961 Plastic Era had a balance of P1,193.41 with the bank
premium, but instead executed an acknowledgment receipt with a promise to pay the 6.   On January 18, 1961 or two days after the insurance premium became due,
premium within 30 days from December 17, 1960, the effectivity date of the policy. at about 4:00 to 5:00 o'clock in the morning, the property insured by Plastic
Capital Insurance duly accepted the receipt. On January 8, 1961, Plastic Era, in Era was destroyed by fire.
partial payment of the premium, delivered a check of P1,000, postdated January 16, 7.   The loss and/or damage was estimated to be P283,875. However, according
1961. Capital Insurance tried to deposit the check only on February 20, 1961, and the to the records the same property has been insured by Plastic Era with the
same was dishonored for lack of funds. But the records show that as of January 19, Philamgen Insurance Company for P200,000.00.
1961, Plastic Era had sufficient funds with the bank to cover the check. On January 8.   Plastic Era demanded from Capital Insurance the payment of the sum of
18, 1961, or two days after the insurance premium became due, the insured P100,000.00 but the latter refused because of Plastic Era’s failure to pay the
properties were destroyed by fire. Plastic Era demanded payment of indemnity but insurance premium. Plastic Era thus filed a complaint for the recover of the
Capital Insurance refused on the ground of non-payment of premium. Plastic Era sum of P100,000 plus attorney’s fees and additional expenses.
sued Capital Insurance and the trial court rendered judgment in favor of the former. 9.   The trial court ordered Capital Insurance to pay Plastic Era P88,325.63.
The Court of Appeals sustained the trial court. The issue is WoN a contract of 10.   CA affirmed the decision of the trial court.
insurance has been duly perfected between Capital Insurance and Plastic Era – YES.
By accepting the Plastic Era’s promise to pay the insurance premium, Capital ISSUE:
Insurance, in effect, had extended credit to the former and did not have the right to 1.   WoN a contract of insurance has been duly perfected between Capital
cancel the policy except by putting Plastic Era in default by giving it personal notice Insurance and Plastic Era – YES. By accepting the promise of Plastic Era to
to that effect. pay the insurance premium within thirty (30) days from the effective date of
policy, Capital Insurance has implicitly agreed to modify the tenor of the
DOCTRINE: Where the insurer accepted the promise of the insured to pay the insurance policy and in effect, waived the provision therein that it would
insurance premium within 30 days from the effectivity date of the policy, the former only pay for the loss or damage in case the same occurs after the payment of
implicitly agreed to modify the tenor of the insurance contract and in effect waived the premium.
the provisions therein that it would only pay for the loss or damage in case the same
occurs after the payment of the premium; and where the policy is silent as to the RULING: Petition denied. CA decision affirmed.
mode of payment, the insurer is deemed to have accepted the promissory note in
payment of the premium. This rendered, the policy is immediately operative. RATIO:
1.   In clear and unequivocal terms the insurance policy32 provides that it is only
upon payment of the premiums by Plastic Era that Capital Insurance agrees
FACTS:
1.   Capital Insurance delivered to Plastic Era its open Fire Policy No.                                                                                                                        
31
22760 wherein the former undertook to insure the latter's building, "This acknowledged receipt of Fire Policy) NO. 22760 Premium. . .) (I promise to pay)
(P2,220.00) (has been paid) THIRTY DAYS AFTER on effective date ______ (Date)”
equipments, raw materials, products and accessories located at Sheridan 32
"This Policy of Insurance Witnesseth That in consideration of PLASTIC ERA hereinafter called the
Street, Mandaluyong, Rizal. Insured, paying to the Capital Insurance hereinafter called the Company, the sum of PESOS TWO
2.   The policy provides that if the property insured would be destroyed or THOUSAND ONE HUNDRED EIGHTY EIGHT the premium for the first period hereinafter mentioned,
damaged by fire after the payment of the premiums, at any time between the for insuring against Loss or Damage by only Fire or Lightning, as hereinafter appears, the Property
hereinafter described and contained, or described herein and not elsewhere, in the several sums following
15th day of December 1960 and one o'clock in the afternoon of the 15th day namely: PESOS ONE HUNDRED THOUSAND ONLY, PHILIPPINE CURRENCY; . . . THE
of December 1961, the insurance company shall make good all such loss or COMPANY HEREBY AGREES with the Insured but subject to the terms and conditions endorsed or
damage in an amount not exceeding P100,000.00. otherwise expressed hereon, which are to be taken as part of this Policy), that if the Property described, or
any part thereof, shall be destroyed or damaged by Fire or Lightning after payment of the Premiums, at
any time between the 15th day of December One Thousand Nine Hundred and Sixty and 1 o'clock in the
afternoon of the 15th day of December One Thousand Nine Hundred and Sixty-One of the last day of any
to insure the properties of the former against loss or damage in an amount claiming a forfeiture of its policy for non-payment even if the check had
not exceeding P100,000.00. been dishonored later.
2.   The crux of the problem then is whether at the time the insurance policy 9.   Finally, it is submitted by Capital Insurance that: "We are here concerned
was delivered to Plastic Era on December 17, 1960, the latter was able to with a case of reciprocal obligations, and Plastic Era having failed to
pay the stipulated premium. comply with its obligation to pay the insurance premium due on the policy
3.   On the day the insurance policy was delivered, Plastic Era did not pay the within thirty days from December 17, 1960, Capital Insurace was relieved
Capital Insurance, but instead executed an acknowledgment receipt wherein of its obligation to pay anything under the policy, without the necessity of
it promised to pay the premium within thirty (30) days from the effectivity first instituting an action for rescission of the contract of ensured entered
date of the policy on December 17, 1960 and Capital Insurance accepted it. into by the parties."
4.   Under Article 124933 of the Civil Code, the mere delivery of a bill of 10.   But precisely in this case, Plastic Era has complied with its obligation to
exchange in payment of a debt does not immediately effect payment. pay the insurance premium and therefore Capital Insurance is obliged to
5.   Here, Capital Insurance accepted the promise of Plastic Era to pay the make good its undertaking to Plastic Era.
insurance premium within thirty (30) days from the effective date of policy. |
By so doing, it has implicitly agreed to modify the tenor of the
insurance policy and in effect, waived the provision therein that it  
would only pay for the loss or damage in case the same occurs after the
payment of the premium. Considering that the insurance policy is silent as
to the mode of payment, Capital Insurance is deemed to have accepted the
promissory note in payment of the premium.
6.   Hence, when the damage of the insured property occurred, the insurance
policy was in full force and effect. The fact that the check issued by
Plastic Era in partial payment of the promissory note was later on
dishonored did not in any way operate as a forfeiture of its rights under
the policy, there being no express stipulation therein to that effect.
7.   The payment of the premium on the insurance policy became an
independent obligation the non-fulfillment of which would entitle Capital
Insurance to recover. It could just deduct the premium due and unpaid upon
the satisfaction of the loss under the policy. It did not have the right to
cancel the policy for nonpayment of the premium except by putting Plastic
Era in default and giving it personal notice to that effect.
8.   Although the check was due for payment on January 16, 1961 and Plastic
Era had sufficient funds to cover it as of January 19, 1961, Capital
Insurance decided to hold the same for thirty-five (35) days before
presenting it for payment. Having held the check for such an
unreasonable period of time, Capital Insurance was estopped from

                                                                                                                                                                                                                                                                                       
subsequent period in respect of which the insured, or a successor in interest to whom the insurance is by
an endorsement hereon declared to be or is otherwise continued, shall pay to the Company and the
Company shall accept the sum required for the renewal of this Policy, the Company will pay or make
good all such loss or Damage, to an amount not exceeding during any one period of the insurance in
respect of the several matters specified, the sum; set opposite thereto respectively, and not exceeding the
whole sum of PESOS, ONE HUNDRED THOUSAND ONLY, PHIL. CUR. . . ."
33
"The delivery of promissory notes payable to order, or bills of exchange or other mercantile documents
shall produce the effect of payment only when they have been cashed, or when through the fault of the
creditor they have been impaired." xxx "In the meantime, the action derived from the original obligation
shall be held in abeyance."
 
001 GULF RESORT INC. v. PHIL. CHARTER (Escalona) 1.   Gulf Resort is the owner of the Plaza Resort situated at Agoo,
May 16, 2005 | Puno, J. | The Policy La Union and had its properties in said resort insured originally
with the American Home Assurance Company (AHAC). In the
PETITIONER: Gulf Resort Inc. first four insurance policies issued by AHAC, the risk of loss
RESPONDENTS: Philippine Charter Insurance Corporation
from earthquake shock was extended only to Gulf Resort’s two
SUMMARY: Gulf Resort, Inc at Agoo, La Union was insured with American swimming pools. Item 5 in those policies referred to the two
Home Assurance Company, which includes loss or damage to shock to any of (2) swimming pools only; that subsequently AHAC issued in
the property insured by this Policy occasioned by or through or in consequence Gulf Resort’s favor Policy No. 206-4182383-0 and in said
of earthquake. Gulf Resort then insured its properties with PCIC based on the policy the earthquake endorsement clause was deleted and
same insurance contract they had with AHAC. Later on, an earthquake struck
Central Luzon and Northern Luzon so the properties and 2 swimming pools in its the entry under Endorsements/Warranties at the time of issue
Agoo Playa Resort were damaged. Gulf then claimed against PCIC for the read that Gulf Resort renewed its policy with AHAC for the
damages the earthquake created on its properties. Gulf's claim was denied on the period of March 14, 1989 to March 14, 1990 under Policy No.
ground that its insurance policy only afforded earthquake shock coverage to the 206-4568061-9, which carried the entry under
two swimming pools of the resort. Gulf contends that pursuant to this rider, no Endorsement/Warranties at Time of Issue, which read
qualifications were placed on the scope of the earthquake shock coverage, thus
the policy extended earthquake shock coverage to all of the insured properties. Endorsement to Include Earthquake Shock in the amount
The RTC ruled in favor of PCIC that the policy only covered the two swimming of P10,700.00 and paid P42,658.14 as premium thereof.
pools. The CA affirmed the decision of the RTC. 2.   Gulf then insured its properties with PCIC using the exact same
contract it used to have with AHAC.
The issue is WoN the insurance policy covers the other properties beyond the 3.   On July 16, 1990, an earthquake struck Central Luzon and
two swimming pools.
Northern Luzon and Gulf Resort’s properties covered by Policy
The Court ruled that the insurance policy did not cover the other properties. The No. 31944 issued by PCIC, including the two swimming pools
Court first ruled that a stipulation in the contract cannot be construed piecemeal, in its Agoo Playa Resort were damaged.
such that the contract should be construed as a whole, including the provisions 4.   After the earthquake, Gulf Resort advised PCIC that it would
and riders. They should all be construed together in order to evidence the be making a claim under its Insurance Policy No. 31944 for
intention of the parties of whether they wanted to insure the other properties
against earthquake shocks. The Court held that there was no intention of the damages on its properties. PCIC instructed Gulf Resort to file a
parties to include such because there were no premiums paid for the other formal claim, then assigned the investigation of the claim to an
properties against earthquake shocks, apart from the two swimming pools. This independent claims adjuster, Bayne Adjusters and Surveyors,
is in consonance with the history of the deletion of the other properties from Inc. On July 30, 1990, PCIC, through its adjuster, requested
insurance against earthquake shocks in the insurance contracts that Gulf had with Gulf Resort to submit various documents in support of its
AHAC.
claim. On August 7, 1990, Bayne Adjusters and Surveyors,
DOCTRINE: All provisions of the insurance policy should be examined and Inc., through its Vice-President A.R. de Leon, rendered a
interpreted in consonance with each other. All its parts are reflective of the true preliminary report finding extensive damage caused by the
intent of the parties. The policy cannot be construed piecemeal. Certain earthquake to the clubhouse and to the two swimming
stipulations cannot be segregated and then made to control; neither do particular pools. Mr. de Leon stated that except for the swimming
words or phrases necessarily determine its character.
pools, all affected items have no coverage for earthquake
FACTS:
shocks. On August 11, 1990, Gulf Resort filed its form
demand for settlement of the damage to all its properties in the
Agoo Playa Resort. On August 23, 1990, PCIC denied Gulf
Resort’s claim on the ground that its insurance policy only
afforded earthquake shock coverage to the two swimming Resort had with PCIC and upon which the subject
pools of the resort. Gulf Resort and PCIC failed to arrive at a insurance contract with Philippine Charter
settlement. Thus, on January 24, 1991, Gulf Resort filed a Insurance Corporation is said to have been based
complaint with the regional trial court of Pasig. and copied, covered an extended earthquake shock
5.   The lower court after trial ruled in favor of the PCIC, stating: insurance on all the insured properties.
a.   The above schedule clearly shows that plaintiff paid ISSUES:
only a premium of P393.00 against the peril of 1.   WoN the insurance policy covered only the two swimming pools, rather
than all the properties insured, against the risk of an earthquake shock –
earthquake shock, the same premium it paid against YES. Only the two swimming pools are covered because the contract that
earthquake shock only on the two swimming pools in reflected the intent of the parties evidenced that only the two swimming
all the policies issued by PCIC. From this fact the Court pools were insured against earthquake shocks. This was proven through the
must consequently agree with the position of PCIC that lack of payment of premiums and testimonies of witnesses.
the endorsement rider means that only the two
RULING: The judgment of the Court of Appeals is affirmed. The petition
swimming pools were insured against earthquake for certiorari is dismissed.
shock.
b.   Gulf Resort correctly points out that a policy of RATIO:
insurance is a contract of adhesion hence, where the 81.  The policy stated:
language used in an insurance contract or application is ANNUAL PAYMENT AGREEMENT ON
such as to create ambiguity the same should be resolved LONG TERM POLICIES
against the party responsible therefor, i.e., the insurance THE INSURED UNDER THIS POLICY HAVING
company which prepared the contract. To the mind of ESTABLISHED AGGREGATE SUMS INSURED IN
the Court, the language used in the policy in litigation is EXCESS OF FIVE MILLION PESOS, IN CONSIDERATION
clear and unambiguous hence there is no need for OF A DISCOUNT OF 5% OR 7 % OF THE NET PREMIUM
interpretation or construction but only application of the x x x POLICY HEREBY UNDERTAKES TO CONTINUE
provisions therein. THE INSURANCE UNDER THE ABOVE NAMED x x x
c.   From the above observations the Court finds that only AND TO PAY THE PREMIUM.
the two (2) swimming pools had earthquake shock Earthquake Endorsement
coverage and were heavily damaged by the earthquake, In consideration of the payment by the Insured to the Company
which struck on July 16, 1990. PCIC having admitted of the sum of P. . . . . . . . . . . . . . . . . additional premium the
that the damage to the swimming pools was appraised Company agrees, notwithstanding what is stated in the printed
by PCIC’s adjuster at P386,000.00, PCIC must, by conditions of this Policy to the contrary, that this insurance
virtue of the contract of insurance, pay Gulf Resort said covers loss or damage (including loss or damage by fire) to
amount. any of the property insured by this Policy occasioned by or
6.   Gulf Resort’s Motion for Reconsideration was denied. Thus, through or in consequence of Earthquake.
Gulf Resort filed an appeal with the Court of Appeals. After
review, the appellate court affirmed the decision of the trial Provided always that all the conditions of this Policy shall
court and ruled, thus: apply (except in so far as they may be hereby expressly varied)
a.   However, after carefully perusing the documentary and that any reference therein to loss or damage by fire should
evidence of both parties, We are not convinced that be deemed to apply also to loss or damage occasioned by or
the last two (2) insurance contracts, which the Gulf through or in consequence of Earthquake.
swimming pools. There is no mention of any premium
82.  Gulf Resort contends that pursuant to this rider, no payable for the other resort properties with regard to
qualifications were placed on the scope of the earthquake earthquake shock. This is consistent with the history of Gulf
shock coverage. Thus, the policy extended earthquake Resort’s previous insurance policies from AHAC.
shock coverage to all of the insured properties. 86.  PCIC, in compliance with the condition set by Gulf Resort,
83.  It is basic that all the provisions of the insurance policy copied AIU Policy No. 206-4568061-9 in drafting its Insurance
should be examined and interpreted in consonance with Policy No. 31944. It is true that there was variance in some
each other. All its parts are reflective of the true intent of terms, specifically in the replacement cost endorsement, but the
the parties. The policy cannot be construed piecemeal. principal provisions of the policy remained essentially similar
Certain stipulations cannot be segregated and then made to to AHAC’s policy. Consequently, we cannot apply the "fine
control; neither do particular words or phrases necessarily print" or "contract of adhesion" rule in this case as the parties
determine its character. Gulf Resort cannot focus on the intent to limit the coverage of the policy to the two swimming
earthquake shock endorsement to the exclusion of the other pools only is not ambiguous.
provisions. All the provisions and riders, taken and interpreted
together, indubitably show the intention of the parties to extend There were a lot of testimonies used as evidence of showing the intent
earthquake shock coverage to the two swimming pools only. of the parties to exclude the other properties from being insured
84.  A careful examination of the premium recapitulation will against earthquake shocks. Here’s the transcript of those testimonies:
show that it is the clear intent of the parties to extend
earthquake shock coverage only to the two swimming 87.   CROSS EXAMINATION OF LEOPOLDO MANTOHAC TSN, November 25, 1991
88.   pp. 12-13
pools. Section 2(1) of the Insurance Code defines a contract of 89.   Q. Now Mr. Mantohac, will it be correct to state also that insofar as your insurance
insurance as an agreement whereby one undertakes for a policy during the period from March 4, 1984 to March 4, 1985 the coverage on
earthquake shock was limited to the two swimming pools only?
consideration to indemnify another against loss, damage or 90.   A. Yes, sir. It is limited to the two swimming pools, specifically shown in the warranty,
liability arising from an unknown or contingent event. Thus, an there is a provision here that it was only for item 5.
91.   Q. More specifically Item 5 states the amount of P393,000.00 corresponding to the two
insurance contract exists where the following elements concur: swimming pools only?
a.   The insured has an insurable interest; 92.   A. Yes, sir.
93.   CROSS EXAMINATION OF LEOPOLDO MANTOHAC TSN, November 25, 1991
b.   The insured is subject to a risk of loss by the happening 94.   pp. 23-26
of the designated peril; 95.   Q. For the period from March 14, 1988 up to March 14, 1989, did you personally
arrange for the procurement of this policy?
c.   The insurer assumes the risk; 96.   A. Yes, sir.
d.   Such assumption of risk is part of a general scheme to 97.   Q. Did you also do this through your insurance agency?
98.   A. If you are referring to Forte Insurance Agency, yes.
distribute actual losses among a large group of persons 99.   Q. Is Forte Insurance Agency a department or division of your company?
bearing a similar risk; 100.   A. No, sir. They are our insurance agency.
101.   Q. And they are independent of your company insofar as operations are concerned?
e.   In consideration of the insurer's promise, the 102.   A. Yes, sir, they are separate entity.
insured pays a premium. 103.   Q. But insofar as the procurement of the insurance policy is concerned they are of
course subject to your instruction, is that not correct?
85.  An insurance premium is the consideration paid an insurer for 104.   A. Yes, sir. The final action is still with us although they can recommend what insurance
undertaking to indemnify the insured against a specified to take.
105.   Q. In the procurement of the insurance police (sic) from March 14, 1988 to March 14,
peril. In fire, casualty, and marine insurance, the premium 1989, did you give written instruction to Forte Insurance Agency advising it that the
payable becomes a debt as soon as the risk attaches. In the earthquake shock coverage must extend to all properties of Agoo Playa Resort in La
Union?
subject policy, no premium payments were made with
regard to earthquake shock coverage, except on the two
106.   A. No, sir. We did not make any written instruction, although we made an oral 137.   WITNESS:
instruction to that effect of extending the coverage on (sic) the other properties of the 138.   We do not normally cover earthquake shock endorsement on stand alone basis. For
company. swimming pools we do cover earthquake shock. For building we covered it for full
107.   Q. And that instruction, according to you, was very important because in April 1987 earthquake coverage which includes earthquake shock
there was an earthquake tremor in La Union? 139.   COURT:
108.   A. Yes, sir. 140.   As far as earthquake shock endorsement you do not have a specific coverage for other
109.   Q. And you wanted to protect all your properties against similar tremors in the [future], things other than swimming pool? You are covering building? They are covered by a
is that correct? general insurance?
110.   A. Yes, sir. 141.   WITNESS:
111.   Q. Now, after this policy was delivered to you did you bother to check the provisions 142.   Earthquake shock coverage could not stand alone. If we are covering building or another
with respect to your instructions that all properties must be covered again by earthquake we can issue earthquake shock solely but that the moment I see this, the thing that
shock endorsement? comes to my mind is either insuring a swimming pool, foundations, they are normally
112.   A. Are you referring to the insurance policy issued by American Home Assurance affected by earthquake but not by fire, sir.
Company marked Exhibit G? 143.   DIRECT EXAMINATION OF JUAN BARANDA III
113.   Atty. Mejia: Yes. 144.   TSN, August 11, 1992
114.   Witness: 145.   pp. 23-25
115.   A. I examined the policy and seeing that the warranty on the earthquake shock 146.   Q. Plaintiffs witness, Mr. Mantohac testified and he alleged that only Exhibits C, D, E
endorsement has no more limitation referring to the two swimming pools only, I was and F inclusive [remained] its coverage against earthquake shock to two (2) swimming
contented already that the previous limitation pertaining to the two swimming pools was pools only but that Exhibits G and H respectively entend the coverage against
already removed. earthquake shock to all the properties indicated in the respective schedules attached to
116.   Petitioner also cited and relies on the attachment of the phrase Subject to: Other Insurance said policies, what can you say about that testimony of plaintiffs witness?
Clause, Typhoon Endorsement, Earthquake Shock Endorsement, Extended Coverage 147.   WITNESS:
Endorsement, FEA Warranty & Annual Payment Agreement on Long Term Policies to 148.   As I have mentioned earlier, earthquake shock cannot stand alone without the other half
the insurance policy as proof of the intent of the parties to extend the coverage for earthquake of it. I assure you that this one covers the two swimming pools with respect to
shock. However, this phrase is merely an enumeration of the descriptive titles of the riders, earthquake shock endorsement. Based on it, if we are going to look at the premium there
clauses, warranties or endorsements to which the policy is subject, as required under Section has been no change with respect to the rates. Everytime (sic) there is a renewal if the
50, paragraph 2 of the Insurance Code. intention of the insurer was to include the earthquake shock, I think there is a substantial
117.   We also hold that no significance can be placed on the deletion of the qualification limiting the increase in the premium. We are not only going to consider the two (2) swimming pools
coverage to the two swimming pools. The earthquake shock endorsement cannot stand alone. of the other as stated in the policy. As I see, there is no increase in the amount of the
As explained by the testimony of Juan Baranda III, underwriter for AHAC-AIU: premium. I must say that the coverage was not broaden (sic) to include the other items.
118.   DIRECT EXAMINATION OF JUAN BARANDA III 149.   COURT:
119.   TSN, August 11, 1992 150.   They are the same, the premium rates?
120.   pp. 9-12 151.   WITNESS:
121.   Atty. Mejia: 152.   They are the same in the sence (sic), in the amount of the coverage. If you are going to
122.   We respectfully manifest that the same exhibits C to H inclusive have been previously do some computation based on the rates you will arrive at the same premiums, your
marked by counsel for defendant as Exhibit[s] 1-6 inclusive. Did you have occasion to Honor.
review of (sic) these six (6) policies issued by your company [in favor] of Agoo Playa 153.   CROSS-EXAMINATION OF JUAN BARANDA III
Resort? 154.   TSN, September 7, 1992
123.   WITNESS: 155.   pp. 4-6
124.   Yes[,] I remember having gone over these policies at one point of time, sir. 156.   ATTY. ANDRES:
125.   Q. Now, wach (sic) of these six (6) policies marked in evidence as Exhibits C to H 157.   Would you as a matter of practice [insure] swimming pools for fire insurance?
respectively carries an earthquake shock endorsement[?] My question to you is, on the 158.   WITNESS:
basis on (sic) the wordings indicated in Exhibits C to H respectively what was the extent 159.   No, we dont, sir.
of the coverage [against] the peril of earthquake shock as provided for in each of the six 160.   Q. That is why the phrase earthquake shock to the two (2) swimming pools only was
(6) policies? placed, is it not?
126.   x x x 161.   A. Yes, sir.
127.   WITNESS: 162.   ATTY. ANDRES:
128.   The extent of the coverage is only up to the two (2) swimming pools, sir. 163.   Will you not also agree with me that these exhibits, Exhibits G and H which you have
129.   Q. Is that for each of the six (6) policies namely: Exhibits C, D, E, F, G and H? pointed to during your direct-examination, the phrase Item no. 5 only meaning to (sic)
130.   A. Yes, sir. the two (2) swimming pools was deleted from the policies issued by AIU, is it not?
131.   ATTY. MEJIA: 164.   x x x
132.   What is your basis for stating that the coverage against earthquake shock as provided for 165.   ATTY. ANDRES:
in each of the six (6) policies extend to the two (2) swimming pools only? 166.   As an insurance executive will you not attach any significance to the deletion of the
133.   WITNESS: qualifying phrase for the policies?
134.   Because it says here in the policies, in the enumeration Earthquake Shock Endorsement, 167.   WITNESS:
in the Clauses and Warranties: Item 5 only (Earthquake Shock Endorsement), sir. 168.   My answer to that would be, the deletion of that particular phrase is inadvertent. Being a
135.   ATTY. MEJIA: company underwriter, we do not cover. . it was inadvertent because of the previous
136.   Witness referring to Exhibit C-1, your Honor.
policies that we have issued with no specific attachments, premium rates and so on. It indeed only Item 3 which were the two swimming pools have coverage for earthquake
was inadvertent, sir. shock.
169.   The Court also rejects petitioners contention that respondents contemporaneous and subsequent 197.   x x x
acts to the issuance of the insurance policy falsely gave the petitioner assurance that the 198.   Q. Now, may we know from you Engr. de Leon your basis, if any, for stating that except
coverage of the earthquake shock endorsement included all its properties in the resort. for the swimming pools all affected items have no coverage for earthquake shock?
Respondent only insured the properties as intended by the petitioner. Petitioners own witness 199.   x x x
testified to this agreement, viz: 200.   A. I based my statement on my findings, because upon my examination of the policy I
170.   CROSS EXAMINATION OF LEOPOLDO MANTOHAC found out that under Item 3 it was specific on the wordings that on the two swimming
171.   TSN, January 14, 1992 pools only, then enclosed in parenthesis (against the peril[s] of earthquake shock only),
172.   pp. 4-5 and secondly, when I examined the summary of premium payment only Item 3 which
173.   Q. Just to be clear about this particular answer of yours Mr. Witness, what exactly did refers to the swimming pools have a computation for premium payment for earthquake
you tell Atty. Omlas (sic) to copy from Exhibit H for purposes of procuring the policy shock and all the other items have no computation for payment of premiums.
from Philippine Charter Insurance Corporation? 201.   In sum, there is no ambiguity in the terms of the contract and its riders. Petitioner cannot rely
174.   A. I told him that the insurance that they will have to get will have the same provisions on the general rule that insurance contracts are contracts of adhesion which should be liberally
as this American Home Insurance Policy No. 206-4568061-9. construed in favor of the insured and strictly against the insurer company which usually
175.   Q. You are referring to Exhibit H of course? prepares it. A contract of adhesion is one wherein a party, usually a corporation, prepares the
176.   A. Yes, sir, to Exhibit H. stipulations in the contract, while the other party merely affixes his signature or his "adhesion"
177.   Q. So, all the provisions here will be the same except that of the premium rates? thereto. Through the years, the courts have held that in these type of contracts, the parties do
178.   A. Yes, sir. He assured me that with regards to the insurance premium rates that they not bargain on equal footing, the weaker party's participation being reduced to the alternative to
will be charging will be limited to this one. I (sic) can even be lesser. take it or leave it. Thus, these contracts are viewed as traps for the weaker party whom the
179.   CROSS EXAMINATION OF LEOPOLDO MANTOHAC courts of justice must protect. Consequently, any ambiguity therein is resolved against the
180.   TSN, January 14, 1992 insurer, or construed liberally in favor of the insured.
181.   pp. 12-14 202.   The case law will show that this Court will only rule out blind adherence to terms where facts
182.   Atty. Mejia: and circumstances will show that they are basically one-sided. Thus, we have called on lower
183.   Q. Will it be correct to state[,] Mr. Witness, that you made a comparison of the courts to remain careful in scrutinizing the factual circumstances behind each case to determine
provisions and scope of coverage of Exhibits I and H sometime in the third week of the efficacy of the claims of contending parties. In Development Bank of the Philippines v.
March, 1990 or thereabout? National Merchandising Corporation, et al., the parties, who were acute businessmen of
184.   A. Yes, sir, about that time. experience, were presumed to have assented to the assailed documents with full knowledge.
185.   Q. And at that time did you notice any discrepancy or difference between the policy 203.   We cannot apply the general rule on contracts of adhesion to the case at bar. Petitioner cannot
wordings as well as scope of coverage of Exhibits I and H respectively? claim it did not know the provisions of the policy. From the inception of the policy, petitioner
186.   A. No, sir, I did not discover any difference inasmuch (sic) as I was assured already that had required the respondent to copy verbatim the provisions and terms of its latest insurance
the policy wordings and rates were copied from the insurance policy I sent them but it policy from AHAC-AIU. The testimony of Mr. Leopoldo Mantohac, a direct participant in
was only when this case erupted that we discovered some discrepancies. securing the insurance policy of petitioner, is reflective of petitioners knowledge, viz:
187.   Q. With respect to the items declared for insurance coverage did you notice any 204.   DIRECT EXAMINATION OF LEOPOLDO MANTOHAC
discrepancy at any time between those indicated in Exhibit I and those indicated in 205.   TSN, September 23, 1991
Exhibit H respectively? 206.   pp. 20-21
188.   A. With regard to the wordings I did not notice any difference because it was exactly the 207.   Q. Did you indicate to Atty. Omlas (sic) what kind of policy you would want for those
same P393,000.00 on the two (2) swimming pools only against the peril of earthquake facilities in Agoo Playa?
shock which I understood before that this provision will have to be placed here because 208.   A. Yes, sir. I told him that I will agree to that renewal of this policy under Philippine
this particular provision under the peril of earthquake shock only is requested because Charter Insurance Corporation as long as it will follow the same or exact provisions of
this is an insurance policy and therefore cannot be insured against fire, so this has to be the previous insurance policy we had with American Home Assurance Corporation.
placed. 209.   Q. Did you take any step Mr. Witness to ensure that the provisions which you wanted in
189.   The verbal assurances allegedly given by respondents representative Atty. Umlas were not the American Home Insurance policy are to be incorporated in the PCIC policy?
proved. Atty. Umlas categorically denied having given such assurances. 210.   A. Yes, sir.
190.   Finally, petitioner puts much stress on the letter of respondents independent claims adjuster, 211.   Q. What steps did you take?
Bayne Adjusters and Surveyors, Inc. But as testified to by the representative of Bayne 212.   A. When I examined the policy of the Philippine Charter Insurance Corporation I specifically
Adjusters and Surveyors, Inc., respondent never meant to lead petitioner to believe that the told him that the policy and wordings shall be copied from the AIU Policy No. 206-4568061-9.
endorsement for earthquake shock covered properties other than the two swimming pools, viz:
191.   DIRECT EXAMINATION OF ALBERTO DE LEON (Bayne
192.   Adjusters and Surveyors, Inc.)
193.   TSN, January 26, 1993
194.   pp. 22-26
195.   Q. Do you recall the circumstances that led to your discussion regarding the extent of
coverage of the policy issued by Philippine Charter Insurance Corporation?
196.   A. I remember that when I returned to the office after the inspection, I got a photocopy
of the insurance coverage policy and it was indicated under Item 3 specifically that the
coverage is only for earthquake shock. Then, I remember I had a talk with Atty. Umlas
(sic), and I relayed to him what I had found out in the policy and he confirmed to me
002 Pineda v. CA (for Gueco c/o Daguman) reduced. The most persuasive rationale for adopting the view that the employer
September 27, 1993 | Davide Jr., J. | Group Policy acts as the agent of the insurer, however, is that the employee has no knowledge
of or control over the employer's actions in handling the policy or its
PETITIONER: LUZ PINEDA, MARILOU MONTENEGRO, VIRGINIA administration. An agency relationship is based upon consent by one person that
ALARCON, DINA LORENA AYO, CELIA CALUMBAG and LUCIA another shall act in his behalf and be subject to his control. It is clear from the
LONTOK, evidence regarding procedural techniques here that the insurer-employer
RESPONDENTS: HON. COURT OF APPEALS and THE INSULAR LIFE relationship meets this agency test with regard to the administration of the
ASSURANCE COMPANY, LIMITED policy, whereas that between the employer and its employees fails to reflect true
agency. The insurer directs the performance of the employer's administrative
SUMMARY: Prime Marine Services, Inc. (PMSI), a crewing/manning outfit, acts, and if these duties are not undertaken properly the insurer is in a position to
procured a Group Policy from Insular Life Assurance Co., Ltd. to provide life exercise more constricted control over the employer's conduct. In the light of the
insurance coverage to its sea-based employees. During the effectivity of the above disquisitions and after an examination of the facts of this case, we hold
policy, six covered employees perished at sea when their vessel sunk. They were that PMSI, through its President and General Manager, Capt. Nuval, acted as the
survived by the complainants-appellees, the beneficiaries under the policy. The agent of Insular Life. The latter is thus bound by the misconduct of its agent.
beneficiaries, except the spouses Alarcon, executed special powers of attorney
authorizing Capt. Nuval, President and General Manager of PMSI, to, among DOCTRINE: Group insurance is a comparatively new form of insurance. In the
others, “follow-up, ask, demand, collect and receive” for their benefit United States, the first modern group insurance policies appear to have been
indemnities of sums of money due them relative to the sinking of the vessel. By issued in 1911 by the Equitable Life Assurance Society. Group insurance is
virtue of these written powers of attorney, complainants-appellees were able to essentially a single insurance contract that provides coverage for many
receive their respective death benefits. Unknown to them, however, PMSI, in its individuals. In its original and most common form, group insurance provides life
capacity as employer and policyholder of the life insurance of its deceased or health insurance coverage for the employees of one employer.
workers, filed with Insular Life formal claims for and in behalf of the
beneficiaries, through Capt. Nuval. On the basis of the five special powers of We are convinced that the employer is the agent of the insurer in performing the
attorney, Insular Life drew against its account six (6) checks, four for duties of administering group insurance policies. It cannot be said that the
P200,000.00 each, one for P50,000.00 and another for P40,000.00 payable to the employer acts entirely for its own benefit or for the benefit of its employees in
order of complainants-appellees. Capt. Nuval, upon receipt of these checks undertaking administrative functions
endorsed and deposited them in his own account.
When Pineda et.al. learned that they were entitled, as beneficiaries, to life FACTS:
insurance benefits under a group policy, they sought to recover these benefits 121.  September 1983, Prime Marine Services Inc. (PMSI), a manning agency,
from Insular Life but the latter denied their claim on the ground that their procured a Group Policy from Insular Life Assurance Co. (Insular Life) to
liability was already extinguished. WoN the CA erred in the appreciation of the provide life insurance coverage to its sea-based employees enrolled under
SPA to include granting of authority to Capt. Nuval to obtain the insurance the plan.
proceeds from Insular Life? YES, because the special powers of attorney "do not
122.  During the effectivity of the policy, six covered employees of the PMSI
contain in unequivocal and clear terms authority to Capt. Nuval to obtain,
perished at sea when their vessel, M/V Nemos sunk in Morocco. They were
receive, receipt from respondent company insurance proceeds arising from the
death of the seaman-insured. On the contrary, the said powers of attorney are survived by Pineda et.al., the beneficiaries under the policy.
couched in terms which could easily arouse suspicion of an ordinary man. We 123.  Pineda et. al, sought to claim death benefits, they approached the President
are convinced that the employer is the agent of the insurer in performing the and General Manager of PMSI, Capt. Nuval. The latter evinced willingness
duties of administering group insurance policies. It cannot be said that the to assist Pineda to recover Overseas Workers Welfare Administration
employer acts entirely for its own benefit or for the benefit of its employees in (OWWA) benefits from the POEA arising frim the deaths of their
undertaking administrative functions. While a reduced premium may result if the husbands/sons.
employer relieves the insurer of these tasks, and this, of course, is advantageous 124.  They executed, with the exception of spouses Alarcon, a special power of
to both the employer and the employees, the insurer also enjoys significant attorney authorizing Capt. Nuval to “follow up, ask, demand, collect and
advantages from the arrangement. The reduction in the premium which results receive” for the benefit indemnities of sums due to them relative to the
from employer-administration permits the insurer to realize a larger volume of sinking of M/V Nemos. By virtue of this authority, they were able to
sales, and at the same time the insurer's own administrative costs are markedly
receive death benefits.
125.  Unknown to them, PMSI, in its capacity as employer and policyholder of released two checks in the amount of P150,000.00 for the three minor
the life insurance of the deceased workers, filed formal claims with Insular children (P50,000.00 each) of complainant, Dina Ayo and another check of
Life in behalf of the beneficiaries through Capt. Nuval. Among the P40,000.00 for minor beneficiary Marissa Lontok, daughter of another
documents for the processing of the claims were the five SPAs executed by complainant Lucia Lontok, there being no showing of any court
Pineda et.al. authorization presented or the requisite bond posted.
126.  Insular Life drew against its bank six checks four for 200k each, one for 50k 130.  On the other hand, the CA ruled that the powers of attorney, Exhibits "1" to
and another for 40k, payable to the order of Pineda et al. These checks were "5," relied upon by Insular Life were sufficient to authorize Capt. Nuval to
released to the treasurer of PMSI, who happened to be Capt. Nuval’s son- receive the proceeds of the insurance pertaining to the beneficiaries.
in-law deposited them in his account with Boston Bank. 131.  Hence, this issue.
127.  Pineda et.al. learned that they were entitled, as beneficiaries under the group
policy with Insular Life sought to recover these but the latter denied their
claim on the ground that the liability of Insular Life was already
ISSUE/s:
extinguished upon deliver to PMSI the six checks issued under their names.
1.   WoN the CA erred in the appreciation of the SPA to include granting of
128.  Pineda et.al. filed an administrative complaint against private respondent
authority to Capt. Nuval to obtain the insurance proceeds from Insular Life?
Insular Life which was filed with the Insurance Commission on 20
YES, because the special powers of attorney "do not contain in unequivocal
September 1989. They prayed therein that after due proceedings, Insular
and clear terms authority to Capt. Nuval to obtain, receive, receipt from
Life "be ordered to pay the claimants their insurance claims" and that
respondent company insurance proceeds arising from the death of the
"proper sanctions/penalties be imposed on" it "for its deliberate, feckless
seaman-insured. On the contrary, the said powers of attorney are couched in
violation of its contractual obligations to the complainants, and of the
terms which could easily arouse suspicion of an ordinary man.
Insurance Code." 5 Insular Life's motion to dismiss the complaint on the
ground that "the claims of complainants are all respectively beyond the
jurisdiction of the Insurance Commission as provided in Section 416 of the RULING: Petition is hereby affirmed.
Insurance Code.
129.  The Insurance Commission made the following conclusions: After taking RATIO:
into consideration the evidences [sic], testimonial and documentary for the
complainants and the respondent, the Commission finds that; First: The 1.   We rule for the petitioners Pineda et. al.
respondent erred in appreciating that the powers of attorney executed by 2.   We have carefully examined the specific powers of attorney, Exhibits "1" to
five (5) of the several beneficiaries convey absolute authority to Capt. "5," which were executed by petitioners Luz Pineda, Lucia B. Lontok, Dina
Nuval, to demand, receive, receipt and take delivery of insurance proceeds Ayo, Celia Calumag, and Marilyn Montenegro, respectively, on 14 May
from respondent Insular Life. A cursory reading of the questioned powers 1986 and uniformly granted to Capt. Rosendo Nuval the following powers:
of authority would disclosed [sic] that they do not contain in unequivocal
and clear terms authority to Capt. Nuval to obtain, receive, receipt from To follow-up, ask, demand, collect and receipt for my benefit
indemnities or sum of money due me relative to the sinking of
respondent company insurance proceeds arising from the death of the
M.V. NEMOS in the vicinity of El Jadida, Casablanca,
seaman-insured. On the contrary, the said powers of attorney are couched in
Morocco on the evening of February 17, 1986; and
terms which could easily arouse suspicion of an ordinary man. Insular Life
did not observe Section 18034 of the Insurance Code, when it issued or
To sign receipts, documents, pertinent waivers of indemnities
or other writings of whatsoever nature with any and all third
                                                                                                                       
                                                                                                                                                                                                                                                                                       
34 court authority or the giving of a bond where the interest of the minor in the
Section 180 is quotes [sic] partly as follows: In the absence of a judicial guardian,
particular act involved does not exceed twenty thousand pesos . . . .
the father, or in the latter's absence or incapacity, the mother of any minor, who is an
insured or a beneficiary under a contract of life, health or accident insurance, may
exercise, in behalf of said minor, any right, under the policy, without necessity of  
persons, concerns and entities, upon terms and conditions Q: What is the reason why policyholders are the ones who file the claim and
acceptable to my said attorney. not the designated beneficiaries of the employees of the policyholders?

3. We agree with the Insurance Commission that the special powers of attorney "do A: Yes because group insurance is normally taken by the employer as an
not contain in unequivocal and clear terms authority to Capt. Nuval to obtain, employee-benefit program and as such, the benefit should be awarded by the
receive, receipt from respondent company insurance proceeds arising from the death policyholder to make it appear that the benefit really is given by the employer
of the seaman-insured. On the contrary, the said powers of attorney are couched in
terms which could easily arouse suspicion of an ordinary man." The holding of the 6. On cross-examination, Urbano further elaborated that even payments, among other
CA to the contrary is principally premised on its opinion that: things, are coursed through the policyholder:

[t]here is nothing in the law which mandates a specific or special Q: What is the corporate concept of group insurance insofar as Insular Life is
power of attorney to be executed to collect insurance proceeds. concerned?
Such authority is not included in the enumeration of art. 1878 of
the New Civil Code. Neither do we perceive collection of
WITNESS:
insurance claims as an act of strict dominion as to require a special
power of attorney.
A: Group insurance is a contract where a group of individuals are covered
under one master contract. The individual underwriting characteristics of each
4. If this be so, then they could not have been meant to be a general power of
individual is not considered in the determination of whether the individual is
attorney since Exhibits "1" to "5" are special powers of attorney. The execution by
insurable or not. The contract is between the policyholder and the insurance
the principals of special powers of attorney, which clearly appeared to be in prepared company. In our case, it is Prime Marine and Insular Life. We do not have
forms and only had to be filled up with their names, residences, dates of execution, contractual obligations with the individual employees; it is between Prime
dates of acknowledgment and others, excludes any intent to grant a general power of
Marine and Insular Life.
attorney or to constitute a universal agency. Being special powers of attorney, they
must be strictly construed.
Q: And so it is part of that concept that all inquiries, follow-up, payment of
claims, premium billings, etc. should always be coursed thru the policyholder?
5. Certainly, it would be highly imprudent to read into the special powers of attorney
in question the power to collect and receive the insurance proceeds due to Pineda
et.al from Group Policy No. G-004694. Insular Life knew that a power of attorney in A: Yes that is our practice.
favor of Capt. Nuval for the collection and receipt of such proceeds was a deviation
from its practice with respect to group policies. Such practice was testified to by Mr. Q: And when you say claim payments should always be coursed thru the
Marciano Urbano, Insular Life's Assistant Manager of the Group Administrative policyholder, do you require a power of attorney to be presented by the
Department policyholder or not?

ATTY. CAGUIOA: a Not necessarily.

Can you explain to us why in this case, the claim was filed by a certain Capt. 7. This practice is usual in the group insurance business and is consistent with the
Noval [sic]? jurisprudence thereon in the State of California — from whose laws our Insurance
Code has been mainly patterned — which holds that the employer-policyholder is
WITNESS: the agent of the insurer.

A: The practice of our company in claim pertaining to group insurance, the 8. Group insurance is a comparatively new form of insurance. In the United States,
policyholder is the one who files the claim for the beneficiaries of the deceased. the first modern group insurance policies appear to have been issued in 1911 by the
At that time, Capt. Noval [sic] is the President and General Manager of Prime Equitable Life Assurance Society. Group insurance is essentially a single insurance
Marine. contract that provides coverage for many individuals. In its original and most
common form, group insurance provides life or health insurance coverage for the sales, and at the same time the insurer’s own administrative costs
employees of one employer. are markedly reduced.

9. The coverage terms for group insurance are usually stated in a master agreement Xxx xxx xxx
or policy that is issued by the insurer to a representative of the group or to an
administrator of the insurance program, such as an employer. The employer acts as a The most persuasive rationale for adopting the view that the
functionary in the collection and payment of premiums and in performing related employer acts as the agent of the insurer, however, is that the
duties. Likewise falling within the ambit of administration of a group policy is the employee has no knowledge of or control over the employer’s
disbursement of insurance payments by the employer to the employees. Most actions in handling the policy or its administration. An agency
policies, such as the one in this case, require an employee to pay a portion of the relationship is based upon consent by one person that another shall
premium, which the employer deducts from wages while the remainder is paid by the act in his behalf and be subject to his control. It is clear from the
employer. This is known as a contributory plan as compared to a non-contributory evidence regarding procedural techniques here that the insurer-
plan where the premiums are solely paid by the employer. employer relationship meets this agency test with regard to the
administration of the policy, whereas that between the employer
10. Although the employer may be the titular or named insured, the insurance is and its employees fails to reflect true agency. The insurer directs
actually related to the life and health of the employee. Indeed, the employee is in the the performance of the employer’s administrative acts, and if these
position of a real party to the master policy, and even in a non-contributory plan, the duties are not undertaken properly the insurer is in a position to
payment by the employer of the entire premium is a part of the total compensation exercise more constricted control over the employer’s conduct.
paid for the services of the employee. Put differently, the labor of the employees is
the true source of the benefits, which are a form of additional compensation to them. 14.   In the light of the above disquisitions and after an examination of the facts
of this case, we hold that PMSI, through its President and General Manager,
11. It has been stated that every problem concerning group insurance presented to a Capt. Nuval, acted as the agent of Insular Life. The latter is thus bound by
court should be approached with the purpose of giving to it every legitimate the misconduct of its agent.
opportunity of becoming a social agency of real consequence considering that the 15.   Insular Life, however, likewise recognized Capt. Nuval as the attorney-in-
primary aim is to provide the employer with a means of procuring insurance fact of the petitioners. Unfortunately, through its official, Mr. Urbano, it
protection for his employees and their families at the lowest possible cost, and in so acted imprudently and negligently in the premises by relying without
doing, the employer creates goodwill with his employees, enables the employees to question on the special power of attorney. In Strong vs. Repide, this Court
carry a larger amount of insurance than they could otherwise, and helps to attract and ruled that it is among the established principles in the civil law of Europe as
hold a permanent class of employees. well as the common law of American that third persons deal with agents at
their peril and are bound to inquire as to the extent of the power of the agent
12. In Elfstrom vs. New York Life Insurance Company, 27 the California Supreme with whom they contract
Court explicitly ruled that in group insurance policies, the employer is the agent of 16.   Even granting for the sake of argument that the special powers of attorney
the insurer. Thus: were in due form, Insular Life was grossly negligent in delivering the
checks, drawn in favor of the petitioners, to a party who is not the agent
We are convinced that the employer is the agent of the insurer in mentioned in the special power of attorney.
performing the duties of administering group insurance policies. It
cannot be said that the employer acts entirely for its own benefit or MINOR ISSUE
for the benefit of its employees in undertaking administrative
functions. While a reduced premium may result if the employer 17.   Nor can we agree with the opinion of the public respondent that since the
relieves the insurer of these tasks, and this, of course, is shares of the minors in the insurance proceeds are less than P50,000.00,
advantageous to both the employer and the employees, the insurer then under Article 225 of the Family Code their mothers could receive such
also enjoys significant advantages from the arrangement. The shares without need of either court appointments as guardian or the posting
reduction in the premium which results from employer- of a bond. It is of the view that said Article had repealed the third paragraph
administration permits the insurer to realize a larger volume of of Section 180 of the Insurance Code. The pertinent portion of Article 225
of the Family Code reads as follows:
Art. 225. The father and the mother shall jointly exercise legal
guardianship over the property of their unemancipated common
child without the necessity of a court appointment. In case of
disagreement, the father's decision shall prevail, unless there is
judicial order to the contrary.

Where the market value of the property or the annual income of the
child exceeds P50,000, the parent concerned shall be required to
furnish a bond in such amount as the court may determine, but not
less than ten per centum (10%) of the value of the property or
annual income, to guarantee the performance of the obligations
prescribed for general guardians.

18.   It is clear from the said Article that regardless of the value of the
unemancipated common child's property, the father and mother ipso
jure become the legal guardian of the child's property. However, if the
market value of the property or the annual income of the child exceeds
P50,000.00, a bond has to be posted by the parents concerned to guarantee
the performance of the obligations of a general guardian.
003 Vda. De Sindayen v. Insular (GUSTILO) DOCTRINE: When the policy is issued and delivered, in the absence of fraud or
September 4, 1935| Butte, J | Delivery of the Policy other grounds for rescission, it is plainly not within the intention of the parties that
there should be any questions held in abeyance or reserved for future determination
PETITIONER: Fortunata Lucero Viuda De Sindayen that leave the very existence of the contract in suspense and doubt. It is therefore in
RESPONDENTS: Insular Life Assurance Co. Ltd the public interest, for the public is profoundly and generally interested in life
insurance, as well as in the interest of the insurance companies themselves by giving
SUMMARY: A. Sindayen was employed as a linotype operator in the Bureau of certainly and security to their policies, that we are constrained to hold, as we, do, that
Printing at Manila.He was in Camiling Tarlac with his wife to spend Christmas the delivery of the policy to the insured by an agent of the company who is
vacation with his aunt, Felicidad Estrada. While there he made a written application authorized to make delivery or without delivery is the final act which binds the
to Insular for a policy of insurance on his life in the sum of P1,000 and he paid to the company and the insured as well in the absence of fraud or other legal ground for
agent Mendoza P15 cash as part of the first premium. It was agreed with the agent, rescission.
Mendoza that the policy, when and if issued, should be delivered to his aunt. Estrada, FACTS:
with whom A. Sindayen left the sum of P26.06 to complete the payment of the first 95.   Arturo Sindayen (A. Sindayen), up to the time of his death on January 19,
annual premium of P40.06. On January 1, 1933, A. Sindayen, was examined by 1933, was employed as a linotype operator in the Bureau of Printing at
Insular’'s doctor who made a favorable report, to Insular. The next day, January 2, A. Manila and had been such for eleven years prior thereto. He and his wife
Sindayen returned to Manila and resumed his work as a linotype operator in the went to Camiling, Tarlac, to spend the Christmas vacation with his aunt,
Bureau of Printing. On January 11, Insular accepted the risk and issued policy No. Felicidad Estrada (Estrada).
47710 dated back to December 1, 1932, and mailed the same to its agent, Mendoza, 96.   While there he made a written application on December 26, 1932, to Insular
for delivery to A Sindayen which was eventually received by Estrada upon her Life Assurance Co., Ltd. (Insular) through its agent, Cristobal Mendoza
payment of the balance of the 1st year premium. (Mendoza), for a policy of insurance on his life in the sum of P1,000 and he
A.  Sindayen died on January 19 and on January 20 Mendoza, learned of the death of paid to the agent Mendoza P15 cash as part of the first premium. It was
A. Sindayen and called on Estrada and asked her to return the policy but Mendoza agreed with the agent, Mendoza that the policy, when and if issued, should
did not return or offer to return the premium paid. Estrada on his aforesaid statement be delivered to his aunt. Estrada, with whom A. Sindayen left the sum of
gave him the policy. Insular obtained from the beneficiary, the widow of A, P26.06 to complete the payment of the first annual premium of P40.06.
Sindayen, her signature to a legal document entitled "ACCORD, SATISFACTION 97.   On January 1, 1933, A. Sindayen, who was then twenty-nine years of age,
AND RELEASE" whereby in consideration of the sum of P40.06 paid to her by a was examined by Insular’'s doctor who made a favorable report, to Insular.
check of Insular, she "assigns, releases and forever discharges said Insular, its The next day, January 2, A. Sindayen returned to Manila and resumed his
successors and assigns, of all claims, obligation in or indebtedness which she, as work as a linotype operator in the Bureau of Printing. On January 11, 1933,
such beneficiary ever had. The said check for P40.06 was never cashed but returned Insular accepted the risk and issued policy No. 47710 dated back to
to Insular. Thereupon this action was brought to enforce payment of the policy. The December 1, 1932, and mailed the same to its agent, Mendoza, for delivery
issue is WoN the policy took effect?- YES because Mendoza was an agent of to A Sindayen. On January 11, 1933, A. Sindayen was at work in the
Insular and his decision to deliver the policy is binding on the company. Bureau of Printing. On January 12, he complained of a severe headache and
The main defense of Insular is that the said policy never took effect because of remained at home. On January 15, he called a physician who found that he
paragraph 3 of the application above quoted, for at the time of its delivery by the was suffering from acute nephritis and uremia. His illness did not yield to
agent as aforesaid the insured was not in good health. The SC held that Mendoza was treatment and on January 19, he died.
duly licensed by the Insurance Commissioner to act as the agent of the Insular. The 98.   The policy which Insular issued and mailed in Manila on January 11 was
well known custom of the insurance business and the evidence in this case prove that received by its agent, Mendoza, in Camiling, Tarlac, on January 16. On
Mendoza was not regarded by Insular as a mere conduit or automaton for the January 18, the agent, Mendoza, in accordance with his agreement with the
performance of the physical act of placing the policy in the hands of the insured. insured, delivered the policy to Estrada upon her payment of the balance of
Further, when the policy is issued and delivered, in the absence of fraud or other the first year's annual premium. The agent, Mendoza, asked Estrada if her
grounds for rescission, it is plainly not within the intention of the parties that there nephew was in good health and she replied that she believed so because she
should be any questions held in abeyance or reserved for future determination that had no information that he was sick and he thereupon delivered to her the
leave the very existence of the contract in suspense and doubt. If this were not so, the policy.
entire business world which deals so voluminously in insurance would be affected by 99.   On January 20 the agent, Mendoza, learned of the death of A. Sindayen and
this uncertainly. called on Estrada and asked her to return the policy but Mendoza did not
return or offer to return the premium paid. Estrada on his aforesaid 3.  WoN there was delivery of the policy even if it was not delivered to the
statement gave him the policy. insured himself? YES because delivery to the insured in person is not necessary.
100.  On February 4, Insular obtained from the beneficiary, the widow of A, Delivery may be made by mail or to a duly constituted agent.
Sindayen, her signature to a legal document entitled "ACCORD,
SATISFACTION AND RELEASE" whereby in consideration of the sum of RULING :In view of the premises, we hold that Insular assumed the risk covered by
P40.06 paid to her by a check of Insular, she "assigns, releases and forever policy No. 47710 on the life of A. Sindayen on January 18, 1933, the date when the
discharges said Insular, its successors and assigns, of all claims, obligation policy was delivered to A Sindayen. The judgment appealed from is therefore
in or indebtedness which she, as such beneficiary ever had or now has, reversed with directions to enter judgment against Insular in the sum of P1,000
hereafter shall, or may have, for, upon, or by reason of said policy of life together with interest at the legal rate from and after May 4, 1933, with costs in both
insurance numbered 47710 upon the life of said A. Sindayen, the latter now instances against Insular.
deceased, or arising therefrom or connected therewith in any manner", RATIO:
101.  The said check for P40.06 was never cashed but returned to Insular. Main issue:
Thereupon this action was brought to enforce payment of the policy. By the 104.  The main defense of Insular is that the said policy never took effect because
terms of the policy, an annual premium of P40.06 is due on the first day of of paragraph 3 of the application above quoted, for at the time of its
December of each year, the first premium already paid by the insured delivery by the agent as aforesaid the insured was not in good health. We
covering the period from December 1, 1932 which is to December 1, 1933. have not heretofore been called upon to interpret and apply this clause in
It is to be noted that the policy was not issued and Insular assumed no actual life insurance application, but identical or substantially identical clauses
risk prior to January 11, 1933. have been construed and applied in a number of cases in the United States
102.  The policy contains the following paragraph: THE CONTRACT. This and the decisions thereon are far from uniform or harmonious. We do not
Policy and the application herefor constitute the entire contract between the find it practicable to attempt to determine where the weight of the authority
parties hereto. All statements made by the Insured shall, in the absence lies and propose to resolve this case on its own facts.
of fraud, be deemed representations and not warranties, and no such 105.  There is one line of cases which holds that the stipulation contained in
statement shall void the Policy unless it is contained in the written paragraph 3 is in the nature of a condition precedent, that is to say, that
application, a copy of which is attached to this Policy. Only the there can be no valid delivery to the insured unless he is in good health at
President, or the Manager, acting jointly with the Secretary or Assistant the time; that this condition precedent goes to the very essence of the
Secretary (and then only in writing signed by them) have power in behalf of contract and cannot be waived by the agent making delivery of the policy.
the Company to issue permits, or to modify this or any contract, or to On the other hand, a number of American decisions hold that an agent to
extend the time for making any premium payment, and the Company shall whom a life insurance policy similar to the one here involved was sent with
bound by any promise or representation heretofore hereafter given by any instructions to deliver it to the insured has authority to bind the company by
person other than the above-named officials, and by them only in writing making such delivery, although the insured was not in good health at the
and signed conjointly as stated. time of delivery, on the theory that the delivery of the policy being the final
103.  The application which A. Sindayen signed in Camiling, Tarlac, on act to the consummation of the contract, the condition as to the insurer's
December 26, 1932, contained among others the following provisions: (a) good health was waived by the company.
That if this application is accepted and a policy issued in my favor, I bind 106.  A number of these cases go to the of holding that the delivery of the policy
myself to accept the same and to pay at least the first year's premium by the agent to the insured consummates the contract even though the agent
thereon in the City of Manila; (b) That the said policy shall not take effect knew that the insured was not in good health at the time, the theory being
until the first premium has been paid and the policy has been delivered to that his knowledge is the company's knowledge and his delivery of the
and accepted by me, while I am in good health; (c) That the agent taking policy is the company's delivery; that when the delivery is made
this application has no authority to make, modify or discharge contracts, or notwithstanding this knowledge of the defect, the company is deemed to
to waive any of the Company's right or requirements. have waived the defect. Although that appears to be the prevailing view in
ISSUE/s: the American decisions and leads to the same conclusion, namely, that the
1.  WoN the policy took effect?- YES because Mendoza was an agent of Insular and act of delivery of the policy in the absence of fraud or other ground for
his decision to deliver the policy is binding on the company recission consummates the insurance, we are inclined to the view that it is
Other issues: more consonant with the well known practice of life insurance companies
2.  WoN the Accord, Satisfaction & Releae made by Insular is fraudulent?-YES and the evidence in the present case to rest our decision on the proposition
it is inequitable that Mendoza was authorized by the company to make the delivery of the
policy when he received the payment of the first premium and he was absence of fraud or other grounds for rescission, it is plainly not within
satisfied that the insured was in good health. the intention of the parties that there should be any questions held in
107.  Mendoza was duly licensed by the Insurance Commissioner to act as abeyance or reserved for future determination that leave the very
the agent of the Insular. The well known custom of the insurance existence of the contract in suspense and doubt. If this were not so, the
business and the evidence in this case prove that Mendoza was not entire business world which deals so voluminously in insurance would be
regarded by Insular as a mere conduit or automaton for the affected by this uncertainly.
performance of the physical act of placing the policy in the hands of the 111.  It is therefore in the public interest, for the public is profoundly and
insured. If Mendoza were only an automaton then the legally effective generally interested in life insurance, as well as in the interest of the
delivery of the policy and the consummation of the contract occurred when insurance companies themselves by giving certainly and security to
Insular expressed its will to release the policy by mailing it to Mendoza, their policies, that we are constrained to hold, as we, do, that the
namely, on January 11, 1933. In such a case Mendoza would perform a delivery of the policy to the insured by an agent of the company who is
purely ministerial act and have no discretion. He could do nothing but make authorized to make delivery or without delivery is the final act which
unconditional delivery. The legal result would be the same as if Insular had binds the company and the insured as well in the absence of fraud or
mailed the policy on January 11, to the insured directly using the post-office other legal ground for rescission. The fact that the agent to whom it has
as its conduit for delivery. On January 11, A. Sindayen was in good health entrusted this duty and corporation can only act through agents is derelict or
performing his regular duties in the Bureau of Printing. negligent or even dishonest in the performance of the duty which has been
108.  But we are not inclined to take such a restrictive view of the agent's entrusted to him would create a liability of the agent to the company but
authority because the evidence in the record shows that Mendoza had the does not resolve the company's obligation based upon the authorized acts of
authority, given him by Insular, to withhold the delivery of the policy to the agent toward a third party who was not in collusion with the agent.
the insured "until the first premium has been paid and the policy has 112.  Paragraph 4 of the application to the effect "that the agent taking this
been delivered to and accepted by the insured while I am in good application has no authority to make, modify or discharge contracts or to
health". Whether that condition had been met or not plainly calls for waive any of the company's rights or requirements" is not in point.
the exercise of discretion. Granted that Mendoza's decision that the Mendoza neither waived nor pretended to waive any right or requirement of
condition had been met by the insured and that it was proper to make a Insular. In fact, his inquiry as to the state of health of the insured discloses
delivery of the policy to him is just as binding on the company as if the that he was endeavoring to assure himself that this requirement of Insular
decision had been made by its board of directors. Granted that Mendoza had been satisfied. In doing so, he acted within the authority conferred
made a mistake of judgement because he acted on insufficient evidence as on him by his agency and his acts within that authority bind the
to the state of health of the insured. But it is not charged that the mistake company. Insular therefore having decided that all the conditions
was induced by any misconduct or omission of duty of the insured. precedent to the taking effect of the policy had been complied with and
109.  It is the interest not only the applicant but of all insurance companies having accepted the premium and delivered the policy thereafter to the
as well that there should be some act which gives the applicant the insured, Insular is now estopped to assert that it never intended that
definite assurance that the contract has been consummated. This sense the policy should take effect.
of security and of peace of mind that one's defendants are provided for Accord, Satisfaction & Release is Fraudulent
without risk either of loss or of litigation is the bedrock of life 113.  Insular does not set up any defense of fraud, misconduct or omission of
insurance. A cloud will be thrown over the entire insurance business if the duty of A. Sindayen or his agent, Estrada or of the beneficiary. In its answer
condition of health of the the insured at the time of delivery of the policy it pleads the "ACCORD, SATISFACTION AND RELEASE" signed by the
may be required into years afterwards with the view to avoiding the policy widow of A. Sindayen.
on the ground that it never took effect because of an alleged lack of good 114.  With respect to the Accord, Satisfaction & Release, it suffices to say
health, at the time of delivery. that this release is so inequitable, not to say fraudulent.
110.  It is difficult to imagine that the insurance company would take such a Delivery of the Policy
position in the face of the common belief of the insuring public that when 115.  It is suggested in Insular’'s brief that fhere was no delivery of the policy in
the policy is delivered, in the absence of fraud or other grounds for this case because the policy was not delivered to and accepted by A.
rescission, the contract of insurance is consummated. The insured rests and Sindayen in person.
acts on that faith. So does the insurance company, for that matter, for from 116.  Delivery to the insured in person is not necessary. Delivery may be made
the date of delivery of the policy it appropriates to its own use the premium by mail or to a duly constituted agent. Insular cites no authorities to support
paid by the insured. When the policy is issued and delivered, in the its proposition and none need be cited to refute it.
Dissenting of Imperial, J.:
117.   The stipulation that the insurance contract shall produce no effect unless the payment of the first premium
and the delivery of the policy be made when the insured is in good health, is not in conflict with any
provision of the Insurance Law now in force, nor with any other law of a general character; neither is said
stipulation contrary to morals or public order, and therefore the same is valid and binding upon the parties.
118.   The majority opinion states that the delivery of the policy by the agent after he has made use of the
discretion conferred upon him by Insular to deliver or to withhold said policy, is binding upon Insular and
the latter cannot evade the consequences thereof. This same legal question has been raised before various
appellate courts of several states of the Union, which made a distinction between agents whose only power
consisted in soliciting insurance and in delivering policies and those who, in addition to such power, were
authorized to issue policies and accept risks on behalf of insurance companies. In the first case the doctrine
is uniform that the acts of agents with limited powers are not binding upon the insurance companies, whereas
in the second case the acts of the agents bind and prejudice the insurance companies represented by them.
119.   It is clear, therefore, that the delivery of the policy by Mendoza does not bind Insular nor is Insulart estopped
from alleging its defense, for the simple reason that Mendoza was not an agent with authority to issue
policies or to accept risks in the name of his principle.
120.   However, it it said that Insular ikewise waived the defense which has hereinbefore been extensively
considered, because it failed to return the first premium collected, and this alleged failure is predicated upon
the statement contained in the penultimate paragraph of the instrument stating that the check for P40.06 was
returned to the widow in consideration of her waiver of any claim whatsoever. A careful reading of the
instrument will convince the mind that what was really meant is that the delivery of the check was another
consideration of the widow's waiver, it being self-evident that said check constituted, in effect, a refund of
the first premium paid by insured and received by the insurer. It is ridiculous to think that such a negligible
amount has been the only consideration of the widow’s waiver of any right or benefit accurring to her from
the policy. A careful perusal of the instrument will show that the real consideration of the widow's waiver
was the unenforceability of the policy due to her husband's illness and the mutual desire of the plaintiff of
the insurer to settle amicably the cases instead of resorting to courts.
121.   In conclusion it is my opinion: (1) That the policy has not produced any effect from which the widow may
derive any right, and (2) that she has expressly waived any all rights accurring from the policy; and for these
reasons I dissent from the majority opinion.
004 Argente v. West Coast Life Insurance Co. (Hilario) 1.   The wife signed a like application for the same policy as her
March 19, 1928 | Malcolm, J. | Concealment husband. Both applications (except names and signatures of the
spouses applicants) were written by Jose Geronimo del Rosario,
PETITIONER: Bernardo Argente an agent for West Coast.
RESPONDENTS: West Coast Life Insurance 2.   But all information contained in applications were furnished to
agent del Rosario by Argente.
SUMMARY: Bernardo Argente and his wife, Vicenta de Ocampo, applied for a 123.  Argente was examined by a medical examiner for Wesst Coast, and the
joint life insurance policy with West Coast Life Insurance Co. They were medically results were recorded in the Medical Examiner’s Report. This was all in the
examined by the company’s doctor, and they supplied the answers to the questions in hand writing of the doctor, but information and answers to questions on the
the Medical Examiner’s Report. Vicenta died of cerebral apoplexy a few months Report were furnished to the doctor by Argente. The same happened with
after the issuance of the policy, prompting Argente to claim on the policy (he was Vicenta, and she gave the answers to the questions to the doctor.
asking for P 15000). Upon the investigation of West Coast, it was discovered that the 124.  The spouses increased the amount of the insurance from P2000 to P15000,
spouses made several fraudulent declarations regarding the condition of their health and asked that the policy be dated May 15, 1925 (they asked this on May 9).
for the last five years, concealing the time the husband was hospitalized and that the 125.  A temporary policy for P 15 000 was issued in favor of the spouses, b ut it
wife had several neurological/phsycological problems (like she was diagnosed with was not delivered to Argente until the first quarterly premium on the policy
alcoholism, then later manic depressive psychosis, then a final diagnosis of psycho- was paid.
neurosis). Thus, West Coast refused to honor Argente’s claim, prompting him to file 1.   Since 30 days had elapsed since the applicants were examined,
an action before the Trial Court. The Trial Court ruled in favor of West Coast, and each of them was required to file a certificate of health before the
the Supreme Court upheld the Trial Court’s decision. policy was delivered on July 2.
126.  On November 18 of the same year (4 months after delivery of policy)
In an action on a life insurance policy where the evidence conclusively shows that Vicenta died of cerebral apoplexy, and her husband Argente presented a
the answers to questions concerning diseases were untrue, the truth of falsity of the claim for P 15 000.
answers become the determining factor. In the policy was procured by fraudulent 1.   Following investigation by West Coast, it was discovered that the
representations, the contract of insurance apparently set forth therein was answers given by the insured in their medical examinations
never legally existent. It can fairly be assumed that had the true facts been with regard to their health and previous illness and medical
disclosed by the assured, the insurance would never have been granted. Assurer in attendance were untrue.
assuming a risk is entitled to know every material fact of which the assured has 127.  For that reason, the West Coast Life Insurance Co. refused to pay the
exclusive or peculiar knowledge, as well as all material facts which directly tend to claim of Bernardo Argente, and on May 25, 1926, wrote him to the effect
increase the hazard or risk which are known by the assured, or which ought to be or that the claim was rejected because the insurance was obtained through
are presumed to be known by him. And a concealment of such facts vitiates the fraud and misrepresentation. This prompted Argente to file an action in the
policy. trial court.
128.  What they concealed (I don’t think you have to read this in detail, but just in
case sir asks!): In the Medical Examiner's Report that Bernardo Argente, in
DOCTRINE: One ground for the rescission of a contract of insurance under the response to the question asked by the medical examiner, "Have you ever
Insurance Act is "a concealment," which in section 25 is defined as "A neglect to consulted a physician for, or have you ever suffered from any ailment or
communicate that which a party knows and ought to communicate." disease of, the brain or nervous system?" answered "No." To the question,
The principal question, therefore, must be, Was the assurer misled or deceived into "Have you consulted a physician for any ailment or disease not included in
entering a contract obligation or in fixing the premium of insurance by a your above answer," answered "Yes. Nature of Ailment, Disease or Injury.
withholding of material information of facts within the assured's knowledge or Scabies, Number of attacks 1, Date 1911. Duration 1 month, Severity Fair,
presumed knowledge? results and, if within five years, name and address of every physician
consulted. Dr. P. Guazon. Cured. Dr. Guazon is dead now." And to the
question, "What physician or physicians, if any, not named above, have you
consulted or been treated by, within the last five years and for what illness
or ailment? (If none, so state)" answered "No." It is, however, not disputed
FACTS: that on January 10, 11, and 13, 1923, Bernardo Argente was confined in
122.  West Coast issued a joint life insurance policy in favor of Bernardo Argente
and his wife, Vicenta de Ocampo.
the Philippine General Hospital where he was treated by Dr. Agerico B. neglect to communicate that which a party knows and ought to
M. Sison for cerebral congestion and Bell's Palsy. communicate."
129.  What they concealed (I don’t think you have to read this in detail, but just in 121.  Argente claims that the alleged concealment was immaterial and
case sir asks!): Vicenta de Ocampo, in response to the question asked by the insufficient to avoid the policy.
medical examiner, "How frequently, if at all, and in what quantity do you 122.  The SC does not buy this.
use beer, wine, spirits or other intoxicants?" answered "Beer only in small 123.  In an action on a life insurance policy where the evidence conclusively
quantities occasionally." To the question, "Have you ever consulted a shows that the answers to questions concerning diseases were untrue, the
physician for or have you ever suffered from any ailment or disease of the truth of falsity of the answers become the determining factor. In the policy
brain or nervous system?" answered "No." To the question, "What was procured by fraudulent representations, the contract of insurance
physician or physicians, if any, not named above, have you consulted or apparently set forth therein was never legally existent. It can fairly be
been treated by, within the last five years and for what illness or ailment? assumed that had the true facts been disclosed by the assured, the
(If none, so state)" answered "None." And to the question, "Are you in good insurance would never have been granted.
health as far as you know and believe?" answered "Yes." 124.  In Joyce, The Law of Insurance, second edition, volume 3, Chapter LV, is
130.  It is, however, not disputed that Vicenta de Ocampo was taken by a found the following: Concealment exists where the assured has
patrolman, at the request of her husband, Bernardo Argente, on May 19, knowledge of a fact material to the risk, and honesty, good faith, and
1924, to the Meisic police station, and from there was transferred to the fair dealing requires that he should communicate it to the assured, but
San Lazaro Hospital. In San Lazaro Hospital, her case was diagnosed by he designated and intentionally with holds the same. Another rule is that
the admitting physician as "alcoholism," but later Doctor Domingo made if the assured undertakes to state all the circumstances affecting the
a diagnosis of probable "manic-depressive psychosis," and still, later in risk, a full and fair statement of all is required.
Mary Chiles Hospital, made a final diagnosis of "phycho-neurosis." 125.  It is also held that the concealment must, in the absence of inquiries, be
131.  Argente concedes and admits most of these facts, yet alleges that both he not only material, but fraudulent, or the fact must have been
and his wife revealed their medical conditions to the doctor who intentionally withheld.
examined them from the insurance company, but that the doctor, 1.   if no inquiries are made and no fraud or design to conceal
presumably acting in collusion with the insurance agent, failed to enters into the concealment the contract is not avoided.
record them in the medical reports. 126.  But it would seem that if a material fact is actually known to the assured, its
132.  The Trial Court ruled in favor of West Coast, and disregarded the concealment must of itself necessarily be a fraud, and if the fact is one
testimonies presented by Argente (his own testimony, his clerk’s, the which the assured ought to know, or is presumed to know, the
doctor’s, and the agent’s). presumption of knowledge ought to place the assured in the same
1.   There appears no motive on the part of the doctor to fasify the position as in the former case with relation to material facts;
Medical Examiner’s Reports. 127.  The basis of the rule vitiating the contract in case of concealment is that it
2.   Further, from the evidence and representations in their applications misleads or deceives the insurer into accepting the risk, or accepting it at the
regarding their health for the last 5 years before they applied for rate of premium agreed upon. The insurer, relying upon the belief that the
insurance, they were totally false. assured will disclose every material within his actual or presumed
133.  He now questions the trial court’s ruling before the SC. knowledge, is misled into a belief that the circumstance withheld does not
exist, and he is thereby induced to estimate the risk upon a false basis that it
ISSUE/s: does not exist.
12.   WoN the Trial Court was correct in saying that West Coast correctly 128.  The principal question, therefore, must be, Was the assurer misled or
refused to pay Argente’s claim – YES, because Argente is guilty of deceived into entering a contract obligation or in fixing the premium of
concealment, on of the grounds for rescission of a contract of insurance insurance by a withholding of material information of facts within the
under the Insurance Act. assured's knowledge or presumed knowledge?
129.  Assurer in assuming a risk is entitled to know every material fact of which
RULING: SC affirmed the lower courts decision. Pwede rin wherefore. the assured has exclusive or peculiar knowledge, as well as all material
facts which directly tend to increase the hazard or risk which are known by
RATIO: the assured, or which ought to be or are presumed to be known by him. And
120.  One ground for the rescission of a contract of insurance under the a concealment of such facts vitiates the policy.
Insurance Act is "a concealment," which in section 25 is defined as "A  
005 EDILLON VS MANILA BANKERS LIFE (HIRANG) FACTS:
30 September 1982 |Vasquez, J. | Concealment 132.  In April 1969, Carmen O. Lapuz applied with Manila Bankers Life (MBL)
for insurance coverage against accident and injuries. She filled up the blank
PETITIONER: REGINA EDILLON application form given to her and filed the same with MBL
RESPONDENTS: MANILA BANKERS LIFE INSURANCE CORPORATION 133.  In the said application form which was dated April 15, 1969, she gave the
date of her birth as July 11, 1904. On the same date, she paid the sum of
SUMMARY: P20.00 representing the premium for which she was issued the
Insured: Carmen Lapuz corresponding receipt signed by an authorized agent of the MBL
Insurer: Manila Bankers Life Insurance Corporation (MBL) 134.  During the effectivity of the Insurance Policy, Carmen Lapuz died, and as
Beneficiary: Regina Edillon such, Regina L. Edillon, the sister of the insured and who was the named
Policy: Insurance against accident and injuries beneficiary in the policy, filed her claim for the proceeds of the insurance
In 1969, Lapuz applied with MBL for a life insurance coverage against accident and 135.  MBL denied the claim of Edillon stating that it is not liable to pay claims
injuries. When she applied for the insurance she was already over 60 years of age under the policy in behalf of "persons who are under the age of sixteen (16)
but despite this, her application was accepted. Under such policy, her beneficiary years of age or over the age of sixty (60) years x x x." It is to note that when
was her sister, Regina Edillon. During the effectivy of the policy, Lapuz died due to Lapuz applied for the policy, she was already over 60 years old
an accident and as such, Edillon, as beneficiary filed a claim to collect the insurance 136.  The trial court ruled in favor of MBL stating that:
proceeds but was denied by MBL contending that in the Insurance Certicifacate a.   The policy of insurance being a contract of adhesion, it was the
provided for a provision which states that it is not liable to pay claims under the duty of the insured to know the terms of the contract he or she is
policy in behalf of "persons who are under the age of sixteen (16) years of age or entering into; the insured in this case, upon learning from its terms
over the age of sixty (60) years x x x." It is to note that when Lapuz applied for the that she could not have been qualified under the conditions stated
policy, she was already over 60 years old. The issue in this case is WoN MBL is in said contract, what she should have done is simply to ask for a
liable and the court ruled in the affirmative. The age of Lapuz was never concealed refund of the premium that she paid
to MBL. In fact, the application clearly showed that at the time of application Lapuz ISSUE/s: WoN Manila Bankers Life is liable – Yes, the court ruled that the fact
was almost 65 yrs old. The accident which resulted in the death of the insured, a that MBL accepted the application, knowing that the age of the applicant is an
risk covered by the policy, occurred on May 31, 1969 - 45 DAYS after the important element, showed that MBL either waived such criteria or it’s employees
insurance coverage was applied for. There was sufficient time for the private are negligence or incompetence for overlooking such fact
respondent to process the application and to notice that the applicant was over 60
years of age and thereby cancel the policy on that ground if it was minded to do RULING: WHEREFORE, the judgment appealed from is hereby REVERSED and
so. If MBL failed to act, it is either because it was willing to waive such SET ASIDE.
disqualification; or, through the negligence or incompetence of its employees for
which it has only itself to blame, it simply overlooked such fact. Under the RATIO:
circumstances, the insurance corporation is already deemed in estoppel 213.  The age of the insured Carmen O. Lapuz was not concealed to the insurance
company. Her application for insurance coverage which was on a printed
DOCTRINE: It is usually held that where the insurer, at the time of the issuance of form furnished by private respondent and which contained very few items
a policy of insurance, has knowledge of existing facts which, if insisted on, would of information clearly indicated her age at the time of filing the same to be
invalidate the contract from its very inception, such knowledge constitutes a waiver almost 65 years of age
of conditions in the contract inconsistent with the known facts, and the insurer is 214.  Despite such information which would hardly be overlooked in the
stopped thereafter from asserting the breach of such conditions. The law is application form, considering its prominence thereon and its materiality to
charitable enough to assume, in the absence of any showing to the contrary, that an the coverage applied for, the respondent insurance corporation received her
insurance company intends to execute a valid contract in return for the premium payment of premium and issued the corresponding certificate of insurance
received; and when the policy contains a condition which renders it voidable at its without question
inception, and this result is known to the insurer, it will be presumed to have 215.  The accident which resulted in the death of the insured, a risk covered by
intended to waive the conditions and to execute a binding contract, rather than to the policy, occurred on May 31, 1969 - 45 DAYS after the insurance
have deceived the insured into thinking he is insured when in fact he is not, and to coverage was applied for. There was sufficient time for the private
have taken his money without consideration respondent to process the application and to notice that the applicant was
over 60 years of age and thereby cancel the policy on that ground if it was
minded to do so. If MBL failed to act, it is either because it was willing to
waive such disqualification; or, through the negligence or incompetence of
its employees for which it has only itself to blame, it simply overlooked
such fact. Under the circumstances, the insurance corporation is already
deemed in estoppel
216.  Its inaction to revoke the policy despite a departure from the exclusionary
condition contained in the said policy constituted a waiver of such
condition. This doctrine was upheld by several cases:
a.   " Que Chee Gan vs. Law Union Rock Insurance Co., Ltd." - In this
case, the requirement of the insurance policy was to have 11 fire
hydrants however, the insurance company in this case, from the
very beginning, knew that the insured only had 2 but nevertheless
accepted the insurance policy and never revoked such policy. The
court in this case said that: “It is usually held that where the
insurer, at the time of the issuance of a policy of insurance, has
knowledge of existing facts which, if insisted on, would invalidate
the contract from its very inception, such knowledge constitutes a
waiver of conditions in the contract inconsistent with the known
facts, and the insurer is stopped thereafter from asserting the
breach of such conditions. The law is charitable enough to
assume, in the absence of any showing to the contrary, that an
insurance company intends to execute a valid contract in return for
the premium received; and when the policy contains a condition
which renders it voidable at its inception, and this result is known
to the insurer, it will be presumed to have intended to waive the
conditions and to execute a binding contract, rather than to have
deceived the insured into thinking he is insured when in fact he is
not, and to have taken his money without consideration”

b.   Capital Insurance & Surety Co., Inc. vs. Plastic Era Co., Inc, -
The company issued the policy upon the execution of a promissory
note for the payment of the premium. A check given subsequently
by the insured as partial payment of the premium was dishonored
for lack of funds. Despite such deviation from the terms of the
policy, the insurer was held liable. The insurance corporation
impliedly agreed to modify the tenor of the insurance policy and in
effect, waived the provision therein that it would only pay for the
loss or damage in case the same occurs after the payment of the
premium. Considering that the insurance policy is silent as to the
mode of payment, Capital Insurance is deemed to have accepted
the promissory note in payment of the premium
006 CANILANG v. CA (LAGUILLES) 17.   On August 5, Jaime Canilang died of congestive heart failure, anemia, and
June 7, 1993 | Feliciano, J. | Concealment chronic anemia. Thelma filed a claim with Great Pacific which the latter
denied, upon the ground that Jaime had concealed material information
PETITIONER: Thelma vda. De Canilang from it.
RESPONDENTS: CA and Great Pacific Life Insurance Corporation 18.   Thelma then filed a complaint against Great Pacific for the recovery of the
insurance proceeds. During the hearing called by the Insurance
SUMMARY: Jaime consulted a doctor and he was diagnosed with sinus Commissioner, Thelma testified that she was not aware of any serious
tachycardia and later on acute bronchitis. He then applied for a non-medical illness suffered by her late husband and that, as far as she knew, her
insurance policy with Great Pacific Life Assurance Company, naming his wife, husband died because of a kidney disorder.
Thelma as his beneficiary. Subsequently, Jaime died of congestive heart failure and 19.   A deposition was given by Dr. Claudio, stating that he previously treated
anemia. Thelma then filed a claim with Great Pacific but the latter refused because Jaime for sinus tachycardia and acute bronchitis. Great Pacific presented Dr.
Jaime allegedly concealed material information from it. Thelma filed a complaint Quismorio, a physician and a medical underwriter working for Great
against Great Pacific for the recovery of the insurance proceeds. Thelma testified Pacific. She testified that as a rule, medical examinations are required only
that she was not aware of any serious illness suffered by Jaime. The Insurance in cases where the applicant has indicated in his application that he has
Commissioner ruled in favor of Thelma on the ground that the ailment of Jaime was previously undergone medical consultation and hospitalization.
not so serious that even if it had been disclosed, it would not have affected Great 20.   The Insurance Commissioner ordered Great Pacific to pay P19,700 plus
Pacific’s decision to insure him. On appeal, the CA reversed the IC’s decision and
legal interest after holding that:
found that the failure to disclose previous medical consultation and treatment a.   The ailment of Jaime was not so serious that even if it had been disclosed, it would
constituted material information which should have been communicated to Great not have affected Great Pacific’s decision to insure him;
Pacific. The issue is WoN Jaime concealed his true state of health. b.   Great Pacific waived its rights to inquire into the health condition of the applicant
by the issuance of the policy despite the lack of answers to some of the pertinent
The SC held in the affirmative. In Jaime’s application for the insurance, he left questions in the insurance application;
c.   There was no intentional concealment on the part of the insured as he had thought
blank the portion for the exceptions that he consulted Dr. Claudio who found him to that he was merely suffering from a minor ailment and simple cold;
be suffering from sinus tachycardia and acute bronchitis. The Court agrees with the d.   BP 874 which voids an insurance contract whether or not concealment was
CA that the information which Jaime Canilang failed to disclose was material to the intentionally made, was not applicable to Jaime’s case because the law only became
ability of Great Pacific to estimate the probable risk he presented as subject of life effective in 1985.
insurance. Had he disclosed his visits to his doctor, the diagnosis made and the 21.   On appeal, the CA reversed and set aside the decision of the Insurance
medicines prescribed, in the insurance application, it may be reasonably assumed Commissioner:
that Great Pacific would have made further inquiries and would have probably a.   It found that the use of the word “intentionally” by the Insurance Commissioner in
refused to issue a non-medical insurance policy or at the very least, required a defining and resolving the issue was not supported by evidence;
b.   That the issue agreed upon by the parties had been whether the deceased insured
higher premium. Sec. 27 of the 1978 Insurance Code is properly read as referring to
made a material concealment as to the state of his health at the time of the filing of
any concealment without regard to whether such is intentional or unintentional. the application.
c.   The CA also held that the failure to disclose previous medical consultation and
DOCTRINE: It is immaterial whether the concealment is intentional or treatment constituted material information which should have been communicated
unintentional. to Great Pacific to enable the latter to make proper inquiries.
22.   Hence, this petition by Thelma Canilang.

ISSUE:
FACTS: WoN Jaime Canilang concealed his true state of health at the time of the filing of the
15.   On June 18, 1982, Jaime Canilang consulted Dr. Claudio and was insurance application – YES, because he did not disclose that he had consulted a
diagnosed as suffering from sinus tachycardia. The doctor prescribed doctor prior to the application for the insurance policy, and that he suffered from an
trazepam, a tranquilizer, and aptin, a beta-blocker drug. Canilang consulted illness.
the doctor again after two months after, and he was found to have acute
bronchitis. RULING: WHEREFORE, the Petition for Review is DENIED for lack of merit and the
16.   Jaime Canilang then applied for a non-medical insurance policy with Great Decision of the Court of Appeals dated 16 October 1989 in C.A-G.R. SP No. 08696 is hereby
Pacific Life Assurance Company naming his wife, petitioner Thelma AFFIRMED. No pronouncement as to costs.
Canilang, as his beneficiary. Jaime Canilang was issued an ordinary life
insurance effective as of August 9, 1982. RATIO:
1.   In addition to the negative statements made by Jaime in paragraphs 1 and 2 certain information to the insurer was not intentional in nature, for the
of the medical declaration, he failed to disclose in the appropriate space, reason that Jaime believed that he was suffering from a common cold.
under the caption “Exceptions,” that he had twice consulted Dr. Claudio 10.   The unspoken theory of the Insurance Commissioner appears to have been
who found him to be suffering from sinus tachycardia and acute bronchitis. that by deleting the phrase “intentional or unintentional,” the Insurance
2.   The relevant statutory provisions as they stood at the time are set out in PD Code of 1978 intended to limit the kinds of concealment which generate a
1460 or the Insurance Code of 1978: right to rescind on the part of the injured party to “intentional
a.   Sec. 26. A neglect to communicate that which a party knows and ought to concealments.” This argument is not persuasive. As a simple matter of
communicate, is called a concealment. grammar, it may be noted that “intentional and unintentional” cancel each
b.   Sec. 28. Each party to a contract of insurance must communicate to the other, in
good faith, all factors within his knowledge which are material to the contract and other out. The net result therefore of the phrase is precisely to leave
as to which he makes no warranty, and which the other has not the means of unqualified the term “concealment.”
ascertaining. 11.   Thus, Sec. 272 of the Insurance Code of 1978 is properly read as
3.   Under those provisions, the information concealed must be information referring to any concealment without regard to whether such is
which the concealing party knew and ought to have communicated, that is intentional or unintentional.
to say, information which was material to the contract. The test of 12.   In the case at bar, the nature of the facts not conveyed to the insurer was
materiality is contained in Sec. 31.1 such that the failure to communicate must have been intentional rather than
4.   Sinus tachycardia is considered present when the heart rate exceeds 100 merely inadvertent. For Jaime Canilang could not have been unaware that
beats per minute. The symptoms include pounding in the chest and his heart beat would at times rise to high and alarming levels and that he
sometimes faintness and weakness. had consulted a doctor twice in the two months before applying for the
5.   The Court agrees with the CA that the information which Jaime insurance.
Canilang failed to disclose was material to the ability of Great Pacific to 13.   The Court finds it difficult to take seriously the argument that Great Pacific
estimate the probable risk he presented as subject of life insurance. Had had waived inquiry into the concealment by issuing the insurance policy
he disclosed his visits to his doctor, the diagnosis made and the medicines notwithstanding Jaime’s failure to set out answers to some of the questions
prescribed, in the insurance application, it may be reasonably assumed that in the insurance application. Such failure precisely constituted concealment
Great Pacific would have made further inquiries and would have probably on the part of Jaime.
refused to issue a non-medical insurance policy or at the very least, required
a higher premium. The medical declaration which was set out in the application for insurance executed by Jaime
6.   The materiality of the information withheld by Great Pacific did not depend Canilang read as follows:
upon the state of mind of Jaime. A man’s state of mind is not capable of "MEDICAL DECLARATION `I hereby declare that:
(1) I have not been confined in any hospital, sanitarium or infirmary, nor received any medical or
proof in our judicial process, except through proof of external acts from
surgical advice/attention within the last five (5) years.
which inferences as to his subjective belief may be reasonably drawn. (2) I have never been treated nor consulted a physician for a heart condition, high blood pressure,
7.   Neither does materiality depend upon the actual or physical events which cancer, diabetes, lung, kidney, stomach disorder, or any other physical impairment.
ensue. Materiality relates rather to the “probable and reasonable influence (3) I am, to the best of my knowledge, in good health.
of the fact” upon the party to whom the communication should have been EXCEPTIONS:
made, in assessing the risk involved in making or omitting to make further
inquiries and in accepting the application for insurance.
8.   The insurance Great Pacific applied for was a non-medical insurance. It was GENERAL DECLARATION
I hereby declare that all the foregoing answers and statements are complete, true and correct. I
held that the waiver of medical examination in a non-medical insurance hereby agree that if there be any fraud or misrepresentation in the above statements material to the
contract renders even more material the information required of the risk, the INSURANCE COMPANY upon discovery within two (2) years from the effective
applicant concerning previous condition of health and diseases suffered, for date of insurance shall have the right to declare such insurance null and void. That the
such information constitutes an important factor which the insurer takes into liabilities of the Company under the said Policy/TA/Certificate shall accrue and begin only from the
consideration in deciding whether to issue the policy or not. date of commencement of risk stated in the Policy/TA/Certificate, provided that the first premium is
paid and the Policy/TA/Certificate is delivered to, and accepted by me in person, when I am in
9.   The Insurance Commissioner also ruled that the failure Jaime to convey
actual good health.
                                                                                                                       
1
  Sec.   31.   Materiality   is   to   be   determined   not   by   the   event,   but   solely   by   the   probable   and  
reasonable  influence  of  the  facts  upon  the  party  to  whom  the  communication  is  due,  in  forming  his                                                                                                                          
2
estimate  of  the  disadvantages  of  the  proposed  contract,  or  in  making  his  inquiries.      Sec.  27.  A  concealment  entitles  the  injured  party  to  rescind  a  contract  of  insurance.    
   
007 SUNLIFE v. CA (Marcos) Sunlife.
June 22, 1995 | Quiason, J. | Concealment 138.  He was issued issued Policy No. 3-903-766-X valued P100,000.00, with
double indemnity in case of accidental death
PETITIONER: Sunlife Assurance Company of Canada 139.  The designated beneficiary was his mother, Bernarda Bacani
RESPONDENTS: The Hon. Court of Appeals and Sps. Rolando and Bernarda 140.  Robert died in a plane crash. Subsequently, Bernarda filed a claim with
Bacani Sunlife, seeking the benefits of the insurance policy taken by her son.
141.  Sunlife conducted an investigation and its findings prompted it to reject the
SUMMARY: Robert Bacani procured a life insurance from Sunlife for himself, claim.
making his mother, Bernarda, the beneficiary. Robert died in a plane crash. 142.  In its letter, Sunlife informed Bernarda that Robert did not disclose material
Bernarda filed a claim with Sunlife, seeking the benefits of the insurance policy. facts relevant to the issuance of the policy, thus rendering the contract
Sunlife, however, rejected the claim of Bernarda given that after conducting an voidable. Attached to the letter was a check representing the total premiums
investigation, it found that Robert did not disclose material facts relevant to the paid.
issuance of the policy. Sunlife claimed that Robert gave false statements 143.  Sunlife claimed that the insured gave false statements in his application
regarding his medical history (See Fact 7 for the questions asked). It found that 2 when he answered the ff. questions:
5. Within the past 5 years have you:
weeks prior to his application, he was examined and confined at the Lung Center a) consulted any doctor or other health practitioner? 

for renal failure and was subject to different kinds of tests. Bernarda filed an b) submitted to: 
ECG? X-rays? blood tests? other tests? 

action for specific performance against Sunlife. The RTC and CA ruled in favor c) attended or been admitted to any hospital or other medical facility?
of Bernards contending that the facts concealed where made in good faith and 6. Have you ever had or sought advice for:
under the belief that they need not be disclosed, further the cause of death was xxx xxx xxx
b) urine, kidney or bladder disorder?"
unrelated to the facts concealed by the insured. The issue is WoN there was
concealment or misrepresentation by Robert that would warrant the rejection of
144.  Robert answered 5(a) in the affirmative but limited his answer to a
Bernarda’s claim in the insurance policy – YES, because Section 26 of the
consultation for flu and cough complications, while the other questions
Insurance Code is explicit in requiring a party to a contract of insurance to
were answered in the negative.
communicate to the other, in good faith, all facts within his knowledge which are
145.  However, it was discovered that two weeks prior to his application for
material to the contract and as to which he makes no warranty, and which the
insurance, Robert was examined and confined at the Lung Center of the
other has no means of ascertaining. Materiality is to be determined not by the
Philippines, where he was diagnosed for renal failure. During his
event, but solely by the probable and reasonable influence of the facts upon the
confinement, he was subject to urinalysis, ultra-sonography and hematology
party to whom communication is due, in forming his estimate of the
tests.
disadvantages of the proposed contract or in making his inquiries. The matters
146.  Bernarda and her husband filed an action for specific performance against
concealed would have definitely affected Sunlife’s action on his application,
Sunlife with the RTC of Valenzuela.
either by approving it with the corresponding adjustment for a higher premium
147.  Sunflife filed its answer with counterclaim and a list of exibits consisting of
or rejecting the same. "Good faith" is no defense in concealment. The insured's
medical records furnished by Lung Center.
failure to disclose the fact that he was hospitalized for two weeks prior to ling his
148.  The RTC ruled in favor of Bernarda and concluded that the facts concealed
application for insurance, raises grave doubts about his bonafides. It appears that
were made in good faith and under the belief that they need not be
such concealment was deliberate on his part.
disclosed. Moreover, it held that the health history of the insured was
immaterial since the insurance policy was “non-medical”.
DOCTRINE: Section 26 of The Insurance Code is explicit in requiring a party
149.  An appeal was made to the CA, which affirmed the decision of the RTC.
to a contract of insurance to communicate to the other, in good faith, all facts
150.  The CA held that the cause of death was unrelated to the facts concealed by
within his knowledge which are material to the contract and as to which he
the insured.
makes no warranty, and which the other has no means of ascertaining. Said
151.  Sunlife’s MR was denied, hence a petition was made to the SC.
Section provides: "A neglect to communicate that which a party knows and
ought to communicate, is called concealment."
ISSUE/s: WoN there was concealment or misrepresentation by Robert that would
warrant the rejection of Bernarda’s claim in the insurance policy – YES, because
Section 26 of the Insurance Code is explicit in requiring a party to a contract of
FACTS: insurance to communicate to the other, in good faith, all facts within his knowledge
137.  Robert John B. Bacani procured a life insurance contract for himself from which are material to the contract and as to which he makes no warranty, and which
the other has no means of ascertaining

RULING: Judgment in question is affirmed.

RATIO:
217.  Section 26 of the Insurance Code is explicit in requiring a party to a
contract of insurance to communicate to the other, in good faith, all facts
within his knowledge which are material to the contract and as to which he
makes no warranty, and which the other has no means of ascertaining.
218.  Said Section provides: "A neglect to communicate that which a party knows
and ought to communicate, is called concealment."
219.  Materiality is to be determined not by the event, but solely by the probable
and reasonable influence of the facts upon the party to whom
communication is due, in forming his estimate of the disadvantages of the
proposed contract or in making his inquiries.
220.  The terms of the contract are clear. The insured is specifically required to
disclose to the insurer matters relating to his health.
221.  The information which the insured failed to disclose were material and
relevant to the approval and the issuance of the insurance policy.
222.  The matters concealed would have definitely affected Sunlife’s action on
his application, either by approving it with the corresponding adjustment for
a higher premium or rejecting the same.
223.  Moreover, a disclosure may have warranted a medical examination of the
insured by Sunlife in order for it to reasonably assess the risk involved in
accepting the application.
224.  "Good faith" is no defense in concealment.
225.  The insured's failure to disclose the fact that he was hospitalized for two
weeks prior to ling his application for insurance, raises grave doubts about
his bonafides. It appears that such concealment was deliberate on his part.
226.  The argument, that Sunlife's waiver of the medical examination of the
insured debunks the materiality of the facts concealed, is untenable.
227.  We reiterate our ruling in Saturnino v. Philippine American Life Insurance
Company, that ". . . the waiver of a medical examination [in a non-medical
insurance contract] renders even more material the information required of
the applicant concerning previous condition of health and diseases suffered,
for such information necessarily constitutes an important factor which the
insurer takes into consideration in deciding whether to issue the policy or
not . . . ."
228.  Anent the finding that the facts concealed had no bearing to the cause of
death of the insured, it is well settled that the insured need not die of the
disease he had failed to disclose to the insurer. It is sufficient that his non-
disclosure misled the insurer in forming his estimates of the risks of the
proposed insurance policy or in making inquiries.
008 Saturnino v. Philam (MATSUMURA) detailed information regarding the applicant’s health and medical
February 28, 1963 | Makalintal, J. | Concealment history is needed.
2.   The policy was issued on the same day upon Saturnino’s payment of the
first year’s premium of P339.25.
PETITIONER: Ignacio Saturnino, in his own behalf and as the judicial guardian of
Carlos Saturnino, minor 3.   On September 19, 1958, Saturnino died of pneumonia, secondly to
RESPONDENTS: The Philippine American Life Insurance Policy influenza.
4.   Estefania’s husband, Ignacio Saturnino, and their minor child Carlos
SUMMARY: Estefania Saturnino (saturnino) applied for a non-medical insurance Saturnino demanded payment of the face value of the policy which is
with Philam. Here, Saturnino was asked to detail her medical history wherein she P5,000.
stated that she never had any of the ailments listen in the application, cancer, or other
tumors. Moreover, that she had not consulted any physician, undergone any operation 5.   It appears that the months prior to the issuance of the policy,
or suffered any injury within the last five years. The insurance policy was issued after Saturnino was operated on for cancer, involving complete removal of
Saturnino paid the first year’s premium. Less than a year later, Saturnino died of the right breast, including the pectoral muscles and the glands found in
pneumonia secondly to influenza. Her husband and child claimed from Philam the the right armpit.
insurance proceeds of P5,000 but was denied. Apparently, months before the issuance
6.   She stayed in the hospital for a period of eight days, after which she was
of the policy, Saturnino was operated on for cancer (her right break was removed) and
discharged, although according to the surgeon who operated on her she
that the doctor said she could not be considered definitely cured since her cancer was
could not be considered definitely cured, her ailment being of the
malignant. Thus, the saturnino family filed this case to recover the insurance.
malignant type.
The SC rueld that first, Saturnino did make false representation of material facts which 7.   Notwithstanding the fact of her operation Saturnino did not disclose this
would avoid the policy. While the Saturnino family argue that the misrepresentation information in her application for insurance.
was not material because the insurance was “non-medical”, the court ruled that the
8.   On the contrary, she stated therein that she did not have, nor had she ever
waiver of the medical examination renders even more material the meedical
had, among other ailments listed in the application, cancer or other
information that was asked from Saturnino. Second, the argument that Philam’s agent,
tumors; that she had not consulted any physician, undergone any
Santos, was aware of Saturnino’s ailment was rejected because it was already settled in
operation or suffered any injury within the preceding five years; and that
the trial court. Such being a question of fact will not be disrupted by the court since
she had never been treated for, nor did she ever have any illness or
what is assailed are pure questions of law. Third, as to the argument that Saturnino was
disease peculiar to her sex, particularly of the breast, ovaries, uterus, and
not aware that she had cancer (that the doctor allegedly didn’t tell her it was cancer),
menstrual disorders.
the court ruled that concealment of the fact of the operation itself was fraudulent.
Moreover, that in avoiding a policy, it is not necessary to show actual fraud. In this 9.   The application also recites that the foregoing declarations constituted "a
jurisdiction, a concealment, whether intentional or unintentional, entitles the insurer to further basis for the issuance of the policy."
rescind the contract of insurance, concealment being defined as "negligence to
10.   Thus, the insurance claim was rejected by Philam., and so the Saturnino
communicate that which a party knows and ought to communicate"
family filed an action to recover the P5,000.
DOCTRINE: A concealment, whether intentional or unintentional, entitles the insurer 11.   The CFI of Manila dismissed the case, but the Saturnino family were
to rescind the contract of insurance, concealment being defined as "negligence to declared to be entitled to the return of the premium already paid plus
communicate that which a party knows and ought to communicate. interest.
12.   Hence, this appeal.
FACTS
1.   Estefania Saturnino (Saturnino) applied for a non-medical insurance with ISSUE/S:
Philippine American Life Insurance Policy (Philam) on November 16, 1.   W/N Saturnino made false representations of material facts which would
1957, witnessed by Edward Santos (Philam’s agent) avoid the policy – YES because Saturnino misrepresented herself as
a.   This kind of policy dispenses with the medical examination of someone who’s never had any cancel or undergone any operation within the
the applicant usually required in ordinary life policies. However, a past 5 years when in fact she did.
9.   In the application for insurance signed by the Saturnino in this case, she
RULING: The judgment appealed from, dismissing the complaint and awarding the agreed to submit to a medical examination by a duly appointed examiner
return to appellants of the premium already paid, with interest at 6% up to January of Philam if in the latter's opinion such examination was necessary as
29, 1959, is affirmed, with costs against appellants. further evidence of insurability. In not asking her to submit to a medical
examination, the Saturnino family argue that Philam was negligent which
RATIO: precluded it from finding about her actual state of health.
1.   There can be no dispute that the information given by Saturnino in 10.   The SC rueld thst No such negligence can be imputed to Philam. It was
her application for insurance was false, namely, that she had never had precisely because the insured had given herself a clean bill of health that
cancer or tumors, or consulted any physician or undergone any operation Philam no longer considered an actual medical checkup necessary.
within the preceding period of five years.
11.   The Saturnino family also contend there was no fraudulent
2.   Are the facts thus falsely represented material? Section 30 of The concealment of the truth as much as Saturnino herself did not know,
Insurance Law provides that "materiality is to be determined not by the since her doctor never told her, that the disease for which she had
event, but solely by the probable and reasonable influence of informing been operated on was cancer.
his estimate of the proposed contract, or in facts upon the party to whom
12.   In the first place the concealment of the fact of the operation itself was
the communication is due, making his inquiries."
fraudulent, as there could not have been any mistake about it, no matter
3.   The Saturnino family argue that the facts subject of the representations what the ailment.
was not material in view of the "non-medical" nature of the insurance
13.   Secondly, in order to avoid a policy, it is not necessary to show actual
applied for, which does away with the usual requirement of medical
fraud on the part of the insured.
examination before the policy is issued.
14.   According to Kasprzyk vs. Metropolitan Insurance Co.,: "Moreover, if it
4.   However, such is without merit. If anything, the waiver of medical
were the law that an insurance company could not defend a policy on the
examination renders even more material the information required of
ground of misrepresentation, unless it could show actual knowledge on
the applicant concerning previous condition of health and diseases
the part of the applicant that the statements were false, then it is plain
suffered, for such information necessarily constitutes an important
that it would be impossible for it to protect itself and its honest
factor which the insurer takes into consideration in deciding whether
policyholders against fraudulent and improper claims. It would be
to issue the policy or not.
wholly at the mercy of any one who wished to apply for insurance, as it
5.   It is logical to assume that if Philam had been properly apprised of the would be impossible to show actual fraud except in the extremest cases.
insured's medical history Saturnino would at least have been made to It could not rely on an application as containing information on which it
undergo medical examination in order to determine her insurability. could act. There would be no incentive to an applicant to tell the truth."
6.   The Saturnino family also argue that due information concerning 15.   In this jurisdiction, a concealment, whether intentional or
Saturnino's previous illness and operation had been given Philam’s unintentional, entitles the insurer to rescind the contract of
agenct, Edward Santos, who filed the application form after it was signed insurance, concealment being defined as "negligence to communicate
in blank by Saturnino. that which a party knows and ought to communicate"
7.   This was denied by Santos in his testimony, and the trial court found such 16.   In Argente vs. West Coast Life Insurance Co, the court said: "'The basis
testimony to be true. This is a finding of fact which is binding upon the of the rule vitiating the contract in cases of concealment is that it misleads
SC, this appeal having been taken upon questions of law alone. or deceives the insurer into accepting the risk, or accepting it at the rate of
8.   The SC did not deem it necessary to consider Philam's additional premium agreed upon. The insurer, relying upon the belief that the
argument, which was upheld by the trial court, that in signing the assured will disclose every material fact within his actual or presumed
application form in blank and leaving it to Santos to fill (assuming that to knowledge, is misled into a belief that the circumstance withheld does not
be the truth) the insured in effect made Santos her agent for that purpose exist, and he is thereby induced to estimate the risk upon a false basis that
and consequently was responsible for the errors in the entries made by it does not exist.'"
him in that capacity.
009 FLORENDO v. PHILAM PLANS (MERILLES) 3.   Ma. Celeste Abcede, Perla’s duaghter, signed tha application as sales
February 22, 2012 | Abad, J. | Concealment counselor
4.   Aside from pension benefits, the comprehensive pension plan also provided
life insurance coverage to Florendo
PETITIONER: Ma. Lourdes S. Florendo
5.   This was covered by a Group Master Policy that Philippine American Life
RESPONDENTS: Philam Plans Inc., Perla Abcede. and Ma. Celeste Abcede
Insurance Company (Philam Life) issued to Philam Plans
a.   Under the master policy, Philam Life was to automatically provide
SUMMARY: Manuel filed an application for a comprehensive pension plan
life insurance coverage, including accidental death, to all who
with Philam Plans through Perla Abcede. Manuel signed the forms and left to
signed up for Philam Plans' comprehensive pension plan
Perla the accomplishment the contents. The pension plan had with it a life
b.   If the plan holder died before the maturity of the plan, his
insurance coverage. Philam Plans subsequently issued a Pension Plan
beneficiary was to instead receive the proceeds of the life
Agreement to Manuel. Manuel paid his quarterly premiums.
insurance, equivalent to the pre-need price. F
c.   urther, the life insurance was to take care of any unpaid premium
11 months after acquiring the pension, Manuel died. His beneficiary, Lourdes
until the pension plan matured, entitling the beneficiary to the
Florendo (his wife), claims for the payment of the benefits but was denied.
maturity value of the pension plan
Philam Plans alleges that Manuel concealed material facts as to his health when
6.   Philam Plans issued Pension Plan Agreement PP 43005584 to Manuel, with
he applied for the pension plans, therefore cannot recover from the insurer.
Ma. Lourdes S. Florendo, his wife, as beneficiary. Manuel paid his
quarterly premiums
Lourdes field an action against Philam plans. The RTC ruled that Manuel was
7.   11 months later, Manuel died of blood poisoning.
not guilty of concealment. The CA reversed saying that nsurance policies are
8.   Lourdes filed a claim with Philam Plans for the payment of the benefits
traditionally contracts uberrimae fidae or contracts of utmost good faith. As
under her husband’s plan.
such, it required Manuel to disclose to Philam Plans conditions affecting the risk
9.   Because Manuel died before his pension plan matured and his wife was to
of which he was aware or material facts that he knew or ought to know.
get only the benefits of his life insurance, Philam Plans forwarded her claim
to Philam Life
The issue before the SC is whether Manuel is guilty of concealing his illness in
10.   On May 3, 1999, Philam Plans wrote Lourdes a letter, declining her claim.
his application forms. The Court ruled that YES, he is guilty. When Manuel
a.   Philam Life found that Manuel was on maintenance medicine for
signed the pension plan application, he adopted as his own the written
his heart and had an implanted pacemaker.
representations and declarations embodied in it. Assuming that it was Perla
b.   Further, he suffered from diabetes mellitus and was taking insulin.
who filled up the application form, Manuel is still bound by what it
11.   Lourdes renewed her demand for payment under the plan
contains since he certified that he authorized her action. Philam Plans had
but Philam Plansrejected it, prompting her to file the present action against
every right to act on the faith of that certification
the pension plan company before the Regional Trial Court (RTC) of
Quezon City
DOCTRINE: (repetition of above) When Manuel signed the pension plan
12.   RTC rendered judgment ordering Philam PLans, Perla and Celeste
application, he adopted as his own the written representations and declarations
solidarity to pay lourdes all the benefits form her husband’s pension plan.
embodied in it. Assuming that it was Perla who filled up the application form,
a.   RTC ruled that Manuel was not guilty of concealing the state of his
Manuel is still bound by what it contains since he certified that he authorized
health from his pension plan application
her action. Philam Plans had every right to act on the faith of that certification
13.   CA reversed the RTC decision holding that insurance policies are
traditionally contracts uberrimae fidae or contracts of utmost good faith. As
such, it required Manuel to disclose to Philam Plans conditions affecting the
FACTS: risk of which he was aware or material facts that he knew or ought to know
1.   Manuel Florendo filed an application for comprehensive pension plan with 14.   Hence, this case.
respondent Philam Plans, Inc. (Philam Plans) after some convincing by ISSUE/s:
respondent Perla Abcede 1.   Whether the CA erred in finding Manuel guilty of concealing his illness
a.   The plan had a pre-need price of P997,050.00, payable in 10 years, when he kept blank and did not answer questions ins pension plan
and had a maturity value of P2,890,000.00 after 20 years application regarding the ailments he suffered - YES. In signing the forms
2.   Manuel signed the application and left to Perla the task of supplying the he attests to the completeness and truthfulness of its contents.
information needed in the application.
5.   Lourdes next points out that it made no difference if Manuel failed to reveal
RULING: WHEREFORE, the Court AFFIRMS in its entirety the decision of the the fact that he had a pacemaker implant in the early 70s since this did not
Court of Appeals in CA-G.R. CV 87085 dated December 18, 2007 fall within the five-year timeframe that the disclosure contemplated
a.   But a pacemaker is an electronic device implanted into the body
RATIO: and connected to the wall of the heart, designed to provide regular,
1.   Lourdes points out that, seeing the unfilled spaces in Manuel's pension plan mild, electric shock that stimulates the contraction of the heart
application relating to his medical history, Philam Plans should have muscles and restores normalcy to the heartbeat
returned it to him for completion. 6.   Assuming that it was Perla who filled up the application form, Manuel
a.   Since Philam Planschose to approve the application just as it was, is still bound by what it contains since he certified that he authorized
it cannot cry concealment on Manuel's part. her action. Philam Plans had every right to act on the faith of that
b.   Further, Lourdes adds that Philam Plans never queried Manuel certification
directly regarding the state of his health. Consequently, it could not 7.   In a final attempt to defend her claim for benefits under Manuel's pension
blame him for not mentioning it plan, Lourdes points out that any defect or insufficiency in the information
2.   However, the Court says that Lourdes forgets that provided by his pension plan application should be deemed waived after the
since Philam Plans waived medical examination for Manuel, it had to rely same has been approved, the policy has been issued, and the premiums have
largely on his stating the truth regarding his health in his application. been collected
a.   For, after all, he knew more than anyone that he had been under a.   The Court cannot agree. The comprehensive pension plan
treatment for heart condition and diabetes for more than five years that Philam Plans issued contains a one-year incontestability period
preceding his submission of that application. b.   The incontestability clause precludes the insurer from disowning
b.   But he kept those crucial facts from Philam Plans liability under the policy it issued on the ground of concealment or
c.   Besides, when Manuel signed the pension plan application, he misrepresentation regarding the health of the insured after a year of
adopted as his own the written representations and its issuance
declarations embodied in it. 8.   Since Manuel died on the eleventh month following the issuance of his
d.   It is clear from these representations that he concealed his chronic plan, the one year incontestability period has not yet set in.
heart ailment and diabetes from PhilamPlans Consequently, Philam Plans was not barred from questioning Lourdes'
3.   Since Manuel signed the application without filling in the details regarding entitlement to the benefits of her husband's pension plan.
his continuing treatments for heart condition and diabetes, the assumption is
that he has never been treated for the said illnesses in the last five years
preceding his application.
a.   This is implicit from the phrase "If your answer to any of the
statements above (specifically, the statement: I have never been
treated for heart condition or diabetes) reveal otherwise, please
give details in the space provided for."
b.   But this is untrue since he had been on "Coumadin," a treatment
for venous thrombosis, and insulin, a drug used in the treatment of
diabetes mellitus, at that time.
4.   Lourdes insists that Manuel had concealed nothing since Perla, the
soliciting agent, knew that Manuel had a pacemaker implanted on his chest
in the 70s or about 20 years before he signed up for the pension plan
a.   Nothing in it implies that someone else may provide the
information that Philam Plans needed.
b.   Manuel cannot sign the application and disown the
responsibility for having it filled up.
c.   If he furnished Perla the needed information and delegated to
her the filling up of the application, then she acted on his
instruction, not on Philam Plans' instruction
010 NG GAN ZEE V ASIAN CRUSADER LIFE INSURANCE (ARMAND) FACTS:
May 30, 1983 | Escolin, J. | Concealment 134.  On May 12, 1962, Kwong Nam applied for a 20-year endowment insurance
PETITIONER: Ng Gan Zee on his life for the sum of P20,000.00, with his wife, appellee Ng Gan Zee as
RESPONDENTS: Asian Crusader Life Assurance Corporation beneficiary. On the same date, Asian Crusader (Asian), upon receipt of the
SUMMARY: In May 1962, Kwong Nam applied for a 20-year policy with Asian required premium from the insured, approved the application and issued the
Crusader Life Assurance Corporation. Asian Crusader asked the following question: corresponding policy. On December 6, 1963, Kwong Nam died of cancer
Has any life insurance company ever refused your application for insurance or for of the liver with metastasis. All premiums had been religiously paid at the
reinstatement of a lapsed policy or offered you a policy different from that applied time of his death.
for? If, so, name company and date. – to which Kwong Nam answered “No”. Kwong 135.  On January 10, 1964, his widow Ng Gan Zee presented a claim in due form
Nam was also examined by Asian Crusader’s medical examiner to whom he to the Asian for payment of the face value of the policy. On the same date,
disclosed that he was once operated and a tumor was removed from his stomach and she submitted the required proof of death of the insured. Asian denied the
such was “associated with ulcer of the stomach.” Kwong Nam’s application was claim on the ground that the answers given by the insured to the questions
approved. In May 1963, he died. His widow, Ng Gan Zee, filed an insurance claim appealing in his application for life insurance were untrue.
but Asian Crusader refused her claim as it insisted that Kwong Nam concealed 136.  Ng Gan Zee brought the matter to the attention of the Insurance
material facts from them when he was applying for the insurance; that he Commissioner, the Hon. Francisco Y. Mandamus, and the latter, after
misrepresented the fact that he was actually denied application by Insular Life when conducting an investigation, wrote that Asian that he had found no material
he was renewing his application with them; that Kwong Nam was actually operated concealment on the part of the insured and that, therefore, Ng Gan Zee
for peptic ulcer. The issue is WoN because of the aforementioned representation, should be paid the full face value of the policy. This opinion of the
the insured has misled, or deceived the insurer in entering the contract or in Insurance Commissioner notwithstanding, appellant refused to settle its
accepting the risk at the rate of premium agreed upon – NO. The representation obligation.
was not material, nor fraudulent, and there was no proof to show that it was 137.  Asian alleged that the insured was guilty of misrepresentation when he
intentionally withheld. Asian Crusader was not able to prove that Kwong Nam’s answeres “No” to the following question: “Has any life insurance company
statement that Insular Life did not deny his insurance renewal with them is untrue. In ever refused your application for insurance or for reinstatement of a lapsed
fact, evidence showed that in April 1962, Insular Life approved Kwong Nam’s policy or offered you a policy different from that applied for? If, so, name
request of reinstatement only with the condition that Kwong Nam’s plan will be company and date.”
lowered from P50,000.00 to P20,000.00 considering his medical history. Kwong 138.  In its brief, Ng Gan Zee stated: “... As pointed out in the foregoing
Nam did not conceal anything from Asian Crusader. His statement that his operation, summary of the essential facts in this case, the insured had in January, 1962,
in which a tumor the size of a hen’s egg was removed from his stomach, was only applied for reinstatement of his lapsed life insurance policy with the Insular
“associated with ulcer of the stomach” and not peptic ulcer can be considered as an Life Insurance Co., Ltd, but this was declined by the insurance company,
expression made in good faith of his belief as to the nature of his ailment and although later on approved for reinstatement with a very high premium as a
operation. Indeed, such statement must be presumed to have been made by him result of his medical examination. Thus notwithstanding the said insured
without knowledge of its incorrectness and without any deliberate intent on his part to answered 'No' to the [above] question propounded to him.”
mislead Asian Crusader.While it may be conceded that, from the viewpoint of a 139.  Lower court found the argument bereft of factual basis, to wit: “On the first
medical expert, the information communicated was imperfect, the same was question there is no evidence that the Insular Life Assurance Co., Ltd. ever
nevertheless sufficient to have induced Asian Crusader to make further inquiries refused any application of Kwong Nam for insurance. Neither is there any
about the ailment and operation of Kwong Nam. It has been held that where, upon the evidence that any other insurance company has refused any application of
face of the application, a question appears to be not answered at all or to be Kwong Nam for insurance… No new policy was issued by the Insular Life
imperfectly answered, and the insurers issue a policy without any further inquiry, Assurance Co., Ltd. to Kwong Nam in connection with said application for
they waive the imperfection of the answer and render the omission to answer more reinstatement and amendment. Such being the case, the Court finds that
fully immaterial. there is no misrepresentation on this matter.”
DOCTRINE: The duty to establish such affirmative defense of fraudulent intent by 140.  Asian further maintains that when the insured was examined in connection
satisfactory and convincing evidence rests upon the defendant. with his application for life insurance, he gave the appellant's medical
It has been held that where, upon the face of the application, a question appears to be examiner false and misleading information as to his ailment and previous
not answered at all or to be imperfectly answered, and the insurers issue a policy operation. The alleged false statements given by Kwong Nam was that that
without any further inquiry, they waive the imperfection of the answer and render the he was operated on for a tumor in the stomach and that it was associated
omission to answer more fully immaterial.
with ulcer of the stomach, operated on at Chinese General Hospital 2 years with ulcer of the stomach, " should be construed as an expression made in
ago. good faith of his belief as to the nature of his ailment and operation. Indeed,
141.  Asian alleges that the physician who treated Kwong Nam at the Chinese such statement must be presumed to have been made by him without
Gen Hospital had diagnosed the patient’s ailment as peptic ulcer. Asian knowledge of its incorrectness and without any deliberate intent on his part
argues that the insured’s statement in his application that a tumor was to mislead the insurer.
removed during said operation constituted material concealment. (So 134.  While it may be conceded that, from the viewpoint of a medical expert, the
basically a discrepancy between “peptic ulcer” and “tumor of hen’s egg information communicated was imperfect, the same was nevertheless
size” sufficient to have induced insurer to make further inquiries about the
ailment and operation of the insured.
ISSUE/s: 135.  Section 32 of the Insurance Law provides:
13.   Whether or not because of the aforementioned representation, the Section 32. The right to information of material facts maybe waived either
insured has misled, or deceived the insurer in entering the contract or by the terms of insurance or by neglect to make inquiries as to such facts
in accepting the risk at the rate of premium agreed upon – NO. The where they are distinctly implied in other facts of which information is
representation was not material, nor fraudulent, and there was no proof to communicated.
show that it was intentionally withheld. 136.  It has been held that where, upon the face of the application, a question
appears to be not answered at all or to be imperfectly answered, and
RULING: Finding no reversible error committed by the trial court, the judgment the insurers issue a policy without any further inquiry, they waive the
appealed from is hereby affirmed, with costs against appellant Asian-Crusader life imperfection of the answer and render the omission to answer more
Assurance Corporation fully immaterial.
137.  As aptly noted by the lower court, "if the ailment and operation of Kwong
RATIO: Nam had such an important bearing on the question of whether the
130.  Section 27 of the Insurance Law (Act 2427) provides: defendant would undertake the insurance or not, the court cannot
Sec. 27. Such party a contract of insurance must communicate to the other, understand why the defendant or its medical examiner did not make any
in good faith, all facts within his knowledge which are material to the further inquiries on such matters from the Chinese General Hospital or
contract, and which the other has not the means of ascertaining, and as to require copies of the hospital records from the appellant before acting on the
which he makes no warranty. application for insurance. The fact of the matter is that the defendant was
131.  Thus, "concealment exists where the assured had knowledge of a fact too eager to accept the application and receive the insured's premium. It
material to the risk, and honesty, good faith, and fair dealing requires that would be inequitable now to allow the defendant to avoid liability under the
he should communicate it to the assurer, but he designedly and intentionally circumstances."
withholds the same." It has also been held "that the concealment must, in
the absence of inquiries, be not only material, but fraudulent, or the
fact must have been intentionally withheld."
132.  Assuming that the aforesaid answer given by the insured is false, as
claimed by the appellant. Sec. 27 of the Insurance Law, above-quoted,
nevertheless requires that fraudulent intent on the part of the insured
be established to entitle the insurer to rescind the contract. And as
correctly observed by the lower court, "misrepresentation as a defense of
the insurer to avoid liability is an 'affirmative' defense. The duty to
establish such a defense by satisfactory and convincing evidence rests
upon the defendant. The evidence before the Court does not clearly and
satisfactorily establish that defense."
133.  It bears emphasis that Kwong Nam had informed the insurer’s medical
examiner that the tumor for which he was operated on was "associated with
ulcer of the stomach." In the absence of evidence that the insured had
sufficient medical knowledge as to enable him to distinguish between
"peptic ulcer" and "a tumor", his statement that said tumor was "associated
011 FIELDMEN’S INSURANCE v. VDA. DE SONGCO (PELIÑO) a.   This time, the jeep had a different Plate No: J-68136
September 23, 1968 | Fernando, J. | Misrepresentation35 3.   On October 29, 1961 (policy was still effective), the jeep was being driven
by Rodolfo, son of Federico and a duly licensed driver.
PETITIONER: Fieldmen’s Insurance Co. a.   The jeep collided with a car in Calumpit, Bulacan, and as a result,
RESPONDENTS: Mercedes Vargas Vda. De Songco, et. al. and Court of Appeals Federico and Rodolfo died, while Carlos (another son), Angelita (wife
of Carlos), and a family friend sustained physical injuries.
SUMMARY: Federico, a person who only reached grade 1, owned a private jeepney. He 4.   The injured parties or the heirs wanted to claim from Fieldmen’s, but
was induced by Fieldmen’s agent, Sambat, to apply for a Common Carrier’s Liability Fieldmen’s denied, saying that what was insured was a private vehicle and
Insurance Policy to cover his motor vehicle. This was insured twice under the same
not a common carrier.
policy. While the policy was still effective, the jeep collided with a car, resulting to the
death of Federico and a son, and injuries to the other passengers who were members of 5.   During trial in the CFI, another son of Federico, Amor, testified that when
his family. The heirs wanted to claim on the insurance policy but Fieldmen’s said that Sambat was inducing Federico to insure the vehicle, he butted in saying that
they can’t since what was insured was a private vehicle and not a common carrier. When they can’t insure the vehicle since it is an “owner” private vehicle, but then
the heirs filed in the CFI, the surviving son, Amor, testified that Sambat induced Sambat said that whether the vehicle was an “owner” type or for
Federico to insure the vehicle despite knowing that the vehicle wasn’t a common carrier. passengers, it could be insured because their company is not owned by the
Lower court ruled in favor of the heirs. On appeal to the CA, the CA also affirmed the government, so they could do as they please whenever they believe that the
lower court. Hence, this petition. The issue in this case is whether or not Fieldmen’s is vehicle is insurable.
liable on the policy despite the fact that the vehicle insured is a private one and not a a.   Fieldmen’s didn’t rebut the testimony of Amor.
common carrier. The SC held in the affirmative. Where inequitable conduct is shown by
b.   Lower court ruled in their favor. Fieldmen’s appealed to the CA, and
an insurance firm, it is estopped from enforcing forfeitures in its favor in order to
forestall fraud or imposition on the insured. After Fieldmen’s led Federico to believe that
the also affirmed the lower court.
he could qualify under the common carrier liability insurance, it could not be permitted 6.   Hence, this case.
to change its stand to the detriment of the heirs of the insured. Estoppel is primarily 7.   Fieldmen’s claims that:
based on the doctrine of good faith and the avoidance of harm that will befall the a.   Estoppel cannot be invoked by the heirs as a bar to the alleged breach
innocent party due to its injurious reliance. A contract of insurance is one of perfect good of warranty and condition in the policy
faith not for the insured alone, but equally so for the insurer; in fact, it is more so for the b.   No legal liability was incurred under the policy by Fieldmen’s.
latter, since its dominant bargaining position carries with it stricter responsibility.
ISSUE/s:
DOCTRINE: Where inequitable conduct is shown by an insurance firm, it is estopped
1.   WON Fieldmen’s is liable on the policy despite the fact that the vehicle is a
from enforcing forfeitures in its favor in order to forestall fraud or imposition on the
insured. Contractual duty of disclosure imposed by utmost good faith is not required of private one and not a common carrier. – YES, after Fieldmen’s led Federico
the insured alone, but it is imposed with equal stringency upon the insurer, moreso upon to believe that he could qualify under the common carrier liability
the latter since his dominant bargaining position carries with it stricter responsibility. insurance, it could not be permitted to change its stand to the detriment of
  the heirs of the insured.
FACTS:
1.   Federico Songco (Federico) was a man of scant education, since he only RULING: WHEREFORE, the decision of the CA is AFFIRMED in its entirety.
reached first grade.
a.   He owned a private jeepney with Plate No. 41-289 for the year 1960. RATIO:
b.   On September 15, 1960, he was induced by Fieldmen’s Insurance On whether Fieldmen’s is liable on the policy
Co. (Fieldmen’s) Pampanga agent, Benjamin Sambat (Sambat) to 1.   Where inequitable conduct is shown by an insurance firm, it is
apply for a Common Carrier’s Liability Insurance Policy covering estopped from enforcing forfeitures in its favor in order to forestall
his motor vehicle upon paying annual premium of P16.50. fraud or imposition on the insured.
c.   4 days after, Fieldmen’s issued the Common Carriers Accident 2.   After Fieldmen’s led Federico to believe that he could qualify under the
Insurance Policy which will be effective for 1 year (Sept. 15, 1960- common carrier liability insurance, it could not be permitted to change its
Sept. 15, 1961). stand to the detriment of the heirs of the insured.
2.   On September 22, 1961, Fieldmen’s renewed the policy upon payment of 3.   Estoppel is primarily based on the doctrine of good faith and the
the premium, this time effective from October 15, 1961-October 15, 1962. avoidance of harm that will befall the innocent party due to its
injurious reliance. The failure to apply in this case would result in a gross
                                                                                                                        travesty of justice.
35
The case never mentioned the words concealment or misrepresentation.
4.   As to the issue on breach, CA said (which the SC also agreed with) that,
some of the conditions contained in the policy were impossible to comply
with under the existing conditions at the time and inconsistent with the
known facts, so the insurer (Fieldmen’s) is estopped from asserting
breach of such conditions.
5.   Even it be assumed that there was an ambiguity, based on Qua Chee Gan v.
Law Union and Rock Insurance, taking into account the well-known rule
that ambiguities or obscurities must be strictly interpreted against the party
that caused them.
a.   Nowadays, monopolies, cartels, those with overwhelming economic
power, manage to impose upon parties dealing with them contracts of
adhesion, in contrast to those entered by parties bargaining on equal
footing. So these kinds of contracts call for greater strictness and
vigilance on the part of courts of justice with a view to protecting the
weaker party.
6.   Contract of insurance is one of perfect good faith (uberima fides);
contractual duty of disclosure imposed by utmost good faith is not for
the insured alone, but equally so for the insurer; in fact, it is more so
for the latter, since its dominant bargaining position carries with it
stricter responsibility.
012 YU PANG CHENG v. CA (PLEYTO)
May 29, 1959 | Bautista Angelo, J. | Misrepresentation FACTS:
1.   On Sept. 5, 1950, Yu Pand Eng submitted parts II and III of his application
PETITIONER: Yu Pang Cheng alias Yu Pang Ching for insurance consisting of the medical declaration made by him to the
RESPONDENTS: The Court of Appeals, et. al. medical examiner of defendant (SORRY, HINDI TALAGA NAKALAGAY SA
CASE SINO SIYA so not sure who kasi may “et. al” so defendant-insurer
SUMMARY: Yu Pand Eng submitted his application for insurance of Sept. 1950. na lang sa next parts), and the medical examiner’s report.
He was issued a policy 3 days after. He entered St. Lukes on Dec. 29, 1950 but he 2.   On Sept. 7, he submitted part I of his application which is the declaration
eventually died of “infiltrating medullary carcinoma, Grade 4, advanced catdiac and made by him to an agent of defendant-insurer
lesser curvature, stomach metastases spleen” on Feb. 1951. Cheng, brother and 3.   Sept. 8: based on said application, and upon payment of the first premium in
beneficiary of Yu Pand Eng, demanded from the defendant-insurer the proceeds of the sum of P591.70, defendant-insurer issued to the insured Policy No.
the insurance but it was denied. Hence, this petition. Defendant-insurer in its answer, 812858
set up the defense that Yu Pand Eng was guilty of misrepresentation and 4.   Dec. 27, 1950: Yu Pand Eng entered St. Luke’s Hospital for medical
concealment of material facts in that he gave false and untruthful answers to certain treatment but he died on Feb. 27, 1951. According to the death certificate,
questions asked him in his application for insurance which were material to the risk he died of “infiltrating medullary carcinoma, Grade 4, advanced catdiac and
against and have the effect of avoiding the insurance policy. Trial court ruled in lesser curvature, stomach metastases spleen.”
favor of Cheng but this was reversed by the CA. The issue in this case is WoN the 5.   Yu Pang Cheng (Cheng), brother and beneficiary of the insured, demanded
insured is guilty of concealment of some facts material to the risk insured against from defendant-insurer the payment of the proceeds of the insurance polic.
which has the effect of avoiding the policy as found by the CA. It appears that Yu 6.   Defendant-insurer refused. Thus, Cheng brought this action to collect from
Pand Eng entered the Chinese General Hospital for medical treatment on January 29, the defendant-insurer the sum of P10,000, value of an insurance policy
1950 having stayed there up to February 11, 1950. Upon entering the hospital, he taken upon the life of one Yu Pand Eng, plus interest thereon at the legal
complained of dizziness, anemia, abdominal paids and tarry stools, and in the rate, the sum of P10,000 as moral damages, P3000 as attorney’s fees, and
evening of his admission he had several abdominal pains and his discharges were the costs of action
with black tarry stools and felt dizzy and weak. Yu Pand Eng's confinement in the 7.   Defendant-insurer in its answer, set up the defense that Yu Pand Eng was
Chinese General Hospital took place from January 29, 1950 to February 11, 1950, guilty of misrepresentation and concealment of material facts in that he
whereas his application for insurance wherein he stated his answers to the questions gave false and untruthful answers to certain questions asked him in his
propounded to him by the examining physician of defendant-insurer was submitted application for insurance which were material to the risk against and
to defendant-insurer on September 5, 1950. (check ratio #1) 5. It is apparent that have the effect of avoiding the insurance policy
when the insured gave his answers regarding his previous ailment, particularly with 8.   After trial, the court rendered judgment ordering defendant-insurer to pay
regard to "Gaztritis, Ulcer of the Stomach or any disease of that organ" and "Vertigo, Cheng the sum of P10k, with legal interest from the filing of the complaint,
Dizziness, Fainting-spells or Unconsciousness", he concealed the ailment of which plus P2k as attorney’s fees, and the costs of the suit
he was treated in the Chinese General Hospital which precisely has direct connection 9.   On appeal, the CA reversed the decision of the trial court, holding that the
with the subject of the questions propounded. This deprived defendant-insurer of the insured was guilty from liability.
opportunity to make the necessary inquiry as to the nature of his past illness so that it 10.   Hence, the present petition for review.
may form its estimate relative to the approval of his application. Had defendant-
insurer been given such opportunity, considering the previous illness of the insured ISSUE/s:
as disclosed by the records of the Chinese General Hospital, defendant-insurer would 1.   WoN the insured is guilty of concealment of some facts material to the risk
probably had never consented to the issuance of the policy in question. (Doctrine) insured against which has the effect of avoiding the policy as found by the
CA – YES. Eng lied on the application despite actually knowing his
DOCTRINE: Concealment is the neglect to communicate that which a party knows sickness.
and ought to communicate. Whenever intentional, the concealment entitles the
insurer to rescind the contract of insurance. The insurance law requires the insured to RULING: the decision appealed from is affirmed, with costs against Cheng
communicate to the insurer all facts within his knowledge which are material to the
contract and which the other party has no means of ascertaining, and the materiality RATIO:
is to be determined not by the event, but solely by the probable and reasonable
influence of the facts upon the party to whom the communication is due.
1.   The insured, in his application for insurance, particularly in his declarations stomach metastases spleen", which may have a direct connection with his
to the examining physician, stated the following in answering the questions previous illness.
propounded to him: 9.   Our Insurance Law provides that A neglect to communicate that which
"14. Have you ever had any of the following diseases or symptoms? Each question a party knows and ought to communicate, is called concealment"
must be read and answered "Yes" or "No." (Section 25, Act No. 2427 or the Insurance Act (not Code)).
xxx xxx xxx 10.   Whether intentional or unintentional, the concealment entitles the
"Gastritis, Ulcer of the Stomach or any disease of that organ? No. insurer to rescind the contract of insurance (Section 26).
"Vertigo, Dizziness, Fainting-spells or Unconsciousness? No. 11.   Our law even requires the insured to communicate to the insurer all
"Cancer, Tumors or Ulcers of any kind? No. facts within his knowledge which are material to the contract and
"15. Have you ever consulted any physician not included in any of the above which the other partty has not the means of ascertaining (Section 27)
answers? Give names and address or physicians list ailments or accidents and date. 12.   the materiality is to be determined not by the event but solely by the
No." probable and reasonable in uence of the facts upon the party to whom
2.   It appears that Yu Pand Eng entered the Chinese General Hospital for the communication is due (Section 30).
medical treatment on January 29, 1950 having stayed there up to February 13.   In the case of Argente vs. West Coast Life Insurance Co., 51 Phil., 725, this
11, 1950. Upon entering the hospital, he complained of dizziness, anemia, Court said:
abdominal paids and tarry stools, and in the evening of his admission he had "One ground for the rescission of a contract of insurance under the
several abdominal pains and his discharges were with black tarry stools and Insurance Act is 'a concealment', which in section 25 is defined 'A neglect
felt dizzy and weak. to communicate that which a party knows and ought to communicate.'
3.   The history of his illness shows that the same "started a year ago as frequent Appellant argues that the concealment was immaterial and isu ceint to avoid
dizziness." An X-ray picture of his stomach was taken and the diagnosis the policy. We cannot agree. In an action on a life insurance policy where
made of him by his doctors showed that his illness was "peptic ulcer, the evidence conclusively shows that the answers to questions concerning
bleeding." diseases were untrue, the truth or falsity of the answers become the
4.   Note: Yu Pand Eng's confinement in the Chinese General Hospital took determining factor. If the policy was procured by fraudulent representations,
place from January 29, 1950 to February 11, 1950, whereas his the contract of insurance apparently set forth therein was never legally
application for insurance wherein he stated his answers to the questions existent. It can fairly be assumed become that had the true facts been
propounded to him by the examining physician of defendant-insurer disclosed by he assured, the insurance would never have been granted."
was submitted to defendant-insurer on September 5, 1950. 14.   CA did not err in declaring the policy ineffective on the ground of
5.   It is apparent that when the insured gave his answers regarding his concealment and in relieving appellee from liability
previous ailment, particularly with regard to "Gaztritis, Ulcer of the
Stomach or any disease of that organ" and "Vertigo, Dizziness,
Fainting-spells or Unconsciousness", he concealed the ailment of which
he was treated in the Chinese General Hospital which precisely has
direct connection with the subject of the questions propounded.
6.   The negative answers given by the insured regarding his previous ailment,
or his concealment of the fact that he was hospitalized and treated for
sometime of peptic ulcer and had suffered form "dizziness, anemia,
abdominal pains and tarry stools", deprived defendant-insurer of the
opportunity to make the necessary inquiry as to the nature of his past
illness so that it may form its estimate relative to the approval of his
application.
7.   Had defendant-insurer been given such opportunity, considering the
previous illness of the insured as disclosed by the records of the Chinese
General Hospital, defendant-insurer would probably had never
consented to the issuance of the policy in question.
8.   In fact, according to the death certificate, the insured died of "infiltrating
medullary carcinoma, Grade 4, advanced cardiac and of lesser curvature,
013 EGUARAS v. GREAT EASTERN (Punsalan) the favorable medical examination made by Dr. Jose Vidal, Great Eastern
Jan. 24, 1916 | Torres, J. | Misrepresentation agreed to the life insurance sought.
3.   The contract of life insurance executed between The Great Eastern Life Assurance
Company, Ltd., and Dominador Albay is set forth in the policy itself and in the
PETITIONER: Francisca Eguaras
original and supplementary applications signed by Albay, it appeared to stipulate
RESPONDENTS: The Great Eastern Life Assurance Co., Ltd., West Smith
that:
a.   "This insurance is granted in consideration of the foregoing statements and
SUMMARY: Great Eastern Life Assurance, Co., Ltd. accepted Dominador Albay’s
agreement in the application presented to obtain this policy, which
application for insurance where P5,000.00 is to be paid by the insurer to Albay’s
application forms a part of the present contract."
beneficiary, Francisca Eguaras (mother-in-law). In perfection of the contract thereof,
4.   Such condition is repeated in Clause VIII of the conditions and privileges
Albay, who was not in good health and by connivance with insurance agent Remigio,
granted to the insured, that:
presented Castor Garcia (fake Albay) to Dr. Vidal who was commissioned by Great
a.   "This policy and the application presented to secure it, taken together,
Eastern to examine applicants for life insurance. In view of the favorable report of Dr.
constitute the whole contract, which cannot be altered except in writing by
Vidal that Albay (who was actually Garcia) was in good health and possessed the
the general manager or some person expressly appointed therefor by the
qualifications required by Great Eastern for perfecting the contract, the company freely
board of directors."
and willingly consented to the execution thereof. Such medical examination and of the
5.   (IMPT) In the supplementary application presented by Albay to the question “Do
favorable professional report induced Great Eastern to indeed believe that Albay was in
you think that you are free from disease and that you have a good
good health but who in fact was Castor Garcia. Actual insured Albay died 1 month and 23
constitution?” He answered “Yes.”
days in Sta. Cruz, Laguna after the insurance had been granted.
6.   (IMPT) Furthermore, in another question “Have you suffered from any affection
Issue: (1) WoN the life insurance obtained by Albay, with the assistance of Remigio of … (c) Chest – Cough, asthma, spitting blood, pleurisy?” he answered “No.”
(insurance agent), is legal and valid – NO 7.   Dr. Vidal, the physician of the insurance company in charge of the physical
The person who was actually examined was not Dominador Albay, but was another examination of the person applying for insurance in Laguna, did the check-up on
(Castor Garcia), in order to get a favorable recommendation on the health of the insured Albay and recorded that the development, expansion, percussion, and auscultation of
which induced Great Eastern to approve of the application and thus issue the contested the applicant's chest were "normal" and recommended to the company that it could
life insurance policy of Albay where Eguaras is the beneficiary. Furthermore, the "take the risk" of insuring the applicant Dominador Albay
signatures that appear on the papers referring to the insurance are so different from a.   The report also had medical officer Lunn placing “O.K.” on it.
those which appear on the other documents which unquestionably bear the 8.   A month after the issuance of the insurance policy, Albay allegedly died of
signature of the real Dominador Albay that SC can do no less than reach the intestinal occlusion (intestinal blockage which causes blockage of bowel) in Sta.
conclusion that there was a person who passed himself off as Dominador Albay and Cruz, Laguna, as proven in Dr. Kamatoy’s certificate.
said person was the one who went on signing the documents relating to the alleged 9.   Great Eastern, despite having received satisfactory proofs of its insured/Albay’s
insurance of Dominador Albay. death, refused to pay the amount of the insurance and alleged that the policy was
(2) WoN Great Eastern Life Assurance is under obligation to pay the value to Francisca secured through fraud and deceit and was therefore illegal and void.
Eguaras – NO
Having proven the misrepresentation by the insurance agent, serious deceit indeed NOTE: this recounts how the insurance policy was applied for and how the misrepresentation went about
occurred in perfecting the insurance contract. The company would not have granted 10.   It appears that Albay had knowledge of the false replies contained in the two
insurance applied for had misrepresentation and deception not occurred. Therefore, it is applications for insurance and knowingly permitted fraud to be practiced upon
improper nor is it permitted by the law to order collection of the P5,000.00 Great Eastern
a.   For in his acknowledgment and consent his mother-in-law was designated
DOCTRINE: Misrepresentation in applying for life insurance, also in connivance with as the beneficiary of the insurance, despite the fact that he had children
an insurance agent, constitutes serious deceit which cannot entitle a party to the proceeds and his mother was still living
of the policy despite being previously issued by the insurance company. 11.   The fraud consisted in the fact that a healthy and robust person was substituted in
  place of the Albay when Dr. Vidal made the physical exam of the one who was
FACTS: seeking to be insured, for Albay was in bad health on and before the date of
1.   Characters: executing the insurance contract, of which facts the insured Albay and the
Francisca Eguaras (Eguaras) – beneficiary; mother-in-law insurance agent Remigio had full knowledge.
Dominador Albay (Albay) – insured a.   Such person was presented to him by insurance agent Remigio who told
The Great Eastern Life Assurance Co., Ltd. (Great Eastern) – insurer him that such was Albay
Ponciano Remigio (Remigio) – insurance agent b.   But in truth, the person was Castor Garcia who presented himself to
2.   Oct. 14, 1912: Albay, through the efforts of Great Eastern’s agent, Ponciano be Dominador Albay.
Remigio, got to insure his life for the sum of P5,000.00, naming Eguaras (who 12.   The insurance company endeavored to prove by means of cross-examination of
was his mother-in-law) as the beneficiary in case of his death. Ponciano Remigioand by means of the declaration of another insurance agent, Jose
a.   That through Albay’s representations and statements in his application and D. Arce, that Remigio had always been in the habit of securing the insurance of
sick persons, who died shortly after it was issued, in fraud and to the serious 2.   But if the real Albay wrote in the Spanish style in the months of Jan-March
injury of the defendant company. 1912, as demonstrated by the signatures affixed to certain documents, it is
a.   Court overruled this attempt of Great Eastern and did not permit proof of impossible to believe that he should have radically changed his form of writing
specific fraudulent acts performed by its agent Remigio 2 months later by adopting a different handwriting as can be seen in the alleged
b.   It is observed that Remigio has already been convicted of the crime of signature in the electors.
estafa and was to restitute The Insular Life Insurance Co. an amount of 3.   The signatures that appear on the papers referring to the insurance are so
P20. different from those which appear on the other documents which
13.   There was also a sworn declaration by Atty. O’Brien indicating that agent Remigio unquestionably bear the signature of the real Dominador Albay that, in
interviewed him in his office telling him that the signatures affixed to the original consideration of the short time which elapsed between the last genuine signature in
and supplemental application for insurance signed before Dr. Vidal were false. March, 1912, when he sold a tract of land, and his oath as elector in May of the
a.   Remigio further told him that the former was disgusted with his same year, 1912, and the great difference that exists between the two signatures, we
accomplices because they could not reach an agreement regarding the can do no less than reach the conclusion already stated that there was a person
distribution among them of the amount of the policy. (scammaz) who passed himself off as Dominador Albay and said person was the one who
14.   Remigio denied all this in his testimony. went on signing the documents relating to the alleged insurance of Dominador
15.   Dr. Reyes also testified and said that Albay consulted him regarding the cough he Albay, who died on December 6, 1912
had and after a medical examination, the conclusion was that Albay was actually 4.   Moreover, Dominador Albay's age, according to the application and the insurance
suffering from tuberculosis in the first stage. policy, was 40 years in 1912, while according to his personal cedulas he was only 32
16.   There is circumstantial evidence in the case that Albay died of tuberculosis because years of age in 1911, so that when he was insured he must have been only 33.
his own mother, Manuela Flores, so affirmed in the affidavit. However, it was not 5.   It is therefore proven that the signatures on the insurance applications reading
presented as evidence because Flores repudiated it in court. "Dominador Albay" are false and forged
a.   It was repudiated because Great Eastern was unwilling to give her money 6.   Under Art. 126936(OLD CIVIL CODE), it is essential to the nature of the deceit that
for the statements they would make in the court in the event that West said deceit be prior to or contemporaneous with the consent that is a necessary
Smith, Great Eastern’s agent, should win the case, and accordingly to prerequisite for perfecting the contract, but not that it may have occurred or
execute “an instrument we can hold to.” happened thereafter.
a.   A contract is therefore deceitful, for the execution whereof the consent
NOTE: at this point, I’ll go back to what happened in the courts; though I think you can skip this sa recit of one of the parties has been secured by means of fraud, because he
17.   CFI rendered judgment where Great Eastern was to pay P5,000.00 to Eguaras with was persuaded by words or insidious machinations, statements or false
legal interest. promises, and a defective consent wrung from him, even though such do
18.   Great Eastern moved for a reopening of the case and a new trial which was denied not constitute estafa or any other criminal act subject to the penal
19.   Great Eastern filed a Bill of exceptions, approved and forwarded to the clerk of the law.
SC. 7.   This fraud consisted in the substitution at the examination of Castor Garcia in
place of the insured Dominador Albay, and as the deceit practiced in the said
ISSUE/s: contract is of a serious nature, the same is ipso facto void and ineffective, in
1.   WoN the life insurance obtained by Albay, with the assistance of Remigio accordance with the provisions of article 1270 of the [old] Civil Code.
(insurance agent), is legal and valid – NO, the person who was examined was not 8.   Serious deceit occurred in perfecting the insurance contract, for had the agent of the
Dominador Albay, but was another (Castor Garcia), in order to get a favorable company not been deceived it would not have granted the insurance applied for by
recommendation on the health of the insured which induced Great Eastern to Albay, nor would it have executed the contract by virtue whereof payment is
approve of the application and thus issue the contested life insurance policy of claimed of the value of the policy obtained through fraud; and consequently on such
Albay where Eguaras is the beneficiary. (misrepresentation) assumptions it is improper, nor is it permitted by the law, to order collection of the
2.   WoN Great Eastern Life Assurance is under obligation to pay the value (P5,000) to amount claimed.
Francisca Eguaras – NO. Having proven the misrepresentation by the insurance
agent, serious deceit indeed occurred in perfecting the insurance contract.

RULING: 1st part of judgment appealed from (regarding payment of P5000) is reversed. The
Great Eastern Life Assurance Company, Ltd. is absolved from the complaint. 2nd part of said
judgment in so far as it absolves West Smith is affirmed and petition for damages dismissed.

RATIO:
1.   Eguaras’ claim is based on the genuineness of the signature of “Dominador Albay”                                                                                                                        
in the elector’s oath on the contention that if the signature on said election oath is 36
Art. 1269: "There is deceit when by words or insidious machinations on the part of one of the
genuine, those which appear on the insurance applications, and that affixed to the contracting parties the other is induced to execute a contract which without them he would not have
Albay’s letter to Great Eastern, must also be true and genuine. made."
014 Soliman v. US (Cristelle)
1958 | N/A | Misrepresentation

Dean Hofi can’t find the original copy of this case, ito lang nasa internet
and super short digest pa ito.

DOCTRINE: One who solicits insurance is an underwriter and not an agent


of the insurance company. If insurer appoints a general agent, then such
agent can bind the company by virtue of the written appointment. On the
other hand, when an underwriter fills up a policy with false answers and
later the insured signs the policy, the false answers become the insured’s
own answers because he signed the policy.

FACTS:
1.   US Life issued a 20 year endowment life policy on the joint lives of
Patricio Soliman and his wife Rosario, each of them being the
beneficiary of the other.
2.   In March 1949, the spouses were informed that the premium for
January 1949 was still unpaid notwithstanding that the 31-day grace
period has already expired, and they were furnished at the same time
long-form health certificates for the reinstatement of the policies.
3.   In April 1949, they submitted the certificates and paid the premiums.
4.   In January 1950, Rosario died of acute dilation of the heart, and
thereafter, Patricio filed a claim for the proceeds of the insurance.
5.   US Life denied the claim and filed action for the rescission of the
contract on the ground that the certificates failed to disclose that
Rosario had been suffering from bronchial asthma for 3 years prior
to their submission.

ISSUES: Whether or not the contract can still be rescinded? Yes, it can
be rescinded.

RULING:

RATIO:
1.   The insurer is once again given two years from the date of
reinstatement to investigate into the veracity of the facts represented
by the insured in the application for reinstatement. When US life
sought to rescind the contract on the ground of
concealment/misrepresentation, two years had not yet
elapsed. Hence, the contract can still be rescinded.
015 Pacific Banking Corp v. CA (Rosales) the foundation does not exist, the superstructure does not arise. Falsehood in
November 28, 1988 | Paras, J. | Misrepresentation such representations is not shown to vary or add to the contract, or to terminate a
contract which has once been made, but to show that no contract has ever
PETITIONER: Pacific Banking Corporation existed. A void or inexistent contract is one which has no force and effect from
RESPONDENTS: Court of Appeals and Oriental Assurance Corporation the very beginning, as if it had never been entered into, and which cannot be
validated either by time or by ratification.
SUMMARY: On October 21,1963, Fire Policy No. F-3770, an open policy, was
issued to the Paramount Shirt Manufacturing Co., by which Oriental Assurance FACTS:
Corporation bound itself to indemnify the insured for any loss or damage, not 1.   On October 21,1963, Fire Policy No. F-3770, an open policy, was issued to
exceeding P61,000.00, caused by fire to its property for a period of one year. the Paramount Shirt Manufacturing Co., by which Oriental Assurance
Paramount Shirt is a debtor of Pacific Banking in the amount of not less than Corporation bound itself to indemnify the insured for any loss or damage,
P800,000.00 and the goods described in the policy were held in trust by the not exceeding P61,000.00, caused by fire to its property consisting of
insured for Pacific Banking under thrust receipts. A fire broke out on the subject stocks, materials and supplies usual to a shirt factory, including furniture,
premises destroying the goods contained in its ground and second floors. Pacific fixtures, machinery and equipment while contained in the ground, second
Banking sent a letter of demand to Oriental Assurance for indemnity due to the and third floors of the building situated at number 256 Jaboneros St., San
loss of property by fire. Oriental Assurance said it is not yet ready to accede to Nicolas, Manila, for a period of one year commencing from that date to
the latter's demand as it is awaiting the final report of the insurance adjuster, October 21, 1964.
H.H. Bayne Adjustment Company. The insurance adjuster notified Pacific 2.   Paramount Shirt was at the time of the issuance of the policy and is up to
Banking that Paramount Shirt under the policy had not filed any claim with it, this time, a debtor of Pacific Banking in the amount of not less than
nor submitted proof of loss which is a clear violation of Policy Condition No.11, P800,000.00 and the goods described in the policy were held in trust by the
and for which reason, determination of the liability of Oriental Assurance could insured for Pacific Banking under thrust receipts.
not be had. For failure of the insurance company to pay the loss as demanded, 3.   Said policy was duly endorsed to Pacific Banking as mortgagee/ trustor of
Pacific Banking on filed in the court an action for a sum of money. Pacific the properties insured, with the knowledge and consent of Oriental
Banking presented in evidence a communication revealing undeclared co- Assurance to the effect that "loss if any under this policy is payable to the
insurances undertaken by Paramount. RTC rendered a decision adjudging Pacific Banking Corporation".
Oriental Assurance liable to Pacific Banking under the said contract of 4.   On January 4, 1964, while the aforesaid policy was in full force and effect,
insurance. The CA reversed. a fire broke out on the subject premises destroying the goods contained in
its ground and second floors.
WoN Paramount Shirt, the insured, was guilty of fraud which rendered the 5.   On January 24, 1964, counsel for Pacific Banking sent a letter of demand to
contract of insurance void? – YES, because Paramount Shirt failed to reveal Oriental Assurance for indemnity due to the loss of property by fire under
before the loss three other insurances. By reason of said unrevealed insurances, the endorsement of said policy.
Paramount Shirt had been guilty of a false declaration; a clear misrepresentation 6.   On January 28, 1964, Oriental Assurance informed counsel for Pacific
and a vital one because where the insured had been asked to reveal but did not, Baking that it was not yet ready to accede to the latter's demand as the
that was deception. Concrete evidence of fraud or false declaration by former is awaiting the final report of the insurance adjuster, H.H. Bayne
Paramount Shirt was furnished by Pacific Banking itself when the facts alleged Adjustment Company.
in the policy under clauses "Co-Insurances Declared" and "Other Insurance 7.   On March 25, 1964, the said insurance adjuster notified counsel for Pacific
Clause" are materially different from the actual number of co-insurances taken Banking that Paramount Shirt under the policy had not filed any claim with
over the subject property. As the insurance policy against fire expressly required it, nor submitted proof of loss which is a clear violation of Policy Condition
that notice should be given by the insured of other insurance upon the same No.11, and for which reason, determination of the liability of Oriental
property, the total absence of such notice nullifies the policy. Undoubtedly, it is Assurance could not be had.
but fair and just that where the insured who is primarily entitled to receive the 8.   On April 24, 1964, Pacific Banking’s counsel replied to aforesaid letter
proceeds of the policy has by its fraud and/or misrepresentation, forfeited said asking the insurance adjuster to verify from the records of the Bureau of
right, with more reason Pacific Banking which is merely claiming as indorsee of Customs the entries of merchandise taken into the customs bonded
said insured, cannot be entitled to such proceeds. warehouse razed by fire as a reliable proof of loss.
9.   For failure of the insurance company to pay the loss as demanded, Pacific
DOCTRINE: Representations of facts are the foundation of the contract and if Banking on filed in the court an action for a sum of money against Oriental
Assurance Corporation in the principal sum of P61,000.00 issued in favor of merit, and the decision appealed from is AFFIRMED. No costs.
Paramount Shirt Manufacturing Co.
10.   Oriental Assurance raised the following defenses in its answer to wit: (a) RATIO:
lack of formal claim by insured over the loss and (b) premature filing of the 1.   Policy Condition No. 3 explicitly provides:
suit as neither Pacific Banking nor Paramount Shirt had submitted any proof The Insured shall give notice to the Company of any insurance
of loss on the basis of which Oriental Assurance would determine its already effected, or which may subsequently be effected, covering
liability and the amount thereof, either to Oriental Assurance or its adjuster any of the property hereby insured, and unless such notice be given
H.H. Bayne Adjustment Co., both in violation of Policy Condition No.11. and the particulars of such insurance or insurances be stated in or
11.   At the trial, Pacific Banking presented in evidence a communication dated endorsed on this Policy by or on behalf of the Company before the
December 22, 1965 of the insurance adjuster, H.H. Bayne Adjustment Co. occurrence of any loss or damage, all benefit under this policy
to Asian Surety Insurance Co., Inc., revealing undeclared co-insurances shall be forfeited.
with the following: P30,000.00 with Wellington Insurance; P25,000. 00 2.   It is not disputed that Paramount Shirt failed to reveal before the loss
with Empire Surety and P250,000.00 with Asian Surety; undertaken by three other insurances. As found by the Court of Appeals, by reason of
insured Paramount on the same property covered by its policy with said unrevealed insurances, the insured had been guilty of a false
Oriental Assurance whereas the only co-insurances declared in the declaration; a clear misrepresentation and a vital one because where
subject policy are those of P30,000.00 with Malayan P50,000.00 with the insured had been asked to reveal but did not, that was deception.
South Sea and P25.000.00 with Victory. Otherwise stated, had Paramount Shirt have known that there were
12.   It will be noted that the defense of fraud and/or violation of Condition No. 3 many co-insurances, it could have hesitated or plainly desisted from
in the Policy, in the form of non-declaration of co-insurances which was entering into such contract. Hence, the Paramount Shirt was guilty of
not pleaded in the answer was also not pleaded in the Motion to clear fraud.
Dismiss. 3.   Concrete evidence of fraud or false declaration by Paramount Shirt was
13.   At any rate the RTC denied Oriental Assurance’s motion on the ground that furnished by Pacific Banking itself when the facts alleged in the policy
the defense of lack of proof of loss or defects therein was raised for the under clauses "Co-Insurances Declared" and "Other Insurance Clause" are
first time after the commencement of the suit and that it must be materially different from the actual number of co-insurances taken over the
deemed to have waived the requirement of proof of loss. subject property.
14.   The case was considered submitted for decision from which order Oriental 4.   Consequently, the whole foundation of the contract fails, the risk does
Assurance filed a motion for reconsideration to set the case or further not attach and the policy never becomes a contract between the parties.
reception of Oriental Assurance’s additional evidence, “in order to prove Representations of facts are the foundation of the contract and if the
that insured has committed a violation of condition No. 3 of the policy in foundation does not exist, the superstructure does not arise. Falsehood
relation to the other Insurance Clause.” in such representations is not shown to vary or add to the contract, or
15.   The case was set for the continuation of the hearing for the reception merely to terminate a contract which has once been made, but to show that no
of the testimony of Alejandro Tan Gatue, Manager of the Adjustment Co., contract has ever existed. A void or inexistent contract is one which has
over the vehement opposition of Pacific Banking. no force and effect from the very beginning, as if it had never been
16.   RTC rendered a decision adjudging Oriental Assurance liable to Pacific entered into, and which cannot be validated either by time or by
Banking under the said contract of insurance. The CA reversed the decision ratification.
of the trial court. 5.   As the insurance policy against fire expressly required that notice
should be given by the insured of other insurance upon the same
ISSUE/s: property, the total absence of such notice nullifies the policy.
3.   WoN Paramount Shirt, the insured, was guilty of fraud which rendered the 6.   The argument that notice of co-insurances may be made orally negates
contract of insurance void? – YES, because Paramount Shirt failed to reveal policy condition No. 20 which requires every notice and other
before the loss three other insurances. By reason of said unrevealed communications to the insurer to be written or printed.
insurances, Paramount Shirt had been guilty of a false declaration; a clear 7.   Particularly referring to the mortgage clause of the policy, Pacific Banking
misrepresentation and a vital one because where the insured had been asked argues that considering the purpose for which the endorsement or
to reveal but did not, that was deception. assignment was made, that is, to protect the mortgagee/assignee against any
untoward act or omission of the insured, it would be absurd to hold that
RULING: PREMISES CONSIDERED, the petition is DISMISSED for lack of
Pacific Banking is barred from recovering the insurance on account of the necessary information to ascertain the particular account of the articles
alleged violation committed by the insured. destroyed by fire as well as the amount of loss. It is noteworthy that
8.   It is obvious that Pacific Banking has missed all together the import of Oriental Assurance and its adjuster notified Pacific Banking that insured
subject mortgage clause which specifically provides: had not yet filed a written claim nor submitted the supporting documents in
Mortgage Clause compliance with the requirements set forth in the policy. Despite the notice,
Loss, if any, under this policy, shall be payable to the PACIFIC the latter remained unheedful. Since the required claim by insured, together
BANKING CORPORATION Manila mortgagee/trustor as its with the preliminary submittal of relevant documents had not been
interest may appear, it being hereby understood and agreed that complied with, it follows that Oriental Assurance could not be deemed to
this insurance as to the interest of the mortgagee/trustor only have finally rejected Pacific Banking’s claim and therefore the latter's cause
herein, shall not be invalidated by any act or neglect—except of action had not yet arisen.
fraud or misrepresentation, or arson—of the mortgagor or 15.   Contracts of insurance are contracts of indemnity upon the terms and
owner/trustee of the property insured; provided, that in case the conditions specified in the policy. The parties have a right to impose
mortgagor or owner/ trustee neglects or refuses to pay any such reasonable conditions at the time of the making of the contract as
premium, the mortgagee/ trustor shall, on demand pay the same. they may deem wise and necessary. The agreement has the force of law
9.   The paragraph clearly states the exceptions to the general rule that between the parties. The terms of the policy constitute the measure of
insurance as to the interest of the mortgagee, cannot be invalidated; the insurer's liability, and in order to recover, the insured must show
namely: fraud, or misrepresentation or arson. As correctly found by the himself within those terms. The compliance of the insured with the
Court of Appeals, concealment of the aforecited co-insurances can easily be terms of the policy is a condition precedent to the light of recovery.
fraud, or in the very least, misrepresentation.
10.   Undoubtedly, it is but fair and just that where the insured who is
primarily entitled to receive the proceeds of the policy has by its fraud
and/or misrepresentation, forfeited said right, with more reason Pacific
Banking which is merely claiming as indorsee of said insured, cannot
be entitled to such proceeds.
11.   Pacific Banking further stressed that fraud which was not pleaded as a
defense in Oriental Assurance’s answer or motion to dismiss, should be
deemed to have been waived.
12.   It will be noted that the fact of fraud was tried by express or at least implied
consent of the parties. Pacific Banking did not only object to the
introduction of evidence but on the contrary, presented the very
evidence that proved its existence.
13.   In the case at bar, policy condition No. 11 specifically provides that the
insured shall on the happening of any loss or damage give notice to the
company and shall within fifteen days after such loss or damage deliver to
Oriental Assurance (a) a claim in writing giving particular account as to the
articles or goods destroyed and the amount of the loss or damage and (b)
particulars of all other insurances, if any. Likewise, insured was required "at
his own expense to produce, procure and give to the company all such
further particulars, plans, specifications, books, vouchers, invoices,
duplicates or copies thereof, documents, proofs and information with
respect to the claim".
14.   The evidence adduced shows that twenty-four days after the fire, Pacific
Banking merely wrote letters to Oriental Assurance to serve as a notice of
loss, thereafter, the former did not furnish the latter whatever pertinent
documents were necessary to prove and estimate its loss. Instead, Pacific
Banking shifted upon Oriental Assurance the burden of fishing out the
016 QUA CHEE GAN v. LAW UNION (Sabaupan) law is charitable enough to assume, in the absence of any showing to the
17 December 1955 | Reyes, J.B.L, J. | Breach of Warranty contrary, that an insurance company intends to execute a valid contract in return
for the premium received; and when the policy contains a condition which
PETITIONER: Qua Chee Gan renders it voidable at its inception, and this result is known to the insurer, it will
RESPONDENTS: Law Union and Rock Insurance Co., Ltd., represented by its be presumed to have intended to waive the conditions and to execute a binding
agent, Warner, Barnes and Co., Ltd. contract, rather than to have deceived the insured into thinking he is insured
when in fact he is not, and to have taken his money without consideration.”
SUMMARY: Qua Chee Gan owned 4 warehouses used for storage of stocks of
copra and of hemp. Bodegas 1, 3, and 4 were gutted by fire of unknown origin. It is well settled that the keeping of inflammable oils on the premises, though
Qua Chee Gan submitted fore claims to the insurance company but the latter prohibited by the policy, does not void it if such keeping is incidental to the
resisted payment claiming that the fire had been deliberately caused by the business.
insured. Qua Chee Gan then instituted a civil action to collect the insurance
proceeds. The insurance company argues among others that the insurance polices  
were avoided for breach of warranty. The issue is whether there was a breach of FACTS:
warranty, specifically the fire hydrant warranty and the hemp warranty. The 20.   Qua Chee Gan, a merchant from Albay, owned four warehouses or bodegas
Supreme Court ruled in the negative. The insurance company is barred by waiver (designated as Bodegas nos. 1-4) used for the storage of stocks of copra and
(or rather estoppel) to claim violation of the fire hydrants warranty, because of hemp, baled and loose. The bodegas, with their contents, were insured
knowing fully all the number of hydrants demanded therein never existed from with the Law Union (insurance company) since 1937, and the loss made
the very beginning, the insurance company nevertheless issued the policies in payable to the Philippine National Bank (PNB) as mortgage of the hemp
question subject to such warranty and received the corresponding premiums. The and copra, to the extent of its interest.
insurance company was aware, even before the policies were issued, that in the 21.   Fire of undetermined origin broke out in July 1940 and lasted for almost
premises insured, there were only 2 fire hydrants installed and 2 others nearby, one week. It gutted and completely destroyed Bodegas Nos. 1, 3, and 4,
owned by the municipality of Tabaco, contrary to the requirements of the with the merchandise stored therein. Qua Chee Gan informed the insurer on
warranty in question. For the hemp warranty, the Court noted that gasoline is not the same day of the fire and the next day, fire adjusters engaged by the
specifically mentioned among the prohibited articles listed in the “hemp insurance company arrived and proceeded to conduct an extensive
warranty.” The cause relied upon by the insurer speaks of “oils” (animal and/or investigation.
vegetable and/or mineral and/or their liquid products having a flash point of 22.   Qua Chee Gan submitted fire claims amounting to P398,562 (but reduced to
below 300°F). This is ambiguous and uncertain, for in ordinary parlance, “oils” the full amount of the insurance, P370,000). The insurance company
mean lubricants and not gasoline or kerosene. And how many insured are in a resisted payment, claiming violation of warranties and conditions, filing of
position to understand or determine “flash point below 300°F.” Here, again, by fraudulent claims, and that the fire had been deliberately caused by the
reason of the exclusive control of the insurance company over the terms and insured or by other persons in connivance with him.
phraseology of the contract, the ambiguity must be held strictly against the 23.   Qua Chee Gan and his brother, Qua Chee Pao, and some employees of his,
insurer and liberally in favor of the insured, specially to avoid forfeiture. were indicted and tried in 1940 for the crime of arson, it being claimed that
Another point that is in favor of the insured is that the gasoline kept in Bodega they had set fire to the destroyed warehouses to collect the insurance. They
No. 2 was only incidental to his business. It should also be noted that the "Hemp were acquitted.
Warranty" forbade storage only "in the building to which this insurance applies 24.   Thereafter, a civil suit to collect the insurance money was filed before the
and/or in any building communicating therewith", and it is undisputed that no CFI to recover the proceeds of certain life insurance policies. The CFI ruled
gasoline was stored in the burned bodegas, and that "Bodega No. 2" which was in favor of Qua Chee Gan. Hence, the instant petition.
not burned and where the gasoline was found, stood isolated from the other 25.   Insurance Company’s arguments:
insured bodegas. a.   The trial court should have held that the policies were avoided for
breach of warranty, specifically the one appearing on a rider pasted
DOCTRINE: It is usually held that where the insurer, at the time of the issuance on the face of the policies. These riders were attached for the first
of a policy of insurance, has knowledge of existing facts which, if insisted on, time in 1939, and the pertinent portions read as follows:
would invalidate the contract from its very inception, each knowledge constitutes
a waiver of conditions in the contract inconsistent with the known facts, and the “Memo. Of Warranty. – The undernoted Appliances for the
insurer is stopped thereafter from asserting the breach of such conditions. The extinction of fire being kept on the premises insured hereby, and it
being declared and understood that there is an ample end constant RATIO:
water supply with sufficient pressure available at all seasons for On the violation of fire hydrant warranty
the same, it is hereby warranted that the said appliances shall be 9.   The insurance company is barred by waiver (or rather estoppel) to claim
maintained in efficient working order during the currency of this violation of the fire hydrants warranty, because knowing fully all the
policy, by reason whereof a discount of 2.5% is allowed on the number of hydrants demanded therein never existed from the very
premium chargeable under this policy. beginning, the insurance company nevertheless issued the policies in
question subject to such warranty and received the corresponding
Hydrants in the compound, not less in number than one for each premiums.
150 feet of external wall measurement of buildings, protected, with 10.   It would perilously close to conniving at fraud upon the insured to allow the
not less than 100 feet of hose piping and nozzles for every 2 insurance company to claim now as void ab initio the policies that it had
hydrants kept under cover in convenient places, the hydrants being issued to Qua Chee Gan without warning of their fatal defect, of which it
supplied with water pressure by a pumping engine, or from some was informed, and after it had misled Qua Chee Gan into believing that the
other source, capable of discharging at the rate of not less than 200 policies were effective.
gallons of water per minute into the upper story of the highest 11.   The insurance company was aware, even before the policies were issued,
building protected, and a trained brigade of not less than 20 mean that in the premises insured, there were only 2 fire hydrants installed and 2
to work the same.” others nearby, owned by the municipality of Tabaco, contrary to the
b.   Given the size of the external wall of the bodegas, Qua Chee Gan requirements of the warranty in question. This fact was established by a
should have 11 fire hydrants in the compound, and that he actually positive testimony of the insurance company’s agents that they inspected
had only two, with a further pair nearby, belonging to the the premises. That such inspection was made is rendered probable by its
municipality of Tabaco. being a prerequisite for the fixing of the discount on the premium to which
c.   The insured violated the “Hemp Warranty” provisions of the the insured was entitled, since the discount depended on the number of
policy covering Bodega No. 2 against the storage of gasoline, since hydrants, and the firefighting equipment available.
Qua Chee Gan admitted that there were 36 cans of gasoline 12.   The law, supported by a long line of cases, is expressed by American
Bodega No. 2. Jurisprudence: “It is usually held that where the insurer, at the time of
d.   There was fraudulent overvaluation and the insured caused the fire the issuance of a policy of insurance, has knowledge of existing facts
so that he can pay the mortgagee bank. which, if insisted on, would invalidate the contract from its very
e.   The burned bodegas could not have contained the quantities of inception, each knowledge constitutes a waiver of conditions in the
copra and hemp stated in the fire claims. contract inconsistent with the known facts, and the insurer is stopped
f.   The insured made false and fraudulent statements that voids the thereafter from asserting the breach of such conditions. The law is
policy. charitable enough to assume, in the absence of any showing to the contrary,
that an insurance company intends to execute a valid contract in return for
ISSUE/s: the premium received; and when the policy contains a condition which
4.   Whether there was a violation of the fire hydrant warranty. – NO, the renders it voidable at its inception, and this result is known to the insurer, it
insurer is barred by estoppel to claim such violation. will be presumed to have intended to waive the conditions and to execute a
5.   Whether there was a violation of the hemp warranty. – NO, gasoline is not binding contract, rather than to have deceived the insured into thinking he is
specifically mentioned among the prohibited articles listed in the “hemp insured when in fact he is not, and to have taken his money without
warranty.” consideration.”
6.   Whether the insured made false and fraudulent statements that voids the 13.   The reason for this rule is as follows: “x x x To allow a company to accept
policy. – NO, the discrepancies were a result of the insured’s erroneous one's money for a policy of insurance which it then knows to be void and of
interpretation of the provisions of the insurance policies and claim forms, no effect, though it knows as it must, that the assured believes it to be valid
caused by his imperfect knowledge of English, and that the misstatements and binding, is so contrary to the dictates of honesty and fair dealing, and so
were innocently made and without intent to defraud. closely related to positive fraud, as to be abhorrent to fairminded men. It
would be to allow the company to treat the policy as valid long enough to
RULING: We find no reversible error in the judgment appealed from, wherefore the get the premium on it and leave it at liberty to repudiate it the next moment.
same is hereby affirmed. x x x”
14.   The inequitableness of the conduct observed by the insurance company in
this case is heightened by the fact that after the insured had incurred the would be unreasonable to expect the insured to maintain for his compound
expense of installing the two hydrants, the company collected the premiums alone a firefighting force that many municipalities in the islands do not even
and issued him a policy so worded that it gave the insured a discount much possess.
smaller than that he was normally entitled to. According to the "Scale of
Allowances," a policy subject to a warranty of the existence of one re On the violation of hemp warranty
hydrant for every 150 feet of external wall entitled the insured to a discount 21.   The Court noted that gasoline is not specifically mentioned among the
of 7.5% of the premium; while the existence of "hydrants, in compound" prohibited articles listed in the “hemp warranty.” The cause relied upon by
(regardless of number) reduced the allowance on the premium to a mere the insurer speaks of “oils” (animal and/or vegetable and/or mineral and/or
2.5%. their liquid products having a flash point of below 300°F). This is
15.   But the insurance company in this case, so worded the policies that while ambiguous and uncertain, for in ordinary parlance, “oils” mean lubricants
exacting the greater number of fire hydrants and appliances, it kept the and not gasoline or kerosene. And how many insured are in a position to
premium discount at the minimum of 2.5%, thereby giving the insurance understand or determine “flash point below 300°F.” Here, again, by reason
company a double benefit. No reason is shown why Qua Chee Gan’s of the exclusive control of the insurance company over the terms and
premises, that had been insured with the insurance company for several phraseology of the contract, the ambiguity must be held strictly against the
years past, should suddenly be regarded in 1939 as so hazardous as to be insurer and liberally in favor of the insured, specially to avoid forfeiture.
accorded a treatment beyond the limits of the insurance company’s own 22.   There is no reason why the prohibition of keeping gasoline in the premises
scale of allowances. Such abnormal treatment of the insured strongly points could not be expressed clearly and unmistakably, in the language and terms
at an abuse of the insurance company’s selection of the words and terms of that the general public can readily understand, without resort to obscure
the contract, over which it had absolute control. esoteric expression. If intended to rely upon a condition of that character, it
16.   Parol evidence is not applicable in the instant case. It is a well settled rule ought to have been plainly expressed in the policy.
of law that an insurer which with knowledge of facts entitling it to treat 23.   Another point that is in favor of the insured is that the gasoline kept in
a policy as no longer in force, receives and accepts a premium on the Bodega No. 2 was only incidental to his business, being no more than a
policy, estopped to take advantage of the forfeiture. x x x It would be customary 2 day's supply for the five or six motor vehicles used for
unconscionable to permit a company to issue a policy under circumstances transporting of the stored merchandise. It is well settled that the keeping
which it knew rendered the policy void and then to accept and retain of inflammable oils on the premises, though prohibited by the policy,
premiums under such a void policy. does not void it if such keeping is incidental to the business. According
17.   Moreover, taking into account the well-known rule that ambiguities or to the weight of authority, even though there are printed prohibitions against
obscurities must be strictly interpreted against the party that caused them, keeping certain articles on the insured premises the policy will not be
the "memo of warranty" invoked by the insurance company bars the latter avoided by a violation of these prohibitions, if the prohibited articles are
from questioning the existence of the appliances called for in the insured necessary or in customary use in carrying on the trade or business
premises, since its initial expression, "the undernoted appliances for the conducted on the premises.
extinction of fire being kept on the premises insured hereby, . . . it is hereby 24.   It should also be noted that the "Hemp Warranty" forbade storage only "in
warranted . . . ", admits of interpretation as an admission of the existence of the building to which this insurance applies and/or in any building
such appliances which appellant cannot now contradict, should the parol communicating therewith", and it is undisputed that no gasoline was stored
evidence rule apply. in the burned bodegas, and that "Bodega No. 2" which was not burned and
18.   The alleged violation of the firehose warranty must be equally rejected, where the gasoline was found, stood isolated from the other insured
since the insurance company’s argument thereon is based on the assumption bodegas.
that the insured was bound to maintain no less than eleven hydrants (one per
150 feet of wall), which requirement the insurance company is estopped On the charge of fraudulent overvaluation
from enforcing. 25.   This charge was found unsubstantiated by the trial court, and no reason has
19.   On the breach of water pressure condition, the court found such claim been shown to alter this finding. The insured gave the insurance examiner
worthless because the witness repeatedly refused and professed inability to all the data he asked for and the examiner even kept and photograph some
estimate the rate of discharge of the water. of the examined books in his possession.
20.   As to the maintenance of a trained fire brigade of 20 mean, the records
shows that the same was organized, and drilled, although not maintained as On the insured’s connivance at the loss
a permanently separate unit, which the warranty did not require. Anyway, it 26.   This defense is predicated on the assumption that the insured was n
financial difficulties and set the fire to defraud the insurance company, under the terms of the policies, and the trial court correctly deducted the
presumably in order to pay off PNB, to which most of the insured hemp and same from its award.
copra was pledged.
27.   This defense is fatally undermined by the established fact that,
notwithstanding the insurer’s refusal to pay the value of the policies,
insured was able to pay off PNB in a short time, and if he was able to do so,
no motive appears for attempt to defraud the insurer.
28.   While the acquittal of the insured in the arson case is not res judicata on the
present civil action, the insurer's evidence, to judge from the decision in the
criminal case, is practically identical in both cases and must lead to the
same result, since the proof to establish the defense of connivance at the re
in order to defraud the insurer "cannot be materially less convincing than
that required in order to convict the insured of the crime of arson.”

On the claim that the burned bodegas could not have contained the quantities of
copra and hemp stated in the fire claims
29.   The basis of the claim was the testimony of the insurer’s adjuster
investigator who examined the premises during and after the fire. His
testimony was however based on inferences from photographs and traces
found after the fire, and must yield to the contradictory testimony of the
engineer, the Chief of the loan department of PNB Legazpi branch, and of
the bank appraiser, who actually saw the contents of the bodegas shortly
before the fire, while inspecting them for the mortgagee bank.

On the claim of false and fraudulent statements by the insured


30.   The trial court found that the discrepancies were a result of the insured’s
erroneous interpretation of the provisions of the insurance policies and
claim forms, caused by his imperfect knowledge of English, and that the
misstatements were innocently made and without intent to defraud.
31.   Considering that all these claims were submitted to the same agent, and that
this same agent had paid the loss caused by the 1939 fire, the Court found
no error in the trial Court's acceptance of the insured's explanation that the
omission in certain policies was due to inadvertance, for the insured could
hardly expect under such circumstances, that the 1939 would pass unnoticed
by the insurance agents.
32.   Moreover, the 20% overclaim on 70% of the hemp stock, was explained by
the insured as caused by his belief that he was entitled to include in the
claim his expected profit on the 70 per cent of the hemp, because the same
as already contracted for and sold to other parties before the fire occurred.
The insured’s overclaim of 20% in the case at bar cannot be regarded as
“more than misstatement, more than inadvertence of mistake, more than a
mere error in opinion, more than a slight exaggeration” that would entitle
the insurer to avoid the policy.
33.   The rule is that to avoid a policy, the false swearing must be willful and
with intent to defraud which was not the cause. Of course, the lack of
fraudulent intent would not authorize the collection of the expected profit
017 FILIPINAS COMPAÑIA v. CA (Saldua) FACTS:
Jan. 31, 1950 | Padilla J. | Breach of Waranty 1.   Filipinas Compania de Seguros is a domestic insurance corp. licensed to
engage in the insurance business in PH. Tan Chauco is the owner of a
PETITIONER: Filipinas Compania de Seguros
building located in Lucena (then w/in Tayabas province). Tan Chauco
RESPONDENTS: Tan Chauco
insured the building for P20,000 and P10,000 in 2 policies issued by
Filipinas Compania.
SUMMARY:
2.   On Jan. 5, 1942, during the term of said 2 policies, the building insured
Tan Chauco insured his building under 2 insurance policies with Filipinas Compania.
was burned and completely destroyed. Notice of proof & loss had been
During the term of the 2 policies, a fire occurred which completely destroyed the
duly made, but since Filipinas Compania refused to pay, Tan Chauco filed
building. Tan Chauco claimed compensation under the policies but Filipinas Compania
an action to recover on the policies.
denied alleging that there was breach of warranties of the insurance policies rendering
3.   After trial, judgment was rendered against Filipinas Compania for the
it not compensable. The breach of warranties alleged are: 1) Sealing of the building by
amount of the 2 policies w/ legal interest from the date of filing of the
the Japanese Forces changed the nature of the building insured which increased the risk
complaint. CA affirmed this.
of loss/damage violating Art. 8 of the policies; 2) The fire occurred was an indirect
4.   Meanwhile, Filipinas Compania went up to the SC raised 3 contentions:
result of the abnormal conditions at that time i.e. Japanese occupation and as per Art. 6
A.   That Art. 8 of the policies provides that if the circumstances
of the policies, since the fire was connected with the abnormal conditions, it is not
affecting the building insured (or its contents) are changed in such a
compensable; and 3) Tan Chauco cannot recover bec. he had made a fraudulent
way as to increase the risk of loss/damage by fire, the insurance
declaration in the policy in w/c he denied a previous fire while in the trial, he admitted
ceases to attach as regards the property affected; UNLESS the
that there had been a previous fire on the land on which the insured building was built,
insured obtains the permission of the company before the occurrence
which was also had been destroyed by fire from neighboring buildings, violating Art.
through an endorsement upon the policy by or on behalf of the
13. Issue is WON there was a breach of warranty. SC said NO. As to the 1st allegation,
company.
the building was actually sealed as a form of precaution since looting was rampant and
the stores were abandoned by the owners, not because the contents of the building were •   Filipinas Compania alleges that the sealing of the subject
owned by enemy nationals. The sealing alone could not have increased the risk/hazard property by the Japanese Forces on Dec. 28, 1941 (before
to which a building is exposed of. As to the 2nd, Abnormal conditions are not just the fire occurred) had the effect of changing the nature of
disorders but also situations which deviated from the normal/ordinary. In this case, it the occupation thereof in a manner which increased the risk
came in the form of the absence of a regularly organized government with its police, of loss. It allegedly resulted to its conversion into an arsenal
health & fire depts. Meanwhile, the fire actually originated from the kitchen of a of war materials that it became the bounden duty of all loyal
neighboring Panciteria. They were next to each other such that while they were forces, whether the regular USAFFE or the guerrilla
separated by a narrow alley, the roofs of both houses almost touched each other and elements to destroy, and so increasing the risk of
loss/damage to the building insured.
covered the alley. Tan Chauco has in fact admitted that the fire could have occurred
even if there was a functioning fire dept. Thus, the risk caused by the proximity of the •   Thus, in accordance w/ the provisions of Art. 8, the
building to the Panciteria is not a new one, and must have already been considered at insurance ceased to attach as of the Dec. 28, 1941.
the time of issuing the policy. The fire could have occurred just as well in times of B.   That under Art. 6, the inferential finding that the fire of Jan. 5, 1942
peace and under normal conditions. Thus, the fire occurred from causes independent of was of accidental origin, and nothing more, could not make Tan
the abnormal conditions. As to the 3rd, SC held that the previous fire is irrelevant as far Chauco’s loss compensable since the contracts of insurance between
as the subject Jan. 5, 1942 fire is concerned. Hence, there was no breach of warranty. the parties specifically required that Tan Chauco prove that the loss
happened independently of the abnormal conditions.Thus, the fire,
DOCTRINE: being an indirect result of the abnormal conditions, is not
As the words of the policy are those of the company, they should be taken most compensable.
strongly against it. Interpretation should be adopted which is most favorable to the C.   That Art. 13 provides for the grounds of forfeiture of the benefits
insured, if such interpretation be not inconsistent with the words used. under the policies among which is: If any false declaration be made
It should mean that the policy covered loss by fire occurring during the existence of or used in support thereof.
any invasion, foreign enemy, rebellion, insurrection, riot, civil commotion, •   Filipinas Compania is alleging that Tan Chauco cannot
military/usurped power, or martial law, in the general locality where the property recover because he had made fraudulent declaration in the
insured was located (so long as the fire was not connected with the said situations). insurance claim that he submitted to the former i.e. that Tan
Chauco denied that there had been a previous fire in the
premises in which he was interested, while at the trial, Tan RE. ART. 6 OF THE POLICY: THE FIRE ACTUALLY OCCURRED FROM
Chauco admitted that there had been a previous fire on the CAUSES INDEPENDENT OF THE ABNORMAL CONDITIONS AT THE
land on which the insured building was built, which had TIME AND IS THUS COMPENSABLE.
also been destroyed by fire from neighboring buildings. 1.   Abnormal Conditions, accdg. to the SC, should be interpreted as to mean “a
ISSUE/s: situation, a condition of things deviating from the normal/ordinary, and
1.   WON there was a breach of warranty in the insurance policies? = NO. produced only by war of invasion, etc. Abnormal conditions do not
First, building was actually sealed as a form of precaution since looting was necessarily imply disorders, fighting, looting, etc. The existence of a
rampant, not because the contents of the building were owned by enemy regularly organized government with its police, health & fire depts. is a sign
nationals. The sealing alone could not have increased the risk/hazard to of normalcy in a community. The absence thereof is a sign of abnormal
which a building is exposed of. The fire actually originated from the kitchen conditions.
of a neighboring Panciteria. They were next to each other such that while 2.   Thus, the fact that the Lucena gov’t. ceased to function even before the
they were separated by a narrow alley, roofs of both houses almost touched entry of the Japs, and that there was no police dept. (Jap. Soldiers kept
each other and covered the alley. Tan Chauco has in fact admitted that the peace & order), the fire dept. had not yet been organized (no equipment as
fire could have occurred even if there was a functioning fire dept.. Thus, the well except for the hose), electrical service was suspended until Feb. 3,
risk caused by the prixmity of the building to the Panciteria is not a new 1942, and that there was a curfew imposed even after the date of the fire on
one, and must have already been considered at the time of issuing the Jan 5, 1942, shows there were abnormal conditions in Lucena on and before
policy. The fire could have occurred just as well in times of peace and under the fire occurred.
normal conditions. 3.   The fire that consumed Tan Chauco’s building was actually caused by a fire
that originated in the kitchen of Perrera’s Panciteria. They were next to each
RULING: The petition for a writ of certiorari is dismissed, with costs against the other such that while they were separated by a narrow alley, the roofs of
petitioner. both houses almost touched each other and covered the alley.
4.   Tan Chauco has in fact admitted that the fire could have occurred even if
RATIO: there was a functioning fire department. Thus, the risk caused by the
RE. ART. 8 OF THE POLICY: SEALING OF THE BUILDING BY THE JAP. prixmity of the building to the Panciteria is not a new one, and must have
FORCES DID NOT CHANGE THE NATURE OF THE BUILDING INSURED already been considered at the time of issuing the policy. The fire could
AND SO IT DID INCREASE THE RISK OF LOSS/DAMAGE have occurred just as well in times of peace and under normal conditions.
1.   At the time that the Japanese forces entered Lucena on Dec. 27, 1941, said
building was closed. On Dec. 28, 1941, all the stores including the stores in RE. ART. 13 OF THE POLICY: FRADULENT CLAIM NOT RELEVANT
Tan Chauco’s building were sealed by the Japanese Forces (except those 1.   SC held that the the previous fire that Tan Chauco failed to mention is
that were open). immaterial and irrelevant so far as the subject Jan. 5, 1942 fire is
2.   SC affirmed the finding of the trial court that the reason for the sealing concerned.
of the building is a form of precaution since looting was rampant and INTERPRETATION OF THE INSURANCE POLICIES/CONTRACTS
the stores were abandoned by the owners, not because the contents of 1.   As the words of the policy are those of the company, they should be taken
the building were owned by enemy nationals. most strongly against it. Interpretation should be adopted which is most
3.   As to the supposed increase in the risk, there were only 3 possible sources favorable to the insured, if such interpretation be not inconsistent with the
of danger to which the building insured could have been exposed to either words used.
by the acts of USAFFE, guerilla or civilian saboteurs. 2.   Thus, it should mean that the policy covered loss by fire occurring during
4.   However, there was no danger. The operations between the Jap. Army and the existence of any invasion, foreign enemy, rebellion, insurrection, riot,
USAFFE Forces shifted to the fortifications around Bataan & Corregidor. civil commotion, military/usurped power, or martial law, in the general
The guerilla units also only begun organizing only after the fall of Bataan locality where the property insured was located (so long as the fire was not
in April 1942 (well after the date of the fire). There was also no acts of connected with the said situations).
sabotage by civilians in Lucena or in Tayabas after the Jap. occupied 3.   In this case, the lower court has correctly found that the loss was occasion
Lucena on Dec. 27, 1941. Thus, except for looting, there was peace & quiet by causes that are independent of the existence of any invasion, foreign
in Lucena upon the Jap. Occupation. enemy, rebellion, insurrection, etc.
5.   Hence, sealing alone could not have increased the risk/hazard to which a
building is exposed of. Besides, the sealing was an act of the enemy over
which Tan Chauco had no influence or control.
018 YOUNG V. MIDLAND TEXTILE INSURANCE (EMAR) 7. Midland Ins alleged that the fireworks were "stored" within the meaning of
March 31, 1915 | Johnson, J. | Breach of Warranty “Warranty B”.
PLAINTIFF-APPELLEE: K.S. Young 8. Young contends that under all the facts and circumstances of the case, they were
DEFENDANT-APPELLANT: The Midland Textile Insurance Company not "stored" in said building, and that placing them in the building was not a
SUMMARY: Young and Midland Insurance entered into an insurance contract violation of the terms of the contract.
where Young would pay P60 for premium and Midland Ins. promised to pay 9. Both parties agree that if they were "hazardous goods," and were "stored," then the
Young P3k, in case the residence, bodega and its contents should be destroyed by Young’s act was a violation of the terms of the contract of Ins. and the Midland Ins
fire. The insurance contract has a condition under “Warranty B” that “during the was justified in repudiating its liability.
pendency of the policy, no hazardous goods [shall be] stored in the building”. 10. Young sought to recover P3k upon the policy.
Young was given 3 boxes of fireworks, intended to be used for the Chinese New 11. The lower court rendered a judgment in favor of the Young; against Midland Ins
Year but the city authorities prohibited the use of such so Young kept them in the for P2,708.78, and costs.
insured premises. The insured residence, bodega and contents were partly 12. Midland Ins appealed to this court.
destroyed by fire but the part where the fireworks were kept was not destroyed.
The issue in this case is WON placing of said fireworks in the building insured is ISSUE: WON the placing of the fireworks in the building insured, under the
a violation of the terms of the insurance contract – YES. The "hazardous goods" conditions enumerated, they being "hazardous goods," is a violation of the terms of
in question were "stored" in the bodega, as that word is generally defined. the contract of Ins. and especially of "warranty B." – YES. The "hazardous goods"
Suppose Midland Ins made an examination of the premises, even in the absence of were "stored" in the bodega, as that word is generally defined.
a fire, and had found he "hazardous goods" there, under the conditions above
described, it would be justified in declaring the policy null and of no effect by RULING: Judgment of the lower court is revoked and Midland Ins. is relieved from
reason of Young’s violation of its terms. Young paid a premium based upon the any responsibility under said complaint, and, without any finding as to costs.
risk at the time the policy was issued. Placing of the firecrackers in the building
insured increased the risk. Young did not pay a premium based upon the increased RATIO:
risk, neither had Midland Ins issued a policy upon the theory of a different risk. 1.   WON a particular article is "stored" must depend upon the intention of the
DOCTRINE: If the "warranty" is a term of the contract, its violation causes a parties.
breach and justify noncompliance or a repudiation. 2.   Most cases cited by the lower court are cases where the article was being put to
FACTS: some reasonable and actual use, which might easily have been permitted by
1. Young has a candy and fruit store in Manila, and occupied a building at Calle the terms of the policy, and within the intent of the parties, and excepted
Claveria, as a residence and bodega (storehouse). from the operation of the warranty, like the present. Said decision are upon cases
2. May 1912: The Midland Textile Insurance Company (Midland Ins), in like:
consideration of the payment of a premium of P60, entered into a contract of Ins. a. Where merchants have had or kept the "hazardous" articles in small
with the plaintiff (policy No. 509105) where Midland Ins. promised to pay Young quantities, and for actual daily use, for safe, such as gasoline, gunpowder, etc.;
P3k, in case the residence, bodega and contents should be destroyed by fire. b. Where such articles have been brought on the premises for actual use thereon,
3. A condition of the insurance contract states: "Waranty B. — It is agreed that and in small quantities, such as oil, paints, etc; and
during the pendency of this policy no hazardous goods stored or kept for sale, and c. Where such articles or goods were used for lighting purpose, and in small
no hazardous trade or process be carried on, in the building to which this Ins. applies, quantities.
or in any building connected therewith." 3.   Century and Standard Dictionaries define “Store”: to be a deposit in a store or
4. Feb 1913: Young was given fireworks, intended to be used for the Chinese New warehouse for preservation/safe keeping; place in a warehouse/other place of
Year but since City authorities prohibited its use, the 3 boxes of fireworks were deposit for safe keeping.
placed in the residence and bodega, in order that Young might later send them to a 4.   The definitions do not include a deposit in a store, in small quantities, for daily
friend in the provinces. use. "Daily use" precludes the idea of a deposit for preservation or safe keeping,
5. March 1913: The residence, bodega and its contents were partially destroyed by as well as a deposit for future consumption, or safe keeping.
fire but the part of the building 5.   In this case, no claim is made that the "hazardous goods" were placed in the
6. Young and Midland Ins agree that fireworks come within the phrase "hazardous bodega for present/daily use. They were placed in the bodega for sake keeping,
goods," mentioned in "warranty B" of the policy and that fireworks were found in a future use or consumption. Young did not claim that he deposited them there
part of the building not destroyed by the fire; that they did not contribute to the fire, with any other idea than "for future use" — for future consumption.
or to the loss occasioned thereby.
6.   The "hazardous goods" in question were "stored" in the bodega, as that word is the terms of the contract. The violation of the terms of the contract, by virtue
generally defined. of the provisions of the policy itself, terminated, at the election of either party,
7.   If Midland Ins examined the premises, even in the absence of a fire, and found he contractual relations. (Kyte vs. Commercial Union Assurance)
"hazardous goods" there, under the conditions above described, it would be 15.   Young paid a premium based upon the risk at the time the policy was issued.
justified, then and there, in declaring the policy null and of no effect because of Placing of the firecrackers in the building insured increased the risk, for which
Young’s violation of its terms. Young did not pay a premium based upon and, neither had Midland Ins issued a
8.   Midland Ins may repudiate is liability, even after the fire. If the "warranty" is a policy upon the theory of a different risk.
term of the contract, its violation causes a breach and justify 16.   Young was enjoying, if his contention may be allowed may be allowed, the
noncompliance/repudiation. benefits of an insurance policy upon one risk, whereas, as a matter of fact, it was
9.   Insurance Contracts are contracts of indemnity upon the terms and conditions issued upon an entirely different risk.
specified in the policy. The parties have a right to impose reasonable conditions 17.   Midland Ins had neither been paid nor had issued a policy to cover the increased
at the time of the making of the contract as they may deem wise and necessary. risk.
10.   The rate of premium is measured by the character of the risk assumed. The 18.   A substantial increase of risk and which is continued for a considerable period
insurance company, for a comparatively small consideration, undertakes to of time, is a direct and certain injury to the insurer, and changes the basis upon
guarantee the insured against loss or damage, upon the terms and conditions which the insurance contract rests. (Kyte vs. Commercial Union Assurance Co;
agreed upon, and upon no other, and when called upon to pay, in case of loss, Frost's Detroit Lumber vs. Millers' Mutual Ins. Co; Moore vs. Phoenix Ins.;
the insurer, therefore, may justly insist upon a fulfillment of these terms. Ferree vs. Oxford Fire & Life Ins.)
11.   If the insured cannot bring himself within the conditions of the policy, he is
not entitled to recover for the loss. Terms of the policy constitute the measure
of the insurer's liability, and in order to recover the insured must show himself
within those terms; and if it appears that the contract has been terminated by a
violation, on the part of the insured, of its conditions, then there can be no right
of recovery.
12.   Insured’s compliance with the terms of the contract is a condition precedent to
the right of recovery. If the insured has violated or failed to perform the
conditions of the contract, and such a violation or want of performance has
not been waived by the insurer, then the insured cannot recover. Courts are
not permitted to make contracts for the parties. The function and duty of the
courts consist simply in enforcing and carrying out he contracts actually made.
13.   GR: Insurance contracts of are construed most favorably to the insured, yet
contracts of Ins., like other contracts, are to be construed according to the sense
and meaning of the terms which the parties themselves have used. If such terms
are clear and unambiguous they must be taken and understood in their
plain, ordinary and popular sense. (Imperial Fire Ins. v. County of Coos; Kyte
v. Commercial Union Assurance) Conditions of insurance contracts, when
plainly expressed in a policy, are binding upon the parties and should be
enforced by the courts, if the evidence brings the case clearly within their
meaning and intent. It tends to bring the law itself into disrepute when, by astute
and subtle distinctions, a plain case is attempted to be taken without the
operation of a clear, reasonable, and material obligation of the contract. (Mack v.
Rochester German Ins.)
14.   Young argues that in view of the fact that the "storing" of the fireworks on the
premises of the insured did not contribute in any way to the damage occasioned
by the fire, he should be permitted to recover — that the "storing" of the
"hazardous goods" in no way caused injury to the defendant company. That
argument, however, is beside the question, if the "storing" was a violation of

You might also like